DIGESTS Remedial Law Review I - Civil Procedure

You might also like

Download as pdf or txt
Download as pdf or txt
You are on page 1of 229

Remedial Law I| Page 1

Jurisdiction
Duero vs. Court of Appeals
G.R. No. 131282, January 4, 2002
Quisumbing, J.

Facts:

Gabriel Duero filed before the RTC a complaint for recovery of possession and ownership against private respondent,
Bernardo Eradel, and two others alleging that Eradel entered and occupied Duero’s land. As shown in the tax
declaration, the land had an assessed value of P5,240. Eradel was not able to file an Answer and he was declared in
default. Duero presented his evidence ex parte and a judgment was rendered ordering Eradel to vacate the land.
Eradel filed a Motion for New Trial explaining that he turned over the complaint and summons to Artemio Laurente
in the honest belief that as landlord, the latter had a better right to the land and was responsible to defend any
adverse claim on it. Meanwhile, RED Conflict Case No. 1029, an administrative case between Duero and applicant-
contestants Romeo, Artemio and Jury Laurente, remained pending with the Office of the Regional Director of the
Department of Environment and Natural Resources in Davao City.

The RTC denied the Motion for New Trial. Eradel filed a Petition for Relief from Judgment which the RTC also
denied. In his Motion for Reconsideration, Eradel alleged that the RTC has no jurisdiction over the case, since the
value of the land was only P5,240 and therefore it was under the jurisdiction of the MTC. The RTC again denied the
said motion. On appeal, the CA reversed the RTC decreeing that Eradel was not estopped from assailing the
jurisdiction of the RTC.

Issue:

Whether or not private respondent Eradel was estopped from assailing the jurisdiction of the RTC?

Held:

No. The fundamental rule is that, the lack of jurisdiction of the court over an action cannot be waived by the parties,
or even cured by their silence, acquiescence or even by their express consent. Further, a party may assail the
jurisdiction of the court over the action at any stage of the proceedings and even on appeal. Even if private
respondent actively participated in the proceedings before said court, the doctrine of estoppel cannot still be properly
invoked against him because the question of lack of jurisdiction may be raised at anytime and at any stage of the
action. Precedents tell us that as a general rule, the jurisdiction of a court is not a question of acquiescence as a
matter of fact, but an issue of conferment as a matter of law. Also, neither waiver nor estoppel shall apply to confer
jurisdiction upon a court, barring highly meritorious and exceptional circumstances.

Here, petitioner, who claims ownership of a parcel of land, filed his complaint before a court without appropriate
jurisdiction. Defendant, a farmer whose tenancy status is still pending before the proper administrative agency
concerned, could have moved for dismissal of the case on jurisdictional grounds. But the farmer as defendant therein
could not be expected to know the nuances of jurisdiction and related issues. This farmer, who is now the private
respondent, ought not to be penalized when he claims that he made an honest mistake when he initially submitted
his motions before the RTC, before he realized that the controversy was outside the RTC’s cognizance but within the
jurisdiction of the municipal trial court. To hold him in estoppel as the RTC did would amount to foreclosing his
avenue to obtain a proper resolution of his case.
Remedial Law I| Page 2

Donato vs. Court of Appeals


G.R. No. 129638, December 8, 2003
Austria-Martinez, J.

Facts:

Petitioner Antonio T. Donato filed a complaint for forcible entry and unlawful detainer against 43 named
defendants. He alleged that the defendants had oral contract of lease that expired at the end of each month but were
impliedly renewed under the same terms by mere acquiescence or tolerance. Sometime in 1992, they stopped paying
rent and in 1994, Donato sent them a written demand to vacate; and the non-compliance with said demand letter
constrained him to file the ejectment case against them. Of the 43 named defendants, only 20 filed a consolidated
Answer. They contended that they cannot be evicted because the Urban Land Reform Law guarantees security of
tenure and priority right to purchase the subject property. The MeTC rendered two separate judgments: a demand
to vacate on the non-answering defendants and a judgment in favor of the 20 respondents sustaining their rights
under the Land Reform Law and thereby, dismissing petitioner’s complaint. Petitioner appealed to the RTC which
affirmed the dismissal. The CA also dismissed the petition for review on two grounds: a) the certification of non-
forum shopping was signed by petitioner’s counsel and not by petitioner himself and, (b) the only annex to the
petition is a certified copy of the questioned decision but copies of the pleadings and other material portions of the
record as would support the allegations of the petition are not annexed. Petitioner’s Motion for Reconsideration on
the order of dismissal was likewise denied. Petitioner then filed a petition for review on certiorari before the
Supreme Court against the CA’s order of dismissal.

Issue:

Whether or not a petition for review (Rule 45) was the proper remedy in assailing the CA’s order of dismissal based
on procedural grounds?

Held:

No. [I]n order to determine whether the recourse of petitioners is proper or not, it is necessary to draw a line
between an error of judgment and an error of jurisdiction. An error of judgment is one which the court may commit
in the exercise of its jurisdiction, and which error is reviewable only by an appeal. On the other hand, an error of
jurisdiction is one where the act complained of was issued by the court, officer or a quasi-judicial body without or in
excess of jurisdiction, or with grave abuse of discretion which is tantamount to lack or in excess of jurisdiction. This
error is correctible only by the extraordinary writ of certiorari.

Inasmuch as the present petition principally assails the dismissal of the petition on ground of procedural flaws
involving the jurisdiction of the court a quo to entertain the petition, it falls within the ambit of a special civil action
for certiorari under Rule 65 of the Rules of Court.
Remedial Law I| Page 3

Gonzaga vs. Court of Appeals


G.R. No. 144025, December 27, 2002.
Corona, J.

Facts:

Petitioner-spouses, Spouses Rene and Lerio Gonzaga, purchased a parcel of land from private respondent Lucky
Homes, Inc. Said lot was specifically denominated as Lot No. 19 and was mortgaged to the Social Security System
(SSS) as security for their housing loan. Spouses Gonzaga then started the construction of their house, not on Lot
No. 19 but on Lot No. 18, as Lucky Homes mistakenly identified Lot No. 18 as Lot No. 19. Upon realizing its error,
Lucky Homes, through its general manager, informed Spouses Gonzaga of such mistake but the latter offered to buy
Lot No. 18 in order to widen their premises. Thus, Spouses Gonzaga continued with the construction of their house.
However, they defaulted in the payment of their housing loan from SSS. Consequently, Lot No. 19 was foreclosed by
SSS. After Lot No. 19 was foreclosed, Spouses Gonzaga offered to swap Lot Nos. 18 and 19 and demanded from
Lucky Homes that their contract of sale be reformed and another deed of sale be executed with respect to Lot No. 18,
considering that their house was built therein. However, Lucky Homes refused. This prompted Spouses Gonzaga to
file an action for reformation of contract and damages with the RTC. The RTC dismissed the complaint. A writ of
execution was then issued by the RTC. Spouses Gonzaga filed an urgent motion to recall writ of execution, alleging
that the RTC had no jurisdiction to try the case as it was vested in the Housing and Land Use Regulatory Board
(HLURB) pursuant to PD 957 (The Subdivision and Condominium Buyers Protective Decree). Conformably, Spouses
Gonzaga filed a new complaint against Lucky Homes with the HLURB. Likewise, Spouses Gonzaga filed before the
CA a petition for annulment of judgment premised on the ground that the RTC had no jurisdiction to try and hear
the case. The CA denied the petition relying mainly on the jurisprudential doctrine of estoppel.

Issue:

Whether or not the petitioners (Spouses Gonzaga) were estopped in assailing the jurisdiction of the RTC?

Held:

Yes. While an order or decision rendered without jurisdiction is a total nullity and may be assailed at any stage,
active participation in the proceedings in the court which rendered the order or decision will bar such party from
attacking its jurisdiction. As held in the leading case of Tijam vs. Sibonghanoy:

“A party may be estopped or barred from raising a question in different ways and for different reasons. Thus we speak
of estoppel in pais, or estoppel by deed or by record, and of estoppel by laches.

“It has been held that a party cannot invoke the jurisdiction of a court to secure affirmative relief against his opponent
and, after obtaining or failing to obtain such relief, repudiate, or question that same jurisdiction x x x x [T]he question
whether the court had jurisdiction either of the subject matter of the action or of the parties was not important in such
cases because the party is barred from such conduct not because the judgment or order of the court is valid and
conclusive as an adjudication, but for the reason that such a practice can not be tolerated—obviously for reasons of
public policy.”

Here, it was petitioners themselves who invoked the jurisdiction of the court a quo by instituting an action for
reformation of contract against private respondents. It appears that, in the proceedings before the trial court,
petitioners vigorously asserted their cause from start to finish. Not even once did petitioners ever raise the issue of
the court’s jurisdiction during the entire proceedings which lasted for two years. It was only after the trial court
rendered its decision and issued a writ of execution against them in 1998 did petitioners first raise the issue of
jurisdiction—and it was only because said decision was unfavorable to them. Petitioners thus effectively waived
their right to question the court’s jurisdiction over the case they themselves filed.
Remedial Law I| Page 4

Escobal vs. Garchitorena


G.R. No. 124644, February 5, 2004.
Callejo, Sr.

Facts:

Arnel Escobal, petitioner, is a member of the Philippine National Police. When petitioner was conducting
surveillance operations on drug trafficking at the Sa Harong Cafè Bar and Restaurant, he somehow got involved in a
shooting incident, resulting in the death of one Rodney Rafael N. Nueca. An Information was then filed with the
RTC charging petitioner and a certain Jun Bombita with murder. When petitioner commenced the presentation of
his evidence, he filed a Motion to Dismiss the case arguing that since he committed the crime in the performance of
his duties, the Sandiganbayan had exclusive jurisdiction over the case. The RTC denied the motion but ordered the
conduct of a preliminary hearing to determine whether or not the crime charged was committed by the petitioner in
relation to his office as a member of the PNP. Thereafter, the RTC issued an Order declaring that the petitioner
committed the crime charged while not in the performance of his official function. The trial court added that upon
the enactment of R.A. No. 7975, the issue had become moot and academic. The amendatory law transferred the
jurisdiction over the offense charged from the Sandiganbayan to the RTC since the petitioner did not have a salary
grade of “27” as provided for in or by Section 4(a)(1), (3) thereof. The RTC, however, reversed itself in the Motion for
Reconsideration and ordered the public prosecutor to file a Re-Amended Information and to allege that the offense
charged was committed by the petitioner in the performance of his duties/functions or in relation to his office; and,
conformably to R.A. No. 7975, to thereafter transmit the same, as well as the complete records with the stenographic
notes, to the Sandiganbayan. The Presiding Justice of the Sandiganbayan ordered the return of the records to the
RTC. It reasoned that the RTC retained jurisdiction over the case since petitioner had a salary grade of “23.”
Furthermore, the prosecution had already rested its case and the petitioner had commenced presenting his evidence
in the RTC; following the rule on continuity of jurisdiction, the latter court should continue with the case and render
judgment therein after trial. Petitioner then filed a petition for certiorari assailing the Order of the Sandiganbayan.

Issue:

Whether or not the Sandiganbayan has jurisdiction over a criminal case of murder against a member of the PNP
with salary grade of 23?

Held:

No. For the Sandiganbayan to have exclusive jurisdiction over crimes committed by public officers in relation to
their office, it is essential that the facts showing the intimate relation between the office of the offender and the
discharge of official duties must be alleged in the Information. It is not enough to merely allege in the Information
that the crime charged was committed by the offender in relation to his office because that would be a conclusion of
law. The amended Information filed with the RTC against the petitioner does not contain any allegation showing the
intimate relation between his office and the discharge of his duties.

Moreover, even if the offender committed the crime charged in relation to his office but occupies a position
corresponding to a salary grade below “27,” the proper Regional Trial Court or Municipal Trial Court, as the case
may be, shall have exclusive jurisdiction over the case. In this case, the petitioner was a Police Senior Inspector,
with salary grade “23.” He was charged with homicide punishable by reclusion temporal. Hence, the RTC had
exclusive jurisdiction over the crime charged.
Remedial Law I| Page 5

Agan, Jr. vs. Philippine International Air Terminals Co., Inc. (PIATCO)
G.R. No. 155001, January 21, 2004.
Puno, J.
Resolution

Facts:

Asia’s Emerging Dragon Corp. (AEDC) submitted an unsolicited proposal to the Philippine Government through the
Department of Transportation and Communication (DOTC) and Manila International Airport Authority (MIAA) for
the construction and development of the NAIA IPT III under a build-operate-and-transfer arrangement. The
DOTC/MIAA invited the public for submission of competitive and comparative proposals to the unsolicited proposal
of AEDC. The Paircargo consortium then submitted their competitive proposal to the Prequalification Bids and
Awards Committee (PBAC). After finding that the Paircargo Consortium submitted a bid superior to the unsolicited
proposal of AEDC and after failure by AEDC to match the said bid, the DOTC issued the notice of award for the
NAIA IPT III project to the Paircargo Consortium, which later organized into herein respondent PIATCO. The
Government, through then DOTC Secretary Arturo T. Enrile, and PIATCO, through its President, Henry T. Go,
signed the “Concession Agreement for the Build- Operate-and-Transfer Arrangement of the Ninoy Aquino
International Airport Passenger Terminal III” (1997 Concession Agreement). This agreement was later on amended
and supplemented. Thereafter, various petitions were filed before the Supreme Court to annul the 1997 Concession
Agreement, the ARCA and the Supplements and to prohibit the public respondents DOTC and MIAA from
implementing them. The SC granted the petitions and declared the 1997 Concession Agreement, the ARCA and the
Supplements null and void. Respondents now seek reversal of the said decision. One of their contentions was that
the principle of hierarchy of courts precludes the SC from taking primary jurisdiction over the case.

Issue:

Whether or not the Supreme Court can take primary jurisdiction over the case?

Held:

Yes. The rule on hierarchy of courts in cases falling within the concurrent jurisdiction of the trial courts and
appellate courts generally applies to cases involving warring factual allegations. For this reason, litigants are
required to repair to the trial courts at the first instance to determine the truth or falsity of these contending
allegations on the basis of the evidence of the parties. Cases which depend on disputed facts for decision cannot be
brought immediately before appellate courts as they are not triers of facts.

It goes without saying that when cases brought before the appellate courts do not involve factual but legal questions,
a strict application of the rule of hierarchy of courts is not necessary. As the cases at bar merely concern the
construction of the Constitution, the interpretation of the BOT Law and its Implementing Rules and Regulations on
undisputed contractual provisions and government actions, and as the cases concern public interest, the Supreme
Court resolved to take primary jurisdiction over them. This choice of action follows the consistent stance of the Court
to settle any controversy with a high public interest component in a single proceeding and to leave no root or branch
that could bear the seeds of future litigation. The suggested remand of the cases at bar to the trial court will stray
away from this policy.
Remedial Law I| Page 6

Liga ng mga Barangay National vs. Atienza, Jr.


G.R. No. 154599, January 21, 2004.
Davide, Jr., CJ.

Facts:

Petitioner Liga ng mga Barangay National (Liga) is the national organization of all the barangays in the Philippines
which constitutes the duly elected presidents of highly-urbanized cities, provincial chapters, the metropolitan
Manila Chapter, and metropolitan political subdivision chapters. Section 493 of the Local Government Code
empowers the Liga to adopt its Constitution and By-Laws to govern its internal organization. In implementing its
Election Code, the Liga set October 21, 2000 for the synchronized elections for highly urbanized city chapters, such
as the Liga Chapter of Manila, together with independent component city, provincial, and metropolitan chapters.
Meanwhile, the respondent City Council of Manila enacted Ordinance No. 8039, Series of 2002, providing, among
other things, for the election of representatives of the District Chapters in the City Chapter of Manila and setting
the elections for both chapters thirty days after the barangay elections. Thereafter, the Liga filed a petition for
certiorari before the Supreme Court assailing the validity of the said ordinance as it contradicts the Liga Election
Code. On the other hand, the respondents seek the dismissal of the petition on the following grounds: (1) the
petitioner availed the wrong remedy as certiorari is available only to one aggrieved by the decision of a tribunal,
officer, or board exercising judicial or quasi-judicial functions; since the City Council and the City Mayor exercise
legislative function and executive functions, respectively, certiorari cannot be availed of; (2) the SC cannot take
cognizance of the case based on the doctrine of hierarchy of courts.

Issues:

(1) Whether or not the petitioner availed of the wrong remedy (Rule 65)?
(2) Whether or not the Supreme Court can take cognizance of the case?

Held:

(1) Yes. The enactment by the City Council of Manila of the assailed ordinance and the issuance by respondent
Mayor of the questioned executive order were done in the exercise of legislative and executive functions,
respectively, and not of judicial or quasi-judicial functions. On this score alone, certiorari will not lie. Second,
although the instant petition is styled as a petition for certiorari, in essence, it seeks the declaration by this Court of
the unconstitutionality or illegality of the questioned ordinance and executive order. It, thus, partakes of the nature
of a petition for declaratory relief over which this Court has only appellate, not original, jurisdiction. As such, this
petition must necessary fail, as this Court does not have original jurisdiction over a petition for declaratory relief
even if only questions of law are involved.

(2) No. The Supreme Court’s original jurisdiction to issue a writ of certiorari (as well as of prohibition, mandamus,
quo warranto, habeas corpus and injunction) is not exclusive, but is concurrent with the Regional Trial Courts and
the Court of Appeals in certain cases. This concurrence of jurisdiction is not, however, to be taken as according to
parties seeking any of the writs an absolute, unrestrained freedom of choice of the court to which application
therefore will be directed. There is after all a hierarchy of courts. That hierarchy is determinative of the venue of
appeals, and also serves as a general determinant of the appropriate forum for petitions for the extraordinary writs.
A becoming regard of that judicial hierarchy most certainly indicates that petitions for the issuance of extraordinary
writs against first level (“inferior”) courts should be filed with the Regional Trial Court, and those against the latter,
with the Court of Appeals. A direct invocation of the Supreme Court’s original jurisdiction to issue these writs
should be allowed only when there are special and important reasons therefor, clearly and specifically set out in the
petition. This is [an] established policy. It is a policy necessary to prevent inordinate demands upon the Court’s time
and attention which are better devoted to those matters within its exclusive jurisdiction, and to prevent further
overcrowding of the Court’s docket.
Remedial Law I| Page 7

Manila Bankers Life Insurance Corporation vs. Ng Kok Wei


G.R. No. 139791, December 12, 2003
Sandoval-Gutierrez, J.

Facts:

Eddy Ng Kok Wei, respondent, sent a Letter of Intent addressed to Manila Bankers Life Insurance Corporation,
petitioner, to purchase a condominium unit at Valle Verde Terraces. When respondent was able to pay 90% of the
purchase price, petitioner’s President executed a Contract to Sell in favor of the respondent. The contract expressly
states that the subject condominium unit “shall substantially be completed and delivered” to the respondent “within
fifteen (15) months” from February 8, 1989 or on May 8, 1990, and that “(S)hould there be no substantial completion
and fail(ure) to deliver the unit on the date specified, a penalty of 1% of the total amount paid (by respondent) shall
be charged against (petitioner)”. In October 1990, the subject condominium unit was still inlivable. Exasperated,
respondent demanded from petitioner payment for damages he sustained. As the demand went unheeded,
respondent filed a complaint for specific performance and damages before the RTC. During the pendency of the case,
respondent finally accepted the condominium unit and therefore his cause of action has been limited to his claim for
damages. The RTC ruled in favor of the respondent which the CA affirmed. In a petition for review before the SC,
petitioner contends that the trial court has no jurisdiction over the instant case.

Issue:

Whether or not petitioner Manila Bankers is estopped in assailing the jurisdiction of the RTC?

Held:

Yes. Based on Presidential Decree No. 1344, as amended, it is the HLURB which has jurisdiction over complaints
for specific performance with damages by a lot or condominium unit buyer against the owner or developer. While it
may be true that the trial court is without jurisdiction over the case, petitioner’s active participation in the
proceedings estopped it from assailing such lack of it. The Court held that it is an undesirable practice of a party
participating in the proceedings and submitting its case for decision and then accepting the judgment, only if
favorable, and attacking it for lack of jurisdiction, when adverse. Here, petitioner failed to raise the question of
jurisdiction before the trial court and the Appellate Court. In effect, petitioner confirmed and ratified the trial
court’s jurisdiction over this case. Certainly, it is now in estoppel and can no longer question the trial court’s
jurisdiction.
Remedial Law I| Page 8

Office of the Court Administrator vs. Sardido


A.M. No. MTJ-01-1370, April 25, 2003 (Formerly A.M. No. 00-11-238-MTC)
Carpio, J.

Facts:

Judge Braulio Hurtado, an RTC Judge, was charged with the crime of Falsification by Private Individual and Use of
Falsified Document which case was assigned to Judge Agustin Sardido, the presiding judge of the Municipal Trial
Court of Koronadal, South Cotabato. The crime arose from a deed of absolute sale which was notarized by Judge
Hurtado at the time when he was still the Clerk of Court of RTC-Koronadal and ex-officio notary public. Judge
Hurtado filed a motion praying that the criminal complaint against him be forwarded to the Supreme Court. Judge
Hurtado claimed that Circular No. 3-89 requires “all cases involving justices and judges of the lower courts, whether
or not such complaints deal with acts apparently unrelated to the discharge of their official functions, such as acts of
immorality, estafa, crimes against persons and property, etc.” to be forwarded to the Supreme Court. Judge Sardido
then excluded the name of Judge Hurtado from the Information and the case against him was forwarded to the
Court Administrator for appropriate action. The Court Administrator returned the case to MTC Koronadal and
opined that Circular No. 3-89 refers only to administrative complaints filed with the IBP against justices and judges
of lower courts. The Circular does not apply to criminal cases filed before trial courts against such justices and
judges.

Issue:

Whether or not the trial court has jurisdiction over a criminal case filed against a judge?

Held:

Yes. Circular No. 3-89 does not refer to criminal cases against erring justices of appellate courts or judges of lower
courts. Trial courts retain jurisdiction over the criminal aspect of offenses committed by justices of appellate courts
and judges of lower courts. This is clear from the Circular directing the IBP, and not the trial courts, to refer all
administrative cases filed against justices of appellate courts and judges of lower courts to the Supreme Court. The
case filed against Judge Hurtado is not an administrative case filed with the IBP. It is a criminal case filed with the
trial court under its jurisdiction as prescribed by law.
Remedial Law I| Page 9

Katon vs. Palanca, Jr.


G.R. No. 151149, September 7, 2004
Panganiban, J.

Facts:

In 1963, George Katon filed a request with the District Office of the Bureau of Forestry in Puerto Princesa, Palawan,
for the re-classification of a piece of real property known as Sombrero Island, located in Tagpait, Aborlan, Palawan.
A survey was conducted and in 1965, the subject land was certified and released as an agricultural land. Records
show that respondents (Fresnillo, Gapilango and Palanca) filed and were issued homestead patents for various
hectares within the Sombrero Island. Petitioner now assails the validity of the homestead patents and asked for the
reconveyance of the lands to him before the RTC. Respondents filed a Motion to Dismiss on the ground of the alleged
defiance by petitioner of the trial court’s Order to amend his Complaint so he could thus effect a substitution by the
legal heirs of the deceased, Respondent Gapilango. The Motion to Dismiss was granted by the RTC in its Order
dated July 29, 1999. Petitioner then filed a Petition for Certiorari before the CA. The CA, instead of ruling only on
the alleged grave abuse of discretion, ruled on the merits. On the MR, the CA acknowledged that it had erred when
it ruled on the merits of the case. But still dismissed the case pursuant to its “residual prerogative” under Section 1
of Rule 9 of the Rules of Court. It ruled that petitioner’s action was brought 24 years after the issuance of Palanca’s
homestead patent. Under the Public Land Act, such action should have been taken within ten years from the
issuance of the homestead certificate of title. Second, it appears from the submission of petitioner himself that
Respondents Fresnillo and Palanca had been occupying six hectares of the island since 1965, or 33 years before he
took legal steps to assert his right to the property. His action was filed beyond the 30-year prescriptive period under
Articles 1141 and 1137 of the Civil Code. Petitioner contends that the CA erroneously invoked its “residual
prerogatives” under Section 1 of Rule 9 of the Rules of Court when it motu proprio dismissed the Petition for lack of
jurisdiction and prescription. According to him, residual prerogative refers to the power that the trial court, in the
exercise of its original jurisdiction, may still validly exercise even after perfection of an appeal. It follows that such
powers are not possessed by an appellate court.

Issue:

Whether or not the Court of Appeals properly exercised its power of residual prerogative?

Held:

Yes. Petitioner has confused what the CA adverted to as its “residual prerogatives” under Section 1 of Rule 9 of the
Rules of Court with the “residual jurisdiction” of trial courts over cases appealed to the CA.

Under Section 1 of Rule 9 of the Rules of Court, defenses and objections not pleaded either in a motion to dismiss or
in the answer are deemed waived, except when (1) lack of jurisdiction over the subject matter, (2) litis pendentia, (3)
res judicata and (4) prescription are evident from the pleadings or the evidence on record. In the four excepted
instances, the court shall motu proprio dismiss the claim or action.

On the other hand, “residual jurisdiction” is embodied in Section 9 of Rule 41 of the Rules of Court. The “residual
jurisdiction” of trial courts is available at a stage in which the court is normally deemed to have lost jurisdiction over
the case or the subject matter involved in the appeal. This stage is reached upon the perfection of the appeals by the
parties or upon the approval of the records on appeal, but prior to the transmittal of the original records or the
records on appeal. In either instance, the trial court still retains its so-called residual jurisdiction to issue protective
orders, approve compromises, permit appeals of indigent litigants, order execution pending appeal, and allow the
withdrawal of the appeal.

In this case, petitioner’s complaint did not sufficiently make a case for declaration of nullity of the free patent and
certificate of title or, alternatively, for reconveyance. Also, the action should be dismissed for being time-barred.
Remedial Law I| Page 10

Figueroa vs. People


G.R. No. 147406, July 14, 2008

Facts:

Petitioner was charged with an information for reckless imprudence resulting in homicide before the RTC of
Bulacan. Petitioner was convicted by the RTC. On appeal, petitioner questioned for the first time the trial court’s
jurisdiction. The Court of Appeals, however, found petitioner estopped from questioning the jurisdiction of the RTC
as he had actively participated in the trial. The CA also affirmed his conviction.

Issue:

Whether or not the petitioner is already estopped in assailing the jurisdiction of the RTC?

Held:

No. As the imposable penalty for the crime charged was less than 6 years, jurisdiction to hear and try the same is
conferred on the Municipal Trial Courts (MTCs). Clearly, therefore, the RTC of Bulacan does not have jurisdiction
over Criminal Case No. 2235-M-94.

The general rule should be, as it has always been, that the issue of jurisdiction may be raised at any stage of the
proceedings, even on appeal, and is not lost by waiver or by estoppel. Estoppel by laches, to bar a litigant from
asserting the court’s absence or lack of jurisdiction, only supervenes in exceptional cases similar to the factual
milieu of Tijam v. Sibonghanoy. Indeed, the fact that a person attempts to invoke unauthorized jurisdiction of a
court does not estop him from thereafter challenging its jurisdiction over the subject matter, since such jurisdiction
must arise by law and not by mere consent of the parties. This is especially true where the person seeking to invoke
unauthorized jurisdiction of the court does not thereby secure any advantage or the adverse party does not suffer
any harm.

Here, the petitioner is in no way estopped by laches in assailing the jurisdiction of the RTC, considering that he
raised the lack thereof in his appeal before the appellate court. At that time, no considerable period had yet elapsed
for laches to attach. True, delay alone, though unreasonable, will not sustain the defense of “estoppel by laches”
unless it further appears that the party, knowing his rights, has not sought to enforce them until the condition of the
party pleading laches has in good faith become so changed that he cannot be restored to his former state, if the
rights be then enforced, due to loss of evidence, change of title, intervention of equities, and other causes.

Estoppel, being in the nature of a forfeiture, is not favored by law. It is to be applied rarely—only from necessity,
and only in extraordinary circumstances. The doctrine must be applied with great care and the equity must be
strong in its favor.
Remedial Law I| Page 11

Serana vs. Sandiganbayan


G.R. No. 162059. January 22, 2008.

Facts:

Petitioner was appointed Student Regent of University of the Philippines and discussed with then President
Estrada the proposed renovation of Vinzon’s Hall. She and her siblings registered with the SEC the Office of the
Student Regent Foundation, Inc. (OSRFI). The OSRFI received from President Estrada 15 million pesos as financial
assistance for the proposed renovation which failed to materialize.

The successor of the petitioner filed a complaint for Malversation of Public Funds with the Office of the
Ombudsman. The Ombudsman found probable cause and charged the petitioner with Estafa. Petitioner filed a
motion to dismiss on the ground that the Sandiganbayan has no jurisdiction over her person as she was not a public
officer.

The Sandiganbayan denied her motion. Hence, this petition.

Issues:

(1) Whether or not the Sandiganbayan has jurisdiction over the subject matter of the offense charged?

(2) Whether or not Serana (a student regent) may be considered as a public officer?

Held:

Yes. Petitioner avers that estafa is not one of the crimes enumerated under Sec. 4(1) of PD 1606, as amended.
Petitioner is mistaken. Sec. 4 must be read in consonance with the other paragraphs of Sec. 4. It must be isolated
with the succeeding paragraphs.
Sec. 4(b) of PD 1606, as amended states -
b. Other offenses or felonies whether simple or
complexed with other crimes committed by the public officials and employees mentioned in subsection a of this
section in relation to their office;

Evidently, the Sandiganbayan has jurisdiction over other felonies committed by public officials in relation to their
office.

(2) Yes.

A public office is the right, authority, and duty created and conferred by law, by which for a given period, either
fixed by law or enduring at the pleasure of the creating power, an individual is invested with some portion of the
sovereign functions of the government, to be exercised by him for the benefit of the public. The individual so invested
is a public officer.

Sec. 4(A)(1)(g) explicitly vests the Sandiganbayan with jurisdiction over Presidents, directors or trustees, or
managers of government owned or controlled corporations, state universities or educational institutions or
foundations. Petitioner falls under this category. The Board of Regents performs functions similar to those of a board
of trustees of a non-stock corporation. By express mandate of the law, petitioner is, indeed, a public officer as
contemplated in PD No. 1606, as amended.

Moreover, compensation is not an essential element of public office.

Petitioner likewise aver that the offense was done in her private capacity. This should also likewise be answered in
the negative. As long as the public office is facilitated to commit the crime, the Sandiganbayan has jurisdiction over
the offense.
Remedial Law I| Page 12

Pat-og, Sr. vs. Civil Service Commission
G.R. No. 198755, June 5, 2013

Facts:

Complainant Bang-on was second year high school student who was punched by the petitioner on his stomach for
not falling in line properly.

Complainant filed an administrative case to the CSC-CAR. CSC-CAR found the petitioner guilty of simple
misconduct. Pat-Og then filed an appeal before the CSC which affirmed the finding of CSC-CAR and ordered the
dismissal of Pat-Og. Pat-Og then filed an MR questioning for the first time the jurisdiction of CSC over the case. He
contended that administrative charges against a public school teacher should have been initially heard by a
committee to be constituted pursuant to the Magna Carta for Public School Teachers.

The CSC dismissed the MR. It ruled that Pat-og was estopped from challenging its jurisdiction considering that he
actively participated in the administrative proceedings against him, raising the issue of jurisdiction only after his
appeal was dismissed by the CSC. On appeal, the appellate court affirmed the decision of the CSC.

Issues:

(1) Whether or not the CSC has jurisdiction over an administrative case involving a public school teacher?
(2) Whether or not petitioner is already estopped in questioning the jurisdiction of CSC?

Held:

(1) Yes. In the case of Puse vs. Santos-Puse, it was held that the CSC, Department of Education and the Board of
Professional Teachers-Professional Regulatory Commission (PRC) have concurrent jurisdiction over administrative
cases against public school teachers.

Under Article IX-B of the 1987 Constitution, the CSC is the body charged with the establishment and
administration of a career civil service which embraces all branches and agencies of the government.

Executive Order (E.O.) No. 292 (the Administrative Code of 1987) and Presidential Decree (P.D.) No. 807 (the Civil
Service Decree of the Philippines) expressly provide that the CSC has the power to hear and decide administrative
disciplinary cases instituted with it or brought to it on appeal. Thus, the CSC, as the central personnel agency of the
government, has the inherent power to supervise and discipline all members of the civil service, including public
school teachers.

Indeed, under Section 9 of R.A. No. 4670, the jurisdiction over administrative cases of public school teachers is
lodged with the investigating committee constituted therein.14 Also, under Section 23 of R.A. No. 7836 (the
Philippine Teachers Professionalization Act of 1994), the Board of Professional Teachers is given the power, after
due notice and hearing, to suspend or revoke the certificate of registration of a professional teacher for causes
enumerated therein.

Concurrent jurisdiction is that which is possessed over the same parties or subject matter at the same
time by two or more separate tribunals. When the law bestows upon a government body the jurisdiction
to hear and decide cases involving specific matters, it is to be presumed that such jurisdiction is
exclusive unless it be proved that another body is likewise vested with the same jurisdiction, in which
case, both bodies have concurrent jurisdiction over the matter.

Where concurrent jurisdiction exists in several tribunals, the body that first takes cognizance of the
complaint shall exercise jurisdiction to the exclusion of the others. In this case, it was CSC which first
acquired jurisdiction over the case because the complaint was filed before it. Thus, it had the authority to proceed
and decide the case to the exclusion of the DepEd and the Board of Professional Teachers.
Remedial Law I| Page 13

In CSC v. Alfonso, it was held that special laws, such as R.A. No. 4670, do not divest the CSC of its inherent power
to supervise and discipline all members of the civil service, including public school teachers. Pat-og, as a public
school teacher, is first and foremost, a civil servant accountable to the people and answerable to the CSC for
complaints lodged against him as a public servant. To hold that R.A. No. 4670 divests the CSC of its power to
discipline public school teachers would negate the very purpose for which the CSC was established and would
impliedly amend the Constitution itself.

To further drive home the point, it was ruled in CSC v. Macud that R.A. No. 4670, in imposing a separate set of
procedural requirements in connection with administrative proceedings against public school teachers, should be
construed to refer only to the specific procedure to be followed in administrative investigations conducted by the
DepEd. By no means, then, did R.A. No. 4670 confer an exclusive disciplinary authority over public school teachers
on the DepEd.

(2) Yes. At any rate, granting that the CSC was without jurisdiction, the petitioner is indeed estopped from raising
the issue. Although the rule states that a jurisdictional question may be raised at any time, such rule admits of the
exception where, as in this case, estoppel has supervened. Here, instead of opposing the CSC’s exercise of
jurisdiction, the petitioner invoked the same by actively participating in the proceedings before the CSC-CAR and by
even filing his appeal before the CSC itself; only raising the issue of jurisdiction later in his motion for
reconsideration after the CSC denied his appeal. This Court has time and again frowned upon the undesirable
practice of a party submitting his case for decision and then accepting the judgment only if favorable, but attacking
it for lack of jurisdiction when adverse.
Remedial Law I| Page 14

Boston Equity Resources, Inc. vs. Court of Appeals


G.R. No. 173946. June 19, 2013.

Facts:

In 1997, Boston Equity Resources, Inc. filed a complaint for sum of money against the spouses Manuel and Lolita
Toledo. In March 1998, Lolita Toleda filed an Answer which she amended in May 1998 alleging that her husband
was already dead. Petitioner then filed a Motion for Substitution. In 2004, after the presentation of evidence of the
petitioner, respondent filed a Motion to Dismiss (instead of a demurrer); one of the grounds alleged being invoked
was that the trial court never acquired jurisdiction over the person of Manuel. The RTC denied the MTD on the
ground that “defendants’ attack on the jurisdiction is now barred by estoppel by laches” since respondent failed to
raise the issue despite several chances to do so. The respondent filed a petition for certiorari before the CA. The CA
granted the petition because according to it, jurisdiction may be raised at any stage of the proceeding, even for the
first time on appeal. By timely raising the issue on jurisdiction in her motion to dismiss, respondent is not estopped
from raising the question on jurisdiction.

Issue:

Whether or not jurisdictional estoppel applies to jurisdiction over the person of the defendant?

Held:

No. The aspect of jurisdiction which may be barred from being assailed as a result of estoppel by laches is
jurisdiction over the subject matter. Here, what respondent was questioning in her motion to dismiss before the trial
court was that court’s jurisdiction over the person of defendant Manuel. Thus, the principle of estoppel by laches
finds no application in this case. Instead, the principles relating to jurisdiction over the person of the parties are
pertinent herein.

Since the defense of lack of jurisdiction over the person of a party to a case is not one of those defenses which are not
deemed waived under Section 1 of Rule 9, such defense must be invoked when an answer or a motion to dismiss is
filed in order to prevent a waiver of the defense. If the objection is not raised either in a motion to dismiss or in the
answer, the objection to the jurisdiction over the person of the plaintiff or the defendant is deemed waived by virtue
of the first sentence of the above-quoted Section 1 of Rule 9 of the Rules of Court.
Remedial Law I| Page 15

People of the Philippines vs. Henry Go.


G.R. No. 168539. March 25, 2014

Facts:

Respondent was charged with violation of RA 3019 for conspiring with then DOTC Sec. Arturo Enrile, now deceased,
in relation to contracts entered by them which is grossly disadvantageous to the government.

Respondent filed a motion to quash on the ground of lack of jurisdiction as he is only a private individual and not a
public officer. Independently of the deceased Secretary Enrile, the public officer with whom he was alleged to have
conspired, respondent, who is not a public officer nor was capacitated by any official authority as a government
agent, may not be prosecuted for violation of Section 3(g) of R.A. 3019.

Issue:

Whether or not respondent, a private person, may be indicted for conspiracy in violating Section 3(g) of R.A. 3019
even if the public officer, with whom he was alleged to have conspired, has died prior to the filing of the Information?

Held:

Yes. It is true that by reason of Secretary Enrile's death, there is no longer any public officer with whom respondent
can be charged for violation of R.A. 3019. It does not mean, however, that the allegation of conspiracy between them
can no longer be proved or that their alleged conspiracy is already expunged. The only thing extinguished by the
death of Secretary Enrile is his criminal liability. His death did not extinguish the crime nor did it remove the basis
of the charge of conspiracy between him and private respondent. Stated differently, the death of Secretary Enrile
does not mean that there was no public officer who allegedly violated Section 3 (g) of R.A. 3019. In fact, the Office of
the Deputy Ombudsman for Luzon found probable cause to indict Secretary Enrile for infringement of Sections 3 (e)
and (g) of R.A. 3019. Were it not for his death, he should have been charged.

The requirement before a private person may be indicted for violation of Section 3(g) of R.A. 3019, among others, is
that such private person must be alleged to have acted in conspiracy with a public officer. The law, however, does
not require that such person must, in all instances, be indicted together with the public officer. If circumstances
exist where the public officer may no longer be charged in court, as in the present case where the public officer has
already died, the private person may be indicted alone.
Remedial Law I| Page 16

City of Manila vs. Judge Cuerdo


G.R. No. 175723 February 4, 2014

Facts:

Respondent City of Manila assessed taxes SM and the other respondents. Because the payment of taxes was a
precondition for the issuance of their business permits, they were constrained to pay the said taxes under protest.
Respondents then filed a Complaint before the RTC of Pasay City for the Refund or Recovery of Illegally and/or
Erroneously-Collected Local Business Tax, Prohibition with Prayer to Issue TRO and Writ of Preliminary
Injunction. The RTC granted private respondents' application for a writ of preliminary injunction. Petitioner filed an
MR but was denied. Thus, the petitioner filed a special civil action for certiorari with the CA assailing the RTC’s
order granting the writ of preliminary injunction. The CA denied the petition on the ground that it has no
jurisdiction over the petition as this is lodged with the CTA.

Issue:

Whether or not the CA has jurisdiction over a special civil action for certiorari which assails the interlocutory order
issued by the RTC in a local tax case?

Held:

No.

Section 5 (1), Article VIII of the 1987 Constitution grants power to the Supreme Court, in the exercise of its original
jurisdiction, to issue writs of certiorari, prohibition and mandamus. With respect to the Court of Appeals, Section 9
(1) of Batas Pambansa Blg. 129 (BP 129) gives the appellate court, also in the exercise of its original jurisdiction, the
power to issue, among others, a writ of certiorari,whether or not in aid of its appellate jurisdiction. As to Regional
Trial Courts, the power to issue a writ of certiorari, in the exercise of their original jurisdiction, is provided under
Section 21 of BP 129.

The foregoing notwithstanding, while there is no express grant of such power, with respect to the CTA, Section 1,
Article VIII of the 1987 Constitution provides, nonetheless, that judicial power shall be vested in one Supreme Court
and in such lower courts as may be established by law and that judicial power includes the duty of the courts of
justice to settle actual controversies involving rights which are legally demandable and enforceable, and to
determine whether or not there has been a grave abuse of discretion amounting to lack or excess of jurisdiction on
the part of any branch or instrumentality of the Government.

On the strength of the above constitutional provisions, it can be fairly interpreted that the power of the CTA
includes that of determining whether or not there has been grave abuse of discretion amounting to lack or excess of
jurisdiction on the part of the RTC in issuing an interlocutory order in cases falling within the exclusive appellate
jurisdiction of the tax court. It, thus, follows that the CTA, by constitutional mandate, is vested with jurisdiction to
issue writs of certiorari in these cases.

Indeed, in order for any appellate court to effectively exercise its appellate jurisdiction, it must have the authority to
issue, among others, a writ of certiorari. In transferring exclusive jurisdiction over appealed tax cases to the CTA, it
can reasonably be assumed that the law intended to transfer also such power as is deemed necessary, if not
indispensable, in aid of such appellate jurisdiction. There is no perceivable reason why the transfer should only be
considered as partial, not total.

If this Court were to sustain petitioners' contention that jurisdiction over their certiorari petition lies with the CA,
this Court would be confirming the exercise by two judicial bodies, the CA and the CTA, of jurisdiction over
basically the same subject matter – precisely the split-jurisdiction situation which is anathema to the
orderly administration of justice. The Court cannot accept that such was the legislative motive, especially
considering that the law expressly confers on the CTA, the tribunal with the specialized competence over tax and
tariff matters, the role of judicial review over local tax cases without mention of any other court that may exercise
such power. Thus, the Court agrees with the ruling of the CA that since appellate jurisdiction over private
Remedial Law I| Page 17

respondents' complaint for tax refund is vested in the CTA, it follows that a petition for certiorari seeking
nullification of an interlocutory order issued in the said case should, likewise, be filed with the same court. To rule
otherwise would lead to an absurd situation where one court decides an appeal in the main case while another court
rules on an incident in the very same case.

Stated differently, it would be somewhat incongruent with the pronounced judicial abhorrence to split
jurisdiction to conclude that the intention of the law is to divide the authority over a local tax case filed
with the RTC by giving to the CA or this Court jurisdiction to issue a writ of certiorari against
interlocutory orders of the RTC but giving to the CTA the jurisdiction over the appeal from the decision
of the trial court in the same case. It is more in consonance with logic and legal soundness to conclude that the
grant of appellate jurisdiction to the CTA over tax cases filed in and decided by the RTC carries with it the power to
issue a writ of certiorari when necessary in aid of such appellate jurisdiction. The supervisory power or jurisdiction
of the CTA to issue a writ of certiorari in aid of its appellate jurisdiction should co-exist with, and be a complement
to, its appellate jurisdiction to review, by appeal, the final orders and decisions of the RTC, in order to have complete
supervision over the acts of the latter.
Remedial Law I| Page 18

St. Mary Crusade Foundation vs. Riel


G.R. No. 176508, January 12, 2015

Facts:

The petitioner applied for the judicial reconstitution of Original Certificate of Title for the issuance of a new OCT
alleging that his copy had been burnt and lost in the fire that gutted the Quezon City Register of Deeds in the late
80s. Initially, the judge gave due course to the petition but upon the opposition of the LRA and UP, the judge
dismissed the complaint. Upon the denial of its MR, the petitioner directly went to the Supreme Court and filed a
petition for certiorari and mandamus contending that the respondent judge acted with grave abuse of discretion in
dismissing its complaint. In their comments, UP and OSG contend that petitioner failed to observe the doctrine of
hierarchy of courts, despite the Court of Appeals (CA) having concurrent jurisdiction with the Court over special
civil actions under Rule 65.

Issue:

Whether or not petitioner violated the doctrine of hierarchy of courts by directly filing a petition for certiorari before
the SC?

Held:

Yes. The filing of the instant special civil action directly in this Court is in disregard of the doctrine of hierarchy of
courts. Although the Court has concurrent jurisdiction with the Court of Appeals in issuing the writ of certiorari,
direct resort is allowed only when there are special, extra-ordinary or compelling reasons that justify the same. The
Court enforces the observance of the hierarchy of courts in order to free itself from unnecessary, frivolous and
impertinent cases and thus afford time for it to deal with the more fundamental and more essential tasks that the
Constitution has assigned to it. There being no special, important or compelling reason, the petitioner thereby
violated the observance of the hierarchy of courts, warranting the dismissal of the petition for certiorari.
Remedial Law I| Page 19

Duncano vs. Sandiganbayan


G.R. No. 191894, July 15, 2015.

Facts:

Danico Duncano, petitioner, is at the time material to the case, the Regional Director of the Bureau of Internal
Revenue (BIR) with Salary Grade 26. The Office of the Special Prosecutor (Office of the Ombudsman) filed a
criminal case against him before the Sandiganbayan for violation of Section 8, in relation to Section 11 of R.A. No.
6713. He filed a Motion to Dismiss contending that the Sandiganbayan has no jurisdiction to try and hear the case
because he is an official of the executive branch occupying the position of a Regional Director but with a
compensation that is classified as below Salary Grade 27. The Sandiganbayan denied the Motion to Dismiss ruling
that the position of Regional Director is one of those exceptions where the Sandiganbayan has jurisdiction even if
such position is not Salary Grade 27. It was opined that Section 4(A)(1) of R.A No. 8249 unequivocally provides that
respondent court has jurisdiction over officials of the executive branch of the government occupying the position of
regional director and higher, otherwise classified as Salary Grade 27 and higher, of R.A. No. 6758, including those
officials who are expressly enumerated in subparagraphs (a) to (g).

Issue:

Whether or not the case against petitioner who is a Regional Director with SG 26 falls within the jurisdiction of the
Sandiganbayan?

Held:

No.

Those that fall within the original jurisdiction of the Sandiganbayan are: (1) officials of the executive branch with
Salary Grade 27 or higher, and (2) officials specifically enumerated in Section 4(A)(1)(a) to (g), regardless of their
salary grades.31 While the first part of Section 4(A) covers only officials of the executive branch with Salary Grade
27 and higher, its second part specifically includes other executive officials whose positions may not be of Salary
Grade 27 and higher but who are by express provision of law placed under the jurisdiction of the Sandiganbayan.

Petitioner is not an executive official with Salary Grade 27 or higher. Neither does he hold any position particularly
enumerated in Section 4(A)(1)(a) to (g).

In the same way, a certification issued by the OIC-Assistant Chief, Personnel Division of the BIR shows that,
although petitioner is a Regional Director of the BIR, his position is classified as Director II with Salary Grade 26.
Remedial Law I| Page 20

CE Casecnan Water and Energy Co. Inc. vs. Province of Nueva Ecija
G.R. No. 196278, June 17, 2015.

Facts:

In 1995, petitioner and the National Irrigation Administration (NIA) entered into a build-operate-transfer (BOT)
contract relative to the construction and development of the Casecnan Multi-Purpose Irrigation and Power Project
in Pantabangan, Nueva Ecija and Alfonso Castaneda, Nueva Vizcaya. The Casecnan Project is a combined irrigation
and hydroelectric power generation facility using the Pantabangan Dam in Nueva Ecija. In 2005, petitioner received
from the Office of the Provincial Assessor a Notice of Assessment of Real Property which indicates that for the years
2002 to 2005, its RPT due was P248,676,349.60. Petitioner assailed the assessment with the Nueva Ecija Local
Board of Assessment Appeals (Nueva Ecija LBAA) which dismissed it on January 26, 2006. Undeterred, petitioner
filed a Notice of Appeal with the Nueva Ecija Central Board of Assessment Appeals (Nueva Ecija CBAA). During the
pendency thereof, respondents collected from petitioner the RPT due under the said assessment as well as those
pertaining to the years 2006 up to the second quarter of 2008, totalling P363,703,606.88. Petitioner paid the
assessed RPT under protest; it also initiated proceedings questioning the validity of the collection with respect to the
years 2006 up to the second quarter of 2008. Petitioner received another notice that it has RPT in arrears. Again,
petitioner questioned this assessment through an appeal before the NE LBAA. While the same was pending,
petitioner received a letter from the respondent demanding payment. Hence, petitioner filed with the RTC for
injunction and damages with application for temporary restraining order (TRO) and preliminary injunction praying
to restrain the collection of the 2008 RPT Reassessment. RTC denied petitioner’s application for a 72-hour TRO.
Subsequently, the RTC granted the 20-day TRO. But later on, it denied petitioner’s application for a writ of
preliminary injunction. Petitioner then filed a Petition for Certiorari before the CA. The CA dismissed the petition
observing that the Petition for Certiorari before it was actually an offshoot of the 2008 RPT Reassessment. And
since in resolving the issue of whether the RTC committed grave abuse of discretion in denying petitioner’s
application for a writ of preliminary injunction, the issue of the validity of the assessment and the collection of the
RPT against petitioner must also be resolved, thus jurisdiction over the case lies within the Court of Tax Appeals
(CTA). Petitioner maintains that its petition relates to an ordinary civil action for injunction and not to a local tax
case.

Issue:

Whether or not the CTA has jurisdiction over an order of the RTC denying an application for writ of injunction
(which seeks to enjoin the assessment and collection of RPT)?

Held:

Yes.

Jurisdiction over the subject matter is required for a court to act on any controversy. It is conferred by law and not
by the consent or waiver upon a court. As such, if a court lacks jurisdiction over an action, it cannot decide the case
on the merits and must dismiss it.

With respect to the CTA, its jurisdiction was expanded and its rank elevated to that of a collegiate court with special
jurisdiction by virtue of Republic Act No. 9282. This expanded jurisdiction of the CTA includes its exclusive
appellate jurisdiction to review by appeal the decisions, orders or resolutions of the RTC in local tax cases originally
decided or resolved by the RTC in the exercise of its original or appellate jurisdiction. In the recent case of City of
Manila v. Grecia-Cuerdo, the Court ruled that the CTA likewise has the jurisdiction to issue writs of certiorari or to
determine whether there has been grave abuse of discretion amounting to lack or excess of jurisdiction on the part of
the RTC in issuing an interlocutory order in cases falling within the CTA’s exclusive appellate jurisdiction.

In praying to restrain the collection of RPT, petitioner also implicitly questions the propriety of the assessment of
such RPT. This is because in ruling as to whether to restrain the collection, the RTC must first necessarily rule on
the propriety of the assessment. In other words, in filing an action for injunction to restrain collection, petitioner
was in effect also challenging the validity of the RPT assessment. As aptly discussed by the CA:
Remedial Law I| Page 21

The original action filed with the RTC is one for Injunction, with an application for Temporary Restraining Order
and a Writ of Preliminary Injunction to enjoin the province of Nueva Ecija from further collecting the alleged real
property tax liability assessed against it. Simply because the action is an application for injunctive relief does not
necessarily mean that it may no longer be considered as a local tax case. The subject matter and the issues, not the
name or designation of the remedy, should control. While an ancillary action for injunction may not be a main case,
the court [still has] to determine, even in a preliminary matter, the applicable tax laws, rules and jurisprudence.
Remedial Law I| Page 22

Ferrer vs. Bautista


G.R. No. 210551. June 30, 2015

Facts:

The Quezon City Council enacted Ordinance No. SP-2095, S-2011 or the Socialized Housing Tax of Quezon City,
which impose one- half percent (0.5%) tax on the assessed value of land in excess of One Hundred Thousand Pesos
(Php100,000.00). They also enacted Ordinance No. SP-2235, S-2013 which impose garbage fees on residential
properties in Quezon City. The petitioner then filed a Petition for Certiorari under Rule 65 before the Supreme
Court seeking to declare unconstitutional and illegal Ordinance Nos. SP-2095, S-2011 and SP-2235, S-2013 on the
Socialized Housing Tax and Garbage Fee, respectively, which are being imposed by the respondents.

Issue:

Whether or not the Supreme Court can take cognizance of the case?

Held:

Yes. Petitioner has adduced special and important reasons as to why direct recourse to the SC should be allowed.
Aside from presenting a novel question of law, this case calls for immediate resolution since the challenged
ordinances adversely affect the property interests of all paying constituents of Quezon City. As well, this petition
serves as a test case for the guidance of other local government units (LGUs). Indeed, the petition at bar is of
transcendental importance warranting a relaxation of the doctrine of hierarchy of courts.

Citing Senator Jaworski v. Phil. Amusement & Gaming Corp., the Court said:

Granting arguendo that the present action cannot be properly treated as a petition for prohibition, the
transcendental importance of the issues involved in this case warrants that we set aside the technical defects and
take primary jurisdiction over the petition at bar. x x x This is in accordance with the well-entrenched principle that
rules of procedure are not inflexible tools designed to hinder or delay, but to facilitate and promote the
administration of justice. Their strict and rigid application, which would result in technicalities that tend to
frustrate, rather than promote substantial justice, must always be eschewed.

Note: A Petition for Certiorari is not the proper remedy because the respondents are not exercising quasi-judicial
functions. And it actually partakes of a declaratory relief over which the Supreme Court has only appellate, not
original, jurisdiction. Despite these, a petition for declaratory relief may be treated as one for prohibition or
mandamus, over which the SC exercises original jurisdiction, in cases with far-reaching implications or one which
raises transcendental issues or questions that need to be resolved for the public good. The judicial policy is that this
Court will entertain direct resort to it when the redress sought cannot be obtained in the proper courts or when
exceptional and compelling circumstances warrant availment of a remedy within and calling for the exercise of Our
primary jurisdiction.
Remedial Law I| Page 23

Lomondot vs. Balindong


G.R. No. 192463. July 13, 2015.

Facts:

Petitioners Omaira and Saripa Lomondot filed with the Shari’a District Court (SDC), Marawi City, a complaint for
recovery of possession and damages with prayer for mandatory injunction and temporary restraining order against
respondents. They alleged that they are the owners by succession of the subject parcel of land. The SDC ruled in
favor of the petitioners and ordered the respondents to vacate. The SDC decision became final and executory.
Petitioner then filed a motion for the issuance of a writ of execution. The SDC granted the motion but it
subsequently issued an order that the resolution for the motion for the issuance of a writ of demolition should be
held in abeyance. Later on, it denied the motion for demolition stating that the respondents were claiming that they
no longer encroach the plaintiff’s land. Upon the denial of their MR, petitioners filed a petition for certiorari with
CA-Cagayan de Oro City. The CA dismissed the petition on the ground of lack of jurisdiction. It opined that it cannot
take cognizance of the instant case because it emanates from the Shari’a Courts, which is not among those courts,
bodies or tribunals enumerated under Chapter 1, Section 9 of BP 129, as amended, over which it can exercise
appellate jurisdiction.

Issue:

Whether or not the Court of Appeals has jurisdiction over decisions of SDC?

Held:

Yes. The Shari’a Appellate Court has yet to be organized with the appointment of a Presiding Justice and two
Associate Justices. Until such time that the Shari’a Appellate Court shall have been organized, however, appeals or
petitions from final orders or decisions of the SDC filed with the CA shall be referred to a Special Division to be
organized in any of the CA stations preferably composed of Muslim CA Justices (Tomawis v. Balindong).
Remedial Law I| Page 24

Municipality of Tangkal vs. Balindong


G.R. No. 193340, January 11, 2017

Facts:

The private respondents, heirs of the late Macalabo Alompo, filed a Complaint with the Shari'a District Court of
Marawi City (Shari'a District Court) against the petitioner, Municipality of Tangkal, for recovery of possession and
ownership of a parcel of land. They alleged that Macalabo was the owner of the land, and that in 1962, he entered
into an agreement with the Municipality of Tangkal allowing the latter to "borrow" the land to pave the way for the
construction of the municipal hall and a health center building. The agreement allegedly imposed a condition upon
the Municipality of Tangkal to pay the value of the land within 35 years, or until 1997; otherwise, ownership of the
land would revert to Macalabo. Private respondents claimed that the Municipality of Tangkal neither paid the value
of the land within the agreed period nor returned the land to its owner. Thus, they prayed that the land be returned
to them as successors-in-interest of Macalabo. The Municipality of Tangkal filed a Motion to Dismiss on the ground
of improper venue and lack of jurisdiction. It argued that since it has no religious affiliation and represents no
cultural or ethnic tribe, it cannot be considered as a Muslim under the Code of Muslim Personal Laws. Moreover,
since the complaint for recovery of land is a real action, it should have been filed in the appropriate Regional Trial
Court of Lanao del Norte. The Shari'a District Court denied the Municipality of Tangkal's motion to dismiss. It held
that since the mayor of Tangkal, Abdulazis A.M. Batingolo, is a Muslim, the case "is an action involving Muslims,
hence, the court has original jurisdiction concurrently with that of regular/civil courts." The Municipality of Tangkal
elevated the case to the SC via a petition for certiorari, prohibition and mandamus.

Issue:

Whether or not the Shari’a District Court has jurisdiction?

Held:

No. The matters over which Shari'a district courts have Jurisdiction are enumerated in the Code of Muslim Personal
Laws, specifically in Article 143. Consistent with the purpose of the law to provide for an effective administration
and enforcement of Muslim personal laws among Muslims, it has a catchall provision granting Shari'a district
courts original jurisdiction over personal and real actions except those for forcible entry and unlawful detainer. The
Shari'a district courts' jurisdiction over these matters is concurrent with regular civil courts, i.e., municipal trial
courts and regional trial courts. There is, however, a limit to the general jurisdiction of Shari'a district courts over
matters ordinarily cognizable by regular courts: such jurisdiction may only be invoked if both parties are Muslims. If
one party is not a Muslim, the action must be filed before the regular courts.

In determining whether the Shari'a District Court has jurisdiction over the case, the threshold question is whether
both parties are Muslims. It is clear from the title and the averments in the complaint that Mayor Batingolo was
impleaded only in a representative capacity, as chief executive of the local government of Tangkal. When an action
is defended by a representative, that representative is not-and neither does he become-a real party in interest. The
person represented is deemed the real party in interest; the representative remains to be a third party to the action.
That Mayor Batingolo is a Muslim is therefore irrelevant for purposes of complying with the jurisdictional
requirement under Article 143(2)(b) that both parties be Muslims. To satisfy the requirement, it is the real party
defendant, the Municipality of Tangkal, who must be a Muslim. Such a proposition, however, is a legal impossibility.
A juridical person exercises no religion. As a government instrumentality, the Municipality of Tangkal can only act
for secular purposes and in ways that have primarily secular effects35-consistent with the non-establishment clause.
Hence, even if it is assumed that juridical persons are capable of practicing religion, the Municipality of Tangkal is
constitutionally proscribed from adopting, much less exercising, any religion, including Islam.
Remedial Law I| Page 25

Presidential Commission on Good Government (PCGG) vs. Dumayas


G.R. No. 209447. August 11, 2015.

Facts:

The Sandiganbayan rendered partial summary judgments in Civil Case No. 0033 revolved around the provisional
takeover by the PCGG of COCOFED, Cocomark, and Coconut Investment Company and their assets and the
sequestration of shares of stock in UCPB CIIF corporations (CIIF oil mills and the 14 CIIF holding companies), or
CIIF companies, so-called for having been either organized, acquired and/or funded as UCPB subsidiaries with the
use of the CIIF levy. SB’s partial judgments were affirmed with modification by the Supreme Court. In 2012, a
petition for declaratory relief was filed by respondent UCPB in the RTC of Makati (Civil Case No. 12-1251) against
the six CIIF oil mills and 14 holding companies (CIIF companies), PCGG and other corporations “similarly situated.”
A similar petition was also filed by respondent United Coconut Planters Life Assurance Corporation (COCOLIFE)
against the same defendants (Civil Case No. 12- 1252). UCPB alleged that the capital or equity used in establishing
the CIIF companies was not exclusively sourced from the coconut levy funds. Like UCPB, COCOLIFE asserted that
the CIIF OMG and 14 CIIF holding companies are not wholly owned by the Government. PCGG filed a Motion to
Dismiss in both cases on the ground of lack of jurisdiction over the subject matter of the case. Both MTDs were
denied.

Issue:

Whether or not the RTC has jurisdiction over suits involving the sequestered coco levy assets and coco levy funds?

Held:

No. Under Section 4(C) of P.D. No. 1606, as amended by R.A. No. 7975 and R.A. No. 8249, the jurisdiction of the
Sandiganbayan included suits for recovery of ill-gotten wealth and related cases.

Under Section 2 of the President’s Executive Order No. 14 issued on May 7, 1986, all cases of the Commission
regarding “the Funds, Moneys, Assets, and Properties Illegally Acquired or Misappropriated by Former President
Ferdinand Marcos, Mrs. Imelda Romualdez Marcos, their Close Relatives, Subordinates, Business Associates,
Dummies, Agents, or Nominees” whether civil or criminal, are lodged within the “exclusive and original jurisdiction
of the Sandiganbayan” and all incidents arising from, incidental to, or related to, such cases necessarily fall likewise
under the Sandiganbayan’s exclusive and original jurisdiction, subject to review on certiorari exclusively by the
Supreme Court.

Respondents’ petitions for declaratory relief filed in the RTC asserted their claim of ownership over the sequestered
CIIF companies and indirectly the CIIF SMC Block of Shares. Undeniably, these are related to the ill-gotten wealth
cases (Civil Case Nos. 0033-A and 0033-F) involving the issue of ownership of the aforesaid sequestered companies
and shares of stock, which have been tried and decided by the Sandiganbayan, and the decision had been appealed
to and finally disposed of by this Court in G.R. Nos. 177857-5831 (COCOFED and Lobregat, et al.’s ownership claim
over the CIIF companies and CIIF SMC Block of Shares) and G.R. No. 18070532 (Eduardo M. Cojuangco, Jr.’s claim
over UCPB shares under an Agreement with PCA).
Remedial Law I| Page 26

Regulus Development, Inc. vs. Dela Cruz


G.R. No. 198172. January 25, 2016.

Facts:

Dela Cruz leased two units of petitioner’s apartment. The contract of lease provides for a period of one-month subject
to automatic renewals. Petitioner sent respondent a letter to terminate the lease of the two subject units. Due to the
respondent’s refusal to vacate the units, the petitioner filed a complaint for ejectment before the Metropolitan Trial
Court (MTC) of Pasay City. The MTC resolved the case in the petitioner’s favor and ordered the respondent to
vacate the premises, and pay the rentals due. The respondent appealed to the Regional Trial Court (RTC). Pending
appeal, the respondent consigned the monthly rentals to the RTC due to the petitioner’s refusal to receive the
rentals. The RTC affirmed the decision of the MTC. The CA reversed the two lower courts and dismissed the
ejectment case. This decision became final and executory. Petitioner then filed a motion praying for the withdrawal
of the rentals consigned with the RTC. The RTC granted the motion. The CA also upheld the RTC’s order and held
that they were issued pursuant to its equity jurisdiction. The CA ruling became final and executory as the SC denied
respondent’s petition for insufficiency in form. The petitioner then returned to the RTC for the issuance of the lease
payments deposited by respondent. However, the withdrawn deposits were insufficient to cover the rentals due from
2001 to 2004. Hence, the petitioner filed a manifestation and motion praying that the RTC to levy upon the
respondent’s property to satisfy the judgment credit. The RTC granted the motion. Respondent went to the CA. The
CA initially denied the petition on MR, the CA reversed and set aside the orders of the RTC directing the levy of the
respondent’s property. The CA held that while the approval of the petitioner’s motion to withdraw the consigned
rentals and the posted supersedeas bond was within the RTC’s jurisdiction, the RTC had no jurisdiction to levy on
the respondent’s real property. The CA explained that the approval of the levy on the respondent’s real property
could not be considered as a case pending appeal, because the decision of the MTC had already become final and
executory. As such, the matter of execution of the judgment lies with the MTC where the complaint for ejectment
was originally filed and presented.

Issue:

Whether or not the RTC had jurisdiction to levy on the respondent’s real property?

Held:

Yes. The appellate jurisdiction of courts is conferred by law. The appellate court acquires jurisdiction over the
subject matter and parties when an appeal is perfected. On the other hand, equity jurisdiction aims to provide
complete justice in cases where a court of law is unable to adapt its judgments to the special
circumstances of a case because of a resulting legal inflexibility when the law is applied to a given
situation. The purpose of the exercise of equity jurisdiction, among others, is to prevent unjust
enrichment and to ensure restitution.

The RTC orders which allowed the withdrawal of the deposited funds for the use and occupation of the subject units
were issued pursuant to the RTC’s equity jurisdiction. The RTC’s equity jurisdiction is separate and distinct from its
appellate jurisdiction on the ejectment case. The RTC could not have issued its orders in the exercise of its appellate
jurisdiction since there was nothing more to execute on the dismissed ejectment case. As the RTC orders explained,
the dismissal of the ejectment case effectively and completely blotted out and cancelled the complaint. Hence, the
RTC orders were clearly issued in the exercise of the RTC’s equity jurisdiction, not on the basis of its appellate
jurisdiction.

The levy of the respondent’s property was made pursuant to the RTC orders issued in the exercise of its equity
jurisdiction, independent of the ejectment case originally filed with the MTC.

An examination of the RTC order dated June 30, 2008, directing the levy of the respondent’s real property shows
that it was based on the RTC order dated July 25, 2003. The levy of the respondent’s property was issued to satisfy
the amounts due under the lease contracts, and not as a result of the decision in the ejectment case. The CA erred
when it concluded that the RTC exercised its appellate jurisdiction in the ejectment case when it directed the levy of
Remedial Law I| Page 27

the respondent’s property. Execution shall be applied for in the court of origin, in accordance with Section 1,45 Rule
39 of the Rules of Court. The court of origin with respect to the assailed RTC orders is the court which issued these
orders. The RTC is the court with jurisdiction to order the execution of the issued RTC orders.
Remedial Law I| Page 28

Rule 1: Commencement of Actions

Alday vs. FGU Insurance Corporation


G.R. No. 138822, January 23, 2001

Facts:

FGU Insurance Insurance Corp. filed a complaint with the RTC of Makati against Evangeline Alday alleging that
the latter owed it owed it P114,650.76, representing unliquidated cash advances, unremitted costs of premiums and
other charges incurred by Alday in the course of her work as an insurance agent for FGU. Alday filed her Answer
and by way of Counterclaim asserted her right for the payment of P104,893.45, representing direct commissions,
profit commissions and contingent bonuses earned and for accumulated premium reserves amounting to
P500,000.00. In addition, Alday prayed for attorney’s fees, litigation expenses, moral damages and exemplary
damages for the allegedly unfounded action filed by FGU. FGU then filed a Motion to Dismiss Alday’s counterclaim
contending that the trial court never acquired jurisdiction over the same because of the non-payment of docket fees.
In response, Alday asked the trial court to declare her counterclaim as exempt from payment of docket fees since it
is compulsory and that FGU be declared in default for having failed to answer such counterclaim. RTC granted the
Motion to Dismiss because it found Alday’s counterclaim to be merely permissive in nature and held that her failure
to pay docket fees prevented the court from acquiring jurisdiction over the same. The Court of Appeals affirmed the
RTC’s decision.

Issue:

Whether or not Alday’s counterclaims are merely permissive and accordingly, the trial court did not acquire
jurisdiction over the same for non-payment of docket fees?

Held:

Alday’s counterclaim for commissions, bonuses, and accumulated premium reserves is merely
permissive. The evidence required to prove her claims differs from that needed to establish FGU’s demands for the
recovery of cash accountabilities from Alday, such as cash advances and costs of premiums. The recovery of FGU’s
claims is not contingent or dependent upon establishing Alday’s counterclaim, such that conducting separate trials
will not result in the substantial duplication of the time and effort of the court and the parties. One would search the
records in vain for a logical connection between the parties’ claims. This conclusion is further reinforced by Aldays’s
own admissions since she declared in her answer that respondent’s cause of action, unlike her own, was not based
upon a Special Agent’s Contract.

However, Alday’s claims for damages, allegedly suffered as a result of the filing of its complaint are
compulsory.

There is no need for petitioner to pay docket fees for her respondent of its complaint, are compulsory.
On the other hand, in order for the trial court to acquire jurisdiction over her permissive counterclaim,
Alday is bound to pay the prescribed docket fees.
In this case, instead of dismissing the counterclaim, trial court should have given Alday a reasonable time, but in no
case beyond the applicable prescriptive or reglementary period, to pay the filing fees for her permissive
counterclaim.

Discussion:

A compulsory counterclaim is one which, being cognizable by the regular courts of justice, arises out of or is
connected with the transaction or occurrence constituting the subject matter of the opposing party’s claim and does
not require for its adjudication the presence of third parties of whom the court cannot acquire jurisdiction.

The criteria or tests that may be used in determining whether a counterclaim is compulsory or permissive,
summarized as follows:
1. Are the issues of fact and law raised by the claim and counterclaim largely the same?
Remedial Law I| Page 29

2. Would res judicata bar a subsequent suit on defendant’s claim absent the compulsory counterclaim rule?
3. Will substantially the same evidence support or refute plaintiffs claim as well as defendant’s counterclaim?
4. Is there any logical relation between the claim and the counterclaim?

Another test is the “compelling test of compulsoriness” which requires “a logical relationship between the claim and
counterclaim, that is, where conducting separate trials of the respective claims of the parties would entail a
substantial duplication of effort and time by the parties and the court.”

Rule on the payment of filing fees:


1. It is not simply the filing of the complaint or appropriate initiatory pleading, but the payment of the
prescribed docket fee, that vests a trial court with jurisdiction over the subject-matter or nature of the
action. Where the filing of the initiatory pleading is not accompanied by payment of the docket fee, the court
may allow payment of the fee within a reasonable time but in no case beyond the applicable prescriptive or
reglementary period.

2. The same rule applies to permissive counterclaims, third- party claims and similar pleadings, which shall
not be considered filed until and unless the filing fee prescribed therefor is paid. The court may allow
payment of said fee within a reasonable time but also in no case beyond its applicable prescriptive or
reglementary period.

3. Where the trial court acquires jurisdiction over a claim by the filing of the appropriate pleading and
payment of the prescribed filing fee but, subsequently, the judgment awards a claim not specified in the
pleading, or if specified the same has been left for determination by the court, the additional filing fee
therefor shall constitute a lien on the judgment. It shall be the responsibility of the Clerk of Court or his
duly authorized deputy to enforce said lien and assess and collect the additional fee.
Remedial Law I| Page 30

Korea Technologies Co., Ltd. vs. Lerma


G.R. No. 143581. January 7, 2008

NOTE: The doctrine in this case is no longer controlling.

Facts:

Korea Technologies Co., Ltd. (KOGIES) and Pacific General Steel Manufacturing Corp. (PGSMC) executed a
Contract in the Philippines, whereby KOGIES would set up an LPG Cylinder Manufacturing Plant in Carmona,
Cavite. Subsequently, the obligations of the parties regarding installment of the plant were complied with. However,
after the installation of the plant, the initial operation could not be conducted as PGSMC encountered financial
difficulties affecting the supply of materials, thus forcing the parties to agree that KOGIES would be deemed to have
completely complied with the terms and conditions of their contract. For the remaining balance for the installation
and initial operation of the plant, PGSMC issued two postdated checks. When KOGIES deposited the checks, these
were dishonored. Thus, KOGIES sent a demand letter to PGSMC. Later on, PGSMC informed KOGIES that it
would cancel their contract on the ground that KOGIES had altered the quantity and lowered the quality of the
machineries and equipment it delivered to PGSMC, among, others. PGSMC filed before the OCP an Affidavit-
Complaint for Estafa against the President of KOGIES. KOGIES sent a letter to PGSMC that it could not
unilaterally rescind the contract and that their dispute should be settled by arbirtration as agreed upon in their
contract. Later, KOGIES instituted an Application for Arbitration before the Korean Commercial Arbitration Board
(KCAB) in Seoul, Korea pursuant to their contract. KOGIES also filed a Complaint for Specific Performance, against
PGSMC before the Regional Trial Court (RTC). PGSMC filed its Answer with Compulsory Counterclaim asserting
that it had the full right to dismantle and transfer the machineries and equipment because it had paid for them in
full as stipulated in the contract; that KOGIES was not entitled to the PhP 9,000,000 covered by the checks for
failing to completely install and make the plant operational; and that KOGIES was liable for damages amounting to
PhP 4,500,000 for altering the quantity and lowering the quality of the machineries and equipment.

The RTC ruled in favor of PGSMC. The Court of Appeals affirmed the trial court and held that the counterclaim of
PGSMC is compulsory in nature which did not require the payment of docket fees.

Issue:

Whether or not the counterclaim of PGSMC is compulsory and is therefore exempt from the payment of the docket
fees?

Held:

No. Effective August 16, 2004 under Sec. 7, Rule 141, as amended by A.M. No. 04-2-04-SC, docket fees are now
required to be paid in compulsory counterclaim or cross-claims.
Remedial Law I| Page 31

Mercado vs. Court of Appeals


G.R. No. 169576, October 17, 2008

Facts:

Mercado had been distributing San Miguel Corporation’s (SMCs) beer products in Quiapo, Manila since 1967. SMC
extended to him a P7.5 million credit line allowing him to withdraw goods on credit. To secure his purchases,
Mercado assigned three China Banking Corporation (CBC) certificates of deposit to SMC and executed a continuing
hold-out agreement. He also submitted three surety bonds from Eastern Assurance and Surety Corporation
(EASCO). Later, SMC notified CBC that Mercado failed to pay for the items he withdrew on credit. Consequently,
citing the continuing hold-out agreement, it asked CBC to release the proceeds of the assigned certificates of deposit.
CBC approved SMBs request and informed Mercado. Mercado then filed an action to annul the continuing hold-out
agreement and deed of assignment in the RTC of Manila. Thereafter, SMC filed its answer with counterclaim
against Mercado. It contended that Mercado delivered only two CBC certificates of deposit. SMC also filed a third-
party complaint with EASCO. However, Mercado filed an urgent manifestation and motion seeking the dismissal of
the complaint. He claimed that he was no longer interested in annulling the continuing hold-out agreement and
deed of assignment. The RTC, however, denied the motion.

During trial, Mercado acknowledged the accuracy of SMCs computation of his outstanding liability as of August
1991. Thus, the RTC dismissed the complaint and ordered Mercado and EASCO to jointly and severally pay SMC.
Aggrieved, Mercado and EASCO appealed to the Court of Appeals (CA) insisting that Mercado did not default in the
payment of his obligations to SMC. On appeal, the CA affirmed the RTC decision in toto. EASCO filed a petition for
review on certiorari before the Supreme Court but eventually agreed to settle its liability with SMC. The petition
was terminated. Meanwhile, Mercado passed away and was substituted by his heirs. The petitioners subsequently
filed a petition asserting that the decision of the CA was void on the ground that the SMC’s counterclaim was
permissive in nature and that inasmuch as SMC did not pay docket fees, the RTC never acquired jurisdiction over
the counterclaim.

Issue:

Whether or not SMC’s counterclaim is permissive in nature which requires payment of docket fees before the court
could acquire jurisdiction of the claim?

Held:

No. SMC’s counterclaim is compulsory in nature. A counterclaim that (1) arises out of (or is necessarily connected
with) the transaction or occurrence that is the subject matter of the opposing party’s claim; (2) falls within the
jurisdiction of the court and (3) does not require for its adjudication the presence of third parties over whom the
court cannot acquire jurisdiction, is compulsory.

When Mercado sought to annul the continuing hold-out agreement and deed of assignment (which he executed as
security for his credit purchases, he in effect sought to be freed from them. While he admitted having outstanding
obligations, he nevertheless asserted that those were not covered by the assailed accessory contracts. For its part,
aside from invoking the validity of the said agreements, SMC therefore sought to collect the payment for the value of
goods Mercado purchased on credit. Thus, Mercado’s complaint and SMCs counterclaim both touched the issues of
whether the continuing hold-out agreement and deed of assignment were valid and whether Mercado had
outstanding liabilities to SMC. The same evidence would essentially support or refute Mercado’s claim and SMCs
counterclaim. Based on the foregoing, had these issues been tried separately, the efforts of the RTC and the parties
would have had to be duplicated. Clearly, SMCs counterclaim, being logically related to Mercado’s claim, was
compulsory in nature. Consequently, the payment of docket fees was not necessary for the RTC to acquire
jurisdiction over the subject matter.
Remedial Law I| Page 32

Proton Pilipinas vs. Banque Nationale de Paris


G.R. No. 151242, June 15, 2005

Facts:

Proton Pilipinas Corporation (Proton) availed of the credit facilities of Banque Nationale de Paris (BNP). To
guarantee the payment of its obligation, its co-petitioners (Automotive, Asea and Autocorp) executed a corporate
guarantee to the extent of US$2,000,000.00. BNP and Proton subsequently entered into three trust receipt
agreements, under which Proton would receive imported passenger motor vehicles and hold them in trust for BNP.
Proton would be free to sell the vehicles subject to the condition that it would deliver the proceeds of the sale to
BNP, to be applied to its obligations to it. In case the vehicles are not sold, Proton would return them to BNP,
together with all the accompanying documents of title. Proton failed to deliver the proceeds of the sale and return
the unsold motor vehicles. BNP then demanded from Automotive, Asea and Autocorp the payment of the amount of
US$1,544,984.40 representing Proton's total outstanding obligations. These guarantors refused to pay. So, BNP filed
before the Makati RTC a complaint against petitioners praying that they be ordered to pay US$1,544,984.40 plus
accrued interest and other related charges subsequent to August 15, 1998 until fully paid.

The Makati RTC Clerk of Court assessed the docket fees which BNP paid at ₱352,116.30. The petitioners filed a
Motion to Dismiss on the ground that BNP failed to pay the correct docket fees and thus prevent the trial court from
acquiring jurisdiction over the case.

The RTC denied petitioners' Motion to Dismiss. It ruled that the docket fees were properly paid since it is the Office
of the Clerk of Court that computes the correct docket fees, and it is their duty to assess the docket fees correctly,
which they did. According to the RTC, even granting arguendo that the docket fees were not properly paid, the court
cannot just dismiss the case. The Court has not yet ordered (and it will not in this case) to pay the correct docket
fees.

The CA affirmed the RTC, ruling that Section 7(a) of Rule 141 of the Rules of Court excludes interest accruing from
the principal amount being claimed in the pleading in the computation of the prescribed filing fees.

Citing Administrative Circular No. 11-94, petitioners argue that BNP failed to pay the correct docket fees as the
said circular provides that in the assessment thereof, interest claimed should be included. There being an
underpayment of the docket fees, the trial court did not acquire jurisdiction over the case.

Issues:

(1) Whether or not BNP paid the correct docket fees?

(2) Whether or not the complaint filed by BNP should be dismissed on the ground that the trial court did not acquire
jurisdiction over the case since it failed to pay the correct docket fees?

Held:

(1) No. When the complaint in this case was filed in 1998, Rule 141 had been amended by Administrative Circular
No. 11-94 which provides for the inclusion of interest, damages of whatever kind, attorney's fees, litigation expenses,
and costs in the total amount claimed as basis for the computation of docket fees. The clerk of court should thus
have assessed the filing fee by taking into consideration "the total sum claimed, inclusive of interest, damages of
Remedial Law I| Page 33

whatever kind, attorney's fees, litigation expenses, and costs, or the stated value of the property in litigation.
(Additionally, the petitioners have adequately proven with documentary evidence that the exchange rate when the
complaint was filed was USD 1 = Php 43.21, and not Php 43.00 as determined by the Clerk of Court. This
overturned the disputable presumption of regularity of the Clerk’s application of the exchange rate.)

(2) No. True, in Manchester Development Corporation v. Court of Appeals, this Court held that the court acquires
jurisdiction over any case only upon the payment of the prescribed docket fees, hence, it concluded that the trial
court did not acquire jurisdiction over the case. It bears emphasis, however, that the ruling in Manchester was
clarified in Sun Insurance Office, Ltd. (SIOL) v. Asuncion when this Court held that in the former, there was clearly
an effort to defraud the government in avoiding to pay the correct docket fees, whereas in the latter the plaintiff
demonstrated his willingness to abide by paying the additional fees as required.

The principle in Manchester could very well be applied in the present case. However, in Manchester, petitioner did
not pay any additional docket fee until the case was decided by this Court. Thus, in Manchester, due to the fraud
committed on the government, this Court held that the court a quo did not acquire jurisdiction over the case and
that the amended complaint could not have been admitted inasmuch as the original complaint was null and void.

In the present case, a more liberal interpretation of the rules is called for considering that, unlike
Manchester, private respondent demonstrated his willingness to abide by the rules by paying the
additional docket fees as required.

Discussion:

1. It is not simply the filing of the complaint or appropriate initiatory pleading, but the payment of the
prescribed docket fee, that vests a trial court with jurisdiction over the subject-matter or nature of the
action. Where the filing of the initiatory pleading is not accompanied by payment of the docket fee, the court
may allow payment of the fee within a reasonable time but in no case beyond the applicable prescriptive or
reglementary period.

2. The same rule applies to permissive counterclaims, third-party claims and similar pleadings, which shall
not be considered filed until and unless the filing fee prescribed therefor is paid. The court may also allow
payment of said fee within a reasonable time but also in no case beyond its applicable prescriptive or
reglementary period.

3. Where the trial court acquires jurisdiction over a claim by the filing of the appropriate pleading and
payment of the prescribed filing fee but, subsequently, the judgment awards a claim not specified in the
pleading, or if specified the same has been left for determination by the court, the additional filing fee
therefor shall constitute a lien on the judgment. It shall be the responsibility of the Clerk of Court or his
duly authorized deputy to enforce said lien and assess and collect the additional fee.
Remedial Law I| Page 34

Ruby Shelter Builders vs. Formaran


G.R. No. 175914, February 10, 2009

Facts:

Petitioner Ruby Shelter Builders obtained a loan of more than Php 95M from respondents Tan and Obiedo, secured
by real estate mortgages over five parcels of land. When petitioner was unable to pay the loan when it became due
and demandable, Tan and Obiedo, through a Memorandum of Agreement, granted petitioner a period within which
to settle its indebtedness, and condoned the interests, penalties and surcharges accruing thereon amounting to more
than Php 74M. The Memorandum required, in turn, that petitioner execute simultaneously, "by way of dacion en
pago," Deeds of Absolute Sale in favor of Tan and Obiedo, covering the same parcels of land subject of the mortgages.
The Deeds of Absolute Sale state that petitioner sold to Tan and Obiedo the parcels of land for the stated purchase
prices. The Memorandum further gives the petitioner an opportunity to redeem any of the parcels of land and
provides for the nullification of the Deeds of Absolute Sale. Otherwise, Tan and Obiedo could already present the
Deeds to the Office of the Register of Deeds to acquire TCTs in their names.

Petitioner defaulted in its obligation, so Tan and Obiedo presented the Deeds of Absolute Sale before the Register of
Deeds of Naga City and were able to secure TCTs over the five parcels of land in their names.

Petitioner filed before the RTC a Complaint against Tan, Obiedo, and Atty. Reyes (the notary public who notarized
the MOA) for declaration of nullity of deeds of sales and damages, with prayer for the issuance of a writ of
preliminary injunction and/or TRO. Among others, petitioner alleged that the Deeds of Absolute Sale over the five
parcels of land were executed merely as security for the payment of its loan to respondents Tan and Obiedo; that the
Deeds of Absolute Sale, executed in accordance with the Memorandum of Agreement, constituted pactum
commisorium and as such, were null and void; and that the acknowledgment in the Deeds of Absolute Sale were
falsified. Upon filing its Complaint with the RTC, petitioner paid the sum of ₱13,644.25 for docket and other legal
fees, as assessed by the Office of the Clerk of Court. The Clerk of Court initially considered the case as an action
incapable of pecuniary estimation and computed the docket and other legal fees due thereon according to Section
7(b)(1), Rule 141 of the Rules of Court.

On the other hand, Tan contended that the civil case involved real properties, the docket fees for which should be
computed in accordance with Section 7(a), not Section 7(b)(1), of Rule 141 of the Rules of Court, as amended by A.M.
No. 04-2-04-SC. Since petitioner did not pay the appropriate docket fees, the RTC did not acquire jurisdiction over
the said case.

The RTC granted Tan’s Omnibus Motion. It was held that both petitioner and respondent Tan must pay the docket
fees in accordance with Section 7(a), Rule 141 of the Rules of Court, as amended. Particularly, petitioner was
ordered to pay additional filing fee.

The RTC Clerk of Court computed the additional docket fees petitioner must pay at ₱720,392.60. Petitioner filed a
Petition for Certiorari with the CA. The CA rendered a decision denying the petition for certiorari. According to the
CA, the objectives of the petitioner in filing the complaint were to cancel the deeds of sale and ultimately, to recover
possession of the same. It is therefore a real action. Consequently, the additional docket fees that must be paid
cannot be assessed in accordance with Section 7(b), but must comply with Section 7(a).

Issue:
Remedial Law I| Page 35

Whether or not the subject case is a real action and therefore the petitioner should be assessed docket fees based on
Section 7(a), Rule 141 of the Rules of Court as amended?

Held:

Yes. A real action is one in which the plaintiff seeks the recovery of real property; or, as indicated in what is now
Section 1, Rule 4 of the Rules of Court, a real action is an action affecting title to or recovery of possession of real
property. While it is true that petitioner does not directly seek the recovery of title or possession of the property in
question, his action for annulment of sale and his claim for damages are closely intertwined with the issue of
ownership of the building which, under the law, is considered immovable property, the recovery of which is
petitioner's primary objective. The prevalent doctrine is that an action for the annulment or rescission of a sale of
real property does not operate to efface the fundamental and prime objective and nature of the case, which is to
recover said real property. It is a real action. A careful examination of respondent’s complaint reveals that it is a
real action involving not only the recovery of real properties, but likewise the cancellation of the titles thereto.

The docket fees under Section 7(a), Rule 141, in cases involving real property, depend on the fair
market value of the same: the higher the value of the real property, the higher the docket fees due. In
contrast, Section 7(b)(1), Rule 141 imposes a fixed or flat rate of docket fees on actions incapable of
pecuniary estimation. Considering that respondent’s complaint is a real action, the Rule requires that
"the assessed value of the property, or if there is none, the estimated value thereof shall be alleged by
the claimant and shall be the basis in computing the fees."
Remedial Law I| Page 36

St. Louis University vs. Cobarrubias


G.R. No. 187104, August 3, 2010

Facts:

Evangeline Cobarrubias is an associate professor of the SLU’s College of Human Sciences. She is an active member
of the Union of Faculty and Employees of Saint Louis University (UFESLU). The relevant Collective Bargaining
Agreements (CBAs) between SLU and UFESLU contain the common provision on forced leave that teaching
employees in college who fail the yearly evaluation shall be on forced leave for one (1) regular semester during which
period all benefits due them shall be suspended.

SLU placed Cobarrubias on forced leave for the first semester of S.Y. 2007-2008 when she failed the evaluation for
S.Y. 2002-2003, 2005-2006, and 2006-2007. Cobarrubias sought recourse from the CBA’s grievance machinery. The
parties failed to settle their dispute, prompting Cobarrubias to file a case for illegal forced leave or illegal suspension
with the National Conciliation and Mediation Board of the DOLE-CAR, Baguio City. When circulation and
mediation again failed, the parties submitted the issues between them for voluntary arbitration before Voluntary
Arbitrator (VA) Fariñas. Cobarrubias argued that the CA already resolved the forced leave issue in a prior case
between the parties, ruling that the forced leave for teachers who fail their evaluation for three (3) times within a
five-year period should be coterminous with the CBA in force during the same five-year period.

VA Fariñas dismissed the case. He found that the CA decision is not yet final because of the pending appeal with the
SC. He also noted that the CBA clearly authorized SLU to place its teaching employees on forced leave.

Cobarrubias filed with the CA a petition for review under Rule 43 of the Rules of Court, but failed to pay the
required filing fees and to attach to the petition copies of the material portions of the record. The CA dismissed the
petition outright for Cobarrubias’ procedural lapses.

Cobarrubias then filed her motion for reconsideration, arguing that the ground cited is technical. She, nonetheless,
attached to her motion copies of the material portions of the record and the postal money orders for ₱4,230.00. She
maintained that the ends of justice and fair play are better served if the case is decided on its merits.

The CA reinstated the petition. It found that Cobarrubias substantially complied with the rules by paying the
appeal fee in full and attaching the proper documents in her motion for reconsideration. The CA brushed aside
SLU’s insistence on the finality of the VA decision and annulled it.

SLU filed a Motion for Reconsideration but it was denied by the CA. Hence, the present petition for review on
certiorari. SLU argues that the CA should not have reinstated the appeal since Cobarrubias failed to pay the docket
fees within the prescribed period, which rendered the VA decision final and executory.

Issue:

Wether or not the CA erred in reinstating Cobarrubias’ petition despite her failure to pay the appeal fee within the
reglementary period?

Held:

Yes. Appeal is not a natural right but a mere statutory privilege, thus, appeal must be made strictly in accordance
with the provision set by law. Rule 43 of the Rules of Court provides that appeals from the judgment of the VA shall
Remedial Law I| Page 37

be taken to the CA, by filing a petition for review within fifteen (15) days from the receipt of the notice of judgment.
Furthermore, upon the filing of the petition, the petitioner shall pay to the CA clerk of court the
docketing and other lawful fees; non-compliance with the procedural requirements shall be a sufficient
ground for the petition’s dismissal. Thus, payment in full of docket fees within the prescribed period is
not only mandatory, but also jurisdictional. It is an essential requirement, without which, the decision
appealed from would become final and executory as if no appeal has been filed.

The payment of the full amount of the docket fee is an indispensable step for the perfection of an appeal. Appeal is
not perfected if only a part of the docket fee is deposited within the reglementary period and the remainder is
tendered after the expiration of the period.

In the present case, Cobarrubias filed her petition for review on December 5, 2007, fifteen (15) days from receipt of
the VA decision on November 20, 2007, but paid her docket fees in full only after seventy-two (72) days, when she
filed her motion for reconsideration on February 15, 2008 and attached the postal money orders for ₱4,230.00.
Undeniably, the docket fees were paid late, and without payment of the full docket fees, Cobarrubias’ appeal was not
perfected within the reglementary period.
Remedial Law I| Page 38

Gipa vs. Southern Luzon Institute


G.R. No. 177425, June 18, 2014

Facts:

SLI (Southern Luzon Institute, an educational institution in Bulan, Sorsogon) asserts absolute ownership of a 7,516-
squaremeter parcel of land situated in Brgy. Poblacion, Bulan, Sorsogon. However, petitioners (defendants in the
court a quo) who were occupying a portion of said land refused to vacate it despite demand prompting SLI to file a
Complaint for Recovery of Ownership and Possession with Damages against petitioners (defendants) Alonzo
Gipa et.al., The defendants asserted in their answer that they did not heed the demand as they believe that they
have the right over the land they occupy and which was also occupied by their predecessors-in-interest which,
according to them, dates back to as early as 1950. Impugning SLI’s claims, the defendants averred that SLI had not
even for a single moment taken possession of the subject property and was merely able to procure a title over the
same thru fraud, bad faith and misrepresentation.

The RTC (court a quo) ruled in favor of SLI. The petitioners appealed to the CA. Their appeal was initially dismissed
since it was not shown that the appellate court docket fees and other lawful fees were paid. Through a Motion for
Reconsideration (MR) with an attached Certification that they already paid the appeal fee, the CA reinstated the
appeal. Subsequently however, the CA required the petitioners to pay within 10 days from receipt of minute
resolution the amount of 30.00 pesos as legal research fund. It was received by their counsel on March 13, 2006.
Even after a lapse of 9 months, the petitioners still failed to comply with this directive resulting in the dismissal of
their appeal.

Petitioners then filed a Petition for Review on Certiorari and are invoking the principle of liberality in the
application of technical rules considering that they have paid the substantial amount of ₱3,000.00 for docket and
other legal fees and fell short only by the meager amount of ₱30.00. They assert that they come under the exception,
in the name of substantial justice and fair play.

Issue:

Whether or not the principle of liberality in the application of the rules should be applied despite the non-payment of
the 30-peso legal research fund to perfect the appeal?

Held:

No. The procedural requirement under Section 4 of Rule 41 is not merely directory, as the payment of the docket
and other legal fees within the prescribed period is both mandatory and jurisdictional. It bears stressing that an
appeal is not a right, but a mere statutory privilege. An ordinary appeal from a decision or final order of the RTC to
the CA must be made within 15 days from notice. And within this period, the full amount of the appellate court
docket and other lawful fees must be paid to the clerk of the court which rendered the judgment or final order
appealed from. Without such payment, the appeal is not perfected. The appellate court does not acquire jurisdiction
over the subject matter of the action and the Decision sought to be appealed from becomes final and executory.
Further, under Section 1 (c), Rule 50, an appeal may be dismissed by the CA, on its own motion or on that of the
appellee, on the ground of the non-payment of the docket and other lawful fees within the reglementary period as
provided under Section 4 of Rule 41. The payment of the full amount of the docket fee is an indispensable step for
the perfection of an appeal.

The liberality which petitioners pray for has already been granted to them by the CA at the outset. Based on the
premise that the questioned Decision of the RTC has already become final and executory due to non-perfection, the
CA could have dismissed the appeal outright. But owing to the fact that only the meager amount of ₱30.00 was
Remedial Law I| Page 39

lacking and considering that the CA may opt not to proceed with the case until the docket fees are paid, it still
required petitioners, even if it was already beyond the reglementary period, to complete their payment of the appeal
fee within 10 days from notice. The CA’s leniency over petitioners’ cause did not end there. Although they were given
only 10 days to remit the ₱30.00 deficiency, the said court allowed an even longer period of nine months to lapse.
Moreover, petitioners’ failure to advance any explanation as to why they failed to pay the correct docket
fees or to complete payment of the same within the period allowed by the CA is fatal to their cause.
Explaining this, the Court stated that "[c]oncomitant to the liberal interpretation of the rules of procedure should be
an effort on the part of the party invoking liberality to adequately explain his failure to abide by the rules." Those
who seek exemption from the application of the rule have the burden of proving the existence of
exceptionally meritorious reason warranting such departure.
Remedial Law I| Page 40


Reyes vs. People
G.R. No. 193034, July 20, 2015

Facts:

Salud Gegato filed a Complaint against Rodging Reyes for grave threats in the MCTC of Bayugan and Sibagat,
Bayugan, Agusan del Sur. The information stated that Reyes uttered these words against Gegato: “Salud, stop your
rumor against my wife because she will be embarrassed. I’m warning you, don't mind our lives for I might kill you.”
Before arraignment, petitioner filed a Motion to Quash based on the ground of jurisdiction and that the crime is not
Grave Threats under Article 282 of the Revised Penal Code, but Other Light Threats under Article 285, paragraph 2
of the same Code which was denied by the MCTC. Reyes then filed a Motion to Inhibit the Presiding Judge on the
ground that Gegato is the Court Interpreter of the same court but it was also denied. The MCTC found Reyes guilty
beyond reasonable doubt of the crime charged.

On appeal to the RTC, Reyes was found guilty only of other light threats. Instead of filing an appeal to the CA
within the reglementary period, Reyes filed a Motion for Extension of time twice which was both denied by the CA.
The first was denied for failure to pay the full amount of the docket fees pursuant to Sec. 1, Rule 42 of the Rules of
Court. The second was denied as no further extension may be granted except for most compelling reason. After the
filing of the second MotEx, Reyes filed this Petition. Accordingly, this petition was NOTED but DISMISSED by the
CA among the grounds for its dismissal is failure to pay the complete docket fees. Reyes filed an MR but it was
denied for failure of the petitioner to furnish copies to the Solicitor General and the private respondent. Thus,
petitioner filed a Second Motion for Reconsideration which was also denied by the CA. In this resolution, the CA set
aside the first resolution denying the 1st MR but also dismissed the Petition for Review. In the Third MR, the CA
RESOLVED to merely NOTE WITHOUT ACTION the petitioner's third Motion for Reconsideration, in view of the
23 November 2009 Resolution dismissing this petition with finality. Now, Reyes insists that the CA erred in
favoring procedural technicalities over his constitutional right to due process.

Issue:

Whether or not the CA ruled correctly in dismissing the petition on the ground of failure to pay the complete docket
fees?

Held:

Yes. Section 1, Rule 42 of the Rules of Court states the need to pay docket fees. The rule is that payment in full of
the docket fees within the prescribed period is mandatory. In Manchester v. Court of Appeals, it was held that a
court acquires jurisdiction over any case only upon the payment of the prescribed docket fee. The strict application of
this rule was, however, relaxed two (2) years after in the case of Sun Insurance Office, Ltd. v. Asuncion, wherein the
Court decreed that where the initiatory pleading is not accompanied by the payment of the docket fee, the court may
allow payment of the fee within a reasonable period of time, but in no case beyond the applicable prescriptive or
reglementary period. This ruling was made on the premise that the plaintiff had demonstrated his willingness to
abide by the rules by paying the additional docket fees required.

Thus, in the more recent case of United Overseas Bank v. Ros, the Court explained that where the party does not
deliberately intend to defraud the court in payment of docket fees, and manifests its willingness to abide by the
rules by paying additional docket fees when required by the court, the liberal doctrine enunciated in Sun Insurance
Office, Ltd., and not the strict regulations set in Manchester, will apply.

Admittedly, this rule is not without recognized qualifications. The Court has declared that in appealed cases, failure
to pay the appellate court docket fee within the prescribed period warrants only discretionary as opposed to
automatic dismissal of the appeal and that the court shall exercise its power to dismiss in accordance with the
tenets of justice and fair play, and with great deal of circumspection considering all attendant circumstances.
Remedial Law I| Page 41

In that connection, the CA, in its discretion, may grant an additional period of fifteen (15) days only within which to
file the petition for review upon proper motion and the payment of the full amount of the docket and other lawful
fees and the deposit for costs before the expiration of the reglementary period and that no further extension shall be
granted except for the most compelling reason and in no case to exceed fifteen (15) days. Therefore, the grant of any
extensions for the filing of the petition is discretionary and subject to the condition that the full amount of the docket
and lawful fees are paid before the expiration of the reglementary period to file the petition.

It is only when persuasive reasons exist that the Rules may be relaxed to spare a litigant of an injustice not
commensurate with his failure to comply with the prescribed procedure.34 In the present case, petitioner failed to
convince this Court of the need to relax the rules and the eventual injustice that he will suffer if his prayer is not
granted.
Remedial Law I| Page 42

NTC vs. Ebesa


G.R. No. 186102, February 24, 2016

Facts:

Early in 2005, NTC filed a case to expropriate the 1,479-square-meter portion of Lot No. 18470, situated in Quiot,
Pardo, Cebu City. It is declared under the co-ownership of the heirs of Teodulo Ebesa, namely, Porferia Ebesa, Efren
Ebesa, Dante Ebesa and Cynthia Ebesa Ramirez (Heirs of Ebesa), but is occupied by Atty. Fortunato Veloso (Veloso)
(respondents), who allegedly purchased the property, as evidenced by an unregistered Deed of Sale. NTC alleged
that the acquisition of an easement right-of-way over a portion of the subject property is necessary for the
construction and maintenance of the Quiot (Pardo) 100MVA Substation Project in Cebu City, an undertaking that
partook of a public purpose.

In his Answer, Veloso, acting as his own counsel in collaboration with Atty. Nilo Ahat, conceded that the project was
indeed intended for a public purpose but disputed its necessity and urgency and alleged that the project will not only
affect a portion of the property but its entirety considering that the construction entails the installation of huge
permanent steel towers and the air space directly above the subject property will be permanently occupied with
transmission lines. The RTC issued an Order of Expropriation. After settling the amount of just compensation to be
paid to Veloso, the RTC rendered a decision directing the NTC to pay Veloso the amount of 35M pesos with interest
and costs and or immediately return the land to Veloso and await the finality of the judgment before paying Veloso.
The MR filed by NTC was denied by the RTC prompting it to file an appeal before the CA. Thereafter, the CA
directed the NTC to submit official receipt or proof of payment of the appeal fees within 10 days from notice. The
NTC filed a Manifestation, alleging that it cannot comply with the order of the CA as it did not pay appeal
docket fees. It asseverated that the receiving clerk of the RTC did not accept its payment for the appeal
fees on the ground that it is exempted from doing so, being a GOCC. The now respondents filed a Motion to
Dismiss stating that the judgment has now attained finality since the payment of docket fees is mandatory and
jurisdictional and non-payment thereof will not toll the running of the appeal period. Afterwards, the NTC filed
another Manifestation informing the CA that it already filed on September 18, 2006 a Manifestation with Urgent
Ex-Parte Motion with the RTC and settled the payment of appeal fees also submitting the official receipts for the
said payment

The CA granted the Motion to Dismiss. The CA held that the NTC's counsel should know that as a GOCC, it is
not exempted from the payment of docket and other legal fees.

Unyielding, the NTC, in its present appeal, contends that the failure to pay appeal docket fees does not
automatically cause the dismissal of the appeal, but lies on the discretion of the court. It asseverates that since its
failure to pay the appeal fees was not willful and deliberate, its omission could be excused in the interest of justice
and equity. It reiterates that it was prepared to pay the docket fees if not for the receiving clerk's advice that the
same was not necessary as it is a GOCC. Even then, it eventually paid the appeal fees, although past the
reglementary period.

Issue:

Whether or not CA was correct in dismissing NTC’s appeal on the ground of non payment of docket fees resulting
into the finality of the RTC’s decision?

Held:

Yes. Verily, the payment of appeal docket fees is both mandatory and jurisdictional. It is mandatory as it is required
in all appealed cases, otherwise, the Court does not acquire the authority to hear and decide the appeal. The failure
to pay or even the partial payment of the appeal fees does not toll the running of the prescriptive period, hence, will
Remedial Law I| Page 43

not prevent the judgment from becoming final and executory. Such was the circumstance in the instant appeal. The
NTC failed to pay the appeal fees without justifiable excuse. That its counsel or his representative was misled by the
advice of the receiving clerk of the RTC is unacceptable as the exercise of ordinary diligence could have avoided such
a blunder. It is apparent from the records that the NTC had ample time to rectify the error or clarify its reservation
regarding the propriety of its supposed exemption from the appeal fees.

In the present case, the NTC failed to present any justifiable excuse for its failure to pay the docket fees like in the
cases of Mactan Cebu International Airport Authority v. Manguhat and Yambao v. CA. In Mactan Cebu
International Airport Authority, the petitioner took the initiative to verify the necessity of paying the docket fees
and paid it outright, albeit six days after the lapse of the period to appeal. Quite the opposite, the NTC in the
present case never lifted a finger until it was required by the CA to present proof of its payment of the docket fees
and paid the same only six months after the period to appeal has prescribed.
Remedial Law I| Page 44

Vargas vs. Estate of Ogsos


G.R. No. 221062, October 5, 2016

Facts:

Ogsos, Sr. and the Heirs of Fermina Pepico (Fermina), represented by their Attorney-in-Fact, Catalino V. Noel,
entered into a Contract of Lease (lease contract) covering five (5) parcels of agricultural land owned by the latter.
Based on the contract, Ogsos, Sr. agreed to pay the Heirs of Fermina 230 piculs or 290.95 liquid-kilogram (lkg.) of
centrifugal sugar every crop year, starting from crop year 1994-1995 to crop year 2000-2001, as lease rental. The
contract was subsequently extended for three years and the lease rental was modified. Petitioner and Kathryn, who
are among the heirs of Fermina, claimed that the lease rentals from crop year 1994-1995 to crop year 1998-1999
were not paid. Thus, they filed a Complaint for Specific Performance and Damages against respondents to recover
the unpaid lease rentals.
In their Answer, respondents alleged that they had faithfully complied with their obligations as embodied in the
lease contract and its subsequent amendments. They denied abandoning the leased premises and claimed that
sometime in December 1998, petitioner and Kathryn unlawfully took possession of the leased premises and
appropriated for themselves the sugarcane ready for harvest under the pretext that they would apply the proceeds
thereof to the unpaid rent. Respondents filed a counterclaim for lost profits plus damages that it had sustained
when petitioner took over the possession of the leased premises and harvesting and appropriating respondents'
crops planted therein. Petitioners filed a motion to dismiss respondents' counterclaim arguing that the same were
permissive and that respondents had not paid the appropriate docket fees. The RTC, denied the said motion,
declaring respondents' counterclaim as compulsory; thus, holding that the payment of the required docket fees was
no longer necessary. The CA affirmed the ruling of the RTC.

Issue:

Whether or not the respondents' counterclaim for damages is compulsory and not permissive in nature, and thus, no
payment of docket fees is required?

Held:

No. In Spouses Mendiola v. CA, the Court had devised tests in determining whether or not a counterclaim is
compulsory or permissive:

(a) Are the issues of fact or law raised by the claim and the counterclaim largely the same?
(b) Would res judicata bar a subsequent suit on defendant's claims, absent the compulsory counterclaim rule?
(c) Will substantially the same evidence support or refute plaintiff's claim as well as the defendant's counterclaim?
(d) Is there any logical relation between the claim and the counterclaim, such that the conduct of separate trials of
the respective claims of the parties would entail a substantial duplication of effort and time by the parties and the
court?

If these tests result in affirmative answers, the counterclaim is compulsory.

Based on the abovementioned standards, the Court finds that the counterclaim of respondents is permissive in
nature. This is because: (a) the issue in the main case, i.e., whether or not respondents are liable to pay lease
rentals, is entirely different from the issue in the counterclaim, i.e., whether or not petitioner and Kathryn are liable
for damages for taking over the possession of the leased premises and harvesting and appropriating respondents'
crops planted therein; (b) since petitioner and respondents' respective causes of action arose from completely
different occurrences, the latter would not be barred by res judicata had they opted to litigate its counterclaim in a
separate proceeding; (c) the evidence required to prove petitioner's claim that respondents failed to pay lease rentals
is likewise different from the evidence required to prove respondents' counterclaim that petitioner and Kathryn are
liable for damages for performing acts in bad faith; and (d) the recovery of petitioner's claim is not contingent or
dependent upon proof of respondents' counterclaim, such that conducting separate trials will not result in the
substantial duplication of the time and effort of the court and the parties.
Remedial Law I| Page 45

In view of the finding that the counterclaim is permissive, and not compulsory as held by the courts a quo,
respondents are required to pay docket fees. However, it must be clarified that respondents' failure to pay the
required docket fees, per se, should not necessarily lead to the dismissal of their counterclaim. It has long been
settled that while the court acquires jurisdiction over any case only upon the payment of the prescribed docket fees,
its non-payment at the time of filing of the initiatory pleading does not automatically cause its dismissal provided
that: (a) the fees are paid within a reasonable period; and (b) there was no intention on the part of the claimant to
defraud the government. Here, respondents cannot be faulted for non-payment of docket fees in connection with
their counterclaim, primarily because RTC and CA had already found such counterclaim to be compulsory.
Respondents' reliance on the findings of the courts a quo, albeit erroneous, exhibits their good faith in not paying the
docket fees, much more their intention not to defraud the government. Thus, the counterclaim should not be
dismissed for nonpayment of docket fees. Instead, the docket fees required shall constitute a judgment lien on the
monetary awards in respondents’ favor.
Remedial Law I| Page 46

Camaso vs. TSM Shipping, Inc.


G.R. No. 223290, November 7, 2016

Facts:

A complaint for disability benefits was filed before the Labor Arbiter by seaman Woodrow Camacho who was
suffering from tonsillar cancer against his employer TSM Shipping (Phils.), Inc., Utkilen, and Jones Tulod who
refused to shoulder his medical expenses. The LA decided in his favor, but NLRC dismissed his complaint. On
certiorari before the CA, latter dismissed his petition "for non-payment of the required docketing fees as required
under Section 3, Rule 46 of the Revised Rules of Court."

Camaso filed a Motion for Reconsideration arguing, inter alia, that a check representing the payment of the required
docket fees was attached to a copy of his petition filed before the CA. He further claimed that upon verification of his
counsel’s messenger, the Division Clerk of Court admitted that it was simply overlooked. CA denied Camaso's
motion. Citing the presumption of regularity of official duties, the CA gave credence to the explanation of the OIC of
the CA Receiving Section, that there was no cash, postal money order, or check attached to HIS petition when it was
originally filed before the CA. In any event, the CA held that assuming that a check was indeed attached to the
petition, such personal check, i.e., Metrobank check dated July 6, 2015 under the personal account of a certain Pedro
L. Linsangan, is not a mode of payment sanctioned by the 2009 Internal Rules of the Court of Appeals (2009 IRCA),
which allows only payment in cash, postal money order, certified, manager's or cashier's checks payable to the CA.

Issue:

Whether or not the CA correctly dismissed Camaso's petition for certiorari despite the fact that a check representing
payment of the docket fees was attached to the copy of his petition?

Held:

No. The failure to pay the required docket fees per se should not necessarily lead to the dismissal of a case. It has
long been settled that while the court acquires jurisdiction over any case only upon the payment of the prescribed
docket fees, its non-payment at the time of filing of the initiatory pleading does not automatically cause its dismissal
provided that: (a) the fees are paid within a reasonable period; and there was no intention on the part of the
claimant to defraud the government.

Here, it appears that when Camaso filed his certiorari petition through his counsel and via mail, a Metrobank check
dated July 6, 2015 under the account name of Pedro L. Linsangan was attached thereto to serve as payment of
docket fees.26 Although this was not an authorized mode of payment under Section 6, Rule VIII27 of the 2009 IRCA,
the attachment of such personal check shows that Camaso exerted earnest efforts to pay the required docket fees.
Clearly, this exhibits good faith and evinces his intention not to defraud the government. In this relation, the
assertion of the Officer-in-Charge of the CA Receiving Section that there was no check attached to
Camaso's certiorari petition is clearly belied by the fact that when it was examined at the Office of the Division
Clerk of Court, the check was found to be still stapled thereto.

In light of the foregoing circumstances, the Court deems it appropriate to relax the technical rules of procedure in
the interest of substantial justice and, hence, remands the instant case to the CA for the resolution of its substantial
merits.
Remedial Law I| Page 47

Rule 2: Causes of Action (Splitting)

Dynamic Builders vs. Presbitero


G.R. No. 174202, April 7, 2015

Facts:

The Municipality of Valladolid, Negros Occidental published an invitation to bid for the construction of a 1,050-
lineal-meter rubble concrete seawall along the municipality’s shoreline. This infrastructure venture is known as the
“Construction Shoreline Protection Project.” The Bids and Awards Committee (BAC) conducted a pre-bid conference
attended by six (6) prospective contractors including Dynamic Builders. The BAC recommended the award in favor
of HLJ Construction and Enterprise for a local infrastructure project. Dynamic Builders questioned the declaration
of its bid by the BAC as “substantially unresponsive”. Upon the denial of its formal protest, it filed a petition for
certiorari before the RTC. Simultaneously, it filed a petition for prohibition with application for temporary
restraining order and/or writ of preliminary injunction before the Supreme Court.

Dynamic Builders submits that Article XVII, Section 58 of Republic Act No. 9184 implicitly allowed it to
simultaneously file a Petition for Certiorari before the Regional Trial Court assailing the protest case on the merits,
and another Petition before this court for injunctive remedies. Dynamic Builders argues that the law prohibiting
lower courts from issuing TRO, preliminary injunctions, or preliminary mandatory injunctions in national
infrastructure projects (R.A. 8975) also prohibits the same in local infrastructure projects. As a result, only the SC
may issue the preliminary injunction sought for in this petition.

Issue:

Whether or not Dynamic Builders violated the rule against splitting of a cause of action when it filed a petition for
certiorari with the RTC and at the same time a petition for prohibition with the SC?

Held:

Yes. Rule 2, Section 3 of the Rules of Court provides that “[a] party may not institute more than one suit for a single
cause of action.” Petitioner’s petition for prohibition seeks to enjoin the execution of public respondent’s Decision and
Resolution on the protest — the same Decision and Resolution sought to be set aside in the Petition before the
Regional Trial Court. In essence, petitioner seeks the same relief through two separate Petitions filed before
separate courts. This violates the rule against forum shopping.

The petitioner is mistaken when it argued that only the Supreme Court can issue injunctive relief against infracture
projects of local governments. The Regional Trial Court can issue injunctive relief against government
infrastructure projects, even those undertaken by local governments, considering that the prohibition in Section 3 of
Republic Act No. 8957 only mentions national government projects. These courts can issue injunctive relief when
there are compelling constitutional violations — only when the right is clear, there is a need to prevent grave and
irreparable injuries, and the public interest at stake in restraining or enjoining the project while the action is
pending far outweighs the inconvenience or costs to the party to whom the project is awarded.
Remedial Law I| Page 48

Rule 3: Parties
Relucio vs. Lopez
G.R. No. 138497, January 16, 2002

Facts:

Angelina Mejia Lopez filed a petition for Appointment as sole administratix of conjugal partnership of properties,
forfeiture, etc. against her husband, Alberto Lopez and petitioner Imelda Relucio. She alleged that sometime in
1968, her husband, who is legally married to her, abandoned the latter and their four legitimate children; that he
arrogated unto himself full and exclusive control and administration of the conjugal properties, spending and using
the same for his sole gain and benefit to the total exclusion of her and their four children; and after abandoning his
family, maintained an illicit relationship and cohabited with Relucio since 1976.

A motion to dismiss was filed by Relucio on the ground that Angelina Lopez has no cause of action against her. The
RTC denied her motion. A Motion for Reconsideration was also denied by the RTC. She filed a petition for certiorari
with the CA which also denied. Hence, this petition.

Issue:

Whether or not Relucio is an indispensable party in this case?

Held:

No. A real party in interest is one who stands “to be benefited or injured by the judgment of the suit.” In this case,
petitioner would not be affected by any judgment in Special Proceedings M-3630. If petitioner is not a real party-in-
interest, she cannot be an indispensable party. An indispensable party is one without whom there can be no final
determination of an action. Petitioner’s participation in Special Proceedings M-3630 is not indispensable. Certainly,
the trial court can issue a judgment ordering Alberto J. Lopez to make an accounting of his conjugal partnership
with respondent, and give support to respondent and their children, and dissolve Alberto J. Lopez’ conjugal
partnership with respondent, and forfeit Alberto J. Lopez’ share in property co-owned by him and petitioner. Such
judgment would be perfectly valid and enforceable against Alberto J. Lopez. Nor can petitioner be a necessary party
in Special Proceedings M-3630. A necessary party as one who is not indispensable but who ought to be joined as
party if complete relief is to be accorded those already parties, or for a complete determination or settlement of the
claim subject of the action. In the context of her petition in the lower court, respondent would be accorded complete
relief if Alberto J. Lopez were ordered to account for his alleged conjugal partnership property with respondent, give
support to respondent and her children, turn over his share in the co-ownership with petitioner and dissolve his
conjugal partnership or absolute community property with respondent.
Remedial Law I| Page 49

De Castro vs. Court of Appeals


July 18, 2002

Facts:

Petitioners Constante A. De Castro and Corazon A. De Castro were co-owners of four lots. Francisco Antigo was
authorized by the De Castros to act as real estate broker in the sale of these lands. 5% of which will be given to him
as commission. Later on, two lots were bought by Times Transit Corporation. However, Artigo sued the De Castros
to collect the unpaid balance of his broker’s commission. One of the defenses advanced by the De Castros is that the
complaint failed to implead their other siblings who were co-owners as well. The De Castros claim that Artigo
always knew that the two lots were co-owned by Constante and Corazon with their other siblings Jose and Carmela
whom Constante merely represented. The De Castros contend that failure to implead such indispensable parties is
fatal to the complaint since Artigo, as agent of all the four coowners, would be paid with funds co-owned by the four
co-owners. RTC held that Artigo is entitled to 5% commission on the purchase price as provided in the contract of
agency. CA affirmed the decision of RTC and held that it is not necessary to implead the other co-owners since the
action is based on contract of agency.

Issue:

Whether or not the other co-owners (other De Castro siblings) are indispensable parties?

Held:

No. An indispensable party is one whose interest will be affected by the courts action in the litigation and without
whom no final determination of the case can be. Joinder of indispensable parties is mandatory and courts cannot
proceed without their presence. However, the rule on mandatory joinder of indispensable parties is not applicable to
the instant case.

There is no dispute that Constante appointed Artigo as agent. Whether Constante appointed Artigo as agent, in
Constante’s individual or representative capacity, or both, the De Castros cannot seek the dismissal of the case for
failure to implead the other co-owners as indispensable parties. The De Castros admit that the other co-owners are
solidarily liable under the contract of agency, citing Article 1915 of the Civil Code, which reads:

Art. 1915. If two or more persons have appointed an agent for a common transaction or undertaking, they shall be
solidarily liable to the agent for all the consequences of the agency.

When the law expressly provides for solidarity of the obligation, as in the liability of co-principals in a contract of
agency, each obligor may be compelled to pay the entire obligation. The agent may recover the whole compensation
from any one of the co-principals, as in this case.
Remedial Law I| Page 50

Urquiola vs. Court of Appeals


G.R. No. 141463. August 6, 2002

Facts:

Pura Kalaw Ledesma was the registered owner of Lot 689, covered by TCT Nos. 111267 and 111266, in Tandang
Sora, Quezon City. This parcel of land was adjacent to certain portions of Lot 707 of the Piedad Estates, namely, Lot
707-A and 707-B, registered in the name of Herminigilda Pedro under TCT Nos. 16951 and 16952, respectively. On
October 29, 1964, Herminigilda sold Lot 707-A and 707-B to Mariano Lising who then registered both lots and Lot
707- C in the name of M.B. Lising Realty and subdivided them into smaller lots.

Certain portions of the subdivided lots were sold to third persons including herein petitioners, spouses Victor and
Honorata Orquiola, who purchased a portion of Lot 707-A-2, Lot 5, Block 1 of the subdivision plan (LRC), Psd-42965.

Sometime in 1969, Ledesma filed a complaint, docketed as Civil Case No. Q-12918, with the Regional Trial Court of
Quezon City against Herminigilda Pedro and Mariano Lising for allegedly encroaching upon Lot 689. During the
pendency of the action, Tandang Sora Development Corporation replaced Ledesma as plaintiff by virtue of an
assignment of Lot 689 made by Ledesma in favor of said corporation. The RTC adjudged defendants Pedro and
Lising jointly and severally liable for encroaching on plaintiff’s land.

As a result, the Deputy Sheriff of Quezon City directed petitioners, through an alias writ of execution, to remove the
house they constructed on the land they were occupying.

To prohibit Judge Baclig of the RTC-QC from issuing a writ of demolition and the Quezon City sheriff from
implementing the alias writ of execution, petitioners filed with the CA a petition for prohibition with prayer for a
restraining order and preliminary injunction on the ground that they bought the subject parcel of land in good faith
and for value, and since they were not impleaded in Civil Case No. Q-12918, the writ of demolition issued in
connection therewith cannot be enforced against them because to do so would amount to deprivation of property
without due process of law. CA dismissed the petition and held that the petitioners were considered privies who
derived their rights from Lising by virtue of the sale and could be reached by the execution order.

Issue:

Whether or not the decision can be enforced against petitioners even if they were not impleaded as parties?

Held:

No. The petitioners are fully entitled to the legal protection of their lot by the Torrens system. Petitioners acquired
the lot before the commencement of Civil Case No. Q-12918 and acquired the registered title in their own names.
They are not successors-in-interest of Mariano Lising, and as such, they cannot be reached by the order of execution
in Civil Case No. Q-12918.

As builders in good faith and innocent purchasers for value, petitioners have rights over the subject property and
hence they are proper parties in interest in any case thereon. Consequently, private respondents should have
impleaded them in Civil Case No. Q-12918. Since they failed to do so, petitioners cannot be reached by the decision
in said case. No man shall be affected by any proceeding to which he is a stranger, and strangers to a case are not
bound by any judgment rendered by the court. In the same manner, a writ of execution can be issued only against a
party and not against one who did not have his day in court. Only real parties in interest in an action are bound by
the judgment therein and by writs of execution and demolition issued pursuant thereto.
Remedial Law I| Page 51

China Banking Corp. vs. Oliver


G.R. No. 135796, October 3, 2002.

Facts:

In August 1995, Pangan Lim, Jr. and a certain Mercedes M. Oliver opened a joint account in China Banking
Corporation at EDSA Balintawak Branch.Thereafter, Lim and Oliver applied for a P17 million loan and offered a
collateral. On November 17, 1995, Lim and Oliver executed in favor of Chinabank a promissory note for
P16,650,000, as well as a Real Estate Mortgage on the property. The mortgage document showed Mercedes Oliver’s
(Oliver One) address to be No. 95 Malakas Street, Diliman, Quezon City.

Later, respondent claiming that she is Mercedes M. Oliver with postal office address at No. 40 J.P. Rizal St., San
Pedro, Laguna, filed an action for annulment of mortgage and cancellation of title with damages against Chinabank
and the Register of Deeds. Respondent (Oliver Two), claimed that she was the registered and lawful owner of the
land subject of the real estate mortgage; that the owners duplicate copy of the title had always been in her
possession; and that she did not apply for a loan or surrender her title to Chinabank. She prayed that: (1) the
owners duplicate copy surrendered to Chinabank as well as the original title with the Registry of Deeds be
cancelled; (2) the mortgage be declared null and void; and (3) the Registry of Deeds be ordered to issue a new and
clean title in her name.

Chinabank moved to dismiss the case for lack of cause of action and non-joinder of an indispensable party, the
mortgagor. The RTC denied the Motion.

Issue:

Whether or not the mortgagor Mercedes Oliver (Oliver One) is an indispensable party to the case?

Held:

No. An indispensable party is a party in interest, without whom no final determination can be had of an action. It is
true that mortgagor Oliver One is a party in interest, for she will be affected by the outcome of the case. She stands
to be benefited in case the mortgage is declared valid, or injured in case her title is declared fake. However,
mortgagor Oliver One’s absence from the case does not hamper the trial court in resolving the dispute between
respondent Oliver Two and petitioner.

Oliver Two’s Complaint was for annulment of mortgage due to petitioner’s negligence in not determining the actual
ownership of the property, resulting in the mortgage’s annotation on TCT No. S-50195 in the Registry of Deeds
custody. This, respondent Oliver Two can do in her complaint without necessarily impleading the mortgagor Oliver
One. Hence, Oliver One is not an indispensable party in the case filed by Oliver Two. Chinabank has interest in the
loan which, however, is distinct and divisible from the mortgagor’s interest, which involves the land used as
collateral for the loan.

A party is also not indispensable if his presence would merely permit complete relief between him and those already
parties to the action, or will simply avoid multiple litigation, as in the case of Chinabank and mortgagor Oliver One.
The latter’s participation in this case will simply enable petitioner Chinabank to make its claim against her in this
case, and hence, avoid the institution of another action. Thus, it was the bank who should have filed a third-party
complaint or other action versus the mortgagor Oliver One.
Remedial Law I| Page 52

Lotte Phils. Co. Inc. vs Dela Cruz


G.R. No. 166302, July 28, 2005

Facts:

Erlinda Dela Cruz, et al. filed an illegal dismissal case against Lotte Phils. Co. Inc and 7J Maintenance and
Janitorial Services. They alleged that they were hired by 7J Maintenance and Janitorial Services, and was assigned
with Lotte Phil, Co., Inc., herein petitioner to perform piece works and as repackers or sealers. They further aver
that due to the expiration of the service contract between 7J and herein petitioner, they were dispensed with their
services, and that they were told by 7J that the latter will call them if they will be needed for work. But they were
never called back. The Labor Arbiter, rendered judgment declaring 7J being the respondents’ employer, and that the
former was guilty of illegal dismissal. The respondents appealed to the NLRC, praying that herein petitioner be
declared as their direct employer because 7J is merely a labor-only contractor. In its decision, NLRC affirmed the
decision of the Labor Arbiter. MR was also denied. Thus, the respondents filed a petition for certiorari before the CA.
Lotte Phils. Co moved to dismiss the petition on the ground that the respondents failed to implead 7J which was the
party-in-interest to the said case. The Court of Appeals reversed and set aside the rulings of the Labor Arbiter and
the NLRC. In its decision, the Court of Appeals declared Lotte as the real employer of respondents and that 7J who
engaged in labor-only contracting was merely the agent of Lotte. Hence, this petition for review on certiorari.

Issue:

Whether or not 7J should have been impleaded in the petition before the CA as an indispensable party?

Held:
Yes. An indispensable party is a party in interest without whom no final determination can be had of an action, and
who shall be joined either as plaintiffs or defendants. The joinder of indispensable parties is mandatory. The
presence of indispensable parties is necessary to vest the court with jurisdiction, which is the authority to hear and
determine a cause, the right to act in a case. Thus, without the presence of indispensable parties to a suit or
proceeding, judgment of a court cannot attain real finality. The absence of an indispensable party renders all
subsequent actions of the court null and void for want of authority to act, not only as to the absent
parties but even as to those present.
In this case, 7J is an indispensable party. It is a party in interest because it will be affected by the outcome of the
case. The Labor Arbiter and the NLRC found 7J to be solely liable as the employer of respondents. The Court of
Appeals however rendered Lotte jointly and severally liable with 7J who was not impleaded by holding that the
former is the real employer of respondents. Plainly, its decision directly affected 7J.
However, non-joinder of indispensable parties is not a ground for the dismissal of an action and the remedy is to
implead the non-party claimed to be indispensable. Parties may be added by order of the court on motion of the party
or on its own initiative at any stage of the action and/or such times as are just. If the petitioner refuses to implead
an indispensable party despite the order of the court, the latter may dismiss the complaint/petition for the
petitioner/plaintiffs failure to comply therefor.
Although 7J was a co-party in the case before the Labor Arbiter and the NLRC, respondents failed to include it in
their petition for certiorari in the Court of Appeals.
Remedial Law I| Page 53

Carabeo vs. Dingco


G.R. No. 190823, April 4, 2011

Facts:

Domingo Carabeo, herein petitioner, entered into a contract denominated as "Kasunduan sa Bilihan ng
Karapatan sa Lupa" (Kasunduan) with Spouses Norberto and Susan Dingco, herein respondents, whereby petitioner
agreed to sell his rights on a parcel of unregistered land situated in Purok III, Tugatog, Orani, Bataan to the
respondents. The respondents tendered the initial payment, while the remaining balance was refused by the
petitioner, proffering as reason therefor that he would register the land first. Subsequently, it came to know to the
respondents that the land was already registered to the petitioner’s name, thus they offered to pay the balance, but
the petitioner declined. They filed a complaint for specific performance before the Regional Trial Court (RTC) of
Balanga, Bataan. The RTC rendered its decision in favor of the respondents. On appeal, the CA affirmed such
decision. MR was denied as well. Hence this petition for review on certiorari. The petitioner’s counsel, avers that the
case should be dismissed due to the death of Domingo Carabeo, being an action personam.

Issue:

Whether or not the case should be dismissed on the ground that the petitioner died?

Held:

No. The question as to whether an action survives or not depends on the nature of the action and the damage sued
for. In the causes of action which survive, the wrong complained [of] affects primarily and principally property and
property rights, the injuries to the person being merely incidental, while in the causes of action which do not
survive, the injury complained of is to the person, the property and rights of property affected being incidental.

In the present case, respondents are pursuing a property right arising from the kasunduan, whereas petitioner is
invoking nullity of the kasunduan to protect his proprietary interest. Assuming arguendo, however, that the
kasunduan is deemed void, there is a corollary obligation of petitioner to return the money paid by respondents, and
since the action involves property rights, it survives.

It bears noting that trial on the merits was already concluded before petitioner died. Since the trial court was not
informed of petitioner’s death, it may not be faulted for proceeding to render judgment without ordering his
substitution. Its judgment is thus valid and binding upon petitioner’s legal representatives or successors-in-interest,
insofar as his interest in the property subject of the action is concerned.

In another vein, the death of a client immediately divests the counsel of authority. Thus, in filing a Notice of Appeal,
petitioner’s counsel of record had no personality to act on behalf of the already deceased client who, it bears
reiteration, had not been substituted as a party after his death. The trial court’s decision had thereby become final
and executory, no appeal having been perfected.
Remedial Law I| Page 54

Dela Cruz vs. Joaquin


G.R. No. 162788, July 28, 2005

Facts:

Pedro Joaquin filed a Complaint for the recovery of possession and ownership, the cancellation of title, and damages,
against petitioners in the Regional Trial Court of Baloc, Sto. Domingo, Nueva Ecija. Respondent alleged that he had
obtained a loan from them in the amount of ₱9,000, payable after 5 years. To secure the payment of the obligation,
he supposedly executed a Deed of Sale in favor of petitioners. The Deed was for a parcel of land in Pinagpanaan,
Talavera, Nueva Ecija. The parties also executed another document entitled "Kasunduan."

Respondent claimed that the Kasunduan showed the Deed of Sale to be actually an equitable mortgage. Spouses De
la Cruz contended that this document was merely an accommodation to allow the repurchase of the property, a right
that he failed to exercise.

RTC issued a Decision in favor of the respondent. The trial court declared that the parties had entered into a sale
with a right of repurchase. It further held that respondent had made a valid tender of payment on two separate
occasions to exercise his right of repurchase.

Sustaining the trial court, the CA noted that petitioners had given respondent the right to repurchase the property
within five (5) years from the date of the sale. The appellate court also found no reason to overturn the finding that
respondent had validly exercised his right to repurchase the land.

In 2004, the CA denied reconsideration and ordered a substitution by legal representatives, in view of respondent’s
death on December 24, 1988. Petitioners assert that the RTC’s Decision was invalid for lack of jurisdiction. They
claim that respondent died during the pendency of the case. There being no substitution by the heirs, the trial court
allegedly lacked jurisdiction over the litigation.

Issue:

Whether or not the RTC Decision should be declared void because there was no substitution made despite the death
of the respondent?

Held:

No. This general rule notwithstanding, a formal substitution by heirs is not necessary when they themselves
voluntarily appear, participate in the case, and present evidence in defense of the deceased. The rule on the
substitution by heirs is not a matter of jurisdiction, but a requirement of due process. Thus, when due process is not
violated, as when the right of the representative or heir is recognized and protected, noncompliance or belated
formal compliance with the Rules cannot affect the validity of a promulgated decision. Mere failure to substitute for
a deceased plaintiff is not a sufficient ground to nullify a trial court’s decision. The alleging party must prove that
there was an undeniable violation of due process.

The records of the present case contain a "Motion for Substitution of Party Plaintiff" dated February 15, 2002, filed
before the CA.

Evidently, the heirs of Pedro Joaquin voluntary appeared and participated in the case. The court stress that the
appellate court had ordered his legal representatives to appear and substitute for him. The substitution even on
appeal had been ordered correctly. In all proceedings, the legal representatives must appear to protect the interests
of the deceased. After the rendition of judgment, further proceedings may be held, such as a motion for
reconsideration or a new trial, an appeal, or an execution.
Remedial Law I| Page 55

Considering the foregoing circumstances, the Motion for Substitution may be deemed to have been granted; and the
heirs, to have substituted for the deceased, Pedro Joaquin. There being no violation of due process, the issue of
substitution cannot be upheld as a ground to nullify the trial court’s Decision.

Navarro vs. Escobido


G.R. No. 153788, November 27, 2009

Facts:

Karen Go, doing business under the trade name KARGO Enterprises, entered into a Lease Agreement with Option
to Repurchase. KARGO Enterprises was represented by Glenn Go (Karen Go’s husband). Subsequently, Karen Go
filed two complaints before the RTC for replevin and/or sum of money with damages against Roger Navarro. In these
complaints, Karen Go prayed that the RTC issue writs of replevin for the seizure of two (2) motor vehicles in
Navarro’s possession. In his Answers, Navarro alleged as a special affirmative defense that the two complaints
stated no cause of action, since Karen Go was not a party to the Lease Agreements with Option to Purchase. RTC
dismissed the case on the ground that the complaints did not state a cause of action.

The RTC issued another order setting aside the order of dismissal. Acting on the presumption that Glenn Go’s
leasing business is a conjugal property, the RTC held that Karen Go had sufficient interest in his leasing business to
file the action against Navarro. However, the RTC held that Karen Go should have included her husband, Glenn Go,
in the complaint based on Section 4, Rule 3 of the Rules of Court (Rules). Thus, the lower court ordered Karen Go to
file a motion for the inclusion of Glenn Go as co-plaintiff.

When the RTC denied Navarro’s motion for reconsideration, Navarro filed a petition for certiorari with the CA,
contending that the RTC committed grave abuse of discretion when it reconsidered the dismissal of the case and
directed Karen Go to amend her complaints by including her husband Glenn Go as co-plaintiff. Navarro alleges that
even if the lease agreements were in the name of KARGO Enterprises, since it did not have the requisite juridical
personality to sue, the actual parties to the agreement are himself and Glenn Go. Since it was Karen Go who filed
the complaints and not Glenn Go, she was not a real party-in-interest and the complaints failed to state a cause of
action. Navarro posits that the RTC erred when it ordered the amendment of the complaint to include Glenn Go as a
co-plaintiff, instead of dismissing the complaint outright because a complaint which does not state a cause of action
cannot be converted into one with a cause of action by a mere amendment or a supplemental pleading. In effect, the
lower court created a cause of action for Karen Go when there was none at the time she filed the complaints.

Issue:

Whether or not Karen Go is a real party in interest and hence, she has a cause of action against Navarro?

Held:

Yes. The 1997 Rules of Civil Procedure requires that every action must be prosecuted or defended in the name of the
real party-in-interest, i.e., the party who stands to be benefited or injured by the judgment in the suit, or the party
entitled to the avails of the suit.

As the registered owner of Kargo Enterprises, Karen Go is the party who will directly benefit from or be injured by a
judgment in this case. Thus, contrary to Navarro’s contention, Karen Go is the real party-in-interest, and it is
legally incorrect to say that her Complaint does not state a cause of action because her name did not appear in the
Lease Agreement that her husband signed in behalf of Kargo Enterprises. Whether Glenn Go can legally sign the
Lease Agreement in his capacity as a manager of Kargo Enterprises, a sole proprietorship, is a question we do not
decide, as this is a matter for the trial court to consider in a trial on the merits.

In a co-ownership, co-owners may bring actions for the recovery of co-owned property without the necessity of joining
all the other co-owners as co-plaintiffs because the suit is presumed to have been filed for the benefit of his
co-owners. Article 487 of the Civil Code, which provides that any of the co-owners may bring an action for
ejectment, covers all kinds of action for the recovery of possession. In sum, in suits to recover properties, all co-
Remedial Law I| Page 56

owners are real parties in interest. However, pursuant to Article 487 of the Civil Code and relevant jurisprudence,
any one of them may bring an action, any kind of action, for the recovery of co-owned properties. Therefore, only one
of the co-owners, namely the co-owner who filed the suit for the recovery of the co-owned property, is an
indispensable party thereto. The other co-owners are not indispensable parties. They are not even necessary parties,
for a complete relief can be accorded in the suit even without their participation, since the suit is presumed to have
been filed for the benefit of all co-owners. Under this ruling, either of the spouses Go may bring an action against
Navarro to recover possession of the Kargo Enterprises-leased vehicles which they co-own.
Remedial Law I| Page 57

Landbank vs. Cacayuran


G.R. No. 191667, April 22, 2015

Facts:

The Sangguniang Bayan of the Municipality Agoo through two Resolutions authorized its Mayor to obtain loans
from the LBP for the implementation of a multi-phased plan (Re-development Plan) and construction of a
commercial center on the Plaza Lot as part of Phase II of the said plan. By virtue of such loan, the Municipality
mortgaged the southern portion of the Agoo Plaza and authorized the assignment of a portion of the Internal
Revenue Allotment (IRA) in favor LBP. Consequently, respondent filed a suit against petitioner including the
Implicated Officers invoking his right as a taxpayer, after he did not get any response from the Municipality’s
officials on his request for the pertinent documents of the Redeveopment Plan. Respondent questioned the validity of
the Subject Loans on the ground that the Plaza Lot used as collateral thereof is property of public dominion and
therefore, beyond the commerce of man.

Petitioner filed a motion to dismiss, but to no avail. In its Answer, Land Bank claimed that it was not privy to the
Implicated Officers’ acts of destroying the Agoo Plaza. It further asserted that Cacayuran did not have a cause of
action against it since he was not privy to any of the Subject Loans.

The RTC declared the Subject Loans null and void, finding that the resolutions approving the procurement of the
same were passed in a highly irregular manner and thus, ultra vires. As such, it pronounced that the Municipality
was not bound by the Subject Loans and that the municipal officers should, instead, be held personally liable for the
same. Further, it ruled that since the Plaza Lot is a property for public use, it cannot be used as collateral for the
Subject Loans. The LBP and the municipal officers appealed to the CA. However, the appeal of the municipal
officers was deemed abandoned and dismissed for their failure to file an appellants' brief.

The Appellate court affirmed RTC’s decision with modification. Petitioner filed a Motion for Reconsideration when in
the meantime, the Municipality filed a Motion for Leave to Intervene and a Motion for Reconsideration in
Intervention contending that as a contracting party to the Subject Loans, it is an indispensable party to the action
filed by Cacayuran. As such, there cannot be any "real disposition" of the instant suit by reason of its exclusion from
the same. However, Cacayuran insists that the Municipality was not a real party-in-interest to the instant case as
his complaint was against the municipal officers in their personal capacity for their ultra vires acts which are not
binding on the Municipality, hence, this case.

Issue:

Whether or not the Municipality of Agoo is an indispensable party?

Held:

Yes. The Municipality is an indispensable party under Sec 7, Rule 3 of the Rules of Court.

Sec 7, Rule 3 mandates that all indispensable parties are to be joined in a suit as it is the party whose interest will
be affected by the court’s action and without whom no final determination of the case can be had. His legal presence
is an absolute necessity. Absence of the indispensable party renders all subsequent actions of the court null and void
for want of authority to act. Failure to implead any indispensable party is not a ground for the dismissal of the
complaint. The proper remedy is to implead them.

In this case, the records reveals that Cacayuran's complaint against LBP and the municipal officers primarily prays
that the commercialization of the Public Plaza be enjoined and also, that the Subject Loans be declared null and void
for having been unlawfully entered into by the said officers. However, Cacayuran failed to implead in his complaint
the Municipality, a real party-in-interest41and an indispensable party that stands to be directly affected by any
judicial resolution on the case, considering that: (a) the contracting parties to the Subject Loans are LBP and the
Remedial Law I| Page 58

Municipality; and (b) the Municipality owns the Public Plaza as well as the improvements constructed thereon,
including the Agoo People's Center.
Remedial Law I| Page 59

Divinagracia vs. Parilla


G.R. No. 196750, March 11, 2015

Facts:

Conrado Nobleza, Sr. owned a parcel of land in Iloilo City. During his lifetime, he contracted two marriages in which
he had two children in his first marriage and seven in his second in which two of his children predeceased him.
Besides that, he also had three illigitimate children.

According to petitioner (Santiago Divinagracia), upon the decedent’s death, he was able to acquire interests over the
subject land when some of the heirs including Felcon (in representation of his father who predeceased the decedent
and his siblings) sold their respective interests to him as embodied in a Deed of Extrajudicial Settlement or
Adjudication with Deed of Sale, which, however, was not signed by the other heirs who did not sell their respective
shares, namely, Ceruleo, Celedonio, and Maude. Subsequently, the same parties executed a Supplemental Contract
whereby the vendors-heirs and Santiago agreed that Santiago would only pay the remaining balance upon the
partition of the subject land. However, Santiago was not able to have the Transfer of Certificate of Title (TCT)
cancelled and the subject document registered because of Ceruleo, Celedonio, and Maude’s refusal to surrender the
said title prompting the filing of a complaint for judicial partition and for receivership.

The RTC found that through the subject document, Santiago became a co-owner of the subject land and, as such,
had the right to demand the partition of the same. However, the RTC held that Santiago did not validly acquire
Mateo, Sr.’s share over the subject land, considering that Felcon admitted the lack of authority to bind his siblings
with regard to Mateo, Sr.’s share thereon.

The CA set aside the RTC Rulings and, consequently, dismissed Santiago’s complaint for judicial partition. It held
that Felcon’s siblings, as well as Maude’s children, are indispensable parties to the judicial partition of the subject
land and, thus, their non-inclusion as defendants in Santiago’s complaint would necessarily result in its dismissal.
The heirs of Santiago moved for reconsideration contending that Santiago had already bought the interests of the
majority of the heirs and, thus, they should no longer be regarded as indispensable parties, but to no avail, hence,
this case.

Issue:

Whether or not all the co-owners are indispensable parties in an action for judicial partition?

Held:

Yes. An indispensable party is one whose interest will be affected by the court’s action in the litigation, and without
whom no final determination of the case can be had. The party’s interest in the subject matter of the suit and in the
relief sought are so inextricably intertwined with the other parties’ that his legal presence as a party to the
proceeding is an absolute necessity. In his absence, there cannot be a resolution of the dispute of the parties before
the court which is effective, complete, or equitable. Thus, the absence of an indispensable party renders all
subsequent actions of the court null and void, for want of authority to act, not only as to the absent parties but even
as to those present.

With regard to actions for partition, Section 1, Rule 69 of the Rules of Court requires that all persons interested in
the property shall be joined as defendants, viz.:

SEC. 1. Complaint in action for partition of real estate. – A person having the right to compel the partition of real
estate may do so as provided in this Rule, setting forth in his complaint the nature and extent of his title and an
adequate description of the real estate of which partition is demanded and joining as defendants all other persons
interested in the property.
Remedial Law I| Page 60

Thus, all the co-heirs and persons having an interest in the property are indispensable parties; as such, an action for
partition will not lie without the joinder of the said parties.

In the instant case, however, a reading of Santiago’s complaint shows that as regards Mateo, Sr.’s interest, only
Felcon was impleaded, excluding therefrom his siblings and co-representatives. Similarly, with regard to Cebeleo,
Sr.’s interest over the subject land, the complaint impleaded his wife, Maude, when pursuant to Article 972 of the
Civil Code; the proper representatives to his interest should have been his children, Cebeleo, Jr. and Neobel. Verily,
Santiago’s omission of the aforesaid heirs renders his complaint for partition defective.

In fine, the absence of the aforementioned indispensable parties in the instant complaint for judicial partition
renders all subsequent actions of the RTC null and void for want of authority to act, not only as to the absent
parties, but even as to those present. Therefore, the CA correctly set aside the November 29, 2002 Decision and the
April 4, 2003 Order of the RTC.
Remedial Law I| Page 61

Rule 4: Venue

Pacific Consultants International Asia, Inc. vs. Schonfeld


G.R. No. 166920. February 19, 2007.
Facts:

Klaus Schonfeld, respondent, filed a Complaint for Illegal Dismissal against Pacific Consultants Asia, Inc. (Pacicon
Philippines, Inc.) before the Labor Arbiter. Petitioners filed a Motion to Dismiss the complaint on the ground that
venue was improperly laid. It averred that under the Employment Contract, complainant and Pacific Consultants
International of Japan (PCIJ) had agreed that any employment-related dispute should be brought before the London
Court of Arbitration. The provision provides that:

Any question of interpretation, understanding or fulfillment of the conditions of employment, as well as any question
arising between the Employee and the Company which is in consequence of or connected with his employment with
the Company and which can not be settled amicably, is to be finally settled, binding to both parties through written
submissions, by the Court of Arbitration in London.

The Labor Arbiter granted the Motion to Dismiss and ruled that since the parties had agreed that any differences
regarding employer-employee relationship should be submitted to the jurisdiction of the court of arbitration in
London, this agreement is controlling. The NLRC affirmed the LA’s decision. The CA reversed the LA and NLRC. It
declared that under the employment contract, the parties were not precluded from bringing a case related thereto in
other venues. While there was, indeed, an agreement that issues between the parties were to be resolved in the
London Court of Arbitration, the venue is not exclusive, since there is no stipulation that the complaint cannot be
filed in any other forum other than in the Philippines. Hence, the present petition before the Supreme Court.

Issue:

Whether or not the venue stipulated in the Employment Contract precludes the filing of the case before other
venue/forum?

Held:

No. The settled rule on stipulations regarding venue is that while they are considered valid and enforceable, venue
stipulations in a contract do not, as a rule, supersede the general rule set forth in Rule 4 of the Revised Rules of
Court in the absence of qualifying or restrictive words. They should be considered merely as an agreement or
additional forum, not as limiting venue to the specified place. They are not exclusive but, rather permissive. If the
intention of the parties were to restrict venue, there must be accompanying language clearly and categorically
expressing their purpose and design that actions between them be litigated only at the place named by them. In the
instant case, no restrictive words like “only,” “solely,” “exclusively in this court,” “in no other court save—,”
“particularly,” “nowhere else but/except—,” or words of equal import were stated in the contract. It cannot be said
that the court of arbitration in London is an exclusive venue to bring forth any complaint arising out of the
employment contract.

Note: Petitioner was also insisting on the application of the principle of forum non conveniens. The Court ruled that
the mere fact that respondent is a Canadian citizen and was a repatriate does not warrant the application of the
principle because forum non conveniens is not a ground for dismissal and the propriety of dismissing a case based on
this principle requires a factual determination; hence, it is properly considered as defense.
Remedial Law I| Page 62

Biaco vs. Philippine Countryside Rural Bank


G.R. No. 161417. February 8, 2007.

Topic: Action in personam, in rem and quasi in rem

Facts:

Ernesto Biaco failed to settle its loans (totaling P1,080,676.50) from Philippine Countryside Rural Bank. PCRB filed
a complaint for foreclosure of mortgage against the spouses Ernesto and Teresa Biaco before the RTC of Misamis
Oriental. Summons was served to the spouses Biaco through Ernesto at his office. Ernesto received the summons
but for unknown reasons, he failed to file an answer. Hence, the spouses Biaco were declared in default upon motion
of the bank. PCRB was allowed to present its evidence ex parte. Judgment was rendered in favor of PCRB. Spouses
Biaco failed to appeal. A writ of execution was issued and the mortgaged property was sold at public auction in favor
of the PCRB in the amount of P150,000. Since the amount was insufficient, PCRB filed an “ex parte motion for
judgment” praying for the issuance of a writ of execution against the other properties of the spouses Biaco for the
full settlement of the remaining obligation. The trial court granted the motion. Ma. Teresa (the wife) then sought
the annulment of the RTC decision contending that extrinsic fraud prevented her from participating in the judicial
foreclosure proceedings. According to her, she came to know about the judgment in the case only after the lapse of
more than six (6) months after its finality. She claimed the that trial court failed to acquire jurisdiction because
summons were served on her through her husband without any explanation as to why personal service could not be
made. The Court of Appeals denied Ma. Teresa’s petition and ruled that judicial foreclosure proceedings are actions
quasi in rem. As such, jurisdiction over the person of the defendant is not essential as long as the court acquires
jurisdiction over the res. Hence, the present petition.

Issue:

Whether or not the trial court acquired jurisdiction over Ma. Teresa?

Held:

No. The question of whether the trial court has jurisdiction depends on the nature of the action, i.e., whether the
action is in personam, in rem, or quasi in rem. The rules on service of summons under Rule 14 of the Rules of Court
likewise apply according to the nature of the action.

An action in personam is an action against a person on the basis of his personal liability. An action in rem is an
action against the thing itself instead of against the person. An action quasi in rem is one wherein an individual is
named as defendant and the purpose of the proceeding is to subject his interest therein to the obligation or lien
burdening the property.

In an action in personam, jurisdiction over the person of the defendant is necessary for the court to validly try and
decide the case. In a proceeding in rem or quasi in rem, jurisdiction over the person of the defendant is not a
prerequisite to confer jurisdiction on the court provided that the court acquires jurisdiction over the res.
Nonetheless, summons must be served upon the defendant not for the purpose of vesting the court with jurisdiction
but merely for satisfying the due process requirements.

In this case, the judicial foreclosure proceeding instituted by respondent PCRB undoubtedly vested the trial court
with jurisdiction over the res. A judicial foreclosure proceeding is an action quasi in rem. As such, jurisdiction over
the person of petitioner is not required, it being sufficient that the trial court is vested with jurisdiction over the
subject matter.

While the trial court acquired jurisdiction over the res, its jurisdiction is limited to a rendition of judgment on the
res. It cannot extend its jurisdiction beyond the res and issue a judgment enforcing petitioner’s personal liability. In
granting respondent PCRB’s ex parte motion for deficiency judgment and ordering the issuance of a writ of
execution against the spouses Biaco to satisfy the remaining balance of the award, without first having acquired
Remedial Law I| Page 63

jurisdiction over the person of petitioner, as it did, the trial court violated her constitutional right to due process,
warranting the annulment of the judgment rendered in the case.

BPI Family Savings Bank, Inc. vs. Spouses Yujuico


G.R. No. 175796. July 22, 2015.

Topic: Venue; real vs. personal actions

Facts:

BPI extrajudicially foreclosed the mortgage constituted on the two parcels of land subject of the Spouses Yujuico’s
loan. Because there was a deficiency, BPI sued the spouses to recover such deficiency in the Makati RTC (where the
principal office of BPI is located). The spouses filed a Motion to Dismiss the complaint on several grounds, namely:
that the suit was barred by res judicata; that the complaint stated no cause of action; and that the plaintiff’s claim
had been waived, abandoned, or extinguished. Makati RTC denied the Motion. The spouses filed a Motion for
Reconsideration while BPI filed its comment/opposition to the Motion. The respondents then filed their reply, in
which they raised for the first time their objection on the ground of improper venue. They contended that the action
for the recovery of the deficiency, being a supplementary action of the extrajudicial foreclosure proceedings, was a
real action that should have been brought in the Manila RTC because Manila was the place where the properties
were located. Makati RTC denied the MR. The CA reversed the ruling of the RTC opining that a suit for recovery of
the deficiency after the foreclosure of a mortgage is in the nature of a mortgage action because its purpose is
precisely to enforce the mortgage contract. As such, the venue of an action for recovery of deficiency must necessarily
be the same venue as that of the extrajudicial foreclosure of mortgage. Thus, the suit for judgment on the deficiency
filed by BPI against the spouses, being an action emanating from the foreclosure of the real estate mortgage contract
between them, must necessarily be filed also at the RTC of Manila, not at the RTC of Makati. Hence, the present
petition.

Issue:

Whether or not the venue was properly laid?

Held:

Yes. It is basic that the venue of an action depends on whether it is a real or a personal action.

Section 1, Rule 4 of the Rules of Court, a real action is one that affects title to or possession of real property, or an
interest therein. Real action is to be commenced and tried in the proper court having jurisdiction over the area
wherein the real property involved, or a portion thereof, is situated, which explains why the action is also referred to
as a local action. In contrast, the venue of a personal action is the place where the plaintiff or any of the principal
plaintiffs resides, or where the defendant or any of the principal defendants resides, or in the case of a nonresident
defendant where he may be found, at the election of the plaintiff, for which reason the action is considered a
transitory one.

Based on the distinctions between real and personal actions, an action to recover the deficiency after the
extrajudicial foreclosure of the real property mortgage is a personal action, for it does not affect title to
or possession of real property, or any interest therein. Accordingly, the proper venue in this case is in Makati
RTC because Makati was the place where the main office of BPI was located.

Moreover, even assuming that the venue was improperly laid, it would be improper to dismiss the case considering
that the spouses had not raised such ground in their Motion to Dismiss. As earlier indicated, they came to raise the
objection of improper venue for the first time only in their reply to the petitioner’s comment on their Motion for
Reconsideration. They did so belatedly. In civil proceedings, venue is procedural, not jurisdictional, and may be
Remedial Law I| Page 64

waived by the defendant if not seasonably raised either in a motion to dismiss or in the answer. In other words,
unless the defendant seasonably objects, any action may be tried by a court despite its being the improper venue.

Rule 6: Counterclaims

Alba vs. Malapajo


G.R. No. 198752. January 13, 2016

Facts:

Petitioner, Arturo Alba, filed a complaint against respondents, Raymund D. Malapajo, Ramil D. Malapajo and the
Register of Deeds of Roxas City for recovery of ownership and/or declaration of nullity or cancellation of
title and damages alleging that the title of a parcel of land was transferred to the respondents by virtue of a Deed
of Sale that was forged in which the respondents co-authored.

In its Answer with counterclaim, the respondents contended that they were innocent purchasers for value and that
the deed was a unilateral document which was presented to them already prepared and notarized and that before
the sale, petitioner had, on separate occasions, obtained loans from them and their mother which were secured by
separate real estate mortgages covering the subject property and that the two real estate mortgages had never been
discharged.

Petitioner in its Reply to the counterclaim, averred among others that, the court had not acquired jurisdiction over
the nature of respondents' permissive counterclaim, thus, there must be payment of docket fees and filing of a
certification against forum shopping, and, that the supposed loan extended by respondents’ mother to petitioner,
must also be dismissed as respondents were not the real parties-in-interest.

The RTC ruled that the counterclaims were compulsory prompting the petitioner to file a petition for certiorari
before the CA. The CA dismissed the petition for certiorari on technical grounds.

Issue:

Whether or not the respondent’s counterclaim was permissive in nature?

Held:

No. A counterclaim is any claim which a defending party may have against an opposing party. A compulsory
counterclaim is one which, being cognizable by the regular courts of justice, arises out of or is connected with the
transaction or occurrence constituting the subject matter of the opposing party's claim and does not require for its
adjudication the presence of third parties of whom the court cannot acquire jurisdiction. Such a counterclaim must
be within the jurisdiction of the court both as to the amount and the nature thereof, except that in an original action
before the Regional Trial Court, necessarily connected with the subject matter of the opposing party's claim or even
where there is such a connection, the Court has no jurisdiction to entertain the claim or it requires for adjudication
the presence of third persons over whom the court acquire jurisdiction. A counterclaim is permissive if it does not
arise out of or is not necessarily connected with the subject matter of the opposing party's claim.

To determine whether a counterclaim is compulsory or permissive, the Court devised the following tests:
(a) Are the issues of fact and law raised by the claim and by the counterclaim largely the same?
(b) Would res judicata bar a subsequent suit on defendants’ claims, absent the compulsory counterclaim rule?;
(c) Will substantially the same evidence support or refute plaintiffs’ claim as well as the defendants’ counterclaim?;
and
(d) Is there any logical relation between the claim and the counterclaim?
Remedial Law I| Page 65

Here, petitioner seeks to recover the subject property by assailing the validity of the deed of sale on the subject
property which he allegedly executed in favor of respondents Malapajo on the ground of forgery. Respondent’s
counterclaimed that, in case the deed of sale is declared null and void, they be paid the loan petitioner obtained from
them plus the agreed monthly interest which was covered by a real estate mortgage on the subject property executed
by petitioner in favor of respondents. There is a logical relationship between the claim and the
counterclaim, as the counterclaim is connected with the transaction or occurrence constituting the
subject matter of the opposing party's claim. Notably, the same evidence to sustain respondents' counterclaim
would disprove petitioner's case. In the event that respondents could convincingly establish that petitioner actually
executed the promissory note and the real estate mortgage over the subject property in their favor then petitioner's
complaint might fail. Petitioner's claim is so related logically to respondents' counterclaim, such that conducting
separate trials for the claim and the counterclaim would result in the substantial duplication of the time and effort
of the court and the parties. Since respondents' counterclaim is compulsory, it must be set up in the same action;
otherwise, it would be barred forever. If it is filed concurrently with the main action but in a different proceeding, it
would be abated on the ground of litis pendentia; if filed subsequently, it would meet the same fate on the ground of
res judicata. There is, therefore, no need for respondents to pay docket fees and to file a certification against forum
shopping for the court to acquire jurisdiction over the said counterclaim.
Remedial Law I| Page 66

Lim Teck Chuan vs. Uy


G.R. No. 155701, March 11, 2015

Facts:

Antonio Lim Tanhu was the original owner of the lot which is the subject matter of the controversy. Allegedly, he
sold the lot to Spouses Cabansag; then Spouses Cabansag sold the same to Serafin Uy (respondent). Serafin then
filed a petition before the RTC praying for the issuance of a new owner’s duplicate TCT in his name. The petition
was initially granted but subsequently nullified because Lim Teck Chuan (petitioner) filed his Opposition alleging
that he is one of the 6 legitimate descendants of Antonio and that the original TCT was not lost and has always been
in his custody. In the meantime, a certain Henry Lim sold the same lot to Leopolda Cecilio by virtue of an Affidavit
of Sole Adjudication/Settlement of the Estate of Antonio Lim Tanhu with Deed of Sale.

Serafin then filed a Complaint for Quieting of Title impleading Leopolda, Henry and the petitioner. Leopolda
averred that she is a buyer in good faith and for value. Petitioner set up a counterclaim against Serafin and a
cross-claim against Lopeolda contending that the property was never transferred and encumbered to any person
during Antonio’s lifetime.

During the proceedings, both and Serafin and Leopolda entered into an amicable settlement and they both filed a
Joint Motion to Dismiss (MTD) on the main ground that the case had become moot and academic since Serafin’s title
to the subject lot had been allegedly quieted. Petitioner opposed the MTD on the ground that he was not included in
the settlement. The RTC, however, granted the MTD and it also dismissed petitioner’s counterclaim and cross-claim.

Petitioner filed directly with the SC a petition for review under Rule 45.

Issue:

Whether or not petitioner’s counterclaim or cross-claim could be prosecuted in the same action despite the dismissal
of the main complaint?

Held:

Yes. Section 2 of Rule 17 provides that:

xxx If a counterclaim has been pleaded by a defendant prior to the service upon him of the plaintiff’s motion for
dismissal, the dismissal shall be limited to the complaint. The dismissal shall be without prejudice to the right of the
defendant to prosecute his counterclaim in a separate action unless within fifteen (15) days from notice of the motion
he manifests his preference to have his counterclaim resolved in the same action….

The RTC erred when it dismissed the case when the present rules state that the dismissal shall be limited only to
the complaint. A dismissal of an action is different from a mere dismissal of the complaint. For this reason, since
only the complaint and not the action is dismissed, the defendant in spite of said dismissal may still prosecute his
counterclaim in the same action. Citing Pinga v. Heirs of German Santiago, the Court said that the dismissal of the
complaint does not necessarily result to the dismissal of the counterclaim.

In the instant case, the petitioner’s preference to have his counterclaim (and cross-claims) be prosecuted in the same
action was timely manifested.
Remedial Law I| Page 67

Metrobank vs. CPR Promotions


G.R. No. 200567, June 22, 2015

Facts:

Because the respondents’ defaulted in their loan obligation to petitioner (MBTC), MBTC filed a petition for extra-
judicial foreclosure of the real estate mortgages securing the loan obligations. According to MBTC, despite the
foreclosure sale, there remained a deficiency balance of PhP2,628,520.73, plus interest and charges as stipulated
and agreed upon in the promissory notes and deeds of real estate mortgages. Despite petitioner’s repeated demands,
however, respondents failed to settle the alleged deficiency. Thus, petitioner filed an action for collection of sum of
money against respondents before the RTC.

The RTC ruled in favor of petitioner that there, indeed, was a balance, and that respondents were liable for the said
amount, as part of their contractual obligation. It also denied the Motion for Reconsideration, prompting for
petitioner to file an appeal. The CA reversed the court a quo and ruled in favor of respondents and ordered that
Metrobank to refund or return to the defendants-appellants the amount representing the remainder of the proceeds
of the foreclosure sale (because there was overpayment). Petitioner filed a Motion for Reconsideration asserting
among others that respondents never set up a counterclaim for refund of any amount, but the same was denied,
hence, this case.

Issue:

Whether or not the CA erred in ordering a refund to the respondent despite their failure to set it up as a
counterclaim?

Held:

Yes. Rule 6 of the Rules of Court define a compulsory counterclaim as follows:

Section 7. Compulsory counterclaim. – A compulsory counterclaim is one which, being cognizable by the regular
courts of justice, arises out of or is connected with the transaction or occurrence constituting the subject matter of the
opposing party’s claim and does not require for its adjudication the presence of third parties of whom the court cannot
acquire jurisdiction. Such a counterclaim must be within the jurisdiction of the court both as to the amount and the
nature thereof. Except that in an original action before the Regional Trial Court, the counterclaim may be considered
compulsory regardless of the amount. A compulsory counterclaim is barred if not set up in the same action.

The court found out that such refund could have also been a compulsory counterclaim. However, it is elementary
that a defending party’s compulsory counterclaim should be interposed at the time he files his Answer, and that
failure to do so shall effectively bar such claim. As it appears from the records, what respondents initially claimed
herein were moral and exemplary damages, as well as attorney’s fees. Then, realizing, based on its computation,
that it should have sought the recovery of the excess bid price, respondents set up another counterclaim, this time in
their Appellant’s Brief filed before the CA. Unfortunately, respondents’ belated assertion proved fatal to their cause
as it did not cure their failure to timely raise such claim in their Answer. Consequently, respondents’ claim for the
excess, if any, is already barred.
Remedial Law I| Page 68

Rule 6: Answers

Valdez vs. Dabon


A.C. No. 7353. November 16, 2015

Facts:

Complainant Nelson charged respondent Atty. Dabon, a Division Clerk of Court of the Court of Appeals (CA), with
gross immorality for allegedly carrying on an adulterous relationship with his wife, Sonia Romero Valdez, which was
made possible by sexual assaults and maintained through threat and intimidation. Respondent Atty. Dabon strongly
refuted the accusation against him claiming that the same was baseless and unfounded and that the complaint for
disbarment was merely calculated to harass, annoy and besmirch his reputation.

In his Comment, Atty. Dabon denied the charges of grossly immoral and unlawful acts through sexual assaults,
abuses, threats and intimidation. He posited that the allegations of spouses Nelson and Sonia in their respective
affidavits were nothing but pure fabrication solely intended to malign his name and honor.

The Court referred the case to the Integrated Bar of the Philippines (IBP) for investigation, report and
recommendation. After the parties had submitted their respective verified position papers, Investigating
Commissioner of the IBP Commission on Bar Discipline (IBP-CBD) rendered his Report and Recommendation,
finding that the charge against respondent Atty. Dabon had been sufficiently proven. The Board of Governors of the
IBP adopted and approved the recommendation. Atty. Dabon filed a motion for reconsideratio but it was denied by
the IBP Board of Governors.

Issue:

Whether or not Atty. Dabon’s denial of the complaint against him constitutes negative pregnant?

Held:

Yes. Atty. Dabon interposed a blanket denial of the romantic involvement but at the same time, he seemed to have
tacitly admitted the illicit affair only that it was not attended by sexual assaults, threats and intimidations. The
Court also observed that he devoted considerable effort to demonstrate that the affair did not amount to gross
immoral conduct and that no sexual abuse, threat or intimidation was exerted upon the person of Sonia, but not
once did he squarely deny the affair itself.

In other words, the respondent's denial is a negative pregnant, a denial coupled with the admission of
substantial facts in the pleading responded to which are not squarely denied. Stated otherwise, a
negative pregnant is a form of negative expression which carries with it an affirmation or at least an
implication of some kind favorable to the adverse party. Where a fact is alleged with qualifying or modifying
language and the words of the allegation as so qualified or modified are literally denied, it has been held that the
qualifying circumstance alone is denied while the fact itself is admitted. It is clear from Atty. Dabon's Comment that
his denial only pertained as to the existence of a forced illicit relationship. Without a categorical denial thereof, he is
deemed to have admitted his consensual affair with Sonia.
Remedial Law I| Page 69

Republic vs. Sandiganbayan


G.R. No. 152154 July 15, 2003

Facts:

Petitioner Republic, through the Presidential Commission on Good Government (PCGG), represented by the Office
of the Solicitor General (OSG), filed a petition for forfeiture before the Sandiganbayan, entitled Republic of the
Philippines vs. Ferdinand E. Marcos, represented by his Estate/Heirs and Imelda R. Marcos.

In said case, petitioner sought the declaration of the aggregate amount of US$356 million deposited in escrow in the
PNB, as ill-gotten wealth. Petitioner filed a motion for summary judgment and/or judgment on the pleadings.
Respondent Mrs. Marcos filed her opposition thereto which was later adopted by respondents Mrs. Manotoc, Mrs.
Araneta and Ferdinand, Jr.

In their Answer, particularly, in paragraph 22, they stated that:

22. Respondents specifically DENY paragraph 23 insofar as it alleges that Respondents clandestinely
stashed the country’s wealth in Switzerland and hid the same under layers and layers of foundations
and corporate entities for being false, the truth being that Respondents’ aforesaid properties were
lawfully acquired.

Issue:
Whether or not paragraph 22 of the Answer constitutes negative pregnant?
Held:
Yes. Evidently, this particular denial had the earmark of what is called in the law on pleadings as a negative
pregnant, that is, a denial pregnant with the admission of the substantial facts in the pleading responded to which
are not squarely denied. It was in effect an admission of the averments it was directed at. Stated otherwise, a
negative pregnant is a form of negative expression which carries with it an affirmation or at least an implication of
some kind favorable to the adverse party. It is a denial pregnant with an admission of the substantial facts alleged
in the pleading. Where a fact is alleged with qualifying or modifying language and the words of the allegation as so
qualified or modified are literally denied, has been held that the qualifying circumstances alone are denied while the
fact itself is admitted.
In the instant case, the material allegations in paragraph 23 of the said petition were not specifically denied by
respondents in paragraph 22 of their answer. The denial contained in paragraph 22 of the answer was focused on
the averment in paragraph 23 of the petition for forfeiture that Respondents clandestinely stashed the country’s
wealth in Switzerland and hid the same under layers and layers of foundations and corporate entities.
Therefore, the allegations in the petition for forfeiture on the existence of the Swiss bank deposits in the sum of
about US$356 million, not having been specifically denied by respondents in their answer, were deemed admitted by
them pursuant to Section 11, Rule 8 of the 1997 Revised Rules on Civil Procedure.
Remedial Law I| Page 70

Caneland Sugar Corp. vs. Alon


G.R. No. 142896 September 12, 2007

Facts:

LBP foreclosed the mortgaged properties belonging to Caneland Sugar Corporation. There was another auction sale
scheduled when Caneland filed with the RTC a complaint for damages, injunction, and nullity of mortgage. RTC
issued an Order holding in abeyance the auction sale but it was rescheduled pursuant to PD 385. P.D. 385 provides
that it shall be mandatory for government financial institution to foreclose collaterals and/or securities for any loan,
credit accommodations and/or guarantees granted by them whenever the arrearages on such account, including
accrued interest and other charges amount to at least 20% of the total outstanding obligation as appearing in the
books of the financial institution. Moreover, no restraining order, temporary or permanent injunction shall be issued
by the court against any government financial institution in any action taken by such institution in compliance with
the mandatory foreclosure provided by said law.

Before the CA, petitioner’s petition was also denied. Hence, this petition. Petitioner contends in the main that the
RTC’s act of authorizing the foreclosure of its property amounts to a prejudgment of the case since it amounts to a
ruling that respondent has a valid mortgage in its favor. Petitioner also argues, among others, that Presidential
Decree (P.D.) No. 385 is not applicable inasmuch as at the time of the lease to Sunnix, Inc., the management and
control of its operations has already been virtually taken over by respondent. On the other hand, respondent
maintains that P.D. No. 385 prohibits the issuance of an injunctive order against government financial institutions;
the CA did not commit any grave abuse of discretion; the RTC Order merely dealt with the propriety of the
injunctive order and not the validity of the mortgage; and the issue of the propriety of the injunctive order has been
rendered moot and academic by the foreclosure sale conducted and the issuance of a certificate of sale by the sheriff.

Issue:
Whether or not the RTC was correct in finding the mortgage valid and not enjoining the foreclosure sale?
Held:
Yes. Petitioner does not dispute its loan obligation with respondent. Petitioner’s bone of contention before the RTC is
that the promissory notes are silent as to whether they were covered by the Mortgage Trust Indenture and Mortgage
Participation on its property covered by TCT No. T-11292. It does not categorically deny that these promissory
notes are covered by the security documents. These vague assertions are, in fact, negative pregnants, i.e.,
denials pregnant with the admission of the substantial facts in the pleading responded to which are not
squarely denied.

As defined in Republic of the Philippines v. Sandiganbayan, a negative pregnant is a "form of negative expression
which carries with it an affirmation or at least an implication of some kind favorable to the adverse party. It is a
denial pregnant with an admission of the substantial facts alleged in the pleading. Where a fact is alleged with
qualifying or modifying language and the words of the allegation as so qualified or modified are literally denied, has
been held that the qualifying circumstances alone are denied while the fact itself is admitted."

Petitioner’s allegations do not make out any justifiable basis for the granting of any injunctive relief. Even when the
mortgagors were disputing the amount being sought from them, upon the non-payment of the loan, which was
secured by the mortgage, the mortgaged property is properly subject to a foreclosure sale. This is in consonance with
the doctrine that to authorize a temporary injunction, the plaintiff must show, at least prima facie, a right to the
final relief.
Remedial Law I| Page 71

Rule 7: Parts of a Pleading

Alma Jose vs. Javellana


G.R. No. 158239, January 25, 2012

Facts:

Margarita Marquez Alma Jose sold to respondent Ramon Javellana by deed of conditional sale two parcels of land.
They agreed that Javellana would pay P80,000.00 upon the execution of the deed and the balance of P80,000.00
upon the registration of the parcels of land under the Torrens System (the registration being undertaken by
Margarita within a reasonable period of time); and that should Margarita become incapacitated, her son and
attorney-in-fact, Juvenal M. Alma Jose (Juvenal), and her daughter, petitioner Priscilla M. Alma Jose, would receive
the payment of the balance and proceed with the application for registration. When Margarita and Juvenal died,
Priscilla did not comply with the undertaking to cause the registration of the properties under the Torrens System,
and, instead, began to improve the properties by dumping filling materials therein with the intention of converting
the parcels of land into a residential or industrial subdivision. Javellana then filed an action for specific
performance, injunction, and damages against her in the Regional Trial Court. Petitioner filed a motion to dismiss,
stating that the complaint was already barred by prescription; and that the complaint did not state a cause of action.
The RTC denied said motion to dismiss, but subsequently in the MR, reversed its own decision, and the complaint
was dismissed. Javellana filed a MR which was denied by the RTC. Javellana filed a Notice of Appeal. It appears
that pending the appeal, Javellana also filed a petition for certiorari in the CA. The CA dismissed the certiorari
petition finding that the RTC did not commit grave abuse of discretion in issuing the orders, and holding that it only
committed, at most, an error of judgment correctible by appeal in issuing the challenged orders.

The CA in its decision on the appeal, reversed the RTC’s decision, it ruled that, the complaint sufficiently stated a
cause of action; that Priscilla, as sole heir, succeeded to the rights and obligations of Margarita with respect to the
parcels of land; that Margaritas undertaking under the contract was not a purely personal obligation but was
transmissible to Priscilla, who was consequently bound to comply with the obligation; that the action had not yet
prescribed due to its being actually one for quieting of title that was imprescriptible brought by Javellana who had
actual possession of the properties. MR was also denied. Hence this petition.

Issue:

Whether or not Javellana committed forum shopping for filing in the CA a petition for certiorari to assail the orders
of the RTC that were the subject matter of their appeal pending in the CA?

Held:

No. Forum shopping happens when, in the two or more pending cases, there is identity of parties, identity of rights
or causes of action, and identity of reliefs sought. Where the elements of litis pendentia are present, and where a
final judgment in one case will amount to res judicata in the other, there is forum shopping. For litis pendentia to be
a ground for the dismissal of an action, there must be: (a) identity of the parties or at least such as to represent the
same interest in both actions; (b) identity of rights asserted and relief prayed for, the relief being founded on the
same acts; and (c) the identity in the two cases should be such that the judgment which may be rendered in one
would, regardless of which party is successful, amount to res judicata in the other. For forum shopping to exist, both
actions must involve the same transaction, same essential facts and circumstances and must raise identical causes
of action, subject matter and issues. Clearly, it does not exist where different orders were questioned, two distinct
causes of action and issues were raised, and two objectives were sought.

The appeal and the petition for certiorari actually sought different objectives. In his appeal in C.A.-G.R.
CV No. 68259, Javellana aimed to undo the RTCs erroneous dismissal of Civil Case No. 79-M-97 to clear
the way for his judicial demand for specific performance to be tried and determined in due course by
the RTC; but his petition for certiorari had the ostensible objective to prevent (Priscilla) from
developing the subject property and from proceeding with the ejectment case until his appeal is finally
resolved, as the CA explicitly determined in its decision in C.A.-G.R. SP No. 60455. Nor were the dangers
that the adoption of the judicial policy against forum shopping designed to prevent or to eliminate attendant. The
Remedial Law I| Page 72

first danger, i.e., the multiplicity of suits upon one and the same cause of action, would not materialize considering
that the appeal was a continuity of Civil Case No. 79-M-97, whereas C.A.-G.R. SP No. 60455 dealt with an
independent ground of alleged grave abuse of discretion amounting to lack or excess of jurisdiction on the part of the
RTC. The second danger, i.e., the unethical malpractice of shopping for a friendly court or judge to ensure a
favorable ruling or judgment after not getting it in the appeal, would not arise because the CA had not yet decided
C.A.-G.R. CV No. 68259 as of the filing of the petition for certiorari. Instead, the court sees the situation of resorting
to two inconsistent remedial approaches to be the result of the tactical misjudgment by Javellana’s counsel on the
efficacy of the appeal to stave off his caretaker’s eviction from the parcels of land and to prevent the development of
them into a residential or commercial subdivision pending the appeal. In the petition for certiorari, Javellana
explicitly averred that his appeal was inadequate and not speedy to prevent private respondent Alma Jose and her
transferee/assignee xxx from developing and disposing of the subject property to other parties to the total
deprivation of petitioners rights of possession and ownership over the subject property, and that the dismissal by the
RTC had emboldened private respondents to fully develop the property and for respondent Alma Jose to file an
ejectment case against petitioners overseer xxx. Thereby, it became far-fetched that Javellana brought the petition
for certiorari in violation of the policy against forum shopping.
Remedial Law I| Page 73

Medado vs. Heirs of Consing


G.R. No. 186720, February 8, 2012

Facts:

Petitioner Spouses Medado and the estate of the late Antonio Consing (Estate of Consing), as represented by
Soledad Consing (Soledad), executed Deeds of Sale with Assumption of Mortgage for the former's acquisition from
the latter of the property in Cadiz City identified as Hacienda Sol. As part of the deal, Spouses Medado undertook to
assume the estate's loan with Philippine National Bank (PNB). Subsequent to the sale, however, the Estate of
Consing offered the subject lots to the government via the Department of Agrarian Reform's Voluntary Offer to Sell
(VOS) program.

The Estate of Consing instituted with the RTC, Branch 44 of Bacolod City an action for rescission and damages,
against Spouses Medado, PNB and the Register of Deeds of Cadiz City, due to the alleged failure of the spouses to
meet the conditions in their agreement. In the meantime that Civil Case No. 00-11320 for rescission was pending,
Land Bank of the Philippines (LBP) issued in favor of the Estate of Consing a certificate of deposit of cash and
agrarian reform bonds, as compensation for the lots covered by the VOS. Spouses Medado feared that LBP would
release the full proceeds thereof to the Estate of Consing. It prompted Spouses Medado to file an action for
injunction with prayer for the issuance of a temporary restraining order, with the RTC. They asked that
the following be issued by the trial court: (a) writ of prohibitory injunction to restrain LBP from releasing the
remaining amount of the VOS proceeds of the lots offered by the Estate of Consing, and restraining the Estate of
Consing from receiving these proceeds; and (b) writ of mandatory injunction to compel LBP to release the remaining
amount of the VOS to the spouses. The RTC granted said petition. Feeling aggrieved, the heirs of the late Antonio
Consing questioned the RTC's order via a petition for certiorari filed with the CA. They sought, among other reliefs,
the dismissal of the complaint for injunction for violation of the rules on litis pendetia. The CA reversed the decision
of the RTC. The MR was denied as well. Hence this petition. The petitioner contended that the consolidated
verification and certification against forum shopping of the petition filed with the CA was defective: first, for being
signed only by Soledad, instead of by all the petitioners, and second, its jurat cites a mere community tax certificate
of Soledad, instead of a government-issued identification card required under the 2004 Rules on Notarial Practice.

Issues:

(1) Whether or not Spouses Medado engaged in forum shopping when they filed an action for injunction despite the
pendency of the action for rescission?

(2) Whether or not the requirement for verification and certification against forum shopping have been
substantially complied?

Held:

(1) Yes. There is forum shopping when the elements of litis pendencia are present, i.e., between actions pending
before the courts, there exist: (1) identity of parties, or at least such parties as represent the same interests in both
actions; (2) identity of rights asserted and relief prayed for, the relief being founded on the same facts; (3) and the
identity of the two proceeding particulars is such that judgment rendered in the other action will regardless of which
party is successful, amount to res judicata in the action under consideration; said requisites are also constitutive of
the requisites for auter action pendant or lis pendens. Applying the foregoing, there was clearly a violation of the
rule against non-forum shopping when Spouses Medado instituted Civil Case No. 797-C for injunction
notwithstanding the pendency of Civil Case No. 00-11320 for rescission of contract and damages.

All elements of litis pendentia are present with the filing of the two cases. There is no dispute that there is identity
of parties representing the same interests in the two actions, both involving the estate and heirs of the late Consing
on one hand, and Spouses Medado on the other. The two other elements are likewise satisfied. There is an identity
of rights asserted and reliefs prayed for in the two cases, with the reliefs being founded on the same set of facts. In
both cases, the parties claim their supposed right as owners of the subject properties. They all anchor their claim of
ownership on the deeds of absolute sale which they had executed, and the law applicable thereto. They assert their
respective rights, with Spouses Medado as buyers and the heirs as sellers, based on the same set of facts that
Remedial Law I| Page 74

involve the deeds of sale’s contents and their validity. Both actions necessarily involve a ruling on the validity of the
same contract as against the same parties. Thus, the identity of the two cases is such as would render the decision
in the rescission case res judicata in the injunction case, and vice versa.

(2) Yes. Records show that Soledad signed the verification and certification against forum shopping on behalf of her
co-petitioners by virtue of a Special Power of Attorney (SPA) attached to the petition filed with the CA. The SPA,
signed by her co-heirs Ma. Josefa Consing Saguitguit, Ma. Carmela Consing Lopez, Ma. Lourdes Consing Gonzales
and Mary Rose Consing Tuason, provides that their attorney-in-fact Soledad is authorized. The authority of Soledad
includes the filing of an appeal before the CA, including the execution of a verification and certification against
forum shopping therefor, being acts necessary to protect, sue, prosecute, defend and adopt whatever action
necessary and proper in relation to their rights over the subject properties. In addition, the allegations and
contentions embodied in the CA petition do not deviate from the claims already made by the heirs in Civil Case Nos.
00-11320 and 797-C, both specifically mentioned in the SPA.

In any case, the Court reiterate that where the petitioners are immediate relatives, who share a common interest in
the property subject of the action, the fact that only one of the petitioners executed the verification or certification of
forum shopping will not deter the court from proceeding with the action.
Remedial Law I| Page 75

Commission on Appointment vs. Paler


G.R. No. 172623, March 3, 2010

Facts:

Respondent Celso M. Paler was a Supervising Legislative Staff Officer II (SG-24) with the Technical Support Service
of the Commission on Appointments. Since he already had an approved leave from June 9 to July 30, 2003, Paler left
for the United States without verifying whether his application for leave (for August 1 November 4, 2003) was
approved or denied. The Commission Chairman informed Paler that he was being dropped from the roll of
employees due to his continuous 30-day absence without leave and in accordance with a CSC Memorandum Order.
Paler moved for reconsideration but this was denied on the ground that it was filed beyond the 15-day reglementary
period. On appeal, the CSC reversed and set aside the Commission Chairman's decision. The Commission filed a
motion for reconsideration but this was denied by the CSC. This constrained petitioner to file with the CA a petition
for review under Rule 43. The CA affirmed the decision of the CSC. Hence, this petition. In his comment, Paler,
aside from arguing that the CA did not commit any error in sustaining the CSC resolutions, also assails Atty. Arturo
L. Tiu's authority to file the petition and sign the verification and certification of non-forum shopping on behalf of
the Commission Chairman.

Issue:

Whether or not Atty. Arturo Tiu has the authority to sign the verification and certification of non-forum shopping?

Held:

As to the verification – YES. As to the certification against forum shopping – NO.

The petitioner in this case is the Commission on Appointments, a government entity created by the Constitution,
and headed by its Chairman. There was no need for the Chairman himself to sign the verification. Its
representative, lawyer or any person who personally knew the truth of the facts alleged in the petition could sign
the verification.

With regard, however, to the certification of non-forum shopping, the established rule is that it must be
executed by the plaintiff or any of the principal parties and not by counsel. In this case, Atty. Tiu failed to
show that he was specifically authorized by the Chairman to sign the certification of non-forum shopping, much less
file the petition in his behalf. There is nothing on record to prove such authority. Atty. Tiu did not even bother to
controvert Palers allegation of his lack of authority. This renders the petition dismissible.
Remedial Law I| Page 76

Basan vs. Coca-Cola Bottlers, Phils.


G.R. Nos. 174365-66, February 4, 2015

Facts:

Petitioners Romeo Basan et al. filed a complaint for illegal dismissal and money claims against Coca-Cola Bottlers
Philippines alleging that the respondent dismissed them without just cause and prior written notice. Coca-Cola, on
the other hand, argues that it hired petitioners as temporary route helpers for a period in anticipation of the volume
of work in their plants or sale offices. The Labor Arbiter ruled in favor of petitioner finding that they are performing
activities necessary and desirable to the usual business of the petitioner for more than the period for regularization.
The NLRC affirmed the Labor Arbiter’s decision only to be reversed by the Court of Appeals on the ground that
Basan, et al. were not regular employees. Basan et al. filed a petition for review before the Supreme Court. Coca-
Cola contends that the petition should be denied since the verification and forum shopping was signed by only one of
the petitioners.

Issue:

Whether or not the petition should fail on the ground that its verification and non-forum shopping has been signed
by only one of the petitioners?

Held:

No. On the procedural issue, we hold that while the general rule is that the verification and certification of non-
forum shopping must be signed by all the petitioners in a case, the signature of only one of them, petitioner Basan in
this case, appearing thereon may be deemed substantial compliance with the procedural requirement.
Jurisprudence is replete with rulings that the rule on verification is deemed substantially complied with when one
who has ample knowledge to swear to the truth of the allegations in the complaint or petition signs the verification,
and when matters alleged in the petition have been made in good faith or are true and correct. Similarly, this Court
has consistently held that when under reasonable or justifiable circumstances, as when all the petitioners share
a common interest and invoke a common cause of action or defense, as in this case, the signature of only
one of them in the certification against forum shopping substantially complies with the certification
requirement. Indeed, the application of technical rules of procedure may be relaxed in labor cases to serve the
demand of justice.
Remedial Law I| Page 77

Uy vs. Court of Appeals


G.R. No. 173186, September 16, 2015.

Facts:

Aniceto Uy previously obtained a favorable judgment for the recovery of possession of the subject land against
Naval-Sai. In July 1999, Naval-Sai filed a complaint for Annulment of Deed with Damages before the same court
against petitioner. Naval-Sai argued that she never sold the lots and that her signature in the purported deed of
sale is spurious. Naval-Sai filed an Amended Complaint dated July 29, 1999. She asserted that the subject TCTs
were already cancelled by virtue of the deed of sale. Unlike the original complaint, however, the Amended
Complaint was not signed by Naval-Sai, but by her counsel.

In his answer with counterclaim, petitioner raised the noncompliance with the requisite certification of non-forum
shopping and prescription. He asserted that jurisdiction has never been acquired over the parties and the subject
matter because the certification against forum shopping in the Amended Complaint was defective, for having been
merely signed by Naval-Sai's counsel. He further claimed that the action for annulment of deed of sale is already
barred by the statute of limitations and that Naval-Sai is guilty of estoppel and laches.

Issue:

Whether or not there was compliance with the requirements on certification for non-forum shopping?

Held:

Yes. A certification against forum shopping is a peculiar and personal responsibility of the party, an assurance given
to the court or other tribunal that there are no other pending cases involving basically the same parties, issues and
causes of action. It must be executed by the party-pleader, not by his counsel. If, however, for reasonable or
justifiable reasons, the party-pleader is unable to sign, he must execute a Special Power of Attorney (SPA)
designating his counsel of record to sign on his behalf.

Here, the original complaint contained a proper verification and certification against forum shopping duly signed by
Naval-Sai as plaintiff. The verification and certification in the amended complaint, on the other hand, was only
signed by her counsel, Atty. Norberta L. Ela. Atty. Ela was not authorized to sign on behalf of Naval-Sai, as in fact,
she assigned one Rodolfo Florentino as agent. The Court of Appeals pointed out that in the certification in the
amended complaint, Atty. Ela specified that it should be taken and read together with the original complaint.

The Court of Appeals took this as a cautionary move on the part of Naval-Sai, justifying the relaxation of the rules
on the ground of substantial compliance. This cautionary move is ineffectual because under
the Rules of Civil Procedure, an amended complaint supersedes the original complaint. For all intents and purposes,
therefore, the original complaint and its verification and certification ceased to exist.

This, notwithstanding, there was still substantial compliance with the Rules. The general rule is that non-
compliance or a defect in the certification is not curable by its subsequent submission or correction. However, there
are cases where we exercised leniency and relaxed the rules on the ground of substantial compliance, the
presence of special circumstances or compelling reasons. The rules on forum-shopping are designed to promote and
facilitate the orderly administration of justice and "should not be interpreted with such absolute literalness as to
subvert its own ultimate and legitimate objective or the goal of all rules of procedure which is to achieve substantial
justice as expeditiously as possible.
Remedial Law I| Page 78

Bandillon vs. La Filipina Uygongco Corporation (LFUC)


G.R. No. 202446. September 16, 2015

Facts:

Petitioners Eduardo Bandillon, et al. (employees) are truck drivers and employees of respondent La Filipina
Uygongco Corporation (LFUC). They filed a complaint for violation of labor standard laws against the latter before
the DOLE Region VI. This case reached the Supreme Court and the employees won the case.

Consequently, the employees filed a Motion for Execution before the DOLE Region VI to enforce the DOLE
Secretary's Order of June 4, 2003. After being served with the writ, LFUC filed a Petition for certiorari and
injunction dated August 15, 2008 with the Court of Appeals, seeking to set aside the writ of execution.

Meanwhile, the DOLE Regional Director had already issued an Order directing LFUC to pay the total amount of
Php3,345,657.94 to the employees. When LFUC received the Order, it filedDOLE Region VI a Motion for
Reconsideration. This MR was, however, denied.

In 2011, the CA rendered its decision remanding the case to DOLE VI Regional Director for re-computation of
awards and reception of evidence of the parties on the ground that that the office of DRD arrived at its computation
without any evidence from the parties.

Before the Supreme Court, LFUC argued that the petition must fail alleging that several of the concerned
employees did not sign the Special Power of Attorney (SPA) authorizing their union president (Payda) to file this
petition, and to sign the verification and certification against forum shopping for such purpose.

Issue:

(1) Whether or not the petition must fail on the ground that several employees concerned did not sign the SPA
authorizing their union president to sign the verification and certification against forum shopping?

(2) Whether or not LFUC committed forum shopping by filing a petition for certiorari with the CA and subsequently
filing a Motion for Reconsideration of the DOLE-Regional Director’s writ of execution?

Held:

(1) No. According to prevailing jurisprudence, neither the fact that the union president alone signed the petition's
verification and certification against forum shopping, nor the fact that the SPA authorizing the filing of the petition
was not signed by all petitioners, invalidate nor render the petition defective, as the present case is one of those
instances when the rules are interpreted more liberally in order to attain substantial justice. Payda’s lone signature
and the SPA signed by most of the petitioners already substantially comply with the requirements for a properly
and validly filed petition.

Indeed, Payda alone signed the verification and certification against forum shopping - as the person authorized in
the SPA to do so - but instead of rendering the petition defective or invalid, this Court, as it has previously ruled
regards the same as already in substantial compliance with the rule.

Rules on non-compliance with the requirements on, or submission of defective, verification and
certification against forum shopping:

1) A distinction must be made between non-compliance with the requirement on or submission of defective
verification, and non-compliance with the requirement on or submission of defective certification against forum
shopping.
2) As to verification, non-compliance therewith or a defect therein does not necessarily render the pleading fatally
defective. The court may order its submission or correction or act on the pleading if the attending circumstances are
such that strict compliance with the Rule may be dispensed with in order that the ends of justice may be served
thereby.
3) Verification is deemed substantially complied with when one who has ample knowledge to swear to the truth of
Remedial Law I| Page 79

the allegations in the complaint or petition signs the verification, and when matters alleged in the petition have
been made in good faith or are true and correct.

4) As to certification against forum shopping, non-compliance therewith or a defect therein, unlike in verification, is
generally not curable by its subsequent submission or correction thereof, unless there is a need to relax the Rule on
the ground of "substantial compliance" or presence of "special circumstances or compelling reasons."
5) The certification against forum shopping must be signed by all the plaintiffs or petitioners in a case; otherwise,
those who did not sign will be dropped as parties to the case. Under reasonable or justifiable circumstances,
however, as when all the plaintiffs or petitioners share a common interest and invoke a common cause of action or
defense, the signature of only one of them in the certification against forum shopping substantially complies with
the Rule.
6) Finally, the certification against forum shopping must be executed by the party-pleader, not by his counsel. If,
however, for reasonable or justifiable reasons, the party- pleader is unable to sign, he must execute a Special Power
of Attorney designating his counsel of record to sign on his behalf.

(2) Yes. LFUC’s filing of a motion for reconsideration before the DOLE-VI Regional Director rendered moot and
academic its petition for certiorari then pending with the Court of Appeals; as such, LFUC’s failure to withdraw the
petition or to even notify the appellate court of the motion for reconsideration filed before the DOLE amounts to a
violation of the rules against forum shopping.

There is no question that as a result of LFUC’s pursuit of the two simultaneous remedies, the rulings of the Court of
Appeals on the petition for certiorari and the DOLE Secretary on LFUC’s motion for reconsideration are now
essentially conflicting, as the former bars any execution and instead directs a further hearing of certain evidence,
while the latter states that such evidence had the chance to be heard and execution should now proceed as a matter
of course. Such conflict is exactly the scenario that the rules against forum shopping try to avert.
Remedial Law I| Page 80

People vs. Arojado


G.R. No. 207041. November 9, 2015

Facts:

In an Information dated March 23, 2009, Jesus Arojado was charged with the crime of murder by the Office of the
City Prosecutor of Roxas City, Capiz. The case was docketed as Criminal Case No. C7509 and was raffled off to
Branch 16 of the Regional Trial Court of Roxas City, Iloilo (RTC of Roxas City). Arojado filed a Motion to Dismiss
the Information filed against him on the ground that the investigating prosecutor who filed the said Information
failed to indicate therein the number and date of issue of her Mandatory Continuing Legal Education
(MCLE) Certificate of Compliance, as required by Bar Matter No.1922 (B.M. No. 1922).

On the other hand, the petitioner contended that: (1) the Information sought to be dismissed is sufficient in form
and substance; (2) the lack of proof of MCLE compliance by the prosecutor who prepared and signed the Information
should not prejudice the interest of the State in filing charges against persons who have violated the law; and (3) an
administrative edict cannot prevail over substantive or procedural law, by imposing additional requirements for the
sufficiency of a criminal information.

The RTC of Roxas City, RTC of Roxas City issued an Order dismissing the subject Information without prejudice.
Respondent filed a Motion for Reconsideration, but the trial court denied it. In its presently assailed Decision, the
CA denied respondent’s petition and affirmed the questioned RTC Orders. Respondent’s Motion for Reconsideration
was likewise denied by the CA. Petitioner contends that: (1) the term “pleadings” as used in B.M. No. 1922 does not
include criminal Informations filed in court; (2) the failure of the investigating prosecutor to indicate in the
Information the number and date of issue of her MCLE Certificate of Compliance is a mere formal defect and is not
a valid ground to dismiss the subject Information which is otherwise complete in form and substance.

Issue:

(1) Whether or not a criminal information is considered as a pleading?

(2) Whether or not the failure of the prosecutor to incidate his MCLE Certificate of Compliance in the Information
renders it defective?

Held:

Yes. An information is, for all intents and purposes, considered an initiatory pleading because it is a written
statement that contains the cause of action of a party, which in criminal cases is the State as represented by the
prosecutor, against the accused. Like a pleading, the Information is also filed in court for appropriate judgment.
Undoubtedly then, an Information falls squarely within the ambit of Bar Matter No. 1922.

An information is a pleading since the allegations therein, which charge a person with an offense, is basically the
same as a complaint in a civil action which alleges a plaintiff’s cause or cause of action. Section 1, Rule 6 of the Rules
of Court, as amended, defines pleadings as the written statements of the respective claims and defenses of the
parties submitted to the court for appropriate judgment.

(2) Yes. However, the Court said that the rule has been revised to the effect that instead of dismissing the
complaint, the lawyer would now be subject to disciplinary action. Failure of a lawyer to indicate in his or her
pleadings the number and date of issue of his or her MCLE Certificate of Compliance will no longer result in the
dismissal of the case and expunction of the pleadings from the records. The trial court should not have dismissed the
information but it should have simply required the investigating prosecutor to indicate therein the number and date
of issue of her MCLE Certificate of Compliance.
Remedial Law I| Page 81

Rule 8: Manner of Making Allegations in Pleadings

Fernando Medical Enterprises, Inc. vs. Wesleyan University


G.R. No. 207970. January 20, 2016
Facts:

Fernando Medical Enterprises, a domestic corporation dealing with medical equipment and supplies, delivered to
and installed medical equipment and supplies at the Wesleyan University's hospital. According to the petitioner, the
Wesleyan University paid only P67,3 57,683.23 of its total obligation of P123,901,650.00, leaving unpaid the sum of
P54,654,195.54. However, on February 11, 2009, petitioner and respondent entered into an agreement whereby the
former agreed to reduce its claim to only P50,400,000.00, and allowed the latter to pay the adjusted obligation on
installment.

Wesleyan University expressly admitted the following (a) the four transactions for the delivery and installation of
various hospital equipment; (b) the total liability of the Wesleyan University; (c) the payments made by the
Wesleyan University; (d) the balance still due to the petitioner; and (e) the execution of the February 11, 2009
agreement. The Wesleyan University denied the rest of the complaint "for lack of knowledge or information
sufficient to form a belief as to the truth or falsity thereof”, inasmuch as the alleged transactions were undertaken
during the term of office of the past officers of defendant Wesleyan University-Philippines.

Fernando Medical then filed its Motion for Judgment Based on the Pleadings, stating that the Wesleyan University
had admitted the material allegations of its complaint and thus did not tender any issue as to such allegations. The
Wesleyan University opposed the Motion for Judgment Based on the Pleadings, arguing that it had specifically
denied the material allegations in the complaint. The RTC issued the order denying the Motion for Judgment Based
on the Pleadings of the petitioner. The CA affirmed the lower court’s decision.

Issue:

Whether or not the respondent sufficiently denied the material allegations in the petitioner’s complaint?

Held:

No. The answer admits the material allegations of ultimate facts of the adverse party’s pleadings not only when it
expressly confesses the truth of such allegations but also when it omits to deal with them at all. The controversion of
the ultimate facts must only be by specific denial.

Section 10, Rule 8 of the Rules of Court recognizes only three modes by which the denial in the answer raises an
issue of fact. The first is by the defending party specifying each material allegation of fact the truth of which he does
not admit and, whenever practicable, setting forth the substance of the matters upon which he relies to support his
denial. The second applies to the defending party who desires to deny only a part of an averment, and the denial is
done by the defending party specifying so much of the material allegation of ultimate facts as is true and material
and denying only the remainder. The third is done by the defending party who is without knowledge or information
sufficient to form a belief as to the truth of a material averment made in the complaint by stating so in the answer.
Any material averment in the complaint not so specifically denied are deemed admitted except an averment of the
amount of unliquidated damages.

In Civil Case No. 09122116, the respondent expressly admitted paragraph nos. 2, 3, 4, 5, 9 and 10 of the complaint.
The admission related to the petitioner’s allegations on: (a) the four transactions for the delivery and installation of
various hospital equipment; (b) the total liability of the respondent; (c) the payments made by the respondents; (d)
the balance still due to the petitioner; and (e) the execution of the February 11, 2009 agreement. The admission of
the various agreements, especially the February 11, 2009 agreement, significantly admitted the petitioner’s
complaint

The respondent denied paragraph nos. 6, 7 and 8 of the complaint “for lack of knowledge or information sufficient to
form a belief as to the truth or falsity thereof, inasmuch as the alleged transactions were undertaken during the
term of office of the past officers of defendant Wesleyan University Philippines.” But the manner of denial effective
Remedial Law I| Page 82

was not a specific denial.
Remedial Law I| Page 83

Go Tong Electrical Supply vs. BPI Family Savings Bank


G.R. No. 187487. June 29, 2015

Facts:

Go Tong Electrical applied and was granted a loan by DBS Bank of the Philippines (formerly BSA) which is covered
by a Promissory Note. Go also executed a Comprehensive Surety Agreement (CSA) covering any and all obligations
undertaken by Go Tong Electrical. Because of its failure to pay the loan, respondent file a complaint for sum of
money against it. In their Answer with Counterclaim (Answer), petitioners merely stated that they "specifically
deny" the allegations under the complaint. The RTC ruled in favor of respondent. The CA sustained the RTC's
ruling.

Issue:

Whether or not petitioner was able to sufficiently deny the allegations in the respondent’s complaint?

Held:

No. The genuineness and due execution of the loan documents in this case were deemed admitted by petitioners
under the parameters of Section 8, Rule 8 of the Rules which provides:

SEC. 8. How to contest such documents. - When an action or defense is founded upon a written instrument, copied in
or attached to the corresponding pleading as provided in the preceding Section, the genuineness and due execution of
the instrument shall be deemed admitted unless the adverse party, under oath, specifically denies them, and sets forth
what he claims to be the facts; but the requirement of an oath does not apply when the adverse party does not appear
to be a party to the instrument or when compliance with an order for an inspection of the original instrument is
refused.

A general denial does not become specific by the use of the word “specifically.” A reading of the Answer
shows that petitioners failed to specifically deny the execution of the Credit Agreement, PN, and CSA under the
auspices of the above quoted rule. The mere statement in paragraph 4 of their Answer, i.e., that they “specifically
deny” the pertinent allegations of the Complaint “for being self-serving and pure conclusions intended to suit
plaintiff’s purposes,” does not constitute an effective specific denial as contemplated by law. Verily, a denial is not
specific simply because it is so qualified by the defendant. Stated otherwise, a general denial does not become
specific by the use of the word “specifically.” Neither does it become so by the simple expedient of coupling the same
with a broad conclusion of law that the allegations contested are “self-serving” or are intended “to suit plaintiff’s
purposes.
Remedial Law I| Page 84

Benguet Exploration, Inc. vs. Court of Appeals


G.R. No. 117434 February 9, 2001

Facts:

Benguet Exploration, Inc. (Benguet) filed a complaint for damages against Seawood Shipping, Inc. (Seawood
Shipping) and Switzerland General Insurance, Co., Ltd. (Switzerland Insurance) in the RTC. The cases were jointly
tried, during which Benguet presented its employees, Rogelio Lumibao and Ernesto Cayabyab, as witnesses.

Rogelio Lumibao, marketing assistant of Benguet, was in charge of exportation. He explained that Seawood
Shipping was chartered by Benguet to transport copper concentrates. The bill of lading stated that the cargo,
consisting of 2,243.496 wet metric tons of copper concentrates, was loaded on board Sangkulirang No. 3 at Poro
Point, San Fernando, La Union. It was insured by Switzerland Insurance. When the cargo was unloaded in Japan,
however, Rogelio Lumibao received a report from a surveyor in Japan stating that the cargo was 355 metric tons
short of the amount stated in the bill of lading. For this reason, petitioner Benguet made a claim of the loss to
Seawood Shipping and Switzerland Insurance.In a letter, Benguet made a formal demand for the value of the
alleged shortage. As both Seawood Shipping and Switzerland Insurance refused the demand, petitioner Benguet
brought these cases against Seawood Shipping and Switzerland Insurance. On cross-examination, Lumibao
admitted that he did not see the actual loading of the cargo at Poro Point and that his knowledge was limited to
what was contained in the bill of lading which he received about two days after the loading.

Ernesto Cayabyab also testified that he was the secretary of Nil Alejandre, manager of Benguet. According to
Cayabyab, he was sent to the warehouse (bodega) at Poro Point, La Union to assist in the loading of the copper
concentrates. Cayabyab said he was present when the cargo was loaded on the ship, as evidenced by the Certificate
of Loading, Certificate of Weight, and the Mates Receipt, all dated July 28, 1985. According to Cayabyab, the Marine
Surveyor and the Chief Mate would go around the boat to determine how much was loaded on the ship. Cayabyab
stated that he saw petitioner Benguet’s representative and his immediate superior, Mr. Alejandre, and the Inspector
of Customs, Mr. Cardenas, sign the Certificate of Weight. Cayabyab also witnessed the ship captain sign the
Certificate of Weight, which stated therein that 2,243.496 wet metric tons of copper concentrates were loaded on the
ship. Cayabyab likewise confirmed the authenticity of the Mates Receipt, saying that he witnessed the Chief Mate
sign the document.

The RTC, however, dismissed Benguet’s complaint. On appeal, the CA affirmed the decision of the RTC. Benguet
moved for reconsideration but was denied. Hence this petition. Benguet contends, among others, that the
genuineness and due execution of the documents presented, i.e., Bill of Lading, Certificate of Loading, Certificate of
Weight, Mates Receipt, were properly established by the testimony of its witness, Ernesto Cayabyab, and that as a
result, there is a prima facie presumption that their contents are true.

Issue:

Whether the genuineness and due execution of the documents presented were properly established by the testimony
of the plaintiff’s witness, resulting to prima facie presumption that their contents are true?

Held:

No. The genuineness and due execution of the documents were not properly established by Benguet’s witnesses. The
court ruled that the admission of the due execution and genuineness of a document simply means that the party
whose signature it bears admits that he signed it or that it was signed by another for him with his authority; that at
the time it was signed it was in words and figures exactly as set out in the pleading of the party relying upon it; that
the document was delivered; and that any formal requisites required by law, such as a seal, an acknowledgment, or
revenue stamp, which it lacks, are waived by him. In another case, the Court held that when the law makes use of
the phrase genuineness and due execution of the instrument it means nothing more than that the instrument is not
spurious, counterfeit, or of different import on its face from the one executed.

It is equally true, however, that execution can only refer to the actual making and delivery, but it cannot involve
other matters without enlarging its meaning beyond reason. The only object of the rule was to enable a plaintiff to
Remedial Law I| Page 85

make out a prima facie, not a conclusive case, and it cannot preclude a defendant from introducing any defense on
the merits which does not contradict the execution of the instrument introduced in evidence.

In this case, respondents presented evidence which casts doubt on the veracity of these documents. Respondent
Switzerland Insurance presented Export Declaration No. 1131/85 which petitioners own witness, Rogelio Lumibao,
prepared, in which it was stated that the copper concentrates to be transported to Japan had a gross weight of only
2,050 wet metric tons or 1,845 dry metric tons, 10 percent more or less. On the other hand, Certified Adjusters, Inc.,
to which Switzerland Insurance had referred petitioners claim, prepared a report which showed that a total of
2,451.630 wet metric tons of copper concentrates were delivered at Poro Point.

Considering the discrepancies in the various documents showing the actual amount of copper concentrates
transported to Poro Point and loaded in the vessel, there is no evidence of the exact amount of copper concentrates
shipped. Thus, whatever presumption of regularity in the transactions might have risen from the genuineness and
due execution of the Bill of Lading, Certificate of Weight, Certificate of Loading, and Mates Receipt was successfully
rebutted by the evidence presented by respondent Switzerland Insurance which showed disparities in the actual
weight of the cargo transported to Poro Point and loaded on the vessel. This fact is compounded by the admissions
made by Lumibao and Cayabyab that they had no personal knowledge of the actual amount of copper concentrates
loaded on the vessel.
Remedial Law I| Page 86

Asian Construction Development Corp. vs. Court of Appeals


G.R. No. 160242 May 17, 2005

Facts:

Monark Equipment Corporation (MEC) filed a Complaint for a sum of money with damages against the Asian
Construction and Development Corporation (ACDC) with the Regional Trial Court (RTC) of QC for non-payment of
rentals of several equipments amounting to P5,071,335.86 despite demand. ACDC filed a motion to file and admit
answer with third-party complaint against Becthel Overseas Corporation (Becthel). In its answer, ACDC admitted
its indebtedness to MEC in the amount of P5,071,335.86 but alleged the various special and affirmative defenses
that the equipment leased were used in the construction project of Becthel and it was not paid of its services that
resulted to the non-payment of rentals of the leased equipments. In its third-party complaint, ACDC prayed to
dismiss the complaint against it and to order that Becthel pay the amount of P456,666.67 plus interest thereon and
attorneys fees. Later on, MEC filed a motion for summary judgment, alleging therein that there was no genuine
issue as to the obligation of ACDC to MEC, which motion was opposed by ACDC. The trial court issued a Resolution
denying the motion of ACDC for leave to file a third-party complaint and granting the motion of MEC, which the
trial court considered as a motion for a judgment on the pleadings. ACDC appealed the resolution to the Court of
Appeals (CA), but was denied. Its MR having denied, petitioner filed the present case.

Issue:

Whether or not ACDC should be allowed to file a third-party complaint against Becthel?

Held:

No. The third-party complaint is improper. The Court, ruled that the defendant may implead another as third-party
defendant (a) on an allegation of liability of the latter to the defendant for contribution, indemnity, subrogation or
any other relief; (b) on the ground of direct liability of the third-party defendant to the plaintiff; or (c) the liability of
the third-party defendant to both the plaintiff and the defendant. There must be a causal connection between the
claim of the plaintiff in his complaint and a claim for contribution, indemnity or other relief of the defendant against
the third-party defendant.

The third-party complaint does not have to show with certainty that there will be recovery against the third-party
defendant, and it is sufficient that pleadings show possibility of recovery. In determining the sufficiency of the
third-party complaint, the allegations in the original complaint and the third-party complaint must be examined. A
third-party complaint must allege facts which prima facie show that the defendant is entitled to contribution,
indemnity, subrogation or other relief from the third-party defendant.

In this case, the claims of the respondent, as plaintiff in the RTC, against the petitioner as defendant therein, arose
out of the contracts of lease and sale; such transactions are different and separate from those between Becthel and
the petitioner as third-party plaintiff for the construction of the latters project in Mauban, Quezon, where the
equipment leased from the respondent was used by the petitioner. The controversy between the respondent and the
petitioner, on one hand, and that between the petitioner and Becthel, on the other, are thus entirely distinct from
each other. There is no showing in the proposed third-party complaint that the respondent knew or approved the use
of the leased equipment by the petitioner for the said project in Quezon. Becthel cannot invoke any defense the
petitioner had or may have against the claims of the respondent in its complaint, because the petitioner admitted its
liabilities to the respondent for the amount of P5,075,335.86. The barefaced fact that the petitioner used the
equipment it leased from the respondent in connection with its project with Becthel does not provide a substantive
basis for the filing of a third-party complaint against the latter. There is no causal connection between the claim of
the respondent for the rental and the balance of the purchase price of the equipment and parts sold and leased to
the petitioner, and the failure of Becthel to pay the balance of its account to the petitioner after the completion of the
project in Quezon. Thus, the denial of the third-party complaint is proper.
Remedial Law I| Page 87

Rule 9: Effect of Failure to Plead

Salvador vs. Rabaja


G.R. No. 199990, February 4, 2015

Facts:

Spouses Rabaja were leasing an apartment in a subject lot registered under the names of the Spouses Salvador.
Later on, the subject property was sold by Spouses Salvador to Spouses Rabaja and personally introduced one
Gonzales as the administrator of the property. Later on, Spouses Rabaja made an initial payment to Gonzales in the
presence of Herminia Salvador. Gonzales also presented to Spouses Rabaja an SPA executed by Rolado Salvador. On
the same day, the parties executed the Contract to Sell which stipulated that for a consideration of ₱5,000,000.00,
Spouses Salvador sold, transferred and conveyed in favor of Spouses Rabaja the subject property. Spouses Rabaja
made several payments totalling ₱950,000.00, which were received by Gonzales pursuant to the SPA provided
earlier as evidenced by the check vouchers signed by Gonzales and the improvised receipts signed by Herminia.

Sometime in June 1999, however, Spouses Salvador complained to Spouses Rabaja that they did not receive any
payment from Gonzales. This prompted Spouses Rabaja to suspend further payment of the purchase price; and as a
consequence, they received a notice to vacate the subject property from Spouses Salvador for non-payment of
rentals. Thereafter, Spouses Salvador instituted an action for ejectment against Spouses Rabaja. In turn, Spouses
Rabaja filed an action for rescission of contract against Spouses Salvador and Gonzales, the subject matter of the
present petition.

In the rescission case, Spouses Rabaja demanded the rescission of the contract to sell praying that the amount of
₱950,000.00 they previously paid to Spouses Salvador be returned to them. Spouses Salvador filed their answer with
counterclaim and cross-claim contending that there was no meeting of the minds between the parties and that the
SPA in favor of Gonzales was falsified. Gonzales filed her answer stating that the SPA was not falsified and that the
payments of Spouses Rabaja amounting to ₱950,000.00 were all handed over to Spouses Salvador. Spouses Salvador
were declared in default for non-appearance in the pre-trial conference and the RTC allowed the respondents to
present evidence ex parte. The RTC ruled in favor of Spouses Rabaja, stating that the contract to sell was actually a
contract of sale and that Gonzales was undoubtedly the attorney-in-fact of Spouses Salvador absent any taint of
irregularity. Spouses Rabaja could not be faulted in dealing with Gonzales who was duly equipped with the SPA
from Spouses Salvador. On appeal, the CA affirmed the decision of the RTC but modifying the same that Gonzales
was not solidarily liable with Sps. Salvador. Their MR was denied, hence, this petition. Spouses Salvador contend
that the order of default must be lifted because reasonable grounds exist to justify their failure to attend the pre-
trial conference

Issue:

Whether or not Spouses Salvador can be validly declared in default for non-appearance in the pre-trial conference?

Held:

No. The failure to attend the pre-trial conference does not result in the default of an absent party. Under the 1997
Rules of Civil Procedure, a defendant is only declared in default if he fails to file his Answer within the reglementary
period. On the other hand, if a defendant fails to attend the pre-trial conference, the plaintiff can present his
evidence ex parte. Still, the non-appearance of the defendant bears the same effect as that of the declaration of
default for failure to file an answer within the prescribed period. The case of Philippine American Life & General
Insurance Company v. Joseph Enario discussed the difference between the non-appearance of a defendant in a pre-
trial conference and the declaration of a defendant in default in the present Rules of Civil Procedure. The decision
instructs:

“Prior to the 1997 Revised Rules of Civil Procedure, the phrase "as in default" was initially included in Rule
20 of the old rules, and which read as follows:
Remedial Law I| Page 88

Sec. 2. A party who fails to appear at a pre-trial conference may be non-suited or considered as in default.”

It was, however, amended in the 1997 Revised Rules of Civil Procedure. Justice Regalado, in his book, REMEDIAL
LAW COMPENDIUM, explained the rationale for the deletion of the phrase "as in default" in the amended
provision, to wit:

“1. This is a substantial reproduction of Section 2 of the former Rule 20 with the change that, instead of
defendant being declared "as in default" by reason of his non-appearance, this section now spells out that the
procedure will be to allow the ex parte presentation of plaintiff’s evidence and the rendition of judgment on
the basis thereof.”

Still, in the same book, Justice Regalado clarified that while the order of default no longer obtained, its effects were
retained, thus:

“Failure to file a responsive pleading within the reglementary period, and not failure to appear at the
hearing, is the sole ground for an order of default, except the failure to appear at a pre-trial conference
wherein the effects of a default on the part of the defendant are followed, that is, the plaintiff shall be allowed
to present evidence ex parte and a judgment based thereon may be rendered against defendant.”

From the foregoing, the failure of a party to appear at the pre-trial has indeed adverse consequences. If it is the
defendant who fails to appear, then the plaintiff is allowed to present his evidence ex parte and the court shall
render judgment based on the evidence presented. Thus, the plaintiff is given the privilege to present his evidence
without objection from the defendant, the likelihood being that the court will decide in favor of the plaintiff, the
defendant having forfeited the opportunity to rebut or present its own evidence.

There is no dispute that Spouses Salvador and their counsel failed to attend the pre-trial conference set on February
4, 2005 despite proper notice. Spouses Salvador aver that their non-attendance was due to the fault of their counsel
as he forgot to update his calendar. This excuse smacks of carelessness, and indifference to the pre-trial stage. It
simply cannot be considered as a justifiable excuse by the Court. As a result of their inattentiveness, Spouses
Salvador could no longer present any evidence in their favor. Spouses Rabaja, as plaintiffs, were properly allowed by
the RTC to present evidence ex parte against Spouses Salvador as defendants. Considering that Gonzales as co-
defendant was able to attend the pre-trial conference, she was allowed to present her evidence. The RTC could only
render judgment based on the evidence presented during the trial.
Remedial Law I| Page 89

BDO vs. Tansipek


G.R. No. 181235 July 22, 2009

Facts:

J. O. Construction, Inc. (JOCI) filed a complaint for sum of money against Philippine Commercial and Industrial
Bank (PCIB). PCIB, in turn, filed a third-party complaint against John Tansipek (respondent). Tansipek failed to
answer the third-party complaint and he was declared in default for failure to do so. He filed a Motion for
Reconsideration from the Order of Default but was denied by the RTC. Tansipek then filed a Petition for Certiorari
with the CA assailing the Default Order and the denial of the Motion for Reconsideration. The CA dismissed the
Petition for failure to attach the assailed Orders. Later, the CA denied respondent Tansipek’s Motion for
Reconsideration for having been filed out of time. He did not appeal said denial to the Supreme Court.

On the third party complaint, Tansipek is ordered to pay PCIB all amounts that the latter shall have to pay to JOCI
on account of this case. Tansipek appealed the Decision to the CA. The CA ruled that it was an error for the trial
court to have acted on PCIB’s motion to declare Tansipek in default. The CA, thus, remanded the case to the RTC
for further proceedings.

Issue:

Whether or not Tansipek’s remedy against the Order of Default is correct?

Held:

No. Tansipek’s remedy against the Order of Default was erroneous from the very beginning. He should have filed a
Motion to Lift Order of Default, and not a Motion for Reconsideration, pursuant to Section 3(b), Rule 9 of the
Rules of Court:

(b) Relief from order of default.—A party declared in default may at any time after notice thereof and before judgment
file a motion under oath to set aside the order of default upon proper showing that his failure to answer was due to
fraud, accident, mistake or excusable negligence and that he has a meritorious defense. In such case, the order of
default may be set aside on such terms and conditions as the judge may impose in the interest of justice.

A Motion to Lift Order of Default is different from an ordinary motion in that the Motion should be verified; and
must show fraud, accident, mistake or excusable neglect, and meritorious defenses. The allegations of (1) fraud,
accident, mistake or excusable neglect, and (2) of meritorious defenses must concur.

Assuming for the sake of argument, however, that Tansipek’s Motion for Reconsideration may be treated as a
Motion to Lift Order of Default, his Petition for Certiorari on the denial thereof has already been dismissed with
finality by the CA. Tansipek did not appeal said ruling of the CA to the Supreme Court. The dismissal of the Petition
for Certiorari assailing the denial of Tansipek’s Motion, thus, constitutes a bar to the retrial of the same issue of
default under the doctrine of the law of the case.

It is important to note that a party declared in default is not barred from appealing from the judgment on the main
case, whether or not he had previously filed a Motion to Set Aside Order of Default, and regardless of the result of
the latter and the appeals therefrom. However, the appeal should be based on the Decision’s being contrary to law or
the evidence already presented, and not on the alleged invalidity of the default order.
Remedial Law I| Page 90

Bitte vs. Jonas


GR No. 212256, December 9, 2015

Facts:

This controversy stemmed from the two civil cases filed by the parties Bitte and Spouses Jonas against each other
relative to a purported contract of sale involving a property in Davao City, registered in the name of Rosa Elsa
Serrano Jonas. Before Rosa Elsa went to Australia, she had executed a Special Power of Attorney (SPA) authorizing
her mother, Andrea, to sell the property. Cipriano, brother of Rosa Elsa, then offered the property for sale to
Spouses Bitte, showing them the authority of Andrea. Spouses Bitte later sought a meeting for final negotiation
with Rosa Elsa. However, shortly after her arrival in the Philippines, Rosa Elsa revoked the SPA and handed a copy
thereof to Andrea. The parties met but no final agreement was reached. Rosa Elsa eventually withdrew from the
transaction.

Spouses Bitte then filed before the RTC a Complaint for Specific Performance with Damages. While the case was
pending, Andrea sold the subject property to Spouses Bitte, through a deed of absolute sale. In response to the
complaint, Rosa Elsa countered that despite her appointment of her mother as her attorney-in-fact/agent, she later
gave her instructions not to sell the property.

During the pre-trial conference, Spouses Bitte failed to appear. So, the RTC dismissed their complaint and set the
reception of Rosa Elsa's counterclaim for hearing. Later on, Benjamin Bitte manifested the withdrawal of their
counsel. The RTC then cancelled the reception of Rosa Elsa's evidence without reconsidering the dismissal of the
complaint.

Spouses Jonas thereafter filed before the RTC a complaint for Annulment of Deed of Absolute Sale, Cancellation of
TCT and Recovery of Possession, Injunction and Damages against Spouses Bitte. The two cases were consolidated.
Spouses Bitte were again declared in default for their failure to attend the pre-trial. The counsel of Spouses Bitte
withdrew and a new one entered his appearance and then filed a verified motion for reconsideration. For the third
time, Spouses Bitte once again failed to appear in the pre-trial and were, thus, declared non-suited. Rosa Elsa then
presented her evidence ex parte.

When the case went up to the Supreme Court, Spouses Jonas claim that the door to any reliefs for Spouses Bitte, be
it through a motion for reconsideration or this subject petition, was closed by the finality and immutability of the
RTC declaration of their default. In other words, it is their stand that the petitioners do not have the right to obtain
recourse from this Court.

Issue:

Whether or not petitioners Bitte no longer have the personality to file the present petition on the basis of the RTC’s
declaration of their default?

Held:

No. The rule is that "right to appeal from the judgment by default is not lost and can be done on grounds that the
amount of the judgment is excessive or is different in kind from that prayed for, or that the plaintiff failed to prove
the material allegations of his complaint, or that the decision is contrary to law."

If a party who has been declared in default has in his arsenal the remedy of appeal from the judgment of default on
the basis of the decision having been issued against the evidence or the law, that person cannot be denied the
remedy and opportunity to assail the judgment in the appellate court. Despite being burdened by the circumstances
of default, the petitioners may still use all other remedies available to question not only the judgment of default but
also the judgment on appeal before this Court. Those remedies necessarily include an appeal by certiorari under
Rule 45 of the Rules of Court.
Remedial Law I| Page 91

Rule 10: Amendment of Pleadings

Yujuico vs. United Resources Asset Management Corp.


GR No. 211113, June 29, 2015

Facts:

To secure the loan of SRADEC from United Resources Asset Management, Inc (URAMI), several stockholders of
STRADEC executed Pledge Agreements whereby they pledged a certain amount of their stocks in the said company
in favor of URAMI. Yujuico is one of the said stockholders. Not having been able to comply with its loan obligations,
a Notice, signed by Atty. Nethercott, informing them about the impending auction sale were sent to STRADEC and
to its stockholders. Nethercott claims that he is URAMI’s attorney-in-fact. As response, Yujuico filed before the RTC
of Pasig an injunction complaint seeking to enjoin the sale at public auction mentioned in Atty. Nethercott's notice.
In his complaint, Yujuico asserts that the planned auction sale is void as Nethercott was not authorized to initiate
the sale in behalf of URAMI.

As the RTC did not issue the injunction, the auction sale pushed through with URAMI emerging as the winning
bidder. Thereafter, the RTC issued a writ of preliminary injunction, which effectively prevented URAMI from
appropriating the stocks it had purchased during the auction sale. Nethercott filed his answer denying the material
allegations of the injucntion complaint. However, URAMI, in its answer, agreed with Yujuico that the sale was
VOID and that it never authorized Nethercott to cause the sale but it pointed out that since URAMI never
sanctioned the sale, it should not be liable for the prejudice caused by the sale and hence, overall it asked for the
dismissal of the case.

Yujuico then filed a Motion for Summary Judgment in view of the admissions made by URAMI in its answer
regarding Atty. Nethercott's lack of authority to cause the auction sale of pledged stocks, there was no longer any
genuine issue left to be resolved in trial.

The resolution of such Motion was however, deferred due to the TRO issued by the SC which remained in effect for
more than a year. URAMI then changed its counsel and then filed an answer with compulsory counterclaim. In
its amended answer, URAMI still vouched for the dismissal of the injunction complaint but reneged
from its previous admissions under the original answer. It now claims that Nethercott was DULY
AUTHORIZED by the Board to cause the sale. The RTC allowed the said amended answer against the
protest of Yujuico. But on MR of Yujuico, the RTC set aside its previous resolution, explaining that it cannot
admit the amended answer because URAMI did not first secure a leave of court. In compliance, URAMI filed the
pertinent motion which was granted by the RTC.

In a petition for certiorari in the CA, the higher court sustained the allowance by the RTC of the amended answer.
Hence, this appeal.

Issue:

Whether or not the amended answer of URAMI may still be allowed?

Held:

Yes. Our rules of procedure allow a party in a civil action to amend his pleading as a matter of right, so long as the
pleading is amended only once and before a responsive pleading is served (or, if the pleading sought to be amended is
a reply, within ten days after it is served). Otherwise, a party can only amend his pleading upon prior leave of court.

As a matter of judicial policy, courts are impelled to treat motions for leave to file amended pleadings
with liberality. Hence, as long as it does not appear that the motion for leave was made with bad faith or with
intent to delay the proceedings, courts are justified to grant leave and allow the filing of an amended pleading. Once
a court grants leave to file an amended pleading, the same becomes binding and will not be disturbed on appeal
unless it appears that the court had abused its discretion.
Remedial Law I| Page 92

In this case, URAMI filed its motion for leave seeking the admission of its amended answer more than two (2) years
after it filed its original answer. Despite the considerable lapse of time between the filing of the original answer and
the motion for leave, the RTC still granted the said motion. Such grant was later affirmed on appeal by the Court of
Appeals.

Matters involving the amendment of pleadings are primarily governed by the pertinent provisions of Rule 10 and
not by Section 4 of Rule 129 of the Rule of Court. Hence, allegations (and admissions) in a pleading—even if not
shown to be made through "palpable mistake"—can still be corrected or amended provided that the amendment is
sanctioned under Rule 10 of the Rules of Court.

Nevertheless, even if we are to apply Section 4 of Rule 129 to the present case, we still find the allowance of
URAMI's amended answer to be in order. To our mind, a consideration of the evidence that URAMI plans to present
during trial indubitably reveals that the admissions made by URAMI under its original answer were a product of
clear and patent mistake.

One of the annexes attached by URAMI is a board resolution authorizing Nethercott to cause the sale. With the
existence of such board resolution, the statement in URAMI's original answer pertaining to the lack of authority of
Atty. Nethercott to initiate the 23 June 2004 auction sale thus appears mistaken, if not entirely baseless and
unfounded.

Also the mere fact that it was filed only after 2 years after the original answer was filed does not discredit the
amended answer as a mere dilatory measure for it should be remembered that the SC issued a TRO which was in
effect for more than a year so even if it wants to, URAMI could not have filed said amended answer.
Remedial Law I| Page 93

Lisam Enterprises vs. Banco De Oro


G.R. No. 143264, April 23, 2012

Facts:

LISAM, in the course of its business transaction, acquired a parcel of residential land with improvement in Legazpi
City. Sometime on March 1996, Lilian and Leandro Soriano obtained a loan from PCIB (now BDO) in their personal
capacity and executed a real estate mortgage as security for such loan. This real estate mortgage, it is being alleged,
was executed, without authority and consent of the board of LISAM and with the use of a falsified board resolution.
The spouses specifically falsified the signature of Lolita Soriano as corporate secretary and director of LISAM. Later
on, Spouses Soriano signed, through instigation of PCIB, a Deed of Assumption of Loans and Mortgage Obligations,
wherein LISAM was made to assume the liabilities of said spouses. After realizing this, Lolita demanded from the
Spouses the compliance of their obligation and to free LISAM from such but it went unanswered. Thereafter,
Soriano commenced a derivative suit against the Spouses before the SEC for Fraudulent Scheme and Unlawful
Machination with Damages. Moreover, the plaintiffs instituted before the RTC a case for the nullification of the
Real Estate Mortgage. The RTC issued a writ of preliminary injunction to enjoin PCIB from disposing the property.
In their Answer, Lilian and the Estate of Leandro stated that they were authorized by LISAM to mortgage the said
property and that the proceeds of the loan were for the benefit and use of LISAM.

PCIB filed a Motion to Dismiss the Complaint on grounds of lack of legal capacity to sue, failure to state cause of
action, and litis pendencia. The RTC then issued an order dismissing the case. Pending resolution of its MR, Lisam
filed a Motion to Admit Amended complaint. Its amended complaint now stating that Lolita exhausted intra-
corporate remedies and that she had asked LISAM’s board to take action but to no avail.

The Trial court issued the questioned Order denying both the Motion for Reconsideration and the Motion to Admit
Amended Complaint. It held that no new argument had been raised by petitioners in their motion for
reconsideration to address the fact of plaintiffs' failure to allege in the complaint that petitioner Lolita A. Soriano
made demands upon the Board of Directors of Lisam Enterprises, Inc. to take steps to protect the interest of the
corporation against the fraudulent acts of the Spouses Soriano and PCIB. The trial court further ruled that the
Amended Complaint can no longer be admitted, because the same absolutely changed petitioners' cause of action.

The petitioner went straight to the SC alleging that the case involves pure questions of law.

Issue:

Whether or not the amended complaint should have been admitted?

Held:

Yes. A responsive pleading having been filed, amendments to the complaint may, therefore, be made only by leave of
court and no longer as a matter of right. The courts should be liberal in allowing amendments to pleadings to avoid a
multiplicity of suits and in order that the real controversies between the parties are presented, their rights
determined, and the case decided on the merits without unnecessary delay. This liberality is greatest in the early
stages of a lawsuit, especially in this case where the amendment was made before the trial of the case, thereby
giving the petitioners all the time allowed by law to answer and to prepare for trial.

Furthermore, amendments to pleadings are generally favored and should be liberally allowed in furtherance of
justice in order that every case, may so far as possible, be determined on its real facts and in order to speed up the
trial of the case or prevent the circuitry of action and unnecessary expense. That is, unless there are circumstances
such as inexcusable delay or the taking of the adverse party by surprise or the like, which might justify a refusal of
permission to amend.

A reading of the amended complaint will reveal that all the foregoing requisites had been alleged therein. Hence, the
amended complaint remedied the defect in the original complaint and now sufficiently states a cause of action.
Remedial Law I| Page 94

Tiu vs. Philippine Bank of Communications


G.R. No. 151932. August 19, 2009.

Facts:

Asian Water Resources, Inc. (AWRI), represented by its officers applied for a real estate loan with Philippine Bank
of Communications (PBCOM). One of its loans was not guaranteed by a collateral, so the bank required all the
members of the Board of Directors of the Corp. to become sureties. A Surety Agreement was executed by its
Directors and acknowledged by a notary public on the same date. All copies of the Surety Agreement, except two,
were kept by bank. (One copy was retained by the notary public for his notarial file and the other was sent to the
Records Management and Archives Office). The bank then demanded payment. Unable to collect, the bank’s counsel
filed a complaint before the RTC for collection against the directors. On their answer, the directors alleged among
other things, that they were not personally liable on the promissory notes, because they signed the Surety
Agreement in their capacities as officers of the corp. They claimed that the Surety Agreement attached to the
complaint were falsified, considering that when they signed the same, the words in his personal capacity did not yet
appear in the document. They attached a copy from the Records Management and Archives Office). Because of this
development, the bank’s counsel searched for and retrieved the file copy of the Surety Agreement. It was discovered
that the insertion was ordered by the bank auditor, in accordance with bank standard operating procedures.
However, the notary public was never informed of the insertion. The bank then filed a Reply and Answer to
Counterclaim with Motion for Leave of Court to Substitute Annex A of the Complaint wherein it attached
the duplicate original copy retrieved from the file of the notary public. RTC issued an Order allowing the
substitution of the altered document with the original Surety Agreement. Aggrieved, petitioners sought recourse
before the CA via a petition for certiorari under Rule 65 of the Rules of Court which dismissed the petition for lack of
merit Hence, the current petition.

Issue:

Whether or not the substitution of the document should have been allowed?

Held:

Yes. With respect to the bank’s right to amend its complaint, including the documents annexed thereto, after
petitioners have filed their answer, Section 3, Rule 10 of the Rules of Court specifically allows amendment by leave
of court.

The granting of leave to file amended pleading is a matter particularly addressed to the sound discretion of the trial
court; and that discretion is broad, subject only to the limitations that the amendments should not substantially
change the cause of action or alter the theory of the case, or that it was not made to delay the action. Nevertheless,
as enunciated in Valenzuela, even if the amendment substantially alters the cause of action or defense, such
amendment could still be allowed when it is sought to serve the higher interest of substantial justice; prevent delay;
and secure a just, speedy and inexpensive disposition of actions and proceedings.

Amendments to pleadings are generally favored and should be liberally allowed in furtherance of justice. That is,
unless there are circumstances such as inexcusable delay or the taking of the adverse party by surprise or the like,
which might justify a refusal of permission to amend.

In the present case, there was no fraudulent intent on the part of PBCOM in submitting the altered surety
agreement. In fact, the bank admitted that it was a mistake on their part to have submitted it in the first place
instead of the original agreement. It also admitted that, through inadvertence, the copy that was attached to the
complaint was the copy wherein the words IN HIS PERSONAL CAPACITY were inserted to conform to the banks
standard practice. This alteration was made without the knowledge of the notary public. The Bank’s counsel had no
idea that what it submitted was the altered document, thereby necessitating the substitution of the surety
agreement with the original thereof, in order that the case would be judiciously resolved.

Moreover, the opposing party could not be prejudiced by the substitution since they can still present the substituted
documents, as part of the evidence of their affirmative defenses. Besides, they are not precluded from filing the
Remedial Law I| Page 95

appropriate criminal action against the bank for attaching the altered copy of the surety agreement to the
complaint. The substitution of the documents would not, in any way, erase the existence of falsification, if any.

Remington Industrial Sales Corp. vs. Court of Appeals


G.R. No. 133657. May 29, 2002

Facts:

Remington filed a complaint for sum of money and damages arising from breach of contract before the RTC.
Impleaded as principal defendant therein was Industrial Steels, Ltd. (ISL), with Ferro Trading GMBH (Ferro) and
respondent British Steel as alternative defendants. ISL and respondent British Steel separately moved for the
dismissal of the complaint on the ground that it failed to state a cause of action against them. RTC denied the
motions to dismiss, as well as the ensuing motion for reconsideration. ISL then filed its answer to the complaint.
On the other hand, respondent British Steel filed a petition for certiorari and prohibition before the Court of Appeals
and claimed that the complaint failed to state a cause of action against it.
Meanwhile, petitioner sought to amend its complaint. Pursuant to Section 2, Rule 10 of the Rules of Court, petitioner
maintained that it can amend the complaint as a matter of right because respondent has not yet filed a responsive
pleading thereto. Subsequently, petitioner prayed that the proceedings in the special civil action be suspended. The
trial court ruled on petitioner’s favor.
Meanwhile, CA granted the writ of certiorari and ordered the respondent judge to dismiss without prejudice the
Complaint against petitioner British Steel (Asia) Ltd.
Petitioner filed a motion for reconsideration of the appellate courts decision, which was denied, Hence this present
petition for review under rule 45.
Issue:
Whether or not the Court of Appeals, by granting the extraordinary writ of certiorari, correctly ordered the dismissal
of the complaint for failure to state a cause of action, despite the fact that petitioner exercised its right to amend the
defective complaint under Section 2, Rule 10 of the Rules of Court?
Held:
No. Section 2, Rule 10 of the Revised Rules of Court explicitly states that a pleading may be amended as a matter of
right before a responsive pleading is served. This only means that prior to the filing of an answer, the plaintiff has
the absolute right to amend the complaint whether a new cause of action or change in theory is introduced. The
reason for this rule is implied in the subsequent Section 3 of Rule 10. Under this provision, substantial amendment
of the complaint is not allowed without leave of court after an answer has been served, because any material change
in the allegations contained in the complaint could prejudice the rights of the defendant who has already set up his
defense in the answer. Conversely, it cannot be said that the defendant’s rights have been violated by changes made
in the complaint if he has yet to file an answer thereto. In such an event, the defendant has not presented any
defense that can be altered or affected by the amendment of the complaint in accordance with Section 2 of Rule 10.
The right granted to the plaintiff under procedural law to amend the complaint before an answer has been served is
not precluded by the filing of a motion to dismiss or any other proceeding contesting its sufficiency. Were we to
conclude otherwise, the right to amend a pleading under Section 2, Rule 10 will be rendered nugatory and
ineffectual, since all that a defendant has to do to foreclose this remedial right is to challenge the adequacy of the
complaint before he files an answer.
Moreover, amendment of pleadings is favored and should be liberally allowed in the furtherance of justice in order to
determine every case as far as possible on its merits without regard to technicalities.
Here, the fact that the other defendants below has filed their answers to the complaint does not bar petitioner’s
right to amend the complaint as against respondent. Indeed, where some but not all the defendants have answered,
the plaintiff may still amend its complaint once, as a matter of right, in respect to claims asserted solely against the
non-answering defendant, but not as to claims asserted against the other defendants.
Remedial Law I| Page 96

Rule 13: Filing and Service of Pleadings

Palileo vs. Planters Development Bank


G.R. No. 193650. October 8, 2014.
Facts:

George Philip Palileo and Jose Dela Cruz (petitioners) filed before the RTC a complaint for specific performance
and/or sum of money and damages with prayer for the issuance of writs of preliminary attachment and preliminary
injunction against Planters Development Bank. Summons was served to PDP and it filed its answer. However, PDP
failed to attend the last pre-trial hearing and it failed to file its pre-trial brief. Thus, Palileo and Dela Cruz were
allowed to present their evidence ex parte. The RTC ruled in favor of Palileo and Dela Cruz. PDP received a copy of
the RTC Decision on July 17, 2006.

On July 31, 2006, PDB filed by private courier service — specifically LBC — an Omnibus Motion for
Reconsideration and for New Trial. Petitioners’ copy of the Omnibus Motion for Reconsideration and for New Trial
was likewise sent on July 31, 2006 by courier service through LBC, but in their address of record — Tupi, South
Cotabato — there was no LBC service at the time.

On August 2, 2006, PDB filed with the RTC another copy of the Omnibus Motion for Reconsideration and for New
Trial via registered mail; another copy thereof was simultaneously sent to petitioners by registered mail as well.

The RTC denied the Omnibus Motion on the ground that it violated the Rule on Motions as it set the hearing on
August 18, 2006 or 16 days after filing (under the Rules; it must not be later than 10 days). Since the Motion was
pro-forma, it did not toll the running of the reglementary period thus making the RTC’s decision final and executory.

The Court of Appeals initially affirmed the decision of the RTC but on MR, the CA decided to relax the rules and
held that the Motion was not pro-forma considering that the address of the trial court as well as that of the opposing
counsel is too distant from the office of the counsel.

Issue:

Whether or not PDP’s Omninus Motion for Reconsideration/New Trial was filed late?

Held:

Yes. PDB’s Omnibus Motion for Reconsideration and for New Trial was filed one day too late. The bank received a
copy of the trial court’s June 15, 2006 Decision on July 17, 2006; thus, it had 15 days — or up to August 1, 2006 —
within which to file a notice of appeal, motion for reconsideration, or a motion for new trial, pursuant to the Rules of
Court. Yet, it filed the omnibus motion for reconsideration and new trial only on August 2, 2006.

Its filing or service of a copy thereof to petitioners by courier service cannot be trivialized. Service and filing of
pleadings by courier service is a mode not provided in the Rules. This is not to mention that PDB sent a
copy of its omnibus motion to an address or area which was not covered by LBC courier service at the time.
Realizing its mistake, PDB refiled and resent the omnibus motion by registered mail, which is the proper mode of
service under the circumstances. By then, however, the 15-day period had expired. Thus, the RTC’s decision had
become final and executory by the failure of PDP to file a timely appeal.
Remedial Law I| Page 97

Heirs of Numeriano Miranda, Sr. vs. Miranda
G.R. No. 179638. July 8, 2013
Facts:

In 1994, the heirs of Numeriano Miranda (petitioners) filed a Complaint for Annulment of Titles and Specific
Performance against the heirs of Pedro Miranda. The RTC ruled in favor of the heirs of Pedro Miranda and sustain
the validity of their titles. Petitioners did not file an appeal and the Decision became final and executory. In 2001,
the RTC issued a Writ of Execution which was not implemented. In 2005, Pablo Miranda (respondent) filed with the
RTC a Petition for Revival of Judgment. The RTC granted the petition on June 20, 2006. On July 13, 2006,
petitioners filed a Notice of Appeal via LBC. The RTC denied the notice of appeal. Aggrieved, petitioners filed a
Petition for Mandamus with the CA. The CA denied the petition on the ground that the Notice of Appeal was filed
out of time. Before the Supreme Court, the petitioners contend that their appeal was perfected on time. They insist
that the Notice of Appeal, which they filed on the 15th day via LBC, was seasonably filed since the law does not
require a specific mode of service for filing a notice of appeal.

Issue:

Whether or not the filing of the Notice of Appeal thru a courier is a valid mode of service for filing a notice of appeal?

Held:

No. Under Section 3, Rule 13 of the Rules of Court, pleadings may be filed in court either personally or by registered
mail. In the first case, the date of filing is the date of receipt. In the second case, the date of mailing is the date of
receipt.

In this case, however, the counsel for petitioners filed the Notice of Appeal via a private courier, a mode of filing not
provided in the Rules. Though not prohibited by the Rules, it cannot be considered the filing of petitioners’ Notice of
Appeal via LBC timely filed. It is established jurisprudence that “the date of delivery of pleadings to a private letter-
forwarding agency is not to be considered as the date of filing thereof in court”; instead, “the date of actual receipt by
the court x x x is deemed the date of filing of that pleading.” Records show that the Notice of Appeal was mailed on
the 15th day and was received by the court on the 16th day or one day beyond the reglementary period. Thus, the
CA correctly ruled that the Notice of Appeal was filed out of time.
Remedial Law I| Page 98

Fortune Life Insurance Company, Inc. vs. Commission on Audit (COA) Proper
G.R. No. 213525. January 27, 2015.

Facts:

The Provincial Government of Antique and Fortune Life Insurance executed a memorandum of agreement
concerning the life insurance coverage of qualified barangay secretaries, treasurers and tanod, the former obligating
P4,393,593.60 for the premium payment, and subsequently submitting the corresponding disbursement voucher to
COA-Antique for pre-audit. The latter office disallowed the payment for lack of legal basis under Republic Act No.
7160 (Local Government Code).

Subsequently, Fortune Insurance filed a petition for money claim in the Commission on Audit (COA).

COA denied its petition holding that under Section 447 and Section 458 of the Local Government Code only
municipal or city governments are expressly vested with the power to secure group insurance coverage for barangay
workers; and for the LGU’s failure to comply with the requirement of publication under Section 21 of Republic Act
No. 9184 (Government Procurement Reform Act).

Fortune Insurance filed a Motion for Reconsideration which was denied on July 14, 2014.

Fortune Insurance filed a petition for certiorari before the Supreme Court on August 12, 2014. The Supreme Court
dismissed the petition for (a) the late filing of the petition; (b) the nonsubmission of the proof of service and
verified declaration; and (c) the failure to show grave abuse of discretion on the part of the respondents. Fortune
Insurance then filed a Motion for Reconsideration.

Issue:

Whether or not Fortune Insurance complied with the rule on proof of service (Section 13, Rule 13)?

Held:

No. Section 13, Rule 13 of the Rules of Court concerns two types of proof of service, namely: the affidavit and the
registry receipt, viz.:

Section 13. Proof of Service.—x x x. If service is made by registered mail, proof shall be made by such affidavit and
the registry receipt issued by the mailing office. The registry return card shall be filed immediately upon its
receipt by the sender, or in lieu thereof the unclaimed letter together with the certified or sworn copy of the notice
given by the postmaster to the addressee.

Section 13 thus requires that if the service is done by registered mail, proof of service shall consist of
the affidavit of the person effecting the mailing and the registry receipt, both of which must be
appended to the paper being served. A compliance with the rule is mandatory, such that there is no
proof of service if either or both are not submitted.

Here, the petition for certiorari only carried the affidavit of service executed by one Marcelino T. Pascua, Jr., who
declared that he had served copies of the petition by registered mail “under Registry Receipt Nos. 70449, 70453,
70458, 70498 and 70524 attached to the appropriate spaces found on pages 64-65 of the petition.” The petition only
bore, however, the cut print-outs of what appeared to be the registry receipt numbers of the registered matters, not
the registry receipts themselves. The rule requires to be appended the registry receipts, not their reproductions.
Hence, the cut print-outs did not substantially comply with the rule.
Remedial Law I| Page 99

Rule 14: Summons

Valmonte vs. Court of Appeals


G.R. No. 131724 February 28, 2000

Facts:

Petitioners Lourdes and Alfredo are husband and wife. They are both residents of the USA. Petitioner Alfredo,
however, is a practicing lawyer in the Philippines with office address at Ermita, Manila. A complaint for partition of
real property and accounting of rentals was filed by private respondent Rosita, who is the sister of petitioner
Lourdes. The subject of the action is a three-door apartment located in Paco, Manila. In her Complaint, Rosita
alleged that in a letter previously sent by petitioner Lourdes to Rosita, the former referred to her husband as the
party to whom all communications intended for her should be sent, whose office address is in the Philippines.

Service of summons was then made upon petitioner Alfredo, who at the time, was at his office in Manila. Alfredo
accepted the summons, insofar as he was concerned, but refused to accept the summons for his wife, on the ground
that he was not authorized to accept the process on her behalf. Accordingly, the process server left without leaving a
copy of the summons and complaint for petitioner Lourdes. Alfredo thereafter filed his Answer with Counterclaim.
Petitioner Lourdes, however, did not file her Answer. For this reason, private respondent moved to declare her in
default. Petitioner Alfredo entered a special appearance in behalf of his wife and opposed the private respondent’s
motion. The RTC, denied the motion of Rosita to declare Petitioner Lourdes in default and subsequently, her Motion
for Reconsideration was denied. On appeal, the CA granted the petition and declared Petitioner Lourdes in default.
The CA also ruled that the Petitioner Lourdes was validly served with summons. Hence, this petition. Petitioner
Lourdes contends, among others, that there was no valid substituted service.

Issue:

Whether or not the summons intended to Petitioner Lourdes was validly served to her husband Alfredo?

Held:

No. There was no valid service of process on Lourdes.

It will be helpful to determine first the nature of the action filed against petitioners Lourdes and Alfredo by private
respondent, whether it is an action in personam, in rem or quasi in rem. This is because the rules on service of
summons embodied in Rule 14 apply according to whether an action is one or the other of these actions.

In an action in personam, personal service of summons or, if this is not possible and he cannot be personally served,
substituted service, as provided in Rule 14, 7-8 is essential for the acquisition by the court of jurisdiction over the
person of a defendant who does not voluntarily submit himself to the authority of the court. If defendant cannot be
served with summons because he is temporarily abroad, but otherwise he is a Philippine resident, service of
summons may, by leave of court, be made by publication. Otherwise stated, a resident defendant in an action in
personam, who cannot be personally served with summons, may be summoned either by means of substituted
service in accordance with Rule 14, 8 or by publication as provided in 17 and 18 of the same Rule.

If the action is in rem or quasi in rem, jurisdiction over the person of the defendant is not essential for giving the
court jurisdiction so long as the court acquires jurisdiction over the res. If the defendant is a nonresident and he is
not found in the country, summons may be served extraterritorially in accordance with Rule 14, Section 17. Service
of summons in the manner provided in Sec. 17 is not for the purpose of vesting it with jurisdiction but for complying
with the requirements of fair play or due process, so that he will be informed of the pendency of the action against
him and the possibility that property in the Philippines belonging to him or in which he has an interest may be
subjected to a judgment in favor of the plaintiff and he can thereby take steps to protect his interest if he is so
minded.
Remedial Law I| Page 100

Applying the foregoing rules to this case, private respondent’s action, which is for partition and accounting under
Rule 69, is in the nature of an action quasi in rem. Such an action is essentially for the purpose of affecting the
defendant’s interest in a specific property and not to render a judgment against him. As petitioner Lourdes A.
Valmonte is a nonresident who is not found in the Philippines, service of summons on her must be in accordance
with Rule 14, Section 17. Such service, to be effective outside the Philippines, must be made either (1) by personal
service; (2) by publication in a newspaper of general circulation in such places and for such time as the court may
order, in which case a copy of the summons and order of the court should be sent by registered mail to the last
known address of the defendant; or (3) in any other manner which the court may deem sufficient.

Since the service of summons upon petitioner Lourdes was not done by means of any of the first two modes, the
question is whether the service on her attorney, petitioner Alfredo, can be justified under the third mode, namely, in
any . . . manner the court may deem sufficient. The Court holds it cannot. This mode of service, like the first two,
must be made outside the Philippines, such as through the Philippine Embassy in the foreign country where the
defendant resides. Moreover, there are several reasons why the service of summons on Atty. Alfredo D. Valmonte
cannot be considered a valid service of summons on petitioner Lourdes. In the first place, service of summons on
petitioner Alfredo D. Valmonte was not made upon the order of the court as required by Rule 14, Sec. 17 and
certainly was not a mode deemed sufficient by the court which in fact refused to consider the service to be valid and,
on that basis, declare petitioner Lourdes in default for her failure to file an answer. In the second place, service in
the attempted manner on petitioner was not made upon prior leave of the trial court as required also in Rule 14,
Section 17. As provided in Section 19, such leave must be applied for by motion in writing, supported by affidavit of
the plaintiff or some person on his behalf and setting forth the grounds for the application. Finally, and most
importantly, because there was no order granting such leave, petitioner Lourdes was not given ample time to file her
Answer which, according to the rules, shall be not less than sixty (60) days after notice.
Remedial Law I| Page 101

Note: The doctrine of substantial compliance is no longer controlling.

Millenium Industrial Commercial Corp. vs. Tan


G.R. No. 131724, February 28, 2000

Facts:
Millenium Industrial Commercial Corporation, executed a Deed of Real Estate Mortgage over its real
property in favor of respondent Tan. The mortgage was executed to secure payment of petitioner's indebtedness to
respondent. Thereafter, respondent filed against petitioner a complaint for foreclosure of mortgage in the Regional
Trial Court. The summons and a copy of the complaint were served upon petitioner through a certain Lynverd
Cinches, described in the sheriff's return, as "a Draftsman, a person of sufficient age and (discretion) working
therein, he is the highest ranking officer or Officer-in-Charge of defendant's Corporation, to receive processes of the
Court."

Petitioner moved for the dismissal of the complaint on the ground that there was no valid service of summons upon
it, as a result of which the trial court did not acquire jurisdiction over it. Petitioner invoked Rule 14, Section13 of the
1964 Rules of Court and contended that service on Lynverd Cinches, as alleged in the sheriff's return, was invalid as
he is not one of the authorized persons on whom summons may be served and that, in fact, he was not even its
employee. The trial court denied its motion to dismiss on the ground that the petitioner is deemed to have
voluntarily submitted itself to the jurisdiction of the Court by interposing an affirmative defense that the obligation
sought to be collected was already paid and extinguished. Later on, Petitioner then filed a petition for certiorari in
the Court of Appeals but was dismissed. The appellate court ruled that although petitioner denied Lynverd Cinches'
authority to receive summons for it, its actual receipt of the summons could be inferred from its filing of a motion to
dismiss, hence, the purpose for issuing summons had been substantially achieved. Hence, this petition. Petitioner
alleges that the enumeration of persons on whom service of summons on a corporation may be effected in Rule 14,
Section 13, is exclusive and mandatory, thus the doctrine of substantial compliance does not apply.

Issue:

Whether or not the service of summons to the Draftsman when the defendant is a corporation is valid?

Held:

No. Petitioner contends that the enumeration in Rule 14, Sec. 13 is exclusive and that service of summons upon one
who is not enumerated therein is invalid. This is the general rule. However, it is settled that substantial compliance
by serving summons on persons other than those mentioned in the above rule may be justified. In G & G Trading
Corporation v. Court of Appeals, the Court ruled that although the service of summons was made on a person not
enumerated in Rule 14, Section 13, if it appears that the summons and complaint were in fact received by the
corporation, there is substantial compliance with the rule as its purpose has been attained.

In Porac Trucking, Inc. v. Court of Appeals, the Court enumerated the requisites for the application of the doctrine
of substantial compliance, to wit: (a) there must be actual receipt of the summons by the person served, i.e.,
transferring possession of the copy of the summons from the Sheriff to the person served; (b) the person served must
sign a receipt or the sheriff's return; and (c) there must be actual receipt of the summons by the corporation through
the person on whom the summons was actually served. The third requisite is the most important for it is through
such receipt that the purpose of the rule on service of summons is attained.

In this case, there is no dispute that the first and second requisites were fulfilled. With respect to the third, the
appellate court held that petitioner's filing of a motion to dismiss the foreclosure suit is proof that it received the
copy of the summons and the complaint. There is, however, no direct proof of this or that Lynverd Cinches actually
turned over the summons to any of the officers of the corporation. For there to be substantial compliance, actual
receipt of summons by the corporation through the person served must be shown. Where a corporation only
learns of the service of summons and the filing of the complaint against it through some person or
means other than the person actually served, the service of summons becomes meaningless. This is
particularly true in the present case where there is serious doubt if Lynverd Cinches, the person on whom service of
Remedial Law I| Page 102

summons was effected, is in fact an employee of the corporation. Except for the sheriff's return, there is nothing to
show that Lynverd Cinches was really a draftsman employed by the corporation. The question is whether it is
allowable to merely infer actual receipt of summons by the corporation through the person on whom summons was
served.
Remedial Law I| Page 103

E.B. Villarosa vs. Benito


G.R. No. 136426 August 6, 1999

Facts:

Petitioner E.B. Villarosa & Partner Co., Ltd. is a limited partnership with principal office address at 102 Juan Luna
St., Davao City and with branch offices at 2492 Bay View Drive, Tambo, Parañaque, Metro Manila and Kolambog,
Lapasan, Cagayan de Oro City. Imperial Development, as plaintiff, filed a Complaint for Breach of Contract and
Damages against petitioner, as defendant, before the Regional Trial Court of Makati allegedly for failure of the
latter to comply with its contractual obligation. Summons, together with the complaint, were served upon E.B.
Villarosa, through its Branch Manager Engr. Wendell Sabulbero at the stated address at Kolambog, Lapasan,
Cagayan de Oro City, but the Sheriffs Return of Service stated that the summons was duly served upon defendant
E. B. Villarosa & Partner Co., Ltd. thru its Branch Manager Engr. WENDELL SALBULBERO at their new office
Villa Gonzalo, Nazareth, Cagayan de Oro City, and evidenced by the signature on the face of the original copy of the
summons.

E.B. Villarosa filed a Motion to Dismiss, which it prayed for the dismissal of the complaint on the ground of
improper service of summons and for lack of jurisdiction over the person of the defendant. E.B. Villarosa contends
that the trial court did not acquire jurisdiction over its person since the summons was improperly served upon its
employee in its branch office at Cagayan de Oro City who is not one of those persons named in Section 11, Rule 14 of
the 1997 Rules of Civil Procedure upon whom service of summons may be made. Imperial Development filed an
Opposition to Defendants Motion to Dismiss alleging that the records show that E.B. Villarosa, through its branch
manager, actually received the summons and the complaint as evidenced by the signature appearing on the copy of
the summons, among others. The trial court issued an Order denying E.B. Villarosa’s Motion to Dismiss. The trial
court stated that since the summons and copy of the complaint were in fact received by the corporation through its
branch manager, there was substantial compliance with the rule on service of summons and consequently, it validly
acquired jurisdiction over the person of the E.B. Villarosa.

E.B. Villarosa, by Special Appearance, filed a Motion for Reconsideration alleging that Section 11, Rule 14 of the
new Rules did not liberalize but, on the contrary, restricted the service of summons on persons enumerated therein;
and that the new provision is very specific and clear in that the word manager was changed to general manager,
secretary to corporate secretary, and excluding therefrom agent and director. Further, in its Reply, it contends that
the changes in the new rules are substantial and not just general semantics. E.B. Villarosa’s Motion for
Reconsideration was denied, hence, the present petition alleging that, respondent court gravely abused its discretion
tantamount to lack or in excess of jurisdiction in denying petitioners motions to dismiss and for reconsideration,
despite the fact that the trial court did not acquire jurisdiction over the person of petitioner because the summons
intended for it was improperly served. Petitioner invokes Section 11 of Rule 14 of the 1997 Rules of Civil Procedure.

Issue:

Whether or not the trial court acquired jurisdiction over the person of the corporation upon service of summons on
its Branch Manager?

Held:

No. The service of summons to the branch manager of the Petitioner is invalid. The Court ruled that when the
complaint was filed by Petitioner on April 3, 1998, the 1997 Rules of Civil Procedure was already in force.

Section 11, Rule 14 of the 1997 Rules of Civil Procedure provides that:

“When the defendant is a corporation, partnership or association organized under the laws of the
Philippines with a juridical personality, service may be made on the president, managing partner, general
manager, corporate secretary, treasurer, or in-house counsel.”
Remedial Law I| Page 104

Petitioner contends that the enumeration of persons to whom summons may be served is restricted, limited and
exclusive following the rule on statutory construction expressio unios est exclusio alterius and argues that if the
Rules of Court Revision Committee intended to liberalize the rule on service of summons, it could have easily done
so by clear and concise language. The Court agrees with petitioner. Notably, under the new Rules, service of
summons upon an agent of the corporation is no longer authorized.

The designation of persons or officers who are authorized to accept summons for a domestic corporation or
partnership is now limited and more clearly specified in Section 11, Rule 14 of the 1997 Rules of Civil
Procedure. The rule now states general manager instead of only manager; corporate secretary instead of secretary;
and treasurer instead of cashier. The phrase agent, or any of its directors is conspicuously deleted in the new rule.

Retired Justice Oscar Herrera, who is also a consultant of the Rules of Court Revision Committee, stated that (T)he
rule must be strictly observed. Service must be made to one named in (the) statute x x x.

It should be noted that even prior to the effectivity of the 1997 Rules of Civil Procedure, strict compliance with the
rules has been enjoined. In the case of Delta Motor Sales Corporation vs. Mangosing, the Court held:

“A strict compliance with the mode of service is necessary to confer jurisdiction of the court over a
corporation. The officer upon whom service is made must be one who is named in the statute; otherwise
the service is insufficient. x x x.”

Accordingly, the Court ruled that the service of summons upon the branch manager of petitioner at its branch office
at Cagayan de Oro, instead of upon the general manager at its principal office at Davao City is
improper. Consequently, the trial court did not acquire jurisdiction over the person of the petitioner.
Remedial Law I| Page 105

Mason vs. Court of Appeals


G.R. No. 144662 October 13, 2003

Facts:

Spouses Mason owned two parcels of land located along EDSA in Pasay City. They entered into a lease contract with
Columbus Philippines Bus Corporation, where Columbus undertook to construct a building worth P10 million at the
end of the third year of the lease. For failure to comply with this stipulation, the Spouses filed a complaint for
rescission of contract with damages against Columbus before the RTC. Summons was served upon Columbus
through a certain Ayreen Rejalde. While the receiving copy of the summons described her as a secretary of
Columbus, the sheriff’s return indicated that she is a secretary to the corporate president, duly authorized to receive
legal processes.

Columbus failed to file its answer, hence, spouses Mason filed a motion to declare it in default. The motion was
granted and the spouses were allowed to present evidence ex-parte. Thereafter, the case was submitted for decision.
The trial court ruled in favor of the spouses. That decision became final on May 12, 1999. The following day,
Columbus filed a motion to lift order of default, which was opposed by the spouses. The trial court ordered the
parties to submit their respective memoranda. However, without waiting for the same, the trial court denied the
motion to lift order of default on the ground that the decision has already become final and executory. Consequently,
the spouses’ Motion for Execution was granted. Columbus filed a motion for reconsideration, which was denied.
Undaunted, it filed a manifestation and motion to lift the writ of execution but was also denied for being dilatory. It
then appealed to the CA, which ruled in its favor. The CA pointed out that Columbus was not properly served with
summons, thus it cannot be faulted if it failed to file an Answer. Spouses Mason filed a motion for reconsideration,
but to no avail. Hence, this petition for review.

Spouses Mason stressed that even though the summons was received by a mere filing clerk of Columbus, there was
substantial compliance with Section 11, Rule 14 because the summons actually reached said corporation. This can
be gleaned from the motion to lift order of default where Columbus did not question the validity of the service of
summons but explained that its failure to answer was due to its impression that the case would not be pursued by
the spouses on the ground that payments were already made to them. The spouses also cited Millenium Industrial
Commercial Corporation v. Tan, and maintained that this Court, by referring to E.B Villarosa & Partner Co., Ltd. v.
Judge Benito, effectively ruled that said provision is the statement of the general rule on service of summons upon
corporation and the substantial compliance rule is the exception.

Issue:

Whether or not there was valid service of summons on Columbus for the trial court to acquire jurisdiction?

Held:

No. The question of whether the substantial compliance rule is still applicable under Section 11, Rule 14 of the 1997
Rules of Civil Procedure has been settled in E.B. Villarosa which applies squarely to the instant case. In the said
case, the Court held that there was no valid service of summons on Villarosa as service was made through a person
not included in the enumeration in Section 11, Rule 14 of the 1997 Rules of Civil Procedure, which revised Section
13, Rule 14 of the 1964 Rules of Court. Also in that case, the Court discarded substantial compliance with the rule
on service of summons, and ruled that the enumeration under the new rule is restricted, limited and exclusive,
following the rule in statutory construction that expressio unios est exclusio alterius. Had the Rules of Court Revision
Committee intended to liberalize the rule on service of summons, it could have easily done so by clear and concise
language. Absent a manifest intent to liberalize the rule, strict compliance with Section 11, Rule 14 of the 1997
Rules of Civil Procedure is stressed. Neither can herein petitioners invoke the ruling in Millenium to support their
position for said case is not on all fours with the instant case. It must be stressed that Millenium was decided when
the 1964 Rules of Court were still in force and effect, unlike the instant case which falls under the new rule. Hence,
the cases cited by spouses Mason where the doctrine of substantial compliance was upheld must be deemed
overturned by Villarosa, which is the later case.
Remedial Law I| Page 106

At this juncture, it is worth emphasizing that notice to enable the other party to be heard and to present evidence is
not a mere technicality or a trivial matter in any administrative or judicial proceedings. The service of summons is a
vital and indispensable ingredient of due process. The Court will deprive Columbus of its right to present its defense
in this multi-million peso suit, if compliance with the rules on service of summons will be disregarded.

Jose vs. Boyon


G.R. No. 147369 October 23, 2003

Facts:

Spouses Jose lodged a complaint for specific performance against Spouses Boyon to compel them to facilitate the
transfer of ownership of a parcel of land subject of a controverted sale. Public respondent Judge Perello issued
summons to Spouses Boyon. As per return of the summons, substituted service was resorted to by the process server
allegedly because efforts to serve the summons personally to Spouses Boyon failed. Spouses Jose filed an Ex-parte
Motion for Leave of Court to Effect Summons by Publication which the RTC granted. Thereafter, the RTC judge,
without a written motion, issued an Order declaring Spouses Boyon in default for failure to file their respective
answers. Spouses Jose were then allowed to submit their evidence ex-parte. Ultimately, the RTC ruled in favor of
Spouses Jose.

Helen Boyon, who was then residing in USA, was surprised to learn from her sister of the resolution issued by the
RTC. Spouses Boyon then filed an Ad Cautelam motion questioning, among others, the validity of the service of
summons effected by the court a quo. The RTC denied the said motion, stating that the defaulted spouses already
lost their standing in court. Spouses Boyon filed a motion for reconsideration but it was denied. Meanwhile, Spouses
Jose moved for the execution of the RTC judgment and the same was granted.

Spouses Boyon filed before the CA a Petition for certiorari questioning the jurisdiction of the RTC. The CA ruled
that the RTC never acquired jurisdiction over Spouses Boyon because of the invalid service of summons upon them.
First, the sheriff failed to comply with the requirements of substituted service of summons because he did not
specify in the Return of Summons the prior efforts he had made to locate them and the impossibility of promptly
serving the summons upon them by personal service. Second, the subsequent summons by publication was equally
infirm, because the Complaint was a suit for specific performance and therefore an action in personam.

Thus, the present Petition for Review on Certiorari under Rule 45. Spouses Jose aver, among others, that the CA
erred in ruling that the service of summons was invalid. They submit that although the case filed before the trial
court was denominated as an action for specific performance, it was actually an action quasi in rem, because it
involved a piece of real property located in the Philippines.

Issue:

Whether or not the service of summons upon Spouses Boyon is valid?

Held:

No.In general, trial courts acquire jurisdiction over the person of the defendant by the service of summons. Where
the action is in personam and the defendant is in the Philippines, such service may be done by personal or
substituted service, following the procedures laid out in Sections 6 and 7 of Rule 14 of the Revised Rules of Court,
which read:

"Section 6. Service in person on defendant. - Whenever practicable, the summons shall be served by handing a copy
thereof to the defendant in person, or, if he refuses to receive and sign for it, by tendering it to him.

"Section 7. Substituted service. - If, for justifiable causes, the defendant cannot be served within a reasonable time as
provided in the preceding section, service may be effected (a) by leaving copies of the summons at the defendant's
residence with some person of suitable age and discretion then residing therein, or (b) by leaving the copies at
defendant’s office or regular place of business with some competent person in charge thereof."
Remedial Law I| Page 107

As can be gleaned from the above-quoted Sections, personal service of summons is preferred to substituted service.
Only if the former cannot be made promptly can the process server resort to the latter. Moreover, the proof of service
of summons must:

(a) indicate the impossibility of service of summons within a reasonable time;

(b) specify the efforts exerted to locate the defendant; and,

(c) state that the summons was served upon a person of sufficient age and discretion who is
residing in the address, or who is in charge of the office or regular place of business, of the
defendant.

It is likewise required that the pertinent facts proving these circumstances be stated in the proof of service or in the
officer’s return. The failure to comply faithfully, strictly and fully with all the foregoing requirements of substituted
service renders the service of summons ineffective.

In the instant case, it appears that the process server hastily and capriciously resorted to substituted service of
summons without actually exerting any genuine effort to locate Spouses Boyon. The only effort he exerted was to go
to No. 32 Ariza Drive, Camella Homes, Alabang, to try to serve the summons personally. While the Return of
Summons states that efforts to do so were ineffectual and unavailing because Helen Boyon was in the United States
and Romeo Boyon was in Bicol, it did not mention exactly what efforts -- if any -- were undertaken to find
respondents. Furthermore, it did not specify where or from whom the process server obtained the information on
their whereabouts.

As to the Service of Summons by Publication or Extraterritorial Service of Summons, it applies only when the action
is in rem or quasi in rem. In the instant case, what was filed before the trial court was an action for specific
performance directed against respondents. While the suit incidentally involved a piece of land, the ownership or
possession thereof was not put in issue, since they did not assert any interest or right over it. Moreover, this Court
has consistently declared that an action for specific performance is an action in personam.
Remedial Law I| Page 108

Manotoc vs. Court of Appeals


G.R. No. 130974 August 16, 2006

Facts:

The Estate of Trajano seeks the enforcement of a foreign court’s judgment rendered by the United States District
Court of Honolulu, Hawaii, USA in a case against Imee Marcos-Manotoc for the wrongful death of deceased
Archimedes Trajano committed by military intelligence officials of the Philippines allegedly under the command,
direction, authority, supervision, tolerance, sufferance and/or influence of Manotoc, pursuant to the provisions of
Rule 39 of the then Revised Rules of Court. The trial court issued a Summons addressed to Manotoc at Alexandra
Condominium Corporation or Alexandra Homes, E2 Room 104, at No. 29 Meralco Avenue, Pasig City. The Summons
and a copy of the Complaint were allegedly served upon (Mr.) Macky de la Cruz, an alleged caretaker of Manotoc at
that condominium unit. When Manotoc failed to file her Answer, the trial court declared her in default through an
Order.

Manotoc, by special appearance of counsel, filed a Motion to Dismiss on the ground of lack of jurisdiction of the trial
court over her person due to an invalid substituted service of summons, averring that: (1) the address of defendant
indicated in the Complaint was not her dwelling, residence, or regular place of business; (2) the party (de la Cruz),
who was found in the unit, was neither a representative, employee, nor a resident of the place; (3) the procedure
prescribed by the Rules on personal and substituted service of summons was ignored; (4) defendant was a resident of
Singapore; and (5) whatever judgment rendered in this case would be ineffective and futile.

During the hearing on the Motion to Dismiss, Manotoc presented Carlos Gonzales, who testified that he saw
defendant Manotoc as a visitor in Alexandra Homes only two times. She also presented her Philippine passport and
the Disembarkation/Embarkation Card issued by the Immigration Service of Singapore to show that she was a
resident of Singapore. She claimed that the person referred to as "Mrs. Manotoc" may not even be her, but the
mother of one Tommy Manotoc. On the other hand, Trajano presented the lead counsel in a case involving the
Marcoses who testified that he participated in the deposition taking of Marcos, Jr. The counsel confirmed that
Marcos, Jr. testified that Manotoc’s residence was at the Alexandra Apartment, Greenhills.

The trial court rejected Manotoc’s Motion to Dismiss, relying on the presumption that the sheriff’s substituted
service was made in the regular performance of official duty, and that such presumption stood in the absence of
proof to the contrary. The trial court likewise discarded Manotoc’s plea for reconsideration for lack of merit.
Manotoc then filed a Petition for Certiorari and Prohibition before the CA. The CA adopted the findings of the trial
court. Manotoc filed a Motion for Reconsideration but was denied. Hence, the present petition for review on
certiorari.

Issue:

Whether or not there is a valid substituted service of summons for the trial court to acquire jurisdiction over
Manotoc?

Held:

No. A meticulous scrutiny of the Sheriff’s Return readily reveals the absence of material data on the serious efforts
to serve the Summons on Manotoc in person. There is no clear valid reason cited in the Return why those
efforts proved inadequate, to reach the conclusion that personal service has become impossible or
unattainable outside the generally couched phrases of "on many occasions several attempts were made to serve the
summons x x x personally," "at reasonable hours during the day," and "to no avail for the reason that the said
defendant is usually out of her place and/or residence or premises."

Besides, apart from the allegation of Manotoc’s address in the Complaint, it has not been shown that respondent
Trajano or Sheriff Cañelas, who served such summons, exerted extraordinary efforts to locate Mantooc.
Certainly, the second paragraph of the Complaint only states that respondents were "informed, and so [they] allege"
Remedial Law I| Page 109

about the address and whereabouts of Manotoc. Before resorting to substituted service, a plaintiff must demonstrate
an effort in good faith to locate the defendant through more direct means. More so, in the case in hand, when the
alleged petitioner’s residence or house is doubtful or has not been clearly ascertained, it would have been better for
personal service to have been pursued persistently.

Also, there is a serious nonconformity from the requirement that the summons must be left with a "person of
suitable age and discretion" residing in defendant’s house or residence.

There are two (2) requirements under the Rules: (1) recipient must be a person of suitable age and discretion; and
(2) recipient must reside in the house or residence of defendant. Both requirements were not met. In this case, the
Sheriff’s Return lacks information as to residence, age, and discretion of Mr. De la Cruz, aside from the sheriff’s
general assertion that de la Cruz is the "resident caretaker" of Manotoc as pointed out by the alleged receptionist
and telephone operator of Alexandra Homes. It is doubtful if Mr. De la Cruz is residing with Manotoc in the
condominium unit considering that a married woman of her stature in society would unlikely hire a male caretaker
to reside in her dwelling. Besides, Mr. De la Cruz’s refusal to sign the Receipt for the summons is a strong indication
that he did not have the necessary "relation of confidence" with Manotoc. To protect Manotoc’s right to due
process by being accorded proper notice of a case against her, the substituted service of summons must be shown to
clearly comply with the rules. Due to non-compliance with the prerequisites for valid substituted service, the
proceedings held before the trial court perforce must be annulled.

Discussion:

Requirements for Substituted Service

Section 8 of Rule 14 of the old Revised Rules of Court which applies to this case provides:

SEC. 8. Substituted service. – If the defendant cannot be served within a reasonable time as provided in the
preceding section [personal service on defendant], service may be effected (a) by leaving copies of the summons at the
defendant’s residence with some person of suitable age and discretion then residing therein, or (b) by leaving the
copies at defendant’s office or regular place of business with some competent person in charge thereof.

We can break down this section into the following requirements to effect a valid substituted service:

(1) Impossibility of Prompt Personal Service

The party relying on substituted service or the sheriff must show that defendant cannot be served promptly or there
is impossibility of prompt service. Section 8, Rule 14 provides that the plaintiff or the sheriff is given a "reasonable
time" to serve the summons to the defendant in person, but no specific time frame is mentioned.

"Reasonable time" is defined as "so much time as is necessary under the circumstances for a reasonably prudent
and diligent man to do, conveniently, what the contract or duty requires that should be done, having a regard for the
rights and possibility of loss, if any[,] to the other party."

To the plaintiff, "reasonable time" means no more than seven (7) days since an expeditious processing of a complaint
is what a plaintiff wants. To the sheriff, "reasonable time" means 15 to 30 days because at the end of the month, it is
a practice for the branch clerk of court to require the sheriff to submit a return of the summons assigned to the
sheriff for service. The Sheriff’s Return provides data to the Clerk of Court, which the clerk uses in the Monthly
Report of Cases to be submitted to the Office of the Court Administrator within the first ten (10) days of the
succeeding month. Thus, one month from the issuance of summons can be considered "reasonable time" with regard
to personal service on the defendant.

Sheriffs are asked to discharge their duties on the service of summons with due care, utmost diligence, and
reasonable promptness and speed so as not to prejudice the expeditious dispensation of justice. Thus, they are
enjoined to try their best efforts to accomplish personal service on defendant. On the other hand, since the defendant
is expected to try to avoid and evade service of summons, the sheriff must be resourceful, persevering, canny,
Remedial Law I| Page 110

and diligent in serving the process on the defendant. For substituted service of summons to be available,
there must be several attempts by the sheriff to personally serve the summons within a reasonable period [of one
month] which eventually resulted in failure to prove impossibility of prompt service.

"Several attempts" means at least three (3) tries, preferably on at least two different dates. In addition, the sheriff
must cite why such efforts were unsuccessful. It is only then that impossibility of service can be confirmed or
accepted.

(2) Specific Details in the Return

The sheriff must describe in the Return of Summons the facts and circumstances surrounding the attempted
personal service. The efforts made to find the defendant and the reasons behind the failure must be clearly narrated
in detail in the Return. The date and time of the attempts on personal service, the inquiries made to locate the
defendant, the name/s of the occupants of the alleged residence or house of defendant and all other acts done,
though futile, to serve the summons on defendant must be specified in the Return to justify substituted service. The
form on Sheriff’s Return of Summons on Substituted Service prescribed in the Handbook for Sheriffs published by
the Philippine Judicial Academy and the SC Administrative Circular No. 5 (1989) require a narration of the efforts
made to find the defendant personally and the fact of failure.

(3) A Person of Suitable Age and Discretion

If the substituted service will be effected at defendant’s house or residence, it should be left with a person of
"suitable age and discretion then residing therein."

A person of suitable age and discretion is one who has attained the age of full legal capacity (18 years old) and
is considered to have enough discernment to understand the importance of a summons.

"Discretion" is defined as "the ability to make decisions which represent a responsible choice and for which an
understanding of what is lawful, right or wise may be presupposed".

Thus, to be of sufficient discretion, such person must know how to read and understand English to comprehend
the import of the summons, and fully realize the need to deliver the summons and complaint to the defendant at the
earliest possible time for the person to take appropriate action. Thus, the person must have the "relation of
confidence" to the defendant, ensuring that the latter would receive or at least be notified of the receipt of the
summons. The sheriff must therefore determine if the person found in the alleged dwelling or residence of defendant
is of legal age, what the recipient’s relationship with the defendant is, and whether said person comprehends the
significance of the receipt of the summons and his duty to immediately deliver it to the defendant or at least notify
the defendant of said receipt of summons. These matters must be clearly and specifically described in the Return of
Summons.

(4) A Competent Person in Charge

If the substituted service will be done at defendant’s office or regular place of business, then it should be served on a
competent person in charge of the place. Thus, the person on whom the substituted service will be made must be the
one managing the office or business of defendant, such as the president or manager; and such individual must
have sufficient knowledge to understand the obligation of the defendant in the summons, its importance, and the
prejudicial effects arising from inaction on the summons. Again, these details must be contained in the Return.
Remedial Law I| Page 111

DOLE Philippines vs. Quilala


G.R. No. 168723, July 9, 2008

Facts:

All Season Farm Corporation sought the recovery of a sum of money, accounting and damages Dole Philippines, Inc.
(Tropifresh Division) (DOLE) and several of its officers in Makati City RTC. According to DOLE, an alias summons
was served upon it through a certain Marifa Dela Cruz, a legal assistant employed by DOLE Pacific General
Services, Ltd., which is an entity separate from Dole. Thus, DOLE filed a Motion to Dismiss the complaint on the
following grounds: (a) the RTC lacked jurisdiction over the person of Dole due to improper service of summons; (b)
the complaint failed to state a cause of action; (c) All Season was not the real party in interest; and (d) the officers of
Dole cannot be sued in their personal capacities for alleged acts performed in their official capacities as corporate
officers of Dole. This was denied by the RTC.

In the CA, DOLE filed a petition for certiorari contending that the alias summons was not properly served. The
appellate court, however, ruled otherwise. It reasoned that DOLE’s president had known of the service of the alias
summons although he did not personally receive and sign it. It also held that in today’s corporate setup, documents
addressed to corporate officers are received in their behalf by their staff.

In this Petition for Review, DOLE assails the validity of the service of summons on it consequently questioning
whether or not jurisdiction over it was acquired by the RTC. nt All Season, for its part, contends that the trial court
had acquired jurisdiction over petitioner, since petitioner received the alias summons through its president. More so,
petitioner had admitted that it received the alias summons in its Entry of Appearance with Motion for Time filed on
May 5, 2003.

Issue:

Whether or not the service of summons was valid?

Held:

No. Considering that the service of summons was made on a legal assistant, not employed by herein petitioner and
who is not one of the designated persons under Section 11, Rule 14, the trial court did not validly acquire
jurisdiction over petitioner. Well-settled is the rule that service of summons on a domestic corporation is restricted,
limited and exclusive to the persons enumerated in Section 11, Rule 14 of the 1997 Rules of Civil Procedure,
following the rule in statutory construction that expressio unios est exclusio alterius. Service must therefore be made
on the president, managing partner, general manager, corporate secretary, treasurer, or in-house counsel.

Contrary to private respondents claim that it was received upon instruction of the president of the corporation as
indicated in the Officers Return, such fact does not appear in the receiving copy of the alias summons which Marifa
Dela Cruz signed. There was no evidence that she was authorized to receive court processes in behalf of the
president.

HOWEVER, under Section 20 of the same Rule, a defendants voluntary appearance in the action is equivalent to
service of summons. Note that on May 5, 2003, petitioner filed an Entry of Appearance with Motion for Time. It was
not a conditional appearance entered to question the regularity of the service of summons, but an appearance
submitting to the jurisdiction of the court by acknowledging the receipt of the alias summons and praying for
additional time to file responsive pleading. Consequently, petitioner having acknowledged the receipt of the
summons and also having invoked the jurisdiction of the RTC to secure affirmative relief in its motion for additional
time, petitioner effectively submitted voluntarily to the jurisdiction of the RTC.
Remedial Law I| Page 112

Santos vs. PNOC


G. R. No. 170943, September 23, 2008

Facts:

PNOC Exploration Corporation filed a complaint for a sum of money against petitioner Pedro T. Santos, Jr. in the
Regional Trial Court of Pasig City. It sought to collect the amount of P698,502.10 representing petitioner’s unpaid
balance of the car loan advanced to him by respondent when he was still a member of its board of directors. Personal
service of summons to petitioner failed because he could not be located in his last known address despite earnest
efforts to do so. Subsequently, on respondent’s motion, the trial court allowed service of summons by publication
which was published in REMATE, a newspaper of general circulation.

When petitioner failed to file his answer within the prescribed period, respondent moved that the case be set for the
reception of its evidence ex parte which the trial court granted. After it has presented evidence ex parte, the case
was submitted for decision. Days after such submission, Santos filed an Omnibus Motion for Reconsideration and to
Admit Attached Answer. He sought reconsideration of the order stating that alleging that the affidavit of service
submitted by respondent failed to comply with Section 19, Rule 14 of the Rules of Court as it was not executed by
the clerk of court. The motion was denied by the RTC. It held that the rules did not require the affidavit of
complementary service by registered mail to be executed by the clerk of court.

Santos elevated the matter to the CA via petition for certiorari. During the pendency of such petition, the RTC
rendered its decision ordering Santos to pay PNOC. Meanwhile, the CA dismissed the petition. Now in this petition
for review, Santos maintains that the RTC did not acquire jurisdiction over his person due to improper service of
summons.

Issue:

Whether or not service by publication is proper in actions in personam?

Held:

Yes. Petitioner invokes the distinction between an action in rem and an action in personam and claims that
substituted service may be availed of only in an action in rem. Petitioner is wrong. The in rem/in personam
distinction was significant under the old rule because it was silent as to the kind of action to which the rule was
applicable. Because of this silence, the Court limited the application of the old rule to in rem actions only. This has
been changed. The present rule expressly states that it applies [i]n any action where the defendant is designated as
an unknown owner, or the like, or whenever his whereabouts are unknown and cannot be ascertained by diligent
inquiry. Thus, it now applies to any action, whether in personam, in rem or quasi in rem.

Service of summons by publication is proved by the affidavit of the printer, his foreman or principal clerk, or of the
editor, business or advertising manager of the newspaper which published the summons. The service of summons by
publication is complemented by service of summons by registered mail to the defendants last known address. This
complementary service is evidenced by an affidavit showing the deposit of a copy of the summons and order for
publication in the post office, postage prepaid, directed to the defendant by registered mail to his last known
address. The rules, however, do not require that the affidavit of complementary service be executed by the clerk of
court. While the trial court ordinarily does the mailing of copies of its orders and processes, the duty to make the
complementary service by registered mail is imposed on the party who resorts to service by publication.

Moreover, even assuming that the service of summons was defective, the trial court acquired jurisdiction over the
person of petitioner by his own voluntary appearance in the action against him. Petitioner voluntarily appeared in
the action when he filed the Omnibus Motion for Reconsideration and to Admit Attached Answer. This was
equivalent to service of summons and vested the trial court with jurisdiction over the person of petitioner.
Remedial Law I| Page 113

Ong vs. Co
G.R. No. 206653, February 25, 2015

Facts:

Petitioner Yuk Ling Ong, a British-Hong Kong national, and respondent Benjamin Co a Filipino citizen, were
married in the Philippines. Respondent filed a petition for declaration of nullity on the ground of psychological
incapacity before the RTC. Respondent indicated that petitioner’s address was 23 Sta. Rosa Street, Unit B-2
Manresa Garden Homes, Quezon City. The RTC issued summons. In his Server’s Return, process server Rodolfo
Torres, Jr. stated that, on August 1, 2002, substituted service of summons with the copy of the petition was effected
after several futile attempts to serve the same personally on petitioner. The said documents were received by Mr.
Roly Espinosa, a security officer. The RTC found respondent’s marriage with petitioner as void ab initio on the
ground of psychological incapacity under Article 36 of the Family Code. It stated that summons was served on
petitioner on August 1, 2002, but she failed to file her responsive pleading within the reglementary period. The
public prosecutor also stated that there were no indicative facts to manifest collusion. Thus, the RTC concluded that
petitioner was psychologically incapacitated to perform her essential marital obligations.

Consequently, petitioner filed a petition for annulment of judgment under Rule 47 of the Rules of Court before the
CA claiming that she was never notified of the cases filed against her. She prayed that the RTC decision be nullified
on the grounds of extrinsic fraud and lack of jurisdiction. CA found the petition for annulment of judgment to be
devoid of merit. It held that there was no sufficient proof to establish that respondent employed fraud to insure
petitioner’s non-participation in the trial of the aforementioned case. Petitioner moved for reconsideration, but her
motion was denied by the CA. Hence, this petition.

Issue:

Whether or not substituted service of summons was validly availed of?

Held:

No. Jurisdiction over the defendant is acquired either upon a valid service of summons or the defendant's voluntary
appearance in court. If the defendant does not voluntarily appear in court, jurisdiction can be acquired by personal
or substituted service of summons as laid out under Sections 6 and 7 of Rule 14 of the Rules of Court,

The landmark case of Manotoc v. CA (Manotoc) thoroughly discussed the rigorous requirements of a substituted
service of summons, to wit: xxx
(1) Impossibility of Prompt Personal Service
For substituted service of summons to be available, there must be several attempts by the sheriff to personally serve
the summons within a reasonable period of one month which eventually resulted in failure to prove impossibility of
prompt service. "Several attempts" means at least three (3) tries, preferably on at least two different dates.
In addition, the sheriff must cite why such efforts were unsuccessful.

(2) Specific Details in the Return


The sheriff must describe in the Return of Summons the facts and circumstances surrounding the attempted personal
service. The efforts made to find the defendant and the reasons behind the failure must be clearly narrated in detail
in the Return. The date and time of the attempts on personal service, the inquiries made to locate the
defendant, the name/s of the occupants of the alleged residence or house of defendant and all other acts
done, though futile, to serve the summons on defendant must be specified in the Return to justify
substituted service.

(3) Person of Suitable Age and Discretion]


The sheriff must therefore determine if the person found in the alleged dwelling or residence of defendant is of legal
age, what the recipient's relationship with the defendant is, and whether said person comprehends the significance of
the receipt of the summons and his duty to immediately deliver it to the defendant or at least notify the defendant of
Remedial Law I| Page 114

said receipt of summons. These matters must be clearly and specifically described in the Return of
Summons. (Emphases and underscoring supplied)

Here, the the process server immediately opted for substituted service of summons after only two (2) days from the
issuance of the summons. The server’s return utterly lacks sufficient detail of the attempts undertaken by the
process server to personally serve the summons on petitioner. The server simply made a general statement that
summons was effected after several futile attempts to serve the same personally. The server did not state the
specific number of attempts made to perform the personal service of summons; the dates and the corresponding time
the attempts were made; and the underlying reason for each unsuccessful service. He did not explain either if there
were inquiries made to locate the petitioner, who was the defendant in the case. These important acts to serve the
summons on petitioner, though futile, must be specified in the return to justify substituted service. The server’s
return did not describe in detail the person who received the summons, on behalf of petitioner. It simply stated that
the summons was received by Mr. Roly Espinosa of sufficient age and discretion, the Security Officer thereat. It did
not expound on the competence of the security officer to receive the summons.

Also, aside from the server’s return, respondent failed to indicate any portion of the records which would describe
the specific attempts to personally serve the summons. Respondent did not even claim that petitioner made any
voluntary appearance and actively participated in Civil Case No. 02-0306.

Given that the meticulous requirements in Manotoc were not met, the Court is not inclined to uphold the CA’s
denial of the petition for annulment of judgment for lack of jurisdiction over the person of petitioner because there
was an invalid substituted service of summons. Accordingly, the decision in Civil Case No. 02-0306 must be declared
null and void.
Remedial Law I| Page 115

Nation Petroleum Gas vs. RCBC


G.R. No. 183370. August 17, 2015

Facts:

Rizal Commercial Banking Corporation filed against Nation Petroleum Gas and its directors/officers a Complaint for
civil damages arising from estafa in relation to violations of the Trust Receipts Law. Thereafter, Sheriff Leodel N.
Roxas served upon petitioners a copy of the summons, as well as other pertinent documents.

Petitioners filed through counsel a Special Appearance with Motion to Dismiss. They asserted that the trial court did
not acquire jurisdiction over the corporation since the summons was improperly served upon Abante, who is a mere
liaison officer and not one of the corporate officers specifically enumerated in Section 11, Rule 14 of the Rules.
Likewise, the individual petitioners argued that the sheriff and/or process server did not personally approach them
at their respective address as stated in the Complaint. Neither did he resort to substituted service of summons, and
that, even if he did, there was no strict compliance with Section 7, Rule 14 of the Rules.

Respondent countered in its Opposition with Motion to Declare Defendants in Default that there was valid service of
summons upon petitioners. With respect to the corporation, Abante received the summons upon the express
authority and instruction of the corporate secretary, petitioner Melinda Ang. As regards the individual petitioners,
the Sheriffs Report reflects that they were served "at their given addresses, but they refused to acknowledge receipt
thereof."

RTC denied petitioners' motion to dismiss and respondent's motion to declare them in default. Petitioner elevated
the jurisdictional issue to the CA via petition for certiorari and prohibition. The appellate court later dismissed the
petition and denied the motion for reconsideration; hence, this petition.

Issue:

Whether or not the court acquired jurisdiction over the person of the (a) corporation and (b) individual officers of the
corporation?

Held:

(a) As to the corporation – YES. When the defendant is a domestic corporation like herein petitioner, service of
summons may be made only upon the persons enumerated in Section 11, Rule 14 of the Rules. The foregoing
notwithstanding, there was a valid and effective service of summons upon petitioner corporation through its liaison
officer who acted as the agent of the corporate secretary. Abante, in so receiving the summons, did so in
representation of Ang who, as corporate secretary, is one of the officers competent under the Rules of Court to
receive summons on behalf of a private juridical person. Thus, while it may be true that there was no direct,
physical handing of the summons to Ang, the latter could at least be charged with having constructively received the
same, which in Our view, amounts to a valid service of summons.

(b) As to the officers – YES. Despite improper service of summons upon their persons, the individual petitioners are
deemed to have submitted to the jurisdiction of the court through their voluntary appearance. The second sentence
of Section 20 Rule 14 of the Rules that "[t]he inclusion in a motion to dismiss of other grounds aside from lack of
jurisdiction over the person of the defendant shall not be deemed a voluntary appearance" clearly refers
to affirmative defenses, not affirmative reliefs.

In the present case, the individual petitioners prayed, among others, for the following: (1) discharge of the writ of
attachment on their properties; (2) denial of the motion to declare them in default; (3) admission of the
Comment/Opposition (to the motion to declare them in default) filed on December 19, 2006; and (4) denial of
respondent’s motion to strike off from the records (their opposition to the motion to declare them in default). By
seeking affirmative reliefs from the trial court, the individual petitioners are deemed to have voluntarily submitted
to the jurisdiction of said court. A party cannot invoke the jurisdiction of a court to secure affirmative relief against
his opponent and after obtaining or failing to obtain such relief, repudiate or question that same jurisdiction.
Remedial Law I| Page 116

Green Star Express vs. Nissin Universal Robina Corp.


G.R. No. 181517 July 6, 2015

Facts:

A Mitsubishi L-300 van which Universal Robina Corporation (URC) owned figured in a vehicular accident with
petitioner Green Star Express, Inc.' s (Green Star) passenger bus, resulting in the death of the van's driver. Thus,
the bus driver, petitioner Fruto Sayson, Jr., was charged with the crime of reckless imprudence resulting in
homicide.

Thereafter, Green Star sent a demand letter to respondent URC for the repair of its passenger bus amounting. URC
denied any liability therefore and argued that the criminal case shall determine the ultimate liabilities of the
parties. Thereafter, the criminal case was dismissed without prejudice, due to insufficiency of evidence. Sayson and
Green Star then filed a complaint for damages against URC before the RTC of San Pedro, Laguna. Francis Tinio,
one of URC's employees, was the one who received the summons. URC filed a Motion to Dismiss claiming lack of
jurisdiction due to improper service. RTC denied URC's motion to dismiss. It ruled that there was substantial
compliance because there was actual receipt of the summons by URC. CA reversed the RTC ruling. Aggrieved,
Green Star and Sayson moved for reconsideration, but the same was denied. Hence, this petition.

Issue:
Whether or not the summons was properly served on URC, vesting the trial court with jurisdiction?

Held:

No. It is a well-established rule that the rules on service of summons upon a domestic private juridical entity must
be strictly complied with. Otherwise, the court cannot be said to have acquired jurisdiction over the person of the
defendant.

Section 11. Service upon domestic private juridical entity. – When the defendant is a corporation, partnership or
association organized under the laws of the Philippines with a juridical personality, service may be made on the
president, managing partner, general manager, corporate secretary, treasurer, or in-house counsel.

Service must, be made only on the person expressly listed in the rules. Here, Tinio, a, member of URC’s accounting
staff, received the summons on January 22, 2004. Green star claims that it was received upon instruction of
Junadette Avedillo. The general manager of the corporation. Such fact, however, does not appear in the Sheriff’s
Return. The Return did not even state whether Avedillo was present at the time the summons was received by
Tinio, the supposed assistant manager. Green Star further avers that the sheriff tendered the summons, but
Avedillo simply refused to sign and receive the same. She then allegedly instructed Tinio to just receive it in her
behalf. However, Green Star never presented said sheriff as witness during the hearing of URC’s motion to dismiss
to attest to said claim. And while the sheriff executed an affidavit which appears to support such allegation, the
same was likewise not presented as evidence. It was only when the case was already before the CA that said
affidavit first surfaced. Since the service of summons was made on a cost accountant, which is not one of the
designated persons under Section 11 of Rule 14, the trial court did not validly acquire jurisdiction over URC,
although the corporation may have actually received the summons. To rule otherwise will be an outright
circumvention of the rules, aggravating further the delay in the administration of justice.
Remedial Law I| Page 117

Guy vs. Gacott


G.R. No. 206147. January 13, 2016
Facts:

Atty. Glenn Gacott purchased two brand new transreceivers from Quantech Systems Corporation (QSC) in Manila
through its employee Rey Medestomas. Due to major defects, Gacott personally returned the transreceivers to QSC
and requested that they be replaced. Medestomas received the returned transreceivers and promised to send him
the replacement units within two (2) weeks. Time passed and Gacott did not receive the replacement units as
promised. QSC informed him that there were no available units and that it could not refund the purchased price.
Gacott then filed a Complaint for Damages before the RTC against QSC. Summons was served upon QSC and
Medestomas, after which they filed their Answer, verified by Medestomas himself and a certain Elton Ong. The RTC
ruled in favor of Gacott. The decision became final. Subsequently, a Writ of Execution was issued. During the
execution stage, Gacott learned that QSC was not a corporation, but was in fact a general partnership registered
with the SEC. In the articles of partnership, Michael Guy was appointed as General Manager of QSC. The Sheriff
then went to DOTC-LTO to check whether QCS, Medestomas and Guy had personal properties registered therein.
Upon learning that Guy had vehicles registered in his name, Gacott instructed the sheriff to proceed with the
attachment of one of the motor vehicles of Guy. The sheriff attached Guy’s vehicle. Guy then filed a Motion to Lift
Attachment Upon Personalty, arguing that he was not a judgment debtor and, therefore, his vehicle could not be
attached. The RTC denied Guy’s motion citing Section 21 of the Corporation Code. Since QSC is an ostensible
corporation, Guy is considered as a general partner and he may be held jointly and severally liable with QSC. Guy
filed a Motion for Reconsideration arguing that he was neither impleaded as a defendant nor validly served with
summons and, thus, the trial court did not acquire jurisdiction over his person. On appeal to the CA, the CA ruled
that Guy, being a partner in QSC, was bound by the summons served upon QSC.

Issue:

Whether or not there was a valid service of summons over QSC?

Held:

No. Under Section 11, Rule 14 of the 1997 Revised Rules of Civil Procedure, when the defendant is a corporation,
partnership or association organized under the laws of the Philippines with a juridical personality, the service of
summons may be made on the president, managing partner, general manager, corporate secretary, treasurer, or in-
house counsel. Jurisprudence is replete with pronouncements that such provision provides an exclusive enumeration
of the persons authorized to receive summons for juridical entities. The records of this case reveal that QSC was
never shown to have been served with the summons through any of the enumerated authorized persons to receive
such, namely: president, managing partner, general manager, corporate secretary, treasurer or in-house counsel.
Service of summons upon persons other than those officers enumerated in Section 11 is invalid. Even substantial
compliance is not sufficient service of summons.

Nevertheless, while proper service of summons is necessary to vest the court jurisdiction over the defendant, the
same is merely procedural in nature and the lack of or defect in the service of summons may be cured by the
defendant’s subsequent voluntary submission to the court’s jurisdiction through his filing a responsive pleading such
as an answer. In this case, it is not disputed that QSC filed its Answer despite the defective summons. Thus,
jurisdiction over its person was acquired through voluntary appearance.

Note: Although the trial court acquired jurisdiction over QSC, this did not extend to the person of Guy
insofar as holding him solidarily liable with the partnership. A suit against the partnership is not
necessarily a suit impleading each and every partner A partnership is a juridical entity that has a
distinct and separate personality from the persons composing it. Since Guy was not impleaded in the
case, the judgment rendered cannot bind nor prejudice him. Therefore, it was improper to levy upon
his property. Moreover, even if he was properly impleaded, his liability is only subsidiary pursuant to
Art. 1816 of the NCC.
Remedial Law I| Page 118

Sunrise Garden Corporation vs. Court of Appeals


G.R. No. 158836. September 30, 2015.

Facts:

Sunrise Garden and the Sangguniang Panlungsod of Antipolo City agreed on a joint undertaking to construct a city
road connecting four barangays in Antipolo City. Sunrise Garden’s contractor began to position its construction
equipment. However, armed guards, allegedly hired by Hardrock Aggregates, Inc., prevented Sunrise Garden’s
contractor from using an access road to move the construction equipment. Sunrise Garden filed a Complaint for
damages with prayer for temporary restraining order and writ of preliminary injunction against Hardrock
Aggregates, Inc. The RTC issued the Writ of Preliminary Injunction. While the Complaint was pending, informal
settlers started to encroach on the area of the proposed city road. Sunrise Garden then filed a Manifestation seeking
the amendment of the Writ to include any and all persons or group of persons from interfering, preventing or
obstructing all of its contractors, equipment personnel and representatives in proceeding with the construction of
the city road. The RTC then issued an Amended Writ of Preliminary Injunction. The construction of the city road
then continued. However, armed guards of K-9 Security Agency, allegedly hired by First Alliance Real Estate
Development, Inc., blocked Sunrise Garden’s contractor’s employees and prevented them from proceeding with the
construction. Sunrise Garden then filed a Motion to cite K-9 Security Agency in contempt. K-9 Security Agency,
joined by First Alliance opposed the Motion to cite them in contempt, raising the defense of lack of jurisdiction over
their persons, since they were not bound by the Amended Writ of Preliminary Injunction. Sunrise Garden filed an
Ex parte Motion to require K-9 Security Agency and First Alliance Real Estate Development, Inc. to comply with the
Amended Writ of Preliminary Injunction. K-9 and First Alliance filed a Motion for Reconsideration reiterating their
arguments that since the trial court did not acquire jurisdiction over them, the Writ of Preliminary Injunction could
not be enforced against them. The MR was denied which prompted First Alliance to file a Petition for Certiorari
with a prayer for temporary restraining order with the Court of Appeals. The CA granted the TRO and it also issued
a Writ of Preliminary Injunction against the RTC Order (for First Alliance to comply with the Amended Writ of
Injunction). Sunrise Garden and the Republic filed a petition before the SC to assail the decision of the CA. While
Sunrise Garden and the Republic’s petitions before the SC were pending, the CA also annulled the Amended Writ of
Preliminary Injunction issued by the trial court.

Issue:

Whether or not the trial court acquired jurisdiction over First Alliance Real Estate Development, Inc. to be bound by
the Amended Writ of Preliminary Injunction?

Held:

No. First Alliance was not a party to the Complaint for Damages filed by Sunrise Garden before the RTC. Both
Sunrise Garden and the Republic did not refute the primary issue of lack of jurisdiction over First Alliance; this is
an admission that the trial court did not acquire jurisdiction over the latter.

On the contention that the trial court acquired jurisdiction because of First Alliance’s voluntary appearance, the
Supreme Court ruled that voluntary appearance in court may not always result in submission to the jurisdiction of a
court. When a party makes a special appearance to challenge, among others, the court’s jurisdiction over his person,
he cannot be considered to have submitted to its authority.

(1) Special appearance operates as an exception to the general rule on voluntary appearance;

(2) Accordingly, objections to the jurisdiction of the court over the person of the defendant must be explicitly made,
i.e., set forth in an unequivocal manner; and

(3) Failure to do so constitutes voluntary submission to the jurisdiction of the court, especially in instances where a
pleading or motion seeking affirmative relief is filed and submitted to the court for resolution.

The appearance of First Alliance and K-9 Security Agency should not be deemed as a voluntary appearance because
it was for the purpose of questioning the jurisdiction of the trial court. The records of this case show that the defense
of lack of jurisdiction was raised at the first instance and repeatedly argued by K-9 Security Agency and respondent
Remedial Law I| Page 119

First Alliance Real Estate Development, Inc. in their pleadings.
Remedial Law I| Page 120

Rule 15: Motions

Acampado vs. Cosmilla


G.R. No. 198531, September 28, 2015

Facts:

Spouses Cosmilla filed a Petition for the Declaration of the Nullity of Document against petitioners (Ethel
Acampado, Emmie Acampado, Elvie Acampado, Earlyn Acampado and Evelyn Acampado) before the RTC of Kalibo,
Aklan, Branch 6. Spouses Cosmilla alleged that the sale of their share on the subject property was effected thru a
forged Special Power of Attorney (SPA) and is therefore null and void. After trial on the merits, the RTC rendered a
Decision dismissing the complaint of the respondents for failure to prove by preponderance of evidence that the
signatures of the respondents in the SPA were forged. Respondents filed a Motion for Reconsideration. The
respondents, however, failed to comply with the requirement of notice of hearing as required under Sections 4 and 5
of Rule 15 of the Revised Rules of Court, the court a quo denied the Motion for Reconsideration. The CA affirmed the
decision but later on reversed its decision in relaxing the rules on notice and hearing required in the motions.

Issue:

Whether or not the Motion for Reconsideration is a contentious motion that needs to comply with the required notice
and hearing and service to the adverse party?

Held:

Yes. The Motion for Reconsideration is a contentious motion that needs to comply with the required notice and
hearing and service to the adverse party as mandated by Sections 4 and 6, Rule 15 of Revised Rules of Court:

It is important, however, to note that these doctrines refer exclusively to a motion, since a motion invariably
contains a prayer, which the movant makes to the court, which is to repeat usually in the interest of the adverse
party to oppose and in the observance of due process, the other party must be given the opportunity to oppose the
motion. In keeping with the principles of due process, therefore, a motion which does not afford the adverse party
the chance to oppose it should simply be disregarded. Failure to comply with the required notice and hearing is a
fatal defect that is deleterious to respondent’s cause.

As born by the records, no notice of hearing was appended to the Motion for Reconsideration of the respondent. As
discussed above, a motion for reconsideration is a litigated motion where the right of the adverse party will be
affected by its admission. The adverse party in this case had the right to resist the motion because it may result to
the reversal of a prior favorable decision. The proof of service was therefore indispensable in order to avoid surprises
on the opposite party. The absence thereof is fatal to the motion.

It bears stressing that a motion without notice and hearing, is pro forma, a mere scrap of paper that cannot be acted
by the court. It presents no question that the court can decide. The court has no reason to consider it and the clerk
has no right to receive it. Indisputably, any motion that does not contain proof of service and notice to the adverse
party is not entitled to judicial cognizance.
Remedial Law I| Page 121

Laude vs. Ginez-Jabalde


G.R. No. 217456. November 24, 2015

Facts:

Jeffrey “Jennifer” Laude (Jennifer) was killed at the Celzone Lodge on Ramon Magsaysay Drive in Olongapo City
allegedly by 19-year-old US Marine Joseph Pemberton. A Complaint for murder was filed by Jennifer’s sibling,
Marilou S. Laude, against Pemberton before the Olongapo City Office of the City Prosecutor. Pemberton was
detained in Camp Aguinaldo, the general headquarters of the Armed Forces of the Philippines. Public Prosecutor
filed an Information for murder against Pemberton before the Regional Trial Court in Olongapo City. A warrant of
arrest against Pemberton. Pemberton surrendered personally to Judge Roline M. Ginez-Jabalde and he was then
arraigned.

On the same day, Marilou S. Laude filed an Urgent Motion to Compel the Armed Forces of the Philippines to
Surrender Custody of Accused to the Olongapo City Jail and a Motion to Allow Media Coverage. The [M]otion was
[scheduled] for hearing on December 22, 2014, at 2 p.m. According to petitioners, they were only able to serve the
Motion on Pemberton’s counsel through registered mail. In any case, they claim to have also “furnished a copy of the
[M]otion personally . . . at the hearing of the [M]otion. On December 23, 2014, Judge Ginez-Jabalde denied
petitioners’ Urgent Motion for lack of merit and the Motion for Reconsideration was also denied.

Petitioners argue that respondent Judge committed grave abuse of discretion tantamount to an excess or absence of
jurisdiction when she dismissed the Urgent Motion to Compel the Armed Forces of the Philippines to Surrender
Custody o[f] Accused to the Olongapo City Jail [based] on mere technicalities. In particular, they argue that the
three-day rule on motions under 1997 Rules of Court is not absolute, and should be liberally interpreted when a case
is attended by exigent circumstances. Petitioners advance that the rationale behind the three-day notice rule is
satisfied when there is an opportunity to be heard, which was present in this case since Pemberton’s counsel and the
Public Prosecutor were present in the hearing of the two Motions filed by petitioners. Petitioners allege that the
court noted their attendance, and were able to make comments during the December 22, 2014 Motion hearing. They
assert that the rights of Pemberton were not compromised in any way earliest possible date.

Issue:

Whether or not the RTC is correct in dismissing the urgent motion filed by the petitioner on the ground of non-
compliance with the three-day notice rule?

Held:

Yes. The failure of petitioners to comply with the three-day notice rule is unjustified. Rule 15, Section 4 of the Rules
of Court clearly makes it a mandatory rule that the adverse party be given notice of hearing on the motion at least
three days prior. Failure to comply with this notice requirement renders the motion defective consistent with
protecting the adverse party’s right to procedural due process. As an integral component of procedural due process,
the three-day notice required by the Rules is not intended for the benefit of the movant. Rather, the requirement is for
the purpose of avoiding surprises that may be sprung upon the adverse party, who must be given time to study and
meet the arguments in the motion before a resolution by the court. Principles of natural justice demand that the right
of a party should not be affected without giving it an opportunity to be heard.
Remedial Law I| Page 122

Rule 16: Motion to Dismiss (Omnibus Motion)

De Guzman vs. Ochoa


G.R. No. 169292. April 13, 2011

Facts:

Respondent spouses Cesar Ochoa and Sylvia Ochoa, through respondent Araceli Azores, ostensibly acting as
attorney in fact, commenced in the Regional Trial Court (RTC) in Pasig City an action seeking the annulment of
contract of mortgage, foreclosure sale, certificate of sale and damages. Petitioners, as defendants in Civil Case No.
68896, filed a motion to dismiss, alleging the sole ground that the complaint did not state a cause of action. The
petitioners’ motion to dismiss was formally opposed by the private respondents. On December 16, 2002, the
respondent RTC Judge denied petitioners’ motion to dismiss and at the same time set for pretrial conference,
directing the parties to submit their respective pre-trial briefs. The petitioners filed a second motion to dismiss,
alleging that the certification against forum shopping attached to the complaint was not executed by the principal
parties (plaintiffs) in violation of Sec. 5, Rule 7, 1997 Rules of Civil Procedure, rendering the complaint fatally
defective and thus dismissible.

The private respondents opposed the second motion to dismiss. The respondent RTC Judge issued her first assailed
order, denying the second motion to dismiss. The petitioners filed their motion for reconsideration, but the
respondent RTC Judge denied the motion through her second assailed order. Aggrieved, petitioners elevated the
order of denial to the CA via a petition for certiorari contending that the RTC should have dismissed the complaint
motu proprio since it was fatally defective. The CA denied the petition for lack of merit.

Issues:
(1) Whether or not the petitioner may be allowed to file second motion to dismiss for defective verification and
certification of forum shopping?

(2) Whether or not an order denying a second motion to dismiss may be assailed by a petition for certiorari?

Held:

(1) No. Section 8, Rule 15 of the Rules of Court defines an omnibus motion as a motion attacking a pleading,
judgment or proceeding. A motion to dismiss is an omnibus motion because it attacks a pleading, that is, the
complaint. For this reason, a motion to dismiss, like any other omnibus motion, must raise and include all objections
available at the time of the filing of the motion because under Section 8, “all objections not so included shall be
deemed waived.” As inferred from the provision, only the following defenses under Section 1, Rule 9, are excepted
from its application: [a] lack of jurisdiction over the subject matter; [b] there is another action pending between the
same parties for the same cause (litis pendentia); [c] the action is barred by prior judgment (res judicata); and [d] the
action is barred by the statute of limitations or prescription. In the case at bench, the petitioners raised the ground
of defective verification and certification of forum shopping only when they filed their second motion to dismiss,
despite the fact that this ground was existent and available to them at the time of the filing of their first motion to
dismiss.

Absent any justifiable reason to explain this fatal omission, the ground of defective verification and certification of
forum shopping was deemed waived and could no longer be questioned by the petitioners in their second motion to
dismiss.

(2) No. An order denying a motion to dismiss is an interlocutory order which neither terminates the case nor finally
disposes of it, as it leaves something to be done by the court before the case is finally decided on the merits. As such,
the general rule is that the denial of a motion to dismiss cannot be questioned in a special civil action for certiorari
which is a remedy designed to correct errors of jurisdiction and not errors of judgment. Therefore, an order denying
a motion to dismiss may only be reviewed in the ordinary course of law by an appeal from the judgment after trial.
The ordinary procedure to be followed in such cases is to file an answer, go to trial, and if the decision is adverse,
reiterate the issue on appeal from the final judgment. Only in exceptional cases where the denial of the motion to
dismiss is tainted with grave abuse of discretion that the Court allows the extraordinary remedy of certiorari.
Remedial Law I| Page 123

Remedial Law I| Page 124

Rule 17: Dismissal of Actions

Lim Teck Chuan vs. Uy


G.R. No. 155701. March 11, 2015
Facts:

Antonio Lim Tanhu was the original owner of the lot which is the subject matter of the controversy. Allegedly, he
sold the lot to Spouses Cabansag; then Spouses Cabansag sold the same to Serafin Uy (respondent). Serafin then
filed a petition before the RTC praying for the issuance of a new owner’s duplicate TCT in his name. The petition
was initially granted but subsequently nullified because Lim Teck Chuan (petitioner) filed his Opposition alleging
that he is one of the 6 legitimate descendants of Antonio and that the original TCT was not lost and has always been
in his custody. In the meantime, a certain Henry Lim sold the same lot to Leopolda Cecilio by virtue of an an
Affidavit of Sole Adjudication/Settlement of the Estate of Antonio Lim Tanhu with Deed of Sale.

Serafin then filed a Complaint for Quieting of Title impleading Leopolda, Henry and the petitioner. Leopolda
averred that she is a buyer in good faith and for value. Petitioner set up a counterclaim against Serafin and a
cross-claim against Lopeolda contending that the property was never transferred and encumbered to any person
during Antonio’s lifetime.

During the proceedings, both and Serafin and Leopolda entered into an amicable settlement and they both filed a
Joint Motion to Dismiss (MTD) on the main ground that the case had become moot and academic since Serafin’s title
to to the subject lot had been allegedly quieted. Petitioner opposed the MTD on the ground that he was not included
in the settlement. The RTC, however, granted the MTD and it also dismissed petitioner’s counterclaim and cross-
claim.

Petitioner filed directly with the SC a petition for review under Rule 45.

Issue:

Whether or not petitioner’s counterclaim or cross-claim could be prosecuted in the same action despite the dismissal
of the main complaint?

Held:

Yes. Section 2 of Rule 17 provides that:

xxx If a counterclaim has been pleaded by a defendant prior to the service upon him of the plaintiff’s motion for
dismissal, the dismissal shall be limited to the complaint. The dismissal shall be without prejudice to the right of the
defendant to prosecute his counterclaim in a separate action unless within fifteen (15) days from notice of the motion
he manifests his preference to have his counterclaim resolved in the same action.

The RTC erred when it dismissed the case when the present rules state that the dismissal shall be limited only to
the complaint. A dismissal of an action is different from a mere dismissal of the complaint. For this reason, since
only the complaint and not the action is dismissed, the defendant in spite of said dismissal may still prosecute his
counterclaim in the same action. Citing Pinga v. Heirs of German Santiago, the Court said that the dismissal of the
complaint does not necessarily result to the dismissal of the counterclaim.

In the instant case, the petitioner’s preference to have his counterclaim (and cross-claims) be prosecuted in the same
action was timely manifested.
Remedial Law I| Page 125

Ching vs. Cheng


G.R. No. 175507. October 8, 2014

Facts:

Antonio Ching owned several businesses and properties, among which was Po Wing Properties, Inc. It is also alleged
that while he was unmarried, he had children from two women. Ramon Ching alleged that he was the only child of
Antonio Ching with his common-law wife, Lucina Santos. Joseph Cheng and Jaime Cheng, on the other hand, claim
to be Antonio Ching’s illegitimate children with his housemaid, Mercedes Igne. When Antonio Ching was murdered,
Ramon Ching allegedly induced Mercedes Igne and her children, Joseph and Jaime, to sign an agreement and
waiver to Antonio Ching’s estate in consideration of ₱22.5 million. Mercedes Igne’s children alleged that Ramon
Ching never paid them. Ramon Ching allegedly executed an affidavit of settlement of estate, naming himself as the
sole heir and adjudicating upon himself the entirety of Antonio Ching’s estate. After a year of investigating Antonio
Ching’s death, the police found Ramon Ching to be its primary suspect. Information was filed against him, and a
warrant of arrest was issued.

First Case: Joseph, Jaime and Mercedes (the Chengs) filed a complaint for declaration of nullity of titles against
Ramon Ching before the Regional Trial Court of Manila. Po Wing Properties was also impleaded in an amended
complaint and Lucina was also allowed to intervene. After the responsive pleadings had been filed, Po Wing
Properties filed a motion to dismiss on the ground of lack of jurisdiction of the subject matter. The RTC granted the
Motion to Dismiss.

Second Case: The Chengs and Lucina Santos filed a complaint for "Annulment of Agreement, Waiver, Extra-
Judicial Settlement of Estate and the Certificates of Title Issued by Virtue of Said Documents with Prayer for
Temporary Restraining Order and Writ of Preliminary Injunction" against Ramon Ching and Po Wing Properties.
The Chengs and Lucina Santos filed a motion to dismiss their complaint in the second case, praying that it be
dismissed without prejudice. The RTC issued an order granting the motion to dismiss. The dismissal was made
without prejudice.

Ching and Po Wing Properties filed a motion for reconsideration. They argue that the dismissal should have been
with prejudice under the "two dismissal rule" of Rule 17, Section 1 of the 1997 Rules of Civil Procedure, in view of
the previous dismissal of the first case.

Third Case: During the pendency of the MR, the Chengs and Lucina Santos filed a complaint for "Disinheritance
and Declaration of Nullity of Agreement and Waiver, Affidavit of Extra-Judicial Agreement, Deed of Absolute Sale,
and Transfer Certificates of Title with Prayer for TRO and Writ of Preliminary Injunction" against Ramon Ching
and Po Wing Properties.

Branch 6 issued an omnibus order resolving both the motion for reconsideration in the second case and the motion to
dismiss in the third case. The trial court denied the motion for reconsideration and the motion to dismiss, holding
that the dismissal of the second case was without prejudice and, hence, would not bar the filing of the third case.

The Court of Appeals rendered the decision in the first certiorari case dismissing the petition. The appellate court
ruled that Ramon Ching and Po Wing Properties’ reliance on the "two-dismissal rule" was misplaced since the rule
involves two motions for dismissals filed by the plaintiff only. In this case, it found that the dismissal of the first
case was upon the motion of the defendants, while the dismissal of the second case was at the instance of the
plaintiffs.

Upon the denial of their MR, Ramon Ching and Po Wing Properties filed this present petition for review.

Issue:

Whether or not the two-dismissal rule applies in this case?


Remedial Law I| Page 126

Held:

No. As a general rule, dismissals under Section 1 of Rule 17 are without prejudice except when it is the second time
that the plaintiff caused its dismissal. Accordingly, for a dismissal to operate as an adjudication upon the merits, i.e,
with prejudice to the re-filing of the same claim, the following requisites must be present:
(1) There was a previous case that was dismissed by a competent court;
(2) Both cases were based on or include the same claim;
(3) Both notices for dismissal were filed by the plaintiff; and
(4) When the motion to dismiss filed by the plaintiff was consented to by
the defendant on the ground that the latter paid and satisfied all the claims of the former.

The purpose of the "two-dismissal rule" is "to avoid vexatious litigation." When a complaint is dismissed a second
time, the plaintiff is now barred from seeking relief on the same claim.

The dismissal of the second case was without prejudice in view of the "two-dismissal rule". Here, the first case was
filed as an ordinary civil action. It was later amended to include not only new defendants but new causes of action
that should have been adjudicated in a special proceeding. A motion to dismiss was inevitably filed by the
defendants on the ground of lack of jurisdiction. The dismissal of the first case was done at the instance of the
defendant under Rule 16, Section 1(b) of the Rules of Civil Procedure.

Thus, when respondents filed the second case, they were merely refiling the same claim that had been previously
dismissed on the basis of lack of jurisdiction. When they moved to dismiss the second case, the motion to
dismiss can be considered as the first dismissal is at the plaintiff’s instance.
Remedial Law I| Page 127

Rule 18: Pre-Trial


People vs. Perez
G.R. No. 142556. February 5, 2003
Facts:
The prosecution alleges that, on January 17, 1997, about noontime, in Sitio Baco, Barangay Macarang, Palauig,
Zambales, six-year old Mayia Ponseca(victim) was walking along Sulok on her way to her house in Sitio Camiling
when appellant Jesus Sebunga Perez approached her. Appellant introduced himself as Johnny and immediately
afterwards, strangled her neck and boxed her abdomen. Still in shock, Mayia fell down. At that point, a dog arrived
and barked at them. Thereafter, the appellant raped the victim. The accused-appellant after being arrested was
charged for the crime of rape. During pre-trial the prosecution and defense stipulated on the following facts:
1. The identity of the accused;
2. The accused was at the time of the incident in the vicinity thereof;
3. The victim in this case, Mayia P. Ponseca, was born on 23 May 1990 as evidenced by her birth certificate;
4. That after the incident, the child was subjected to a medico-legal examination to which a medico-legal certificate
was issued by Dr. Editha Divino.

The prosecution marked in evidence the birth certificate of the victim Mayia O. Ponseca as Exhibit A, and the
medico-legal certificate issued by Dr. Editha Divino as Exhibit B. As a defense, appellant avers during the incident,
he left the fishpond and walked home to Barangay Alwa which was about thirty meters from the fishpond. After
trial, the trial Court convicted him for the crime of statutory rape. Hence, this automatic review.

In his Reply Brief, appellant contends that even assuming that the guilt of appellant has been proven beyond
reasonable doubt, the trial court erred in imposing the death penalty. Appellant maintains that the death penalty
cannot be imposed on him for failure of the prosecution to prove Mayia’s age by independent evidence. Appellant
points out that while Mayia’s birth certificate was duly marked during the pre-trial, it was not presented and
identified during the trial. Appellant asserts that Mayia’s minority must not only be specifically alleged in the
Information but must also be established beyond reasonable doubt during the trial.

Issue:

Whether or not the stipulation during the pre-trial on the victim’s age is binding on the accused?

Held:
Yes. The Court ruled that, during the pre-trial, the prosecution marked in evidence Mayia’s birth certificate as
Exhibit A. The prosecution submitted its Offer of Evidence which included Exhibit A, a certified true copy of Mayia’s
birth certificate. The trial court admitted Exhibit A without any objection from the defense.
The purpose of pre-trial is to consider the following: (a) plea bargaining; (b) stipulation of facts; (c) marking for
identification of evidence of the parties; (d) waiver of objections to admissibility of evidence; (e) modification of the
order of trial if the accused admits the charge but interposes lawful defenses; and (f) such matters as will promote a
fair and expeditious trial of the criminal and civil aspects of the case. Facts stipulated and evidence admitted during
pre-trial bind the parties. Section 4, Rule 118 of the Revised Rules of Criminal Procedure provides:

SEC. 4. Pre-trial order. - After the pre-trial conference, the court shall issue an order reciting the actions taken, the
facts stipulated, and evidence marked. Such order shall bind the parties, limit the trial to matters not disposed
of, and control the course of the action during the trial, unless modified by the court to prevent manifest injustice.

Moreover, Mayia herself testified in open court as to her age. During the trial on December 15, 1998, which was
about twenty-three (23) months after the rape incident occurred on January 17, 1997, Mayia testified on cross-
examination that she was 8 years old last May 23. Thus, by deduction, since Mayia was born on May 23, 1990 as
shown in her birth certificate, she was about six (6) years and seven (7) months old on January 17, 1997, the day the
crime took place. The Court ruled that the prosecution has indisputably proven that Mayia was below seven years
Remedial Law I| Page 128

old at the time appellant raped her. Thus, the trial court was correct in imposing the death penalty on appellant.
Under Article 335 of the Revised Penal Code, as amended by Section 11 of Republic Act No. 7659, the death penalty
shall be imposed if the crime of rape is committed against a child below seven (7) years old. Mayia was six (6) years
and seven (7) months old when appellant raped her.
Remedial Law I| Page 129

Rule 19: Intervention

Office of the Ombudsman vs. Sison


G.R. No. 185954 February 16, 2010
Facts:

The Isog Han Samar Movement, represented by Fr. Noel Labendia of the Diocese of Calbayog, Catbalogan, Samar,
filed a letter-complaint addressed to Ombudsman accusing Governor Tan and other local public officials of the
Province of Samar, including Sison, the Provincial Budget Officer, of highly anomalous transactions entered into by
them amounting to several millions of pesos. The letter-complaint stemmed from the audit conducted by the Legal
and Adjudication Office (LAO), Commission on Audit (COA), which found, that various purchases totaling PhP 29.34
million went without proper bidding procedures and documentations; that calamity funds were expended without a
State of Calamity having been declared by the President; and that purchases for rice, medicines, electric fans, and
cement were substantially overpriced.mThe Office of the Ombudsman found basis to proceed with the
administrative case against the impleaded provincial officials of Samar. In his counter-affidavit, Sison denied the
accusations contained in the letter-complaint and claimed his innocence on the charges. He asserted that his
function is limited to the issuance of a certification that an appropriation for the requisition exists, that the
corresponding amount has been obligated, and that funds are available. He did not, in any way, vouch for the
truthfulness of the certification issued by the requesting parties.

The Office of the Ombudsman rendered a Decision, finding Sison and several other local officials of the Province of
Samar guilty of grave misconduct, dishonesty, and conduct prejudicial to the best interest of the service and
dismissing him from service. Sison appealed to the CA via a Petition for Review under Rule 43. CA rendered a
decision reversing and setting aside the decision of the Office of the Ombudsman against Sison. It held that the
Office of the Ombudsman failed to adduce substantial evidence in order to convict Sison. Moreover, it reasoned that
Sison’s responsibility as Provincial Budget Officer was to ensure that appropriations exist in relation to the
emergency purchase being made and that he had no hand or discretion in characterizing a particular purchase as
emergency in nature. Hence, he cannot be held administratively liable for simply attesting to the existence of
appropriations for a certain purpose, save if such certification is proved to be false.

The Office of the Ombudsman filed an Omnibus Motion for Intervention and to Admit Attached Motion for
Reconsideration, which was subsequently denied by the CA. Hence, this petition.

Issue:

Whether or not the Office of the Ombudsman may be allowed to intervene and seek reconsideration of the adverse
decision rendered by the CA?

Held:

No. To warrant intervention under Rule 19 of the Rules of Court, two requisites must concur: (1) the movant has a
legal interest in the matter in litigation; and (2) intervention must not unduly delay or prejudice the adjudication of
the rights of the parties, nor should the claim of the intervenor be capable of being properly decided in a separate
proceeding. The interest, which entitles one to intervene, must involve the matter in litigation and of such direct and
immediate character that the intervenor will either gain or lose by the direct legal operation and effect of the
judgment.

In support of its argument that it has legal interest, the Office of the Ombudsman cites Philippine National Bank v.
Garcia, Jr. In the said case, PNB imposed upon its employee, Garcia, the penalty of forced resignation for gross
neglect of duty. On appeal, the CSC exonerated Garcia from the administrative charges against him. However,
Office of the Ombudsman cannot use Garcia to support its intervention in the appellate court for the following
reasons:
Remedial Law I| Page 130

First, Sison was not exonerated from the administrative charges against him, and was, in fact, dismissed for grave
misconduct, dishonesty, and conduct prejudicial to the best interest of the service by the Office of the Ombudsman in
the administrative case, OMB-C-A-05-0051-B. Thus, it was Sison who appealed to the CA being, unquestionably, the
party aggrieved by the judgment on appeal.

Second, the issue here is the right of the Office of the Ombudsman to intervene in the appeal of its decision, not its
right to appeal.

And third, Garcia should be read along with Mathay, Jr. v. Court of Appeals and National Appellate Board of the
National Police Commission v. Mamauag (Mamauag), in which this Court qualified and clarified the exercise of the
right of a government agency to actively participate in the appeal of decisions in administrative cases.

Clearly, the Office of the Ombudsman is not an appropriate party to intervene in the instant case. It must remain
partial and detached. More importantly, it must be mindful of its role as an adjudicator, not an advocate.

Furthermore, the Rules provides explicitly that a motion to intervene may be filed at any time before rendition of
judgment by the trial court. In the instant case, the Omnibus Motion for Intervention was filed only on July 22,
2008, after the Decision of the CA was promulgated on June 26, 2008. The Office of the Ombudsman was aware of
the appeal filed by Sison. The Rules of Court provides that the appeal shall be taken by filing a verified petition for
review with the CA, with proof of service of a copy on the court or agency a quo. Clearly, the Office of the
Ombudsman had sufficient time within which to file a motion to intervene. As such, its failure to do so should not
now be countenanced.
Remedial Law I| Page 131

Ombudsman vs. Chavez


G.R. No. 172206 July 3, 2013

Facts:

The Batangas State University Board of Regents (BSU-BOR) received an Order from Deputy Ombudsman directing
the former to enforce the Office of the Ombudsman's Joint Decision and Supplemental Resolution finding herein
respondents guilty of dishonesty and grave misconduct and imposing the penalty of dismissal from service. Pursuant
to said Order, the BSU-BOR issued Resolution resolving to implement the Order of the Office of the Ombudsman.
Thus, herein respondents filed a petition for injunction with prayer for issuance of a temporary restraining order or
preliminary injunction before the RTC. The gist of the petition before the RTC is that the BSU-BOR should be
enjoined from enforcing the Ombudsman's Joint Decision and Supplemental Resolution because the same are still
on appeal and, therefore, are not yet final and executory.

The RTC ordered the dismissal of herein respondents' petition for injunction on the ground of lack of cause of action.
Respondents filed their notice of appeal and promptly filed a Motion for Issuance of a Temporary Restraining Order
and/or Injunction with the CA. CA granted respondents' prayer for a temporary restraining order enjoining the
BSU-BOR from enforcing its Resolution No. 18, series of 2005.

Thereafter, the Office of the Ombudsman filed a Motion to Intervene and to Admit Attached Motion to Recall
Temporary Restraining Order, with the Motion to Recall Temporary Restraining Order attached thereto.
Respondents opposed said motion and then filed an Urgent Motion for Issuance of a Writ of Preliminary Injunction.
CA denied the Office of the Ombudsman’s motion. Hence, this petition.

Issue:

Whether or not the Office of the Ombudsman has legal personality to intervene in the appealed case before the CA?

Held:

Yes. The CA should have allowed the Office of the Ombudsman to intervene in the appeal pending with the lower
court. As held in the case of Office of the Ombudsman v. Samaniego, the Ombudsman is in a league of its own. It is
different from other investigatory and prosecutory agencies of the government because the people under its
jurisdiction are public officials who, through pressure and influence, can quash, delay or dismiss investigations
directed against them. Its function is critical because public interest (in the accountability of public officers and
employees) is at stake. It is true that under our rule on intervention, the allowance or disallowance of a motion to
intervene is left to the sound discretion of the court after a consideration of the appropriate circumstances. However,
such discretion is not without limitations. One of the limits in the exercise of such discretion is that it must not be
exercised in disregard of law and the Constitution. The CA should have considered the nature of the Ombudsman's
powers as provided in the Constitution and RA 6770.

Here, since its power to ensure enforcement of its Joint Decision and Supplemental Resolution is in danger of being
impaired, the Office of the Ombudsman had a clear legal interest in defending its right to have its judgment carried
out. The CA patently erred in denying the Office of the Ombudsman's motion for intervention.
Remedial Law I| Page 132

Anonuevo vs. Intestrate of Jalandoni


G.R. No. 178221 December 1, 2010

Facts:

Rodolfo Jalandoni died intestate. Bernardino, the brother of Rodolfo, filed petition for issuance of letters of
administration with the Court of First Instance of Negros Occidental to commence the judicial settlement of the
estate. May Anonuevo and their siblings introduced themselves as children of Sylvia Blee Desantis, who is the
daughter of Isabel Blee with one John Desantis. Isabel (their grandmother) was at the time of Rodolfo’s death is the
legal spouse of the latter. For this reason, Isabel is entitled to a share in estate of Rodolfo.

Petitioners pray that they be allowed to intervene in the intestate proceedings because Sylvia and Isabel have
already passed away. Respondents opposed because the evidence showed by the Petitioners reveal that Isabel has a
subsisting marriage with John Desantis at the time she was purportedly married to Rodolfo. Thus, her marriage
with Rodolfo was void ab initio.

The intestate court issued an order allowing petitioners to take part in settlement proceedings. The CA reversed the
decision of the intestate court and sided with the respondents.

Issue:

Whether or the not the CA erred when it nullified the orders of intestate court allowing petitioners to intervene in
settlement proceedings?

Held:

No. A court’s power to allow or deny intervention, albeit discretionary in nature, is circumscribed by the basic
demand of sound judicial procedure that only a person with interest in an action or proceeding may be allowed to
intervene. Otherwise stated, a court has no authority to allow a person, who has no interest in an action or
proceeding, to intervene therein.

Petitioners and their siblings failed to offer sufficient evidence to establish that Isabel was the legal spouse of
Rodolfo. The very evidence of the petitioners and their siblings negates their claim that Isabel has interest in
Rodolfo’s estate. The birth certificate of Sylvia precisely serves as the competent evidence of marriage between
Isabel and John Desantis. The inability of the petitioners and their siblings to present evidence to prove that Isabel’s
prior marriage was dissolved results in a failure to establish that she has interest in the estate of Rodolfo. Clearly,
an intervention by the petitioners and their siblings in the settlement proceedings cannot be justified.
Remedial Law I| Page 133

Fernandez vs. Court of Appeals


A.M. OCA IPI No. 12-201-CA-J. February 19, 2013

Facts:

Complainants Ethelwoldo Fernandez and Antonio Henson were elected to the board of directors of NADECOR. In a
regular stockholder’s meeting where two groups were vying for control over the company, Calalang, De Jesus,
Romulo, Ayala, Lazatin, Fernandez, Nitorreda, Engle were elected. Gatmaitan was also elected as Corporate
Secretary.

Thereafter, the Ricaforts, claiming to be stockholders of record, sought to annul the said meeting held. They filed a
complaint before the RTC of Pasig. The Ricaforts alleged that they were not given due notice of the said meeting
thus they were not present and were not able to exercise their right. The RTC agreed with the Ricaforts.

Four separate Petition for Certiorari were filed by the members of the board with the CA, all with application for a
TRO and/or preliminary injunction. The CA denied such applications, but on the same day nevertheless, the 11th
division issued a TRO. During the effectivity of the TRO, the old Board of Directors assumed the functions of the
new one in order to prevent any hiatus and not to prejudice the corporation. All the CA petitions were consolidated
as well as the other cases.

The respondents Ricafort filed their Comment Ad Cautelam to the petition in CA-G.R. No. 122784. The petitioners
therein thereafter filed three (3) urgent motions to resolve their application for writ of preliminary injunction, on
March 8, on May 22, and again on June 6, 2012. The Writ of Preliminary Injunction was granted by the CA 14th
Division, which not for long was questioned.

Complainants filed with the Supreme Court a Petition for Certiorari and Prohibition, seeking to annul the writ of
preliminary injunction issued by the CA’s Special 14th Division. Complainants also filed an Administrative case
against the Justices of the 14th Division of the CA.

Issue:

Whether or not the some of the petitioners who were not parties in the cases in the Court of Appeals have a legal
personality to assail the writ of preliminary injunction issued by the CA 14th Division?

Held:

No. Section 1 of Rule 19 of the Rules of Court provides that a person who has a legal interest in the matter in
litigation, or in the success of either of the parties, or an interest against both, or is so situated as to be adversely
affected by a distribution or other disposition of property in the custody of the court or of an officer thereof may, with
leave of court, be allowed to intervene in the action. Conversely, a person who is not a party in the main suit cannot
be bound by an ancillary writ, such as a preliminary injunction. Indeed, he cannot be affected by any proceeding to
which he is a stranger. Moreover, a person not an aggrieved party in the original proceedings that gave rise to the
petition for certiorari, will not be permitted to bring the said action to annul or stay the injurious writ. Such is the
clear import of Sections 1 and 2 of Rule 65 of the Rules of Court. Thus, a person not a party to the proceedings in the
trial court or in the CA cannot maintain an action for certiorari in the Supreme Court to have the judgment
reviewed. Stated differently, if a petition for certiorari or prohibition is filed by one who was not a party in the lower
court, he has no standing to question the assailed order.

The complainants, who at various times served as elected members of the Board of NADECOR, did not bother to
intervene in the CA petitions, hence, they are not entitled to the service of pleadings and motions therein.
Complainant Fernandez was himself a defendant in SEC Case No. 11-164 in the RTC, but he cose not to join any of
the four CA petitions.
Remedial Law I| Page 134

Persons who are not parties to any of the consolidated petitions have no personality to assail the said injunctive
writ. They should first have intervened below, and then filed a motion for reconsideration from the questioned CA
order.
Remedial Law I| Page 135

Rodriguez vs. Court of Appeals


G.R. No. 184589, June 13, 2013

Facts:

Purita Landicho by virtue of November 16, 1965 decision of CFI of Rizal was able to register to her name the
property subject of the case. However, a TCT was issued in her name instead of OCT. The subject property in this
case, after being sold several times, was brought by Philippine Chinese Charitable Association, Inc. (PCCAI), herein
respondent from WPFI on November 13, 1973 and was issued TCT on July 15, 1975. But it appeared that on
November 14, 1996, Landicho executed a DOAS in favor of Deogenes O. Rodriguez, herein petitioner. Seven years
after, Rodriguez filed with Omnibus Motion with the RTC, praying for the Land Registration Authority(LRA) to
issue OCT in his name. During trial, TCT which was issued in the name of PCCAI was presented by the petitioner,
and alleged that it was fictitious. PCCAI now filed a motion for leave to intervene, alleging that it has a substantial
legal interest over the said land. PCCAI filed an MR. Both motion for leave to intervene, and MR was denied by the
RTC, it ruled that intervention would not be allowed after the decision has become final and executory. The LRA
upon receipt of the order, filed a manifestation to the RTC that it cannot comply with the said order, since two TCT
were issued covering the same land. At the same time, PCCAI filed a petition for Certiorari and Prohibition before
the CA, it argued that even though the issuance of execution for final and executory order is a ministerial duty, it
should refrain from issuing such if it will be unjust and inequitable, and that since it was an indispensable party,
they should have been allowed to intervene by the RTC. The CA decided in favor of PCCAI, MR was denied as well.
Hence, this petition.

Issue:

Whether or not the petitioner should be allowed to intervene even the November 16, 1965 judgment CFI of Rizal was
already final and executory?

Held:

Yes. The subject property is presently covered by TCT No. 482970 in the name of PCCAI. As the registered owner,
PCCAI clearly has a legal interest in the subject property. The issuance of another certificate of title to Rodriguez
will adversely affect PCCAI, constituting a cloud on its TCT No. 482970.

Although Rule 19 is explicit on the period when a motion to intervene may be filed, the Court allowed exceptions in
several cases, viz:

This rule, however, is not inflexible. Interventions have been allowed even beyond the period prescribed in the Rule,
when demanded by the higher interest of justice. Interventions have also been granted to afford indispensable
parties, who have not been impleaded, the right to be heard even after a decision has been rendered by the trial
court, when the petition for review of the judgment has already been submitted for decision before the Supreme
Court, and even where the assailed order has already become final and executory. In Lim v. Pacquing, the motion
for intervention filed by the Republic of the Philippines was allowed by this Court to avoid grave injustice and injury
and to settle once and for all the substantive issues raised by the parties.

In fine, the allowance or disallowance of a motion for intervention rests on the sound discretion of the court after
consideration of the appropriate circumstances. Rule 19 of the Rules of Court is a rule of procedure whose object is
to make the powers of the court fully and completely available for justice. Its purpose is not to hinder or delay, but
to facilitate and promote the administration of justice.

The particular circumstances of this case similarly justify the relaxation of the rules of procedure on intervention.
First, the interests of both PCCAI and Rodriguez in the subject property arose only after the CFI Decision dated
November 16, 1965 in Land Reg. Case No. N-5098 became final and executory. PCCAI bought the subject property
from WPFI on November 13, 1973 and was issued TCT No. 482970 for the same on July 15, 1975; while Rodriguez
Remedial Law I| Page 136

bought the subject property from Landicho on November 14, 1996. Second, as previously discussed herein, both
PCCAI and Rodriguez trace their titles back to Landicho. Hence, the intervention of PCCAI could not unduly delay
or prejudice the adjudication of the rights of Landicho, the original party in Land Reg. Case No. N-5098. Third, the
latest proceedings in Land Reg. Case No. N-5098 involved Rodriguez's Omnibus Motion, filed before the RTC on
May 18, 2005, in which he prayed for the execution of the November 16, 1965 Decision of the CFI. PCCAI moved to
intervene in the case only to oppose Rodriguez's Omnibus Motion on the ground that the subject property is already
registered in its name under TCT No. 482970, which originated from Landicho's TCT No. 167681. And fourth, after
learning of Rodriguez's Omnibus Motion in Land Reg. Case No. N-5098 via the November 3, 2006 subpoena issued
by the RTC, PCCAI was reasonably expected to oppose the same. Such action was the most opportune and expedient
remedy available to PCCAI to prevent the RTC from ordering the issuance of a decree of registration and OCT in
Rodriguez's name. For this reason, the RTC should have allowed the intervention of PCCAI.
Remedial Law I| Page 137

Yao vs. Perello


G.R. No. 153828, October 24, 2003

Facts:

HLURB rendered a decision rescinding the contract to sell between petitioner (Yao) and PR Builders, and ordering
PR Builders to refund petitioner the amount of ₱2,116,103.31, as well as to pay damages in the amount of ₱250,000.
Thereafter, the HLURB issued a writ of execution against PR Builders and its managers, and referred the writ to
the office of the Clerk of Court of Muntinlupa for enforcement. Pursuant to the writ, the deputy sheriff levied on a
parcel of land in Canlubang, Calamba, Laguna, registered in the names of spouses Pablito Villarin and private
respondent, Bernadine Villarin. The property was scheduled for public auction on March 20, 2002.

Private respondent filed before the RTC of Parañaque City, a petition for prohibition with prayer for temporary
restraining order and/or writ of preliminary injunction, seeking to enjoin Sheriff Melvin T. Bagabaldo from
proceeding with the public auction. Private respondent alleged that she co-owned the property subject of the
execution sale; that the property regime between private respondent and her husband was complete separation of
property, and that she was not a party in the HLURB case, hence, the subject property could not be levied on to
answer for the separate liability of her husband.

On April 25, 2002, or more than a month after public respondent judge issued the resolution of March 22, 2002,
petitioner filed a motion for intervention. The RTC denied the motion. Aggrieved, petitioner filed the instant petition
for certiorari imputing grave abuse of discretion to public respondent judge in: (a) declaring the subject property
exempt from execution and therefore could not be sold to satisfy the obligation of private respondent’s husband, and
(b) denying petitioner’s motion for intervention on the ground that the same was filed late.

Issue:

Whether or not petitioner can validly intervene to question the resolution of the RTC exempting the property of
Bernadine Villarin from execution?

Held:

No. Petitioner’s claim that he had the right to intervene is without basis. Nothing in the Rules require the inclusion
of a private party as respondent in petitions for prohibition. On the other hand, to allow intervention, it must be
shown that (a) the movant has a legal interest in the matter in litigation or otherwise qualified, and (b)
consideration must be given as to whether the adjudication of the rights of the original parties may be delayed or
prejudiced, or whether the intervenor’s rights may be protected in a separate proceeding or not. Both requirements
must concur as the first is not more important than the second.

In the case at bar, it cannot be said that petitioner’s right as a judgment creditor was adversely affected by the
lifting of the levy on the subject real property. Records reveal that there are other pieces of property exclusively
owned by the defendants in the HLURB case that can be levied upon.

Moreover, even granting for the sake of argument that petitioner indeed had the right to intervene, he must exercise
said right in accordance with the rules and within the period prescribed therefor.

As provided in the Rules of Court, the motion for intervention may be filed at any time before rendition of judgment
by the trial court. Petitioner filed his motion only on April 25, 2002, way beyond the period set forth in the rules. The
court resolution granting private respondent’s petition for prohibition and lifting the levy on the subject property
was issued on March 22, 2002. By April 6, 2002, after the lapse of 15 days, the said resolution had already become
final and executory.
Remedial Law I| Page 138

Pinlac vs. Court of Appeals


G.R. No. 91486. September 10, 2003

Facts:

Petitioners filed a Petition for Quieting of Title over three vast parcels of land known as Lot Nos. 1, 2 & 3. Lot No. 1
was covered by TCT No. 5690, while Lot Nos. 2 and 3 were originally covered by OCT No. 614 and OCT No. 333,
respectively. The trial court rendered a Partial Decision in favor of petitioners and against the defendants who were
declared in default, including respondent owners of Vilmar-Maloles (Vilma) Subdivision whose properties were
within Lot No. 2. The defaulted title owners of Vilma filed with the Court of Appeals a Petition to Annul the Partial
Decision of the trial court, which was granted.

The appellate court ruled that the court a quo did not acquire jurisdiction over the person of respondents because of
defective service of summons by publication. Petitioners motion for reconsideration of the said decision was also
denied. Consequently, it filed a petition for certiorari before the Supreme Court, but the Court only affirmed CA’s
decision. Aggrieved, petitioner then, filed a Motion for Reconsideration contending among others that the disposition
of the trial court with respect to Lot No. 3, should not have been annulled by the Court of Appeals because the
petition for annulment of judgment filed by the respondents concerned only Lot No. 2. The Court then, issued a
Resolution partially granting petitioners motion for reconsideration by reinstating paragraphs 4 and 5 of the
dispositive portion of the trial court’s Partial Decision pertaining to Lot No. 3. However, the Republic of the
Philippines, represented by the Land Registration Authority (LRA), thru the Office of the Solicitor General (OSG),
filed a motion for intervention and a Petition-In-Intervention praying that judgment be rendered declaring among
other that, the OCT No. 333 was a valid and existing title in line with the decisions this Honorable Court had
already rendered, hence, this case.

Issue:

Whether a Motion for intervention and a Petition-In-Intervention filed by the Republic were proper taking into
account that a partial Resolution by the Supreme Court has already been rendered?

Held:
Yes. The rule on intervention, like all other rules of procedure is intended to make the powers of the Court fully and
completely available for justice. It is aimed to facilitate a comprehensive adjudication of rival claims overriding
technicalities on the timeliness of the filing thereof. Indeed, in exceptional cases, the Court has allowed intervention
notwithstanding the rendition of judgment by the trial court. In one case, intervention was allowed even when the
petition for review of the assailed judgment was already submitted for decision in the Supreme Court. In Mago v.
Court of Appeals intervention was granted even after the decision became final and executory, thus:

The permissive tenor of the provision on intervention shows the intention of the Rules to give to the court the full
measure of discretion in permitting or disallowing the same. But needless to say, this discretion should be exercised
judiciously and only after consideration of all the circumstances obtaining in the case.

The intervention of the Republic is necessary to protect public interest as well as government properties located and
projects undertaken on Lot No. 3. The Constitutional mandate that no person shall be deprived of life, liberty, or
property without due process of law can certainly be invoked by the Republic which is an indispensable party to the
case at bar. As correctly pointed out by the Solicitor General, while the provision is intended as a protection of
individuals against arbitrary action of the State, it may also be invoked by the Republic to protect its properties.
Remedial Law I| Page 139

Chipongian vs. Benitez-Lirio


G.R. No. 162692. August 26, 2015

Facts:
The late Vicente Benitez was married to Isabel Chipongian, the petitioner's sister. Isabel had predeceased Vicente.
The couple had no offspring that was why Vicente and the petitioner had executed a deed of extrajudicial settlement
respecting the estate of Isabel, whereby the latter waived all his rights to the estate of Isabel in favor of Vicente.
According to the petitioner, however, Vicente executed an affidavit on the same date whereby he affirmed that the
waiver did not extend to the paraphernal properties of Isabel. Upon the death of Vicente, Victoria Benitez Lirio
(Victoria), a sister of Vicente, and Feodor Benitez Aguilar (Feodor), a nephew of Vicente, initiated proceedings for
the settlement of the estate of Vicente in the Regional Trial Court and it issued the letters of administration to
Feodor.
The petitioner intervened in the Special Proceedings and sought the partial revocation of the issuance of the Letters
of Administration order for the exclusion of the paraphernal properties of Isabel from the estate of Vicente. He cited
the affidavit of Vicente in support of the partial revocation. However, Feodor countered with the request that he be
allowed to continue to administer all the properties left by Vicente, including the paraphernal properties of Isabel.
Consequently, petitioner specifically moved for the exclusion of the paraphernal properties of Isabel from Vicente's
estate, but he withdrew the motion even before the RTC could rule on it. Instead, he filed a Motion for Leave to
Intervene and to Admit Complaint-in-Intervention. Respondents Victoria and Feodor opposed the complaint-in-
intervention, but the RTC granted the Motion for Leave to Intervene and to Admit Complaint-in-Intervention, and
admitted the complaint-in-intervention of the petitioner.
The RTC rendered judgment dismissing the complaint-in-intervention, for it took petitioner 12 years to assert the
purported affidavit allegedly executed in his favor by Vicente O. Benitez. The Petitioner filed a Motion for
Reconsideration for the judgment, but the same was denied. A notice of appeal was also filed, but was denied for it
was filed beyond the reglementary period. Another Motion for Reconsideration on the denial of notice of appeal was
again denied due to petitioner’s failure to pay the court docket fees, although the Court conceded that the notice of
appeal was filed on time. As his last resort, petitioner filed a Motion to Set Aside the order denying his Motion for
Reconsideration, but the same was denied, prompting him to file a petition for Certiorari under Rule 65 before the
CA.
The CA dismissed the petition for certiorari opining that petitioner paid the appeal fees only on March 31, 1999, but
as admitted by him in his Motion for Reconsideration, the last day to perfect his appeal was on August 21, 1998.
This prompted petitioner to file a petition for certiorari before the Supreme Court. While in the Supreme Court,
respondents Victoria and Feodor sought the denial of the petition for review because the petitioner did not file a
record on appeal, as mandated under Section 2(a) Rule 41 of the Rules of Court. However, in his reply to
respondent’s comment, petitioner submitted that it had to be noted that the appeal was from the decision of the trial
court to dismiss petitioner's complaint-in-intervention and not “the final order or judgment rendered in the case”,
obviously referring to the main case, that was, the intestate estate case. Since the intervention was not an
independent proceeding but only ancillary or supplemental to the main case, the rule on multiple appeals does not
apply and the filing of a record on appeal is not a pre-requisite to the acceptance and consideration of the appeal by
the appellate court.
Issue:
Whether or not the rule on multiple appeals (filing of a record on appeal) is applicable in Intervention?
Held:
Yes. Intervention is "a remedy by which a third party, not originally impleaded in the proceedings, becomes a
litigant therein to enable him, her or it to protect or preserve a right or interest which may be affected by such
proceedings." If an intervention makes a third party a litigant in the main proceedings, his pleading-in-intervention
should form part of the main case. Accordingly, when the petitioner intervened in Special Proceedings No. SP-797,
his complaint-in-intervention, once admitted by the RTC, became part of the main case, rendering any final
disposition thereof subject to the rules specifically applicable to special proceedings, including Rule 109 of the Rules
of Court, which deals with appeals in special proceedings.
Remedial Law I| Page 140

The dismissal of the petitioner's intervention constituted "a final determination in the lower court of the rights of
the party appealing," that is, his right in the paraphernal properties of his deceased sister. As such, it fell under
paragraph (c) of Section 1 because it had the effect of disallowing his claim against the estate of Vicente, as well
as under paragraph (e) of Section 1 because it was a final determination in the trial court of his intervention.
Conformably with either or both paragraphs, the dismissal was the proper subject of an appeal in due course by
virtue of its nature of completely disposing of his intervention. The proper mode of appealing a judgment or
final order in special proceedings is by notice of appeal and record on appeal.

As the foregoing rules further indicate, a judgment or final order in special proceedings is appealed by record on
appeal. A judgment or final order determining and terminating a particular part is usually appealable, because it
completely disposes of a particular matter in the proceeding, unless otherwise declared by the Rules of Court. The
ostensible reason for requiring a record on appeal instead of only a notice of appeal is the multi-part nature of nearly
all special proceedings, with each part susceptible of being finally determined and terminated independently of the
other parts. An appeal by notice of appeal is a mode that envisions the elevation of the original records to the
appellate court as to thereby obstruct the trial court in its further proceedings regarding the other parts of the case.
In contrast, the record on appeal enables the trial court to continue with the rest of the case because the original
records remain with the trial court even as it affords to the appellate court the full opportunity to review and decide
the appealed matter. Considering that the petitioner did not submit a record on appeal in accordance with Section 3
of Rule 41, he did not perfect his appeal of the judgment dismissing his intervention. As a result, the dismissal
became final and immutable. He now has no one to blame but himself. The right to appeal, being statutory in
nature, required strict compliance with the rules regulating the exercise of the right. As such, his perfection of his
appeal within the prescribed period was mandatory and jurisdictional, and his failure to perfect the appeal within
the prescribed time rendered the judgment final and beyond review on appeal.
Remedial Law I| Page 141

Rules 23-29: Modes of Discovery

Dasmariñas Garments vs. Reyes


G.R. No. 108229 August 24, 1993

Facts:

American President Lines (APL) instituted an action against Dasmariñas Garments to recover the sum of US
$53,228.45 as well as an amount equivalent to twenty-five percent (25%) thereof as attorney's fees and litigation
expenses. Dasmariñas Garments filed an Answer with Counterclaim. APL on the other hand, filed a motion during
the hearing praying that it intended to take the depositions of Kenneth H. Lee and Yeong Fang Yeh in Taipei,
Taiwan and prayed that for this purpose, a commission or letters rogatory be issued addressed to the consul, vice-
consul or consular agent of the Republic of the Philippines in Taipei.

Five days later, APL filed an amended motion stating that since the Philippine Government has no consulate office
in Taiwan in view of its “one China policy,” there being in lieu thereof an office set up by the President “presently
occupied by Director Joaquin Roces which is the Asian Exchange Center, Inc.,” it was necessary—and it therefore
prayed—“that commission or letters rogatory be issued addressed to Director Joaquin Roces, Executive Director,
Asian Executive Exchange Center, Inc., Room 901,112 Chunghsiao, E. Road, Section 1, Taipei, Republic of China,
to hear and take the oral deposition of the forenamed persons.

Dasmariñas Garments opposed the motion on the grounds that the motion was fatally defective in that it does not
seek that a foreign court examine a person within its jurisdiction, the Issuance of letters rogatory was unnecessary
because the witnesses can be examined before the Philippine Court and the Rules of Court expressly require that
the testimony of a witness must be taken orally in open court and not by deposition

The RTC ruled in favor of the motion of the APL and affirmed by the CA.

Issue:

Whether or not the motion to take the testimonies of plaintiffs Taiwanese witnesses, Kenneth H. Lee and Yeong
Fah Yeh, by deposition (upon written interrogatories) should be allowed?

Held:

Yes. Depositions are chiefly a mode of discovery. They are intended as a means to compel disclosure of facts resting
in the knowledge of a party or other person which are relevant in some suit or proceeding in court. Depositions, and
the other modes of discovery (interrogatories to parties; requests for admission by adverse party; production or
inspection of documents or things; physical and mental examination of persons) are meant to enable a party to
learn all the material and relevant facts, not only known to him and his witnesses but also those known to the
adverse party and the latter’s own witnesses. In fine, the object of discovery is to make it possible for all the parties
to a case to learn all the material and relevant facts, from whoever may have knowledge thereof, to the end that
their pleadings or motions may not suffer from inadequacy of factual foundation, and all the relevant facts may be
clearly and completely laid before the Court, without omission or suppression.

It is apparent then that the deposition of any person may be taken wherever he may be, in the Philippines or
abroad. If the party or witness is in the Philippines, his deposition “shall be taken before any judge, municipal or
notary public” (Sec. 10, Rule 24, Rules of Court). If in a foreign state or country, the deposition “shall be taken: (a)
on notice before a secretary or embassy or legation, consul general, consul, vice consul, or consular agent of the
Republic of the Philippines, or (b) before such person or officer as may be appointed by commission or under letters
rogatory”.

Where the deposition is to be taken in a foreign country where the Philippines has no “secretary or embassy or
legation, consul general, consul, vice-consul, or consular agent,” then obviously it may be taken only “before such
person or officer as may be appointed by commission or under letters rogatory.”
Remedial Law I| Page 142

A commission may be defined as “(a)n instrument issued by a court of justice, or other competent tribunal, to
authorize a person to take depositions, or do any other act by authority of such court or tribunal”. Letters rogatory,
on the other hand, may be defined as “(a)n instrument sent in the name and by the authority of a judge or court to
another, requesting the latter to cause to be examined, upon interrogatories filed in a cause pending before the
former, a witness who is within the jurisdiction of the judge or court to whom such letters are addressed”.

Dasmariñas also contends that the “taking of deposition is a mode of pretrial discovery to be availed of before the
action comes to trial.” Not so. Depositions may be taken at any time after the institution of any action, whenever
necessary or convenient. There is no rule that limits deposition-taking only to the period of pre-trial or before it; no
prohibition against the taking of depositions after pre- trial. Indeed, the law authorizes the taking of depositions of
witnesses before or after an appeal is taken from the judgment of a Regional Trial Court “to perpetuate their
testimony for use in the event of further proceedings in the said court” (Rule 134, Rules of Court), and even during
the process of execution of a final and executory judgment.
Remedial Law I| Page 143

Go vs. People
G.R. No. 185527. July 18, 2012

Facts:

Petitioners Harry Go, Tonny Ngo, Jerry Ngo and Jane Go were charged before the MeTC of Manila for Other
Deceits under Article 318 of the RPC. Upon arraignment, petitioners pleaded not guilty to the charge. The
prosecution’s complaining witness, Li Luen Ping, a frail old businessman from Laos, Cambodia, traveled from his
home country back to the Philippines in order to attend the hearing held on September 9, 2004. However, trial
dates were subsequently postponed due to his unavailability.

The private prosecutor filed with the MeTC a Motion to Take Oral Deposition of Li Luen Ping, alleging that he was
being treated for lung infection at the Cambodia Charity Hospital in Laos, Cambodia and that, upon doctor’s
advice, he could not make the long travel to the Philippines by reason of ill health. Notwithstanding petitioners’
Opposition, the MeTC granted the motion after the prosecution complied with the directive to submit a Medical
Certificate of Li Luen Ping. The RTC reversed the MeTC. The RTC held that Section 17, Rule 23 on the taking of
depositions of witnesses in civil cases cannot apply suppletorily to the case since there is a specific provision in the
Rules of Court with respect to the taking of depositions of prosecution witnesses in criminal cases, which is
primarily intended to safeguard the constitutional rights of the accused to meet the witness against him face to
face.

The CA reversed the RTC and affirmed the MeTC’s decision ruling that no rule of procedure expressly disallows the
taking of depositions in criminal cases and that, in any case, petitioners would still have every opportunity to cross
examine the complaining witness and make timely objections during the taking of the oral deposition either
through counsel or through the consular officer who would be taking the deposition of the witness.

Issue:

Whether or not the deposition of a prosecution’s main witness outside the country should be allowed?

Held:

No. The procedure under Rule 23 to 28 of the Rules of Court allows the taking of depositions in civil cases, either
upon oral examination or written interrogatories, before any judge, notary public or person authorized to
administer oaths at any time or place within the Philippines; or before any Philippine consular official,
commissioned officer or person authorized to administer oaths in a foreign state or country, with no additional
requirement except reasonable notice in writing to the other party. But for purposes of taking the deposition in
criminal cases, more particularly of a prosecution witness who would forseeably be unavailable for trial, the
testimonial examination should be made before the court, or at least before the judge, where the case is pending as
required by the clear mandate of Section 15, Rule 119 of the Revised Rules of Criminal Procedure.

Since the conditional examination of a prosecution witness must take place at no other place than the court where
the case is pending, the RTC properly nullified the MeTC’s orders granting the motion to take the deposition of Li
Luen Ping before the Philippine consular official in Laos, Cambodia.

The condition of the private complainant being sick and of advanced age falls within the provision of Section 15
Rule 119 of the Rules of Court. However, said rule substantially provides that he should be conditionally examined
before the court where the case is pending. Thus, this Court concludes that the language of Section 15 Rule 119
must be interpreted to require the parties to present testimony at the hearing through live witnesses, whose
demeanor and credibility can be evaluated by the judge presiding at the hearing, rather than by means of
deposition. No where in the said rule permits the taking of deposition outside the Philippines whether the deponent
is sick or not.
Remedial Law I| Page 144

Vda. de Manguerra vs. Risos


G.R. No. 152643, August 28, 2008

Facts:

Respondents (Risos, et. al.) were charged with Estafa Through Falsification of Public Document before the RTC.
The case, arose from the falsification of a deed of real estate mortgage allegedly committed by respondents where
they made it appear that petitioner (Concepcion De Manguerra), the owner of the mortgaged property known as the
Gorordo property, affixed her signature to the document. Concepcion who was a resident of Cebu City, while on
vacation in Manila, was unexpectedly confined at the Makati Medical Center due to upper gastro-intestinal
bleeding; and was advised to stay in Manila for further treatment. The counsel of Concepcion filed a motion to take
the latter’s deposition due to her weak physical condition and old age, which limited her freedom of mobility. The
RTC of Cebu granted the motion and directed that Concepcion’s deposition be taken before the Clerk of Court of
Makati City. After several motions for change of venue of the deposition-taking, Concepcion’s deposition was finally
taken at her residence.

Aggrieved, respondents assailed the RTC orders in a special civil action for certiorari before the CA. The CA
nullified the RTC orders, declaring that the examination of prosecution witnesses, as in the present case, is
governed by Section 15, Rule 119 of the Revised Rules of Criminal Procedure and not Rule 23 of the Rules of
Court. The latter provision, said the appellate court, only applies to civil cases. Pursuant to the specific provision of
Section 15, Rule 119, Concepcion’s deposition should have been taken before the judge or the court where the case
is pending, which is the RTC of Cebu, and not before the Clerk of Court of Makati City; and thus, in issuing the
assailed order, the RTC clearly committed grave abuse of discretion. MR was denied. Hence, this petition.

Issue:

Whether or not the deposition of a prosecution witness not before the court where the action is pending may be
allowed?

Held:

No. Rule 119 specifically states that a witness may be conditionally examined: 1) if the witness is too sick or infirm
to appear at the trial; or 2) if the witness has to leave the Philippines with no definite date of
returning. Undoubtedly, the procedure set forth in Rule 119 applies to the case at bar. It is thus required that the
conditional examination be made before the court where the case is pending. It is also necessary that the accused be
notified, so that he can attend the examination, subject to his right to waive the same after reasonable notice. As to
the manner of examination, the Rules mandate that it be conducted in the same manner as an examination during
trial, that is, through question and answer.

Petitioners further insist that Rule 23 applies to the instant case, because the rules on civil procedure apply
suppletorily to criminal cases. It is true that Section 3, Rule 1 of the Rules of Court provides that the rules of civil
procedure apply to all actions, civil or criminal, and special proceedings. In effect, it says that the rules of civil
procedure have suppletory application to criminal cases. However, it is likewise true that the criminal proceedings
are primarily governed by the Revised Rules of Criminal Procedure. Considering that Rule 119 adequately and
squarely covers the situation in the instant case, we find no cogent reason to apply Rule 23 suppletorily or
otherwise.

Rule 119 categorically states that the conditional examination of a prosecution witness shall be made before the
court where the case is pending. Contrary to petitioner’s contention, there is nothing in the rule which may
remotely be interpreted to mean that such requirement applies only to cases where the witness is within the
jurisdiction of said court and not when he is kilometers away, as in the present case. Therefore, the court may not
introduce exceptions or conditions.
Remedial Law I| Page 145

People vs. Webb


G.R. No. 132577. August 17, 1999

Facts:

Hubert Webb, an accused in the crime of Rape with Homicide, filed a Motion to Take Testimony by Oral Deposition,
to take the testimonies of some vital witnesses residing in the U.S., before the proper Philippine consular
authorities since the Philippine court had no jurisdiction over them and may not therefore be compelled by
subpoena to testify. Respondent further alleged that the taking of the oral depositions of the aforementioned
individuals whose testimonies are allegedly ‘material and indispensable’ to establish his innocence of the crime
charged is sanctioned by Section 4, Rule 24 of the Revised Rules of Court. The prosecution thereafter filed an
opposition to the said motion averring that Rule 24, Section 4 of the Rules of Court has no application in criminal
cases. The trial court denied the motion but was thereafter reversed by the CA on appeal.

Issue:

Whether or not deposition of the defense’s witnesses outside the country may be allowed in this case?

Held:

No. It need not be overemphasized that the factual circumstances only serves to underscore the immutable fact
that the depositions proposed to be taken from the five U.S. based witnesses would be merely corroborative or
cumulative in nature and in denying respondent’s motion to take them, the trial court was but exercising its
judgment on what it perceived to be a superfluous exercise on the belief that the introduction thereof will not
reasonably add to the persuasiveness of the evidence already on record.

It is pointed out that the defense has already presented at least fifty-seven (57) witnesses and four hundred sixty
four (464) documentary exhibits, many of them of the exact nature as those to be produced or testified to by the
proposed foreign deponents. Under the circumstances, we sustain the proposition that the trial judge commits no
grave abuse of discretion if she decides that the evidence on the matter sought to be proved in the United States
could not possibly add anything substantial to the defense evidence involved.

The use of discovery procedures is directed to the sound discretion of the trial judge. The deposition taking can not
be based nor can it be denied on flimsy reasons. Discretion has to be exercised in a reasonable manner and in
consonance with the spirit of the law.
Remedial Law I| Page 146

Disini vs. Sandiganbayan


G.R. No. 175730, July 5, 2010

Facts:

Disini, herein petitioner, was charged for two information for corruption of public officials, Art 212 in relation to
Art 210 (RPC) and violation of RA 3019. For failure to file a responsive pleading, petitioner was declared in default.
Subsequently, petitioner filed a Motion to Lift Order of Default and for Leave to File and Admit Attached Answer,
together with an Answer to Amended Complaint with Compulsory Counterclaims. He maintained that he was
unaware of the civil case pending against him because he never received summons or other processes from the
court, nor any pleadings from the parties of the case. His only fault, he averred, was that he was ignorant of the
proceedings in the case because of the absence of a proper notice. Petitioner asked the respondent court to look at
his meritorious defenses. He then invoked the liberality of the courts in lifting default orders to give both parties
every opportunity to defend their cases, and pointed out that the proceedings, being in their pre-trial stage, would
not be delayed by petitioner’s participation therein. Sandiganbayan denied his motion to lift order of default. Thus,
petitioner filed a Petition for Certiorari before the Supreme Court.

In his Petition, petitioner originally sought the nullification of the proceedings before the Sandiganbayan on the
theory of lack of jurisdiction over his person, premised on the alleged impropriety in the service of summons.
However, petitioner subsequently filed several motions with the Sandiganbayan which sought various affirmative
reliefs from that court, sans any qualification of the nature of its appearance and without reserving or reiterating
its previous objection on the ground of lack of jurisdiction over the person. One of the motions is a Motion for Leave
To Take Deposition.

Issue:

Whether or not by filing a Motion for Leave To Take Deposition, the petitioner is deemed to have waived his
objections to the lack of jurisdiction of the Sandiganbayan?

Held:

Yes. It is important to note that there are two instances when the defendant can take depositions under Section 1
of Rule 23: (1) after the court has acquired jurisdiction over the defendant or the property subject of the action; and
(2) after an answer has been served. Both instances presuppose that the court has already acquired jurisdiction
over the defendant. By seeking this relief (Motion for Leave To Take Deposition), petitioner is deemed to have
voluntarily submitted himself to the jurisdiction of the Sandiganbayan. Thus, petitioner may be held to have
waived his objections regarding the lack of jurisdiction over his person by seeking affirmative relief through the
said provision.

While petitioner bewailed the mode of service of summons on him and questioned the Sandiganbayan’s jurisdiction
over his person, he has rendered his own arguments moot by his voluntary appearance or submission to the
jurisdiction of the Sandiganbayan. Jurisprudence holds that an objection based on lack of jurisdiction over the
person is waived when the defendant files a motion or pleading which seeks affirmative relief other than the
dismissal of the case.
Remedial Law I| Page 147

Afulugencia vs. Metro Bank


G.R. No. 185145. February 5, 2014.

Facts:

Petitioners, Spouses Vicente and Leticia Afulugencia, filed a Complaint for nullification of mortgage, foreclosure,
auction sale, certificate of sale and other documents, with damages, against respondents Metropolitan Bank &
Trust Co. (Metrobank) and Emmanuel L. Ortega (Ortega) before the RTC of Malolos City. With the conclusion of
pre-trial, petitioners filed a Motion for Issuance of Subpoena Duces Tecum Ad Testificandum to require
Metrobank’s officers to appear and testify as the petitioners’ initial witnesses during the August 31, 2006 hearing
for the presentation of their evidence-in-chief, and to bring the documents relative to their loan with Metrobank, as
well as those covering the extrajudicial foreclosure and sale of petitioners’ 200- square meter land in Meycauayan,
Bulacan.

Metrobank filed an Opposition arguing that for lack of a proper notice of hearing, the Motion must be denied; that
being a litigated motion, the failure of petitioners to set a date and time for the hearing renders the Motion
ineffective and pro forma; that pursuant to Sections 1 and 6 of Rule 25 of the Rules, Metrobank’s officers – who are
considered adverse parties – may not be compelled to appear and testify in court for the petitioners since they were
not initially served with written interrogatories; that petitioners have not shown the materiality and relevance of
the documents sought to be produced in court; and that petitioners were merely fishing for evidence.

Issue:

Whether or not a party not served with written interrogatories may be compelled by the adverse party to give
testimony in open court?

Held:

No. As a rule, in civil cases, the procedure of calling the adverse party to the witness stand is not allowed, unless
written interrogatories are first served upon the latter. This is embodied in Section 6, Rule 25 of the Rules, which
provides – Sec. 6. Effect of failure to serve written interrogatories. Unless thereafter allowed by the court for good
cause shown and to prevent a failure of justice, a party not served with written interrogatories may not be compelled
by the adverse party to give testimony in open court, or to give a deposition pending appeal.

One of the purposes of the above rule is to prevent fishing expeditions and needless delays; it is there to maintain
order and facilitate the conduct of trial. It will be presumed that a party who does not serve written interrogatories
on the adverse party beforehand will most likely be unable to elicit FACTS useful to its case if it later opts to call
the adverse party to the witness stand as its witness.

In the present case, petitioners seek to call Metrobank’s officers to the witness stand as their initial and main
witnesses, and to present documents in Metrobank’s possession as part of their principal documentary evidence.
This is tantamount to building their whole case from the evidence of their opponent. The burden of proof and
evidence falls on petitioners, not on Metrobank; if petitioners cannot prove their claim using their own evidence,
then the adverse party Metrobank may not be pressured to hang itself from its own defense.
Remedial Law I| Page 148

Allied Agri-Business Development Co. vs. Court of Appeals


G.R. No. 118438. December 4, 1998

Facts:

Cherry Valley Farms Limited (Cherry Valley), a foreign company based in England, filed against petitioner Allied
Agri-Business Development Co. Inc. (Allied) a complaint with the Regional Trial Court of Makati City for collection
of sum of money for failure of Allied to pay several duck hatching eggs and ducklings which in value totaled
51,245.12. Allied filed an answer denying the material allegations of the complaint. Cherry Valley served on
Allied’s counsel a Request for Admission. Allied filed its Comments/Objections alleging that: (a) the admissions
requested were matters which the private respondent had the burden to prove through its own witness during the
trial and thus petitioner need not answer; and, (b) the request for admission regarding the ownership set-up of
petitioner corporation was immaterial and improper for not having been pleaded in the complaint. In its
Reply to Comments/Objections to Request for Admission, Cherry Valley maintained that there was no need on its
part to produce a witness to testify on the matters requested for admission, for these pertained to incidents
personal to and within the knowledge of petitioner alone. Thereafter, Cherry Valley filed a motion with the trial
court to resolve the objections of Allied to the request for admission. Trial court issued an Order disregarding
Allied’s Comments/Objections to Request for Admission in view of its non-compliance with Sec. 2, Rule 26, of the
Rules of Court and directing Allied to answer the request for admission within ten (10) days from receipt of the
order, otherwise, the matters contained in the request would be deemed admitted. Allied moved to reconsider the
order but was denied. Allied’s motion for reconsideration and directed the latter to answer the request for
admission within a non-extendible period of five (5) days from receipt of the order. Allied failed to submit a sworn
answer to the request for admission within the additional period of five (5) days granted by the trial court. Hence,
Cherry Valley filed a motion for summary judgment alleging that there was already an implied admission on the
matters requested for admission pursuant to Rule 26 of the Rules of Court. Trial court rendered judgment against
petitioner. Allied appealed to the Court of Appeals which affirmed the summary judgment. Hence, this petition.
Issue:
Whether or not Allied’s failure to answer the request for admission resulted in its admission of the matters stated
in the request?

Held:

Yes. The purpose of the rule governing requests for admission of facts and genuineness of documents is to expedite
trial and to relieve parties of the costs of proving facts which will not be disputed on trial and the truth of which can
be ascertained by reasonable inquiry. Each of the matters of which an admission is requested shall be deemed
admitted unless within a period designated in the request which shall not be less than fifteen (15) days after service
thereof, or within such further time as the court may allow on motion, the party to whom the request is directed files
and serves upon the party requesting the admission a sworn statement either denying specifically the matters of
which an admission is requested or setting forth in detail the reasons why he cannot truthfully either admit or deny
those matters. Upon service of request for admission, the party served may do any of the following acts: (a) he may
admit each of the matters of which an admission is requested, in which case, he need not file an answer; (b) he may
admit the truth of the matters of which admission is requested by serving upon the party requesting a written
admission of such matters within the period stated in the request, which must not be less than ten (10) days after
service, or within such further time as the court may allow on motion and notice; (c) he may file a sworn statement
denying specifically the matter of which an admission is requested; or, (d) he may file a sworn statement setting
forth in detail the reasons why he cannot truthfully either admit or deny the matters of which an admission is
requested.
The records show that although petitioner filed with the trial court its comments and objections to the request for
admission served on it by private respondent, the trial court disregarded the objections and directed petitioner after
denying its motion for reconsideration, to answer the request within five (5) days from receipt of the directive;
otherwise, the matters of which the admission was requested would be deemed admitted. Petitioner failed to submit
the required answer within the period. The matter set forth in the request were therefore deemed admitted by
petitioner. The burden of affirmative action is on the party upon whom notice is served to avoid the admission
rather than upon the party seeking the admission. Hence, when petitioner failed to reply to a request to admit, it
Remedial Law I| Page 149

may not argue that the adverse party has the burden of proving the facts sought to be admitted. Petitioners’ silence
is an admission of the facts stated in the request.
Remedial Law I| Page 150

Philippine Health Insurance Corporation vs. Our Lady of Lourdes Hospital


G.R. No. 193158, November 11, 2015

Facts:

Petitioner Philippine Health Insurance Corporation (PHIC) filed a complaint with its Legal Sector-Prosecution
Department against Our Lady of Lourdes Hospital (OLLH) for the administrative offense of filing multiple claims,
which was penalized under one of the provisions of the Implementing Rules and Regulations (IRR) of R.A. No.
7875. Allegedly, OLLH filed two claims of the same amount of PhilHealth benefits involving the same patient for
the same diagnosis and covering the same period of confinement. Summons was duly served, then, an answer was
filed by OLLH. The parties were then, directed to file their respective Position Papers, but only PHIC complied
with the order. OLLH moved to defer the submission of its position paper pending the answer of the PHIC
President and CEO to the written interrogatories as well as the inspection and copying of the original
transmittal letter and all other claims that accompanied Annex B11 of the Complaint. According to
OLLH, these modes of discovery were availed of because its representatives were denied and/or not given access to
documents and were not allowed to talk to PHIC personnel with regard to the charge. PHIC filed its Comment on
OLLH’s motion. In its defense, PHIC asserted that OLLH’s overt acts clearly revealed its intent to delay the
administrative proceedings. It stressed that the material points which OLLH sought to establish in its resort to
modes of discovery were already presented in the pleadings and documents it submitted for consideration of the
Arbiter. Specifically, the subject information and documents sought to be examined were the same information and
documents which OLLH itself prepared, produced, and submitted to the PHIC. Likewise, the PHIC procedure
subject of the interrogatories, apart from being publicly accessible and already known to OLLH, was immaterial to
the case given OLLH’s sole defense that it inadvertently attached the wrong document that led to the processing of
two separate claims.

PHIC Arbitration Department ruled that the interrogatories and motion for production and inspection of
documents filed by OLLH could not be given be given due course. The Court of Appeals reversed the Resolution of
the PHIC Arbiter and ruled that petitioner OLLH has shown good cause for its resort to the modes of discovery as
the same was anchored on its being able to intelligently prepare a position paper considering that it was not
allowed access to some pertinent documents or talk to PHIC personnel with regard the charge of filing multiple
claims.

Issue:

Whether or not OLLH’s resort to modes of discovery, specifically the written interrogatories and production and
inspection of documents, was proper?

Held:

No. Through written interrogatories, a party may elicit from the adverse party or parties any facts or matter that
are not privileged and are material and relevant to the subject of the pending action. Like other modes of discovery
authorized by the Rules, the purpose of written interrogatories is to assist the parties in clarifying the issues and in
ascertaining the facts involved in a case. On the other hand, the provision on production and inspection of
documents is to enable not only the parties but also the court (in this case, the PHIC Arbitration Department) to
discover all the relevant and material facts in connection with the case pending before it. It must be shown,
therefore, that the documents sought to be produced, inspected and/or copied/photographed are material or contain
evidence relevant to an issue involved in the action.

In this case, the questions contained in the written interrogatories filed and received on July 28, 2009 sought to
elicit facts that could already be seen from the allegations as well as attachments of the Complaint and the Verified
Answer. Specifically, the entries in the three (3) Validation Report that OLLH sought to be identified and/or
explained by PHIC are either immaterial or irrelevant (to the issue of whether OLLH is guilty of filing multiple
claims and OLLH’s defense that it inadvertently attached a second copy of the subject PhilHealth Claim Form 2 to
the Transmittal Letter filed on June 19, 2007) or, even if material or relevant, are self-explanatory and need no
further elaboration from PHIC. Thus, the interrogatories were frivolous and need not be answered. Aside from this,
the PHIC Arbitration Department correctly observed that the written interrogatories were mistakenly addressed to
Remedial Law I| Page 151

the President and CEO of PHIC, who could not competently answer, either based on his job description or firsthand
experience, issues that arose from and related to the filing and processing of claims. We likewise find as self-
serving the allegation of OLLH that its representatives were denied access to the documents pertaining to the
subject PhilHealth claim and, at the same time, were not allowed to talk to any of the PhilHealth personnel. No
iota of evidence, documentary or testimonial, was submitted to substantiate this convenient excuse.
Remedial Law I| Page 152

Rule 30: Trial


People vs. Bustamante
G.R. No. 189836, June 5, 2013

Note: The case illustrated the order in the examination of witness (direct, cross, redirect, recross) under
Rule 132. This is different in the order of trial (in civil cases, Section 5, Rule 30 and in criminal cases,
Sec. 11, Rule 119)

Facts:

Romeo Bustamante was charged with the crime of rape by means of threat and intimidation for having sexual
intercourse with his own daughter, the herein offended party, AAA for several times, starting from the time that the
offended party was only eleven (11) years of age.

[AAA] testified that she lived with his father, the [appellant] in this case, mother and younger siblings, 3 brothers
and a sister, in x x x, Alcala, Cagayan. At about lunch time or thereafter on February 17, 1997, she was alone in the
second floor in their house when the [appellant] arrived. Her younger brother Jayjun was playing outside while her
mother went to clean their ginger garden. The [appellant] laid her down on the floor and removed her shorts and
panty. He then removed his pants, went on top of her and inserted his penis into her vagina. [Appellant] removed
his penis after he ejaculated and told her not to report what had happened. [Appellant] forced her and she was not
able to resist because she was still young during that time. She reported the incident to her mother and the police.

On re-direct examination, [AAA] testified that she filed the case against the [appellant] so that the latter would no
longer box and maltreat her and because he raped her. On re-cross, it was revealed that [appellant] was neither
armed during the incident nor covered her mouth when he laid her down. She did not shout because she was afraid.
[Appellant] threatened her before he raped her.

xxxx

[Appellant] testified that in the early morning on February 7, 1997, he went to Tuguegarao with his daughter,
[AAA]. He went to Mrs. Lolit Casauay, his employer, and Sgt. Poli to tell them his problem regarding [AAA] and her
cousin having sex. Sgt. Poli advised [appellant] to go to Alcala Police to have his problem entered in the blotter and
to go back to him after. They stayed in Tuguegarao the whole day and went back to x x x, Alcala, Cagayan about
7:00 o’clock in the evening. When they were approaching their house, Purita Torrado called for [AAA] and told
[appellant] that he was a traitor. Purita Torrado and brothers, Rogelio and Amador Torrado, then entered his house,
mauled him and tied his hands. Thereafter, policemen arrived and brought him to the Municipal Hall of Alcala,
Cagayan without informing him why. His daughter [AAA] charged him of the heinous crime of rape because his wife
and brothers-in-law harbored ill feelings against him, blaming him to have spread the rumor that Rogelio Torrado
was the father of the child of his own sister Purita Torrado. Before February 17, 1997, his daughter [AAA] admitted
to him that she had sexual relations with her cousin Randy Torrado for which reason he went to Tuguegarao to help
[AAA] file a complaint against said Randy Torrado. It was after they came from Tuguegarao that his daughter
[AAA] charged him with rape.

On cross-examination, [appellant] testified that he did not report any barangay official that Randy Torrado sexually
molested his daughter x x x, but went to a person Ernie Fiesta who was not a barangay official. He admittedly told
his problem to Sgt. Poli who asked [AAA] questions but the same was not entered in the blotter of the Cagayan
Police Provincial Office.

On re-direct, [appellant] further testified that it was his daughter [AAA] who told him that Randy Torrado molested
her so he brought her to Tuguegarao the following day, February 17, 1997. He was not able to enter it in the blotter
of Alcala police as directed by Sgt. Poli because when they arrived in Maraburab, Alcala from Tuguegarao, his
brothers-in-law mauled him. He did not file any charges against his brothers-in-law.

Police Inspector Carlos T. Poli testified as follows: He was then the Assistant Chief Investigator at the Cagayan
Police Provincial Headquarters on February 17, 1997. [Appellant], with his daughter [AAA] went to him. [Appellant]
told him that his daughter was molested by a nephew of his wife but he could not recall the name. He advised
Remedial Law I| Page 153

[appellant] to have the incident reported to, and entered in the blotter of, the Alcala Police Station where the
incident took place and to return for investigation. He talked to [AAA] who admitted that there was truth to the
report that she was molested and that there was a second occasion. He did not enter the report in the blotter
because they did not have a blotter so he advised [appellant] to have the case entered in the blotter of Alcala Police.
The report was not recorded because [appellant] only ought his advice and that he would first talk to his wife as the
suspect was her relative. Admittedly, he invited the wife of [appellant] to his office upon the request of her in-laws
who pitied and considered the [appellant] as their son. He asked the wife if she could help but the latter could not do
it because her brother and sister were interested to pursue the case.

The trial court convicted him of the said crime. In his appeal, he claims that, as evidenced by the victim’s own
testimony, AAA filed a false complaint of rape against him mainly due to her ill feelings towards him brought about
by his purported repeated physical maltreatment of the victim.

Issue:

Whether or not the accused should be convicted of the crime of rape?

Held:

Yes. The Court rule that, it is settled in jurisprudence that in a prosecution for rape, the accused may be convicted
solely on the basis of the testimony of the victim that is credible, convincing, and consistent with human nature and
the normal course of things. Jurisprudence is likewise instructive that the factual findings of the trial court,
especially on the credibility of the rape victim, are accorded great weight and respect and will not be disturbed on
appeal.

In the case at bar, both the trial court and the Court of Appeals found AAA to be a credible witness and her
testimony worthy of full faith and credit.

In his defense, appellant interposes denial while also ascribing ill motive on the part of the victim, his own biological
daughter, for accusing him of rape. However, it is well-settled that denial, if unsubstantiated by clear and
convincing evidence, is a self-serving assertion that deserves no weight in law because denial cannot prevail over the
positive, candid and categorical testimony of the complainant, and as between the positive declaration of the
complainant and the negative statement of the appellant, the former deserves more credence. Likewise, the
testimonies of the witnesses presented by appellant failed to buttress his defense of denial as they merely related to
tangential matters which do not seriously affect the issue of AAA’s credibility.
Remedial Law I| Page 154

Rule 31: Consolidation and Severance

Metrobank vs. Sandoval


G.R. No. 169677 February 18, 2013

Facts:

On July 17, 1987, the Republic brought a complaint for reversion, reconveyance, restitution, accounting and
damages in the Sandiganbayan against Andres V. Genito, Jr., Ferdinand E. Marcos, Imelda R. Marcos and other
defendants. The action was to recover allegedly ill-gotten wealth of the Marcoses, their nominees, dummies and
agents. Among the properties subject of the action were 2 parcels of commercial land located in Tandang Sora (Old
Balara), Quezon City, registered in the names of Spouses Andres V. Genito, Jr. and Ludivina L. Genito. Republic
later on amended its complaint to implead Asian Bank as an additional defendant since the latter claimed
ownership of the 2 parcels of land as the registered owner and it was also in possession of the properties by virtue of
the writ of possession issued by the RTC QC.

When the Republic was about to terminate its presentation of evidence against the original defendants, it moved to
hold a separate trial against Asian Bank. Asian Bank sought the deferment of any action on the motion and
contended that it would be deprived of its day in court if a separate trial were to be held against it without having
been sufficiently apprised about the evidence the Republic had adduced before it was brought in as an additional
defendant. Republic maintained that a separate trial for Asian Bank was proper because its cause of action against
Asian Bank was entirely distinct and independent from its cause of action against the original defendants; and that
the issue with respect to Asian Bank was whether Asian Bank had actual or constructive knowledge at the time of
the issuance of the TCTs for the properties in its name that such properties were the subject of the complaint in
Civil Case No. 0004, while the issue as to the original defendants was whether they had "committed the acts
complained of as constituting illegal or unlawful accumulation of wealth which would, as a consequence, justify
forfeiture of the said properties or the satisfaction from said properties of the judgment that may be rendered in
favor of the Republic." Asian Bank’s rejoinder to the reply asserted that the issue in Civil Case No. 0004 was
intimately related to the issue delving on the character of the properties as the ill-gotten wealth of the original
defendants; that it thus had a right to confront the evidence presented by the Republic as to the character of the
properties; and that the Sandiganbayan had no jurisdiction to decide Asian Bank’s ownership of the properties
because the Sandiganbayan, being a special court with limited jurisdiction, could only determine the issue of
whether or not the properties were illegally acquired by the original defendants. Sandiganbayan granted the
Republic’s motion for separate trial. Asian Bank moved for the reconsideration of the resolution, but the
Sandiganbayan denied its motion.

Hence, Metrobank commenced this special civil action for certiorari as the successor-in-interest of Asian Bank and
transferee of the properties.

Issue:

Whether or not a separate trial against Asian Bank was proper?

Held:

No. The rule on separate trials in civil actions is found in Section 2, Rule 31 of the Rules of Court, which reads:

Section 2. Separate trials. – The court, in furtherance of convenience or to avoid prejudice, may order a separate trial
of any claim, cross-claim, counterclaim, or third-party complaint, or of any separate issue or of any number of claims,
cross-claims, counterclaims, third-party complaints or issues.

The text of the rule grants to the trial court the discretion to determine if a separate trial of any claim, cross-claim,
counterclaim, or third-party complaint, or of any separate issue or of any number of claims, cross-claims,
counterclaims, third-party complaints or issues should be held, provided that the exercise of such discretion is in
furtherance of convenience or to avoid prejudice to any party.
Remedial Law I| Page 155

Sandiganbayan committed grave abuse of its discretion in ordering a separate trial as to Asian Bank (Metrobank) on
the ground that the issue against Asian Bank was distinct and separate from that against the original defendants.
Thereby, the Sandiganbayan veered away from the general rule of having all the issues in every case tried at one
time, unreasonably shunting aside the dictum in Corrigan, supra, that a "single trial will generally lessen the delay,
expense, and inconvenience to the parties and the courts." Exceptions to the general rule are permitted only when
there are extraordinary grounds for conducting separate trials on different issues raised in the same case, or when
separate trials of the issues will avoid prejudice, or when separate trials of the issues will further convenience, or
when separate trials of the issues will promote justice, or when separate trials of the issues will give a fair trial to
all parties. Otherwise, the general rule must apply. The justification of the Sandiganbayan for allowing the separate
trial did not constitute a special or compelling reason like any of the exceptions.

To begin with, the issue relevant to Asian Bank was not complicated. In that context, the separate trial would not be
in furtherance of convenience. And, secondly, the cause of action against Asian Bank was necessarily connected with
the cause of action against the original defendants. Should the Sandiganbayan resolve the issue against Spouses
Genito in a separate trial on the basis of the evidence adduced against the original defendants, the properties would
be thereby adjudged as ill-gotten and liable to forfeiture in favor of the Republic without Metrobank being given the
opportunity to rebut or explain its side. The outcome would surely be prejudicial towards Metrobank.
Remedial Law I| Page 156

Republic vs. Heirs of Enrique Oribello


G.R. No. 199501 March 6, 2013

Facts:

This case involves a parcel of land situated in Nagbaculao, Kalaklan, Olongapo City, which was once classified as
forest land by the Bureau of Forest Development. The property was originally occupied by a certain Valentin
Fernandez (Valentin) in 1968 by virtue of a Residential Permit issued by the same government office. Upon
Valentin’s death, his son Odillon continued to pay the property, together with Spouses Apog. Odillon sold the
property to Mrs. Balcita who later on sold the property to Oribello. Oribello then filed a Miscellaneous Sales
Application which as initially denied but on his second try, a sales patent was issued to him. Spouses Apog protested
with the DENR the grant of the patent and the DENR recommended the filing of a reversion proceeding suit. The
OSG instituted a complaint for reversion and cancellation of title before RTC Olongapo. The case was thereafter
consolidated with the complaint for recovery of possession filed by Spouses Apog.

On July 25,1997, the court issued an order stating that on several occasions when these cases were set for trial ,
neither of the Republic’s counsels appeared., constraining the court to postpone the hearing. The actuations of both
lawyers result to delay in the early termination of these cases which have been pending since 1992.

The trial court dismissed the consolidated cases without prejudice for non-substitution of the deceased plaintiff
(Oribello) and his counsel. Republic moved for reconsideration, contending that the order applied exclusively to civil
case for recovery of possession and did not affect case for reversion of property. Petitioner prayed that it allowed to
present its evidence. The Trial Court allowed the continuation of the presentation of petitioner’s evidence in its
Order dated June 2005. Heirs of Oribello filed then a Manifestation and Motion. They pointed out that from the time
the Republic received the Order in 1997, it did nothing to question the same, making the order final. In its
resolution in July 12, 2006, the RTC recalled its order and dismissed the case.

The Republic appealed to the CA. The CA agreed with respondents that petitioner has lost its right to participate in
the proceedings of the case when it failed to question the RTC’s September 12, 1997 order declaring it to have
abandoned the case as a consequence of petitioner’s inaction, such order inevitably became final. Republic, however,
contends that the order, deeming it to have abandoned the cases, is interlocutory in nature; thus is not appealable.
Respondents argue otherwise, maintaining that such order is a dismissal of the complaint on the ground of failure to
prosecute which is under the Rules, considered an adjudication on the merits, and hence, appealable.

Issue:

Whether or not the consolidated cases can be the subject of multiple appeals (or in other words, whether or not the
judgment in one of the consolidates cases can be appealed independently of the other case)?

Held:

Yes. Section 1, Rule 31 of the Rules of Court provides:

SECTION 1. Consolidation. When actions involving a common question of law or fact are pending before the court, it
may order a joint hearing or trial of any or all the matters in issue in the actions; it may order all the actions
consolidated, and it may make such orders concerning proceedings therein as may tend to avoid unnecessary costs or
delay.

Consolidation is a procedural device to aid the court in deciding how cases in its docket are to be tried so that the
business of the court may be dispatched expeditiously and with economy while providing justice to the parties. To
promote this end, the rule allows the consolidation and a single trial of several cases in the court’s docket, or the
consolidation of issues within those cases.

In the context of legal procedure, the term “consolidation” is used in three different senses:
Remedial Law I| Page 157

(1) Where all except one of several actions are stayed until one is tried, in which case the judgment in the one trial is
conclusive as to the others. This is not actually consolidation but is referred to as such. (quasi-consolidation) (2)
Where several actions are combined into one, lose their separate identity, and become a single action in which a
single judgment is rendered. This is illustrated by a situation where several actions are pending between the same
parties stating claims which might have been set out originally in one complaint. (actual consolidation)

(3) Where several actions are ordered to be tried together but each retains its separate character and requires the
entry of a separate judgment. This type of consolidation does not merge the suits into a single action, or cause the
parties to one action to be parties to the other. (consolidation for trial)

In the present case, the complaint for reversion filed by petitioner (Civil Case No. 225-0-92) was consolidated with
the complaint for recovery of possession filed by Oribello (Civil Case No. 223-0-91). While these two cases involve
common questions of law and fact, each action retains its separate and distinct character. The reversion suit settles
whether the subject land will be reverted to the State, while the recovery of possession case determines which
private party has the better right of possession over the subject property. These cases, involving different issues and
seeking different remedies, require the rendition and entry of separate judgments. The consolidation is merely for
joint trial of the cases. Notably, the complaint for recovery of possession proceeded independently of the reversion
case, and was disposed of accordingly by the trial court.

Since each action does not lose its distinct character, severance of one action from the other is not necessary to
appeal a judgment already rendered in one action. There is no rule or law prohibiting the appeal of a judgment or
part of a judgment in one case which is consolidated with other cases. Further, severance is within the sound
discretion of the court for convenience or to avoid prejudice. It is not mandatory under the Rules of Court that the
court sever one case from the other cases before a party can appeal an adverse ruling on such case.
Remedial Law I| Page 158

Maraño vs. Pryce Gases, Inc.
G.R. No. 196592. April 6, 2015

Facts:

Petitioners (Spouses Juvy and Maria Luisa Maraño) filed for free patent application for a parcel of land in
Damulaan, Albuera, Leyte which was granted. Consequently, they then filed an ejectment complaint against
respondent (Pryce Gases, Inc.) alleging that the latter illegally entered the subject lot and constructed a building
thereon. The MTC granted the petition, but was later on reversed by the RTC. On appeal, the Court of Appeals
remanded the case to the MTC for trial as a reivindicatory action under the ordinary rules of civil procedure.

However, in the interim, the respondent filed a protest on the free patent application filed by the petitioners. The
petitioners filed an action to quiet title against the respondent with the RTC, 8th Judicial Region, Branch 14,
Baybay City, Leyte and a month later, the respondent also filed a complaint for reconveyance against the petitioners
before the same RTC. The petitioners moved to dismiss the respondent’s complaint, but the RTC denied their
motion. The respondent later moved to amend its complaint from reconveyance to the cancellation of the petitioners’
certificate of title. The petitioners again moved to dismiss the respondent’s amended complaint on the ground of litis
pendentia in view of the then pending reivindicatory action with the MTC.

The RTC dismissed the petitioners’ motion. The petitioners moved for reconsideration but their motion was likewise
denied by the RTC. The petitioners questioned the RTC’s resolution in a petition for certiorari with the CA.

The CA held that no litis pendentia existed between the reivindicatory action (then pending before the MTC) and
the amended complaint for cancellation of certificate of title filed by the respondent with the RTC. The petitioners
moved to reconsider the CA’s decision but their motion was denied, hence, this petition.

Issue:

Whether or not there was no litis pendentia existed between the reivindicatory action (then pending before the MTC)
and the amended complaint for cancellation of certificate of title filed by the respondent with the RTC?

Held:

Yes. Instead of ordering the dismissal of the respondent’s complaint for cancellation of certificate of title, the
consolidation of the reivindicatory action and the cancellation of certificate of title case is the appropriate remedy in
the present situation. Consolidation is proper when two or more actions pending, not necessarily, before
the same court involve a common question of law or fact. In such cases, the court may, order a joint hearing or
trial of any or all the matters in issue in the actions, order all the actions consolidated, and make such orders
concerning the proceedings therein for the purpose of avoiding unnecessary costs and delay. Considering that the
validity of the petitioners’ certificate of title is the crucial issue in both the reivindicatory action pending appeal
before the RTC and the cancellation of certificate of title case filed by the respondent, these two cases should be
consolidated in order to avoid the possibility of rendering conflicting decisions and for the orderly administration of
justice. And since the issue of validity of the petitioners’ certificate of title has been subjected to a full-blown trial
before the MTC and is now the subject of appeal before the RTC, allowing the cancellation of certificate of title case
to proceed independently and separately would be needlessly circuitous and would necessarily delay the resolution
of the present issue.
Remedial Law I| Page 159

Rule 33: Demurrer to Evidence

Republic vs. Gimenez


G.R. No. 174673, January 11, 2016

Facts:

The Republic, through the Presidential Commission on Good Government (PCGG), instituted a Complaint for
Reconveyance, Reversion, Accounting, Restitution and Damages against the Gimenez Spouses before the
Sandiganbayan. The Complaint seeks to recover ill-gotten wealth acquired by the Gimenez Spouses as dummies,
agents, or nominees of former President Ferdinand E. Marcos and Imelda Marcos. Documentary and testimonial
evidence were presented by the Republic and accordingly, the Sandiganbayan gave the Republic 30 days or until
March 29, 2006 to file its formal offer of evidence. However, despite the repeated extensions and the lapse of 75 days
from the date of the termination of the presentation of evidence, Republic failed to submit its Formal Offer of
Evidence. The Sandiganbayan issued a Resolution declaring that the Republic waived the filing of its Formal Offer
of Evidence.

Ignacio Gimenez then filed a Motion to Dismiss on Demurrer to Evidence. He argued that the Republic showed no
right to relief as there was no evidence to support its cause of action. Later, Fe Roa Gimenez filed a Motion to
Dismiss on the ground of failure to prosecute. Through her own Motion to Dismiss, she joined Ignacio Gimenez’s
demurrer to evidence. Two days after, Fe Roa Gimenez’s filing of the Motion to Dismiss, the Republic filed a Motion
for Reconsideration [of the first assailed Resolution] and to Admit Attached Formal Offer of Evidence. The
Sandiganbayan on its second Resolution denied the Republic’s Motion for Reconsideration and granted the Gimenez
Spouses’ Motion to Dismiss. Hence, this petition. The Republic contends, among others, that the Sandiganbayan
erred when it granted the demurrer to evidence filed by respondents and dismissed the case despite a "prima
facie foundation (based on the pleadings and documents on record) that spouses Gimenezes amassed enormous
wealth grossly disproportionate to their lawful income or declared lawful assets."

Issue:

Whether or not the Sandiganbayan erred in granting the Demurrer to Evidence?

Held:

Yes. To erroneously grant a dismissal simply based on the delay to formally offer documentary evidence essentially
deprives one party of due process. Weighing the amount of time spent in litigating the case against the number of
delays petitioner incurred in submitting its Formal Offer of Evidence and the state’s policy on recovering ill-gotten
wealth, this court is of the belief that it is but only just that the Rules be relaxed and petitioner be allowed to submit
its written Formal Offer of Evidence. The Sandiganbayan’s Resolutions should be reversed.

Likewise, the court laid down the guidelines in resolving a demurrer to evidence:

A demurrer to evidence may be issued when, upon the facts and the law, the plaintiff has
shown no right to relief. Where the plaintiff’s evidence together with such inferences and conclusions
as may reasonably be drawn therefrom does not warrant recovery against the defendant, a demurrer
to evidence should be sustained. A demurrer to evidence is likewise sustainable when, admitting
every proven fact favorable to the plaintiff and indulging in his favor all conclusions fairly and
reasonably inferable therefrom, the plaintiff has failed to make out one or more of the material
elements of his case, or when there is no evidence to support an allegation necessary to his claim. It
should be sustained where the plaintiff’s evidence is prima facie insufficient for a recovery.

Furthermore, this court already clarified what the trial court determines when acting on a motion to dismiss based
on demurrer to evidence:
Remedial Law I| Page 160

“What should be resolved in a motion to dismiss based on a demurrer to evidence is whether
the plaintiff is entitled to the relief based on the facts and the law. The evidence contemplated by the
rule on demurrer is that which pertains to the merits of the case, excluding technical aspects such as
capacity to sue. . . .”

What should be determined now by the Sandiganbayan is whether petitioner’s evidence is sufficient to entitle it to
the relief it seeks after the Sandiganbayan rested its case. The Court ruled that the evidences presented by the
petitioner is sufficient to establish a presumption that respondents acquired ill-gotten wealth during respondent Fe
Roa Gimenez’s incumbency as public officer and which total amount or value was manifestly out of proportion to her
and her husband’s salaries and to their other lawful income or properties and that the Sandiganbayan brushed off
the totality of evidence on which petitioner built its case.

Even assuming that no documentary evidence was properly offered, this court finds it clear from the second assailed
Resolution that the Sandiganbayan did not even consider other evidence presented by petitioner during the 19 years
of trial. The Sandiganbayan erred in ignoring petitioner’s testimonial evidence without any basis or justification.
Numerous exhibits were offered as part of the testimonies of petitioner’s witnesses.

A liberal application of the Rules is in line with the state’s policy to recover ill-gotten wealth. In case of doubt, courts
should proceed with caution in granting a motion to dismiss based on demurrer to evidence. An order granting
demurrer to evidence is a judgment on the merits. This is because while a demurrer "is an aid or instrument for the
expeditious termination of an action," it specifically "pertains to the merits of the case." To reiterate, "[d]emurrer to
evidence authorizes a judgment on the merits of the case without the defendant having to submit evidence on his [or
her] part, as he [or she] would ordinarily have to do, if plaintiff’s evidence shows that he [or she] is not entitled to
the relief sought." The order of dismissal must be clearly supported by facts and law since an order granting
demurrer is a judgment on the merits.
Remedial Law I| Page 161

Doctrine: There is no oral motion for leave to file a demurrer. This doctrine may be deemed superseded by
the New Rules on Continuous Trial Act effective September 2017.

Bernardo vs. Court of Appeals


G.R. No. 119010 September 5, 1997

Facts:

Paz Bernardo was originally charged with four counts of violation of BP 22 before the RTC of Quezon City.
Subsequently, two of the cases were dismissed due to the Affidavit of Desistance executed by private respondent
Florlita, thus the remaining two cases were left to be disposed of by the trial court.

After presenting its last witness, the prosecution rested its case and formally offered its exhibits. After the
prosecution had formally offered its evidence, in open court, the counsel of the defense respectfully ask for leave of
court to file demurrer to evidence on the ground that the prosecutor failed to elicit facts where the checks were
issued and where they were actually dishonored. However, the prosecution objected and showed the properly
marked exhibits and testimonial evidences for that matter. In view of such objections, the court denied the grounds
of the defense for demurrer. The defense, however, reiterated that they will file their demurrer to evidence without
leave of court and the court considered the motion dilatory. Further, the court considered that the defense waived
their right to present evidence and that the case is deemed submitted for decision.

Bernardo assailed the order of the respondent judge before the Court of Appeals by way of certiorari, prohibition and
mandamus. She argued that the trial court committed grave abuse of discretion in considering her to have waived
her right to present evidence after the denial of her motion for leave to file demurrer to evidence. The CA rendered
a decision modifying in effect the order of the lower court by directing the trial court to set the case for trial for
reception of evidence for the petitioner. Petitioner moved for partial reconsideration but was denied. Hence, this
petition. The petitioner submits, among others, that when the trial court denied her motion, it was in effect a denial
only of the motion for leave to file demurrer to evidence and not the demurrer to evidence itself and, therefore, the
order of the respondent appellate court allowing petitioner to present evidence was premature.

Issue:

Whether or not the petitioner should be allowed to present evidence after the denial of her demurrer to evidence?

Held:

No. Under the new rule on demurrer to evidence, the accused has the right to file a demurrer to evidence after the
prosecution has rested its case. If the accused obtained prior leave of court before filing his demurrer, he can still
present evidence if his demurrer is denied. However, if he demurs without prior leave of court, or after his motion
for leave is denied, he waives his right to present evidence and submits the case for decision on the basis of the
evidence for the prosecution. This power to grant leave to the accused to file a demurrer is addressed to the sound
discretion of the trial court. The purpose is to determine whether the accused in filing his demurrer is merely
stalling the proceedings.

In this case, petitioner admits that in the hearing, the trial court denied her motion for leave to file a demurrer to
evidence. In such case, the only right petitioner has under Section 15, Rule 119 of the Rules of Court after having
been denied leave to submit a demurrer is to adduce evidence in her defense. However, even without express leave of
trial court, nay, after her motion for leave was denied, petitioner insisted on a filing a demurrer instead of
presenting evidence in her defense.

Once prior leave is denied and the accused still files his demurrer to evidence or motion to dismiss, the court no
longer has discretion to allow the accused to present evidence. The only recourse left for the court is to decide the
case on the basis of the evidence presented by the prosecution. And, unless there is grave abuse thereof amounting
to lack or excess of jurisdiction, which is not present in the instant case, the trial courts denial of prior leave to file
demurrer to evidence or motion to dismiss may not be disturbed.
Remedial Law I| Page 162

Radiowealth Finance Co. vs. Del Rosario


G.R. No. 138739 July 6, 2000

Facts:

Spouses Del Rosario jointly and severally executed, signed and delivered in favor of Radiowealth Finance Company
a Promissory Note for P138,948. The spouses defaulted on the monthly installments and despite repeated demands,
they failed to pay their obligations. Radiowealth then filed a Complaint for the collection of a sum of money. During
the trial, the credit and collection officer of Radiowealth presented in evidence the check payments, the demand
letters, the customer’s ledger card for the spouses and dishonor slips. The officer admitted that he did not have
personal knowledge of the transaction or the execution of any of these pieces of documentary evidence, which had
merely been endorsed to him. The trial court issued an Order terminating the presentation of evidence for
Radiowealth. Thus, Radiowealth formally offered its evidence and exhibits and rested its case. The spouses filed a
Demurrer to Evidence for alleged lack of cause of action. The RTC dismissed the complaint for failure of Radiowealth
to substantiate its claims, the evidence it had presented being merely hearsay. However, the CA reversed the trial
court and remanded the case for further proceedings.

Issue:
Whether or not remand of the case to the trial court was necessary when the CA reversed the RTC’s decision
granting the demurrer to evidence?
Held:
No. If the trial court denies the dismissal motion, i.e., finds that plaintiffs evidence is sufficient for an award of
judgment in the absence of contrary evidence, the case still remains before the trial court which should then proceed
to hear and receive the defendant’s evidence so that all the facts and evidence of the contending parties may be
properly placed before it for adjudication as well as before the appellate courts, in case of appeal. Nothing is
lost. The doctrine is but in line with the established procedural precepts in the conduct of trials that the trial court
liberally receive all proffered evidence at the trial to enable it to render its decision with all possibly relevant proofs
in the record, thus assuring that the appellate courts upon appeal have all the material before them necessary to
make a correct judgment, and avoiding the need of remanding the case for retrial or reception of improperly
excluded evidence, with the possibility thereafter of still another appeal, with all the concomitant delays. The rule,
however, imposes the condition by the same token that if his demurrer is granted by the trial court, and the order of
dismissal is reversed on appeal, the movant loses his right to present evidence in his behalf and he shall have been
deemed to have elected to stand on the insufficiency of plaintiff’s case and evidence. In such event, the appellate
court which reverses the order of dismissal shall proceed to render judgment on the merits on the basis of plaintiff’s
evidence.
In other words, defendants who present a demurrer to the plaintiff’s evidence retain the right to present their own
evidence, if the trial court disagrees with them; if the trial court agrees with them, but on appeal, the appellate court
disagrees with both of them and reverses the dismissal order, the defendants lose the right to present their own
evidence. The appellate court shall, in addition, resolve the case and render judgment on the merits, inasmuch as a
demurrer aims to discourage prolonged litigations.
In the case at bar, the trial court, acting on the spouses’ demurrer to evidence, dismissed the Complaint on the
ground that Radiowealth had adduced mere hearsay evidence. However, on appeal, the appellate court reversed the
trial court because the genuineness and the due execution of the disputed pieces of evidence had in fact been
admitted by the spouses. Applying Rule 33, Section 1 of the 1997 Rules of Court, the CA should have rendered
judgment on the basis of the evidence submitted by the petitioner. In fact, the CA had sufficient evidence on record
to decide the collection suit. A remand is not only frowned upon by the Rules, it is also logically unnecessary on the
basis of the facts on record.
Remedial Law I| Page 163

Cabador vs. People


G.R. No. 186001, October 2, 2009
Facts:
Antonio Cabador was charged before the RTC of Quezon City of murdering, in conspiracy with others, Atty. Valerio.
After presenting only five witnesses over five years of intermittent trial, the RTC declared at an end the
prosecution’s presentation of evidence and required the prosecution to make a written or formal offer of its
documentary evidence within 15 days from notice. The public prosecutor, however, asked for three extensions of
time, the last of which was to end on July 28, 2006. Still, the prosecution did not make the required written offer.
Cabador then filed a motion to dismiss the case, complaining of a turtle-paced proceeding since his arrest and
detention in 2001 and invoking his right to a speedy trial. Unknown to Cabador, four days earlier or on July 28,
2006, the prosecution asked the RTC for another extension of the period for its formal offer, which offer it eventually
made on August 1, 2006, the day Cabador filed his motion to dismiss. The RTC issued an Order treating Cabador’s
motion to dismiss as a demurrer to evidence. And, since he filed his motion without leave of court, the RTC declared
him to have waived his right to present evidence in his defense. The trial court deemed the case submitted for
decision. Cabador filed a motion for reconsideration but it was denied. The CA denied Cabador’s petition and
affirmed the lower court’s actions. Hence, the present petition for review on certiorari.
Issue:
Whether or not Cabador’s motion to dismiss before the trial court was in fact a demurrer to evidence filed without
leave of court?
Held:
No. In Enojas, Jr. v. Commission on Elections, the Court held that, to determine whether the pleading filed is a
demurer to evidence or a motion to dismiss, the Court must consider (1) the allegations in it made in good faith; (2)
the stage of the proceeding at which it is filed; and (3) the primary objective of the party filing it.
Here, the pertinent portions of Cabador’s motion to dismiss shows that he took pains to point how trial in the case
had painfully dragged on for years. The gaps between proceedings were long, with hearings often postponed because
of the prosecutor’s absence. This was further compounded, Cabador said, by the prosecutions repeated motions for
extension of time to file its formal offer and its failure to file it within such time.
In criminal cases, a motion to dismiss may be filed on the ground of denial of the accused’s right to speedy trial. This
denial is characterized by unreasonable, vexatious, and oppressive delays without fault of the accused, or by
unjustified postponements that unreasonably prolonged the trial. The fact is that Cabador did not even bother to do
what is so fundamental in any demurrer. He did not state what evidence the prosecution had presented against him
to show in what respects such evidence failed to meet the elements of the crime charged. His so-called demurrer did
not touch on any particular testimony of even one witness. He cited no documentary exhibit. Indeed, he could not
because, he did not know that the prosecution finally made its formal offer of exhibits on the same date he filed his
motion to dismiss.
Besides, a demurrer to evidence assumes that the prosecution has already rested its case. Here, after the
prosecution filed its formal offer of exhibits, the same day Cabador filed his motion to dismiss, the trial court still
needed to give him an opportunity to object to the admission of those exhibits. It also needed to rule on the formal
offer. And only after such a ruling could the prosecution be deemed to have rested its case. Since Cabador filed his
motion to dismiss before he could object to the prosecution’s formal offer, before the trial court could act on the offer,
and before the prosecution could rest its case, it could not be said that he had intended his motion to dismiss to serve
as a demurrer to evidence.
Remedial Law I| Page 164

Claudio vs. Saraza


G.R. No. 213286, August 26, 2015

Facts:

Mamerta Lopez et. al., filed an action for annulment of sale, power of attorney and mortgage with prayer for
damages before the RTC against Florentino Claudio and Spouses. Saraza. They allege that Florentino made it
appear that his parents sold to him the lot subject of the action through an Absolute Deed of Sale. Furthermore,
they allege that the signatures of the vendors were forged. Later on, Florentino executed a deed of real estate
mortgage in favor of Spouses Saraza to secure his loan from them. That Spouses Saraza were mortgagees in bad
faith because they knew fully well that Florentino could not have acquired the subject property from his parents
because Porfirio had long been deceased while Mamerta was in the United States of America at the time of the
alleged sale and lastly, that it was only a few days after the execution of the mortgage that the TCT under
Florentino’s name was issued.

Spouses Saraza filed a Demurrer to Evidence after the presentation of the evidence in chief of the prosecution which
was granted. On appeal, the Court of Appeals affirmed the RTC’s grant of the demurrer. However, the Supreme
Court reversed both the RTC and the CA.

Issue:

Whether or not remand of the proceedings to the RTC is proper when the Order grating the demurrer to evidence
was subsequently reversed by the Supreme Court?

Held:

Yes. Petitioners' evidence, standing alone and in the absence of controverting evidence, would afford sufficient basis
for a judgment in their favor and against Spouses Saraza. Despite the fact that Spouses Saraza are deemed to have
waived their right to present evidence before the RTC pursuant to Section 1, Rule 33 of the Rules of Court, still this
disposition is without prejudice to the judgment on the merits to be handed down by the RTC.
Remedial Law I| Page 165

Felipe vs. MGM Motor Trading Corporation


GR No. 191849, September 23, 2015

Facts:

Frederick Felipe filed a Complaint for Specific Performance and Damages against MGM Motors, and Ayala General
Insurance Corporation. He claims that he purchased on installment basis a Nissan Terrano Wagon through MGM
Motors' authorized representative Jane Sarmiento. He allegedly paid 200,000 pesos as downpayment and P5,000.00
reservation fee to Sarmiento. He further issued seven (7) Allied Bank checks, each bearing the amount of P24,165.00
payable to MGM Motors. After it was delivered to him, he insured it with Ayala Insurance. However, the vehicle
was reportedly stolen while parked in Adriatico Street. As a result, he tried to claim from Ayala Insurance but it
was refused. On the other hand, despite his repeated demands, MGM Motors refused to issue him the document of
sale on installment.

In its Answer, MGM stated that it offered Felipe a discount of 200k if he would pay in cash but since the latter failed
to pay in cash, MGM did not give him the registration papers. Also, MGM Motors stated that petitioner was able to
fraudulently register the vehicle with the Land Transportation Office in his name and insure the same with Ayala
Insurance. During a negotiation, MGM and Felipe agreed to the amount of 1,020,000 as full payment and as a
consequence, Felipe’s mother issued a post dated check in such amount but it bounced. A case for BP 22 was filed
against Felipe’s mother.

During the trial of this case, Only two pieces of evidence were admitted by the trial court: (1) the Official Receipt
dated 7 May 1998 issued by MGM Motors wherein it acknowledged receipt of P200,000.00 from petitioner; and (2)
the testimony of his father Alberto that he was present when petitioner paid P200,000.00 to MGM Motors.

MGM Motors and Ayala Insurance filed their respective Motions to Dismiss on demurrer to evidence. The RTC
granted it and dismissed the case. This was likewise affirmed by the CA. Hence, this petition.

Issue:

Whether or not the demurrer to evidence was correctly granted?

Held:

Yes. The essential question to be resolved in a demurrer to evidence is whether the plaintiff has been able to show
that he is entitled to his claim, and it is incumbent upon the trial court judge to make such a determination.

Well-established is the rule that the burden of proof lies on the party who makes the allegations. There is no dispute
that the only pieces of evidence admitted in court are the testimony of Alberto and the receipt showing MGM Motors
receiving P200,000.00 from petitioner as partial payment of the subject car. The allegation that the purchase of the
vehicle was on an installment basis was not supported by any evidence. The receipt of a partial payment does not
suffice to prove that the purchase was made on an installment basis. Petitioner did not present any document to
prove said allegation while MGM Motors produced a sales invoice wherein it was stated that the mode of payment is
"COD" or cash on delivery.

In the same vein, petitioner failed to substantiate his allegation against Ayala Insurance. Petitioner has the burden
of proof to show that a loss occurred and said loss was covered by his insurance policy. Considering that the trial
court only admitted two pieces of evidence in petitioner's favor and none of those tend to prove loss of the subject car
and coverage thereof under the insurance policy, petitioner is not entitled to the reliefs he had prayed for.
Remedial Law I| Page 166

Rule 34-35: Judgment on the Pleadings/Summary Judgment

Republic vs. Pilipinas Shell Petroleum Corp.


Facts:

Filipino Way Industries assigned Tax Credit Certificates (TCCs) to Pilipinas Shell Petroleum Corp. (PSPC).
Believing that the TCCs were actually good and valid, the Bureau of Customs (BOC) accepted and allowed PSPC to
use the above TCCs to pay the customs duties and taxes due on its oil importations. However, the TCCs were later
on invalidated for having been fraudulently issued and transferred. The Republic of the Philippines represented by
the BOC then filed a collection suit in the RTC against PSPC. On the other hand, PSPC filed with the Court
of Tax Appeals a petition for review questioning the factual and legal bases of BOC's collection efforts. However,
upon reaching the SC, that case was eventually dismissed.

With the resumption of proceedings in the RTC, PSPC filed a motion for summary judgment arguing that there is no
basis for the Republic's claims considering that the subject TCCs were already fully utilized for the payment of
PSPC's customs duties and taxes, and that EXCOM Resolution No. 03-05-99, the basis of the cancellation of the
TCCs, was declared void and invalid in Pilipinas Shell Petroleum Corporation v. CIR, where the Supreme Court
likewise ruled that the subject TCCs cannot be cancelled on the basis of post-audit since a post-audit is not allowed
and not a suspensive condition.

In its Comment/Opposition, BOC argued that rendition of summary judgment is inappropriate in this case in view of
disputed facts that necessitate a full-blown trial where both parties can present evidence on their respective claims.

RTC denied the motion for summary judgment. However, on MR by PSPC, the RTC reversed its earlier ruling and
granted the motion for summary judgment. With the denial of its motion for reconsideration, BOC appealed to the
CA. CA denied the appeal and affirmed the questioned orders of the RTC. BOC's motion for reconsideration was
likewise denied by the CA. Hence, this petition for review under Rules 45.

Issues:

(1) Whether or not summary judgment is proper?

(2) What is the proper remedy against a summary judgment?

Held:

(1) No. Under Rule 35 of the 1997 Rules of Civil Procedure, as amended, except as to the amount of damages, when
there is no genuine issue as to any material fact and the moving party is entitled to a judgment as a matter of law,
summary judgment may be allowed. Even if on their face the pleadings appear to raise issues, when the affidavits,
depositions and admissions show that such issues are not genuine, then summary judgment as prescribed by the
Rules must ensue as a matter of law. The determinative factor, therefore, in a motion for summary judgment, is the
presence or absence of a genuine issue as to any material fact.

For a full-blown trial to be dispensed with, the party who moves for summary judgment has the burden of
demonstrating clearly the absence of genuine issues of fact, or that the issue posed is patently insubstantial as to
constitute a genuine issue. Genuine issue means an issue of fact which calls for the presentation of evidence as
distinguished from an issue which is fictitious or contrived. Trial courts have limited authority to render summary
judgments and may do so only when there is clearly no genuine issue as to any material fact. When the facts as
pleaded by the parties are disputed or contested, proceedings for summary judgment cannot take the place of trial.
As certain facts pleaded are contested by the parties in this case, rendition of summary judgment is not proper.

(2) Petition for Review Under Rule 45. Any review by the appellate court of the propriety of the summary
judgment rendered by the trial court based on these pleadings would not involve an evaluation of the probative
value of any evidence, but would only limit itself to the inquiry of whether the law was properly applied given the
facts and these supporting documents. Therefore, what would inevitably arise from such a review are pure questions
Remedial Law I| Page 167

of law, and not questions of fact, which are not proper in an ordinary appeal under Rule 41, but should be raised by
way of a petition for review on certiorari under Rule 45.

Petitioner raised as sole issue in its brief filed with the CA the RTC's erroneous grant of summary judgment in favor
of PSPC based on its finding that there exists no genuine factual issue. Obviously, it availed of the wrong mode of
appeal when it filed a notice of appeal in the RTC under Section 2(a), Rule 41, instead of a petition for review on
certiorari in this Court under Rule 45. Notwithstanding petitioner's wrong mode of appeal, the CA should not have
so easily dismissed the petition, considering that the parties involved are local government units and that what is
involved is the determination of their respective territorial jurisdictions.
Remedial Law I| Page 168

Comglas Corp. vs. Santos Car Check Center


G.R. No. 202989. March 25, 2015.

Facts:

Santos Car Check Center leased a space to Comglasco for a period of five years. However, the latter advised Santos
through a letter that it was pre-terminating their lease contract. Santos refused to accede to the pre-termination,
reminding Comglasco that their contract was for five years. Thereafter, Comglasco vacated the leased premises and
stopped paying any further rentals. Santos sent several demand letters, which Comglasco completely ignored.
Eventually, Santos sent its final demand letter, which Comglasco again ignored which led Santos to file a suit for
breach of contract. Comglasco filed its Answer. Santos then moved for a judgment on the pleadings, which the RTC
granted. Judgment was rendered in favor of Santos and against Comglasco. The CA affirmed the RTC’s Order.
Hence, this petition. Comglasco maintains that the RTC was wrong to rule that its answer to Santos’ complaint
tendered no issue, or admitted the material allegations therein; that the court should have heard it out on the
reason it invoked to justify its action to pre-terminate the parties’ lease; that therefore a summary judgment would
have been the proper recourse, after a hearing.

Issue:

Whether or not judgment on the pleadings is proper in this case?

Held:

Yes. Relying on Article 1267 of the Civil Code to justify its decision to pre-terminate its lease with Santos,
Comglasco invokes the 1997 Asian currency crisis as causing it much difficulty in meeting its obligations. Article
1267 speaks of a prestation involving service which has been rendered so difficult by unforeseen subsequent events
as to be manifestly beyond the contemplation of the parties. To be sure, the Asian currency crisis befell the region
from July 1997 and for sometime thereafter, but Comglasco cannot be permitted to blame its difficulties on the said
regional economic phenomenon because it entered into the subject lease only on August 16, 2000, more than three
years after it began, and by then Comglasco had known what business risks it assumed when it opened a new shop
in Iloilo City.

As found by the CA, Comglasco’s Answer admitted the material allegations in the complaint, to wit: a) that Santos
holds absolute title to a showroom space; b) that Comglasco leased the said showroom from Santos; c) that after a
little over a year, Comglasco pre-terminated the lease; d) that, disregarding Santos’ rejection of the pre-termination
of their lease, Comglasco vacated the leased premises on January 15, 2002; e) that Comglasco never denied the
existence and validity of the parties’ lease contract. Specifically, the CA noted that Paragraph 2 of the Answer
admitted the allegations in Paragraphs 2, 3 and 4 of the complaint that the lease was for five years, starting on
August 16, 2000 and to expire on August 15, 2005, at a monthly rental of 60,000.00 on the first year, 66,000.00 on
the second year, and 72,600.00 on the third up to the fifth year.

The RTC acted correctly in resorting to Section 1 of Rule 34, on Judgment on the Pleadings, to cut short a needless
trial. Comglasco cannot cite Article 1267 of the Civil Code, and that it must be deemed to have admitted the
material allegations in the complaint. Section 1, Rule 34 reads:

A judgment on the pleadings is a judgment on the facts as pleaded, and is based exclusively upon the
allegations appearing in the pleadings of the parties and the accompanying annexes. It is settled that
the trial court has the discretion to grant a motion for judgment on the pleadings filed by a party if
there is no controverted matter in the case after the answer is filed. A genuine issue of fact is that
which requires the presentation of evidence, as distinguished from a sham, fictitious, contrived or false
issue.

Under Rule 35, on Summary Judgments, Comglasco had recourse to move for summary judgment, wherein it could
have adduced supporting evidence to justify its action on the parties' lease, but it did not do so. Section 2 of Rule 35
provides:
Remedial Law I| Page 169

Sec. 2. Summary judgment for defending party. - A party against whom a claim, counterclaim, or cross-
claim is asserted or a declaratory relief is sought may, at any time, move with supporting affidavits,
depositions or admissions for a summary judgment in his favor as to all or any part thereof.
Remedial Law I| Page 170

Philippine Bank of Communications vs. Go


G.R. No. 175514. February 14, 2011.
Facts:

Jose Go obtained two loans from PBCom, evidenced by two promissory notes, embodying his commitment to pay
P17,982,222.22 for the first loan, and P80 million for the second loan, within a ten-year period from September 30,
1999 to September 30, 2009. To secure the loans, he executed pledge agreements covering the shares of stock of Ever
Gotesco Resources and Holdings, Inc. Two years later, however, the market value of the said shares of stock plunged
to less than P0.04 per share. Thus, PBCom, as pledgee, notified Go in writing that it was renouncing the pledge
agreements. Later, PBCom filed before the RTC a complaint for some of money against the Spouses Go. PBCom
alleged that Spouses Go defaulted on the two promissory notes, having paid only three installments on interest
payments—covering the months of September, November and December 1999. Consequently, the entire balance of
the obligations of Go became immediately due and demandable. Spouses Go filed their Answer with Counterclaim
denying the material allegations in the complaint and stating (1) the loan obligation is payable within the period of
10 years so it is not yet due and demandable; (2) they made substantial payments so there is a need for accounting;
and (3) there was no prior demand.

PBCom filed a Motion for Summary Judgment alleging that the Answer interposed no specific denials on the
material averments in paragraphs 8 to 11 of the complaint such as the fact of default, the entire amount being
already due and demandable by reason of default, and the fact that the bank had made repeated demands for the
payment of the obligations. The RTC granted the Motion and rendered its judgment ordering the Spouses Go to pay
the obligation.

The Court of Appeals reversed the RTC. It found the supposed admission (as found by the RTC) to be insufficient to
justify a rendition of summary judgment in the case for sum of money, since there were other allegations and
defenses put up by Spouses Go in their Answer which raised genuine issues on the material facts in the action.

Hence, this petition.

Issue:

Whether or not a Summary Judgment was proper?

Held:

No. The CA correctly ruled that there exist genuine issues as to three material facts, which have to be addressed
during trial: first, the fact of default; second, the amount of the outstanding obligation, and third, the existence of
prior demand.

Under Rule 35 of the 1997 Rules of Procedure, except as to the amount of damages, when there is no genuine issue
as to any material fact and the moving party is entitled to a judgment as a matter of law, summary judgment may
be allowed. Summary or accelerated judgment is a procedural technique aimed at weeding out sham claims or
defenses at an early stage of litigation thereby avoiding the expense and loss of time involved in a trial.

Under the Rules, summary judgment is appropriate when there are no genuine issues of fact which call for the
presentation of evidence in a full- blown trial. Even if on their face the pleadings appear to raise issues, when the
affidavits, depositions and admissions show that such issues are not genuine, then summary judgment as prescribed
by the Rules must ensue as a matter of law. The determinative factor, therefore, in a motion for summary judgment,
is the presence or absence of a genuine issue as to any material fact.

A “genuine issue” is an issue of fact which requires the presentation of evidence as distinguished from a sham,
fictitious, contrived or false claim. When the facts as pleaded appear uncontested or undisputed, then there is no
real or genuine issue or question as to he facts, and summary judgment is called for. The party who moves for
summary judgment has the burden of demonstrating clearly the absence of any genuine issue of fact, or that the
issue posed in the complaint is patently unsubstantial so as not to constitute a genuine issue for trial. Trial courts
have limited authority to render summary judgments and may do so only when there is clearly no genuine issue as
to any material fact. When the facts as pleaded by the parties are disputed or contested, proceedings for summary
judgment cannot take the place of trial.
Remedial Law I| Page 171

Juxtaposing the Complaint and the Answer discloses that the material facts here are not undisputed so as to call for
the rendition of a summary judgment. While the denials of Spouses Go could have been phrased more strongly or
more emphatically, and the Answer more coherently and logically structured in order to overthrow any shadow of
doubt that such denials were indeed made, the pleadings show that they did in fact raise material issues that have
to be addressed and threshed out in a full-blown trial. The matters in contention are, as the CA stated, whether or
not respondents were in default, whether there was prior demand, and the amount of the outstanding loan. These
are the matters that the parties disagree on and by which reason they set forth vastly different allegations in their
pleadings which each will have to prove by presenting relevant and admissible evidence during trial.
Remedial Law I| Page 172

Adolfo vs. Adolfo
G.R. No. 201427. March 18, 2015.
Facts:

[MAIN CASE] Civil Case No. MAN-4821 – In 2004, Teofilo Adolfo filed with the RTC Mandaue a Petition for judicial
separation of property against his estranged wife, respondent Fe Adolfo. He alleged that the two are married and
that during their marriage, they acquired through conjugal funds Lot 1087-A-2-E, a 3,652-square-meter property
(the subject property). However, due to irreconcilable differences and since reunion is no longer possible, he prays for
a decree of separation of conjugal property. In her Answer, respondent contended that while she remained married
to petitioner, she is the sole owner of the subject property, the same being her paraphernal property which she
inherited from her mother. She thus prayed for the petition to be dismissed.

[OTHER CASE] Civil Case No. MAN-2683 – In 1996, respondent’s sister, Florencia Tudtud and her husband
(Spouses Gingoyons), filed a case for partition with damages against respondent. The Complaint alleged that the
respondent executed a deed of sale in favor of Spouses Gingoyons over a 300-square-mater portion of the subject
property but she refused to partition/subdivide the same despite payment. For her defense, respondent claimed in
her Answer that when the sale was made, the subject property constituted conjugal property of her marriage with
petitioner; since petitioner did not sign the deed of sale as he knew nothing about the sale, the sale was null and
void. The trial court ruled that the subject property constituted conjugal property of the marriage. Thus, it nullified
the deed of sale. This case was the subject of an appeal with the CA, which was docketed as C.A.-G.R. CV No. 78971.

[MAIN CASE] During the pretrial conference, petitioner submitted as evidence the copies of the Complaint, the
respondent’s answer and the Decision in Civil Case No. Man-2683. Petitioner then filed a Request for Admission of
the genuineness and due execution of the copies of the Complaint, Answer and Decision. Respondent failed to file
her response to the request for admission. Consequently, petitioner filed a Motion for Judgment Based on the
Pleadings stating that since respondent failed to answer his request for admission, the matters contained in the
request are deemed admitted; that as a consequence of the application of the rule, respondent is in effect considered
to have admitted that the subject property is a conjugal asset of their subsisting marriage which may thus be the
subject of his petition for judicial separation of property; and that on account of said admission, a hearing on the
merits becomes unnecessary. The trial court then directed the transfer of the case to Branch 55 (the court which
decided Civil Case No. MAN-2683). Branch 55 then granted petitioner’s Motion; it treated the Motion for Judgment
on the Pleadings as one for Summary Judgment. It held that the failure of respondent to respond to the Request for
Admission was an admission that Lot 1087-A-2-E is a conjugal property, thus, petitioner is entitled for partition.
Respondent appealed the case to the CA docketed as C.A.-G.R. CV No. 01783.

[OTHER CASE] C.A.-G.R. CV No. 78971: The CA reversed the RTC and declared that the subject property was
respondent’s paraphernal property. It ordered the partition of the property. This decision became final and
executory.

[MAIN CASE] C.A.-G.R. CV No. 01783: The CA held that the trial court cannot treat petitioner’s motion for
judgment on the pleadings as one for summary judgment. It then remanded the case back to the RTC. The petitioner
then filed a petition for review before the SC.

Issue:

Whether or not the trial court erred in rendering a summary judgment on Civil Case No. MAN-4821 despite the
pendency of the appeal in Civil Case No. MAN-2683?

Held:

Yes. Judgment on the pleadings is proper “where an answer fails to tender an issue, or otherwise admits the
material allegations of the adverse party’s pleading.” Summary judgment, on the other hand, will be granted “if the
pleadings, supporting affidavits, depositions, and admissions on file, show that, except as to the amount of damages,
there is no genuine issue as to any material fact and that the moving party is entitled to a judgment as a matter of
law.

The existence or appearance of ostensible issues in the pleadings, on the one hand, and their sham or fictitious
character, on the other, are what distinguish a proper case for summary judgment from one for a judgment on the
pleadings. In a proper case for judgment on the pleadings, there is no ostensible issue at all because of the failure of
the defending party’s answer to raise an issue. On the other hand, in the case of a summary judgment, issues
apparently exist — i.e., facts are asserted in the complaint regarding which there is as yet no admission, disavowal
Remedial Law I| Page 173

or qualification; or specific denials or affirmative defenses are in truth set out in the answer — but the issues thus
arising from the pleadings are sham, fictitious or not genuine, as shown by affidavits, depositions, or admissions.

In rendering summary judgment, the trial court relied on respondent’s failure to reply to petitioner’s request for
admission, her admission in Civil Case No. MAN-2683, as well as its May 15, 2002 Decision declaring that the
subject property is a conjugal asset. It took judicial notice of the proceedings in said case. While there is nothing
irregular with this — as courts may “take judicial notice of a decision or the facts prevailing in another case sitting
in the same court, the trial court however disregarded the fact that its decision was then the subject of a pending
appeal in C.A.- G.R. CV No. 78971. It should have known that until the appeal is resolved by the appellate court, it
would be premature to render judgment on petitioner’s motion for judgment on the pleadings; that it would be
presumptuous to assume that its own decision would be affirmed on appeal. One of the issues raised in the appeal is
precisely whether the subject property is conjugal, or a paraphernal asset of the respondent. Thus, instead of
resolving petitioner’s motion for judgment on the pleadings, the trial court should have denied it or held it in
abeyance.

On the part of petitioner, it must be said that he could not have validly resorted to a motion for judgment on the
pleadings or summary judgment. While it may appear that under Rules 34 and 35 of the 1997 Rules, he may file a
motion for judgment on the pleadings or summary judgment as a result of the consequent admission by respondent
that the subject property is conjugal, this is not actually the case. Quite the contrary, by invoking the proceedings
and decision in Civil Case No. MAN-2683, petitioner is precluded from obtaining judgment while the appeal in said
case is pending, because the result thereof determines whether the subject property is indeed conjugal or
paraphernal. He may not preempt the appeal in C.A.-G.R. CV No. 78971.

Finally, the Court notes that the appellate court overlooked the May 30, 2007 Decision in C.A.-G.R. CV No. 78971,
which became final and executory on June 23, 2007. Because of this, petitioner’s case is left with no leg to stand on.
There being no conjugal property to be divided between the parties, Civil Case No. MAN-4821 must be dismissed.
Remedial Law I| Page 174

Rule 37: Motion for Reconsideration/New Trial

Mendezona vs. Ozamis


G.R. No. 143370. February 6, 2002

Facts:

The petitioners (Spouses Mario Mendezona and Teresita Mendezona, et al.) alleged that they own a parcel of
land in Lahug, Cebu City and that they traced their titles of ownership over their respective properties from a
notarized Deed of Absolute Sale executed in their favor by Carmen Ozamiz. They initiated a suit for quieting of
title to remove a cloud on their respective titles caused by the inscription thereon of a notice
of lis pendens, which came about as a result of an incident in special proceeding for guardianship over the
person and properties of Carmen Ozamiz initiated by the respondents (Julio Ozamis, et al.) in the RTC.
Respondents instituted the petition for guardianship, alleging that Carmen Ozamiz, then 86 years old, after an
illness, had become disoriented and could no longer take care of herself nor manage her properties. Respondent
Paz O. Montalvan was designated as guardian over the person of Carmen Ozamiz while petitioner Mario
J. Mendezona, respondents Roberto J. Montalvan and Julio H. Ozamiz were designated as joint guardians over
the properties of the said ward. As guardians, respondents filed their inventories and accounts, listing therein
Carmen Ozamiz’s properties, including the said Lahug property. In their Answer, the respondents opposed the
petitioners’ claim of ownership of the Lahug property and alleged that the titles issued in the petitioners’
names are defective and illegal, and the ownership of the said property was acquired in bad faith and without
value. Respondents further alleged that at the time of the sale, Carmen Ozamiz was already ailing and not in
full possession of her mental faculties.

Trial on the merits ensued. The trial court rendered its decision in favor of the petitioners. On appeal to the CA,
the appellate court reversed the factual findings of the trial court. Petitioners filed a motion for reconsideration
of the decision of the appellate court. Subsequent thereto, the petitioners filed a motion for a new trial and/or
for reception of evidence. They contended, among other things, that the appellate court totally ignored the
testimony of Judge Teodorico Durias regarding the mental condition of Carmen Ozamiz a month before the
execution of the Deed of Absolute Sale in question. The said testimony was taken in the special proceeding.
However, Judge Durias was not presented as a witness. Petitioners alleged that Judge Duriass testimony is a
newly-discovered evidence which could not have been discovered prior to the trial in the court below by the
exercise of due diligence. The appellate court denied both motions.

Issue:

Whether or not the testimony of Judge Durias in the Special Proceeding can be considered as newly discovered
evidence?

Held:

No. A motion for new trial upon the ground of newly discovered evidence is properly granted only where there is
concurrence of the following requisites, namely: (a) the evidence had been discovered after trial; (b) the evidence
could not have been discovered and produced during trial even with the exercise of reasonable diligence; and (c)
the evidence is material and not merely corroborative, cumulative or impeaching and is of such weight that if
admitted, would probably alter the result. All three (3) requisites must characterize the evidence sought to be
introduced at the new trial.

The requirement of reasonable diligence has not been met by the petitioners. As early as the pre-trial of the
case, the name of Judge Durias has already cropped up as a possible witness for the defendants,
herein respondents. Judge Durias testimony in another case cannot be considered as newly discovered evidence
since the facts to be testified to by Judge Durias which were existing before and during the trial, could have
been presented by the petitioners at the trial below. The testimony of Judge Durias has been in existence
waiting only to be elicited from him by questioning.
Remedial Law I| Page 175

It has been held that a lack of diligence is exhibited where the newly discovered evidence was necessary or
proper under the pleadings, and its existence must have occurred to the party in the course of the preparation
of the case, but no effort was made to secure it; there is a failure to make inquiry of persons who were likely to
know the facts in question, especially where information was not sought from co-parties; there is a failure to
seek evidence available through public records; there is a failure to discover evidence that is within the control
of the complaining party; there is a failure to follow leads contained in other evidence; and, there is a failure to
utilize available discovery procedures. Thus, the testimony of Judge Durias cannot be considered as newly
discovered evidence to warrant a new trial.
Remedial Law I| Page 176

Chua vs. People


G.R. No. 196853. July 13, 2015

Facts:

Robert Chua issued several postdated PS Bank checks to Philip See pursuant to their rediscounting
arrangement. However, See claimed that when he deposited the checks, they were dishonored either due to
insufficient funds or closed account. Despite demands, Chua failed to make good the checks. See filed a
complaint for violations of BP 22. He attached thereto a demand letter. Chua was charged with 54 counts of
violation of BP 22 before the MeTC. The prosecution formally offered as its evidence the demand letter dated
December 10, 1993. Chua, however, objected to its admissibility on the grounds that it is a mere photocopy and
that it does not bear any proof that he actually received it. Chua filed a Motion to Submit Demurrer to
Evidence. However, the MeTC failed to act on his motion since the judge of said court vacated his post.

Several years later, the prosecution filed a Motion to Re-Open Presentation of Prosecution's Evidence and
Motion to Allow Prosecution to Submit Additional Formal Offer of Evidence. It averred that while See was still
trying to locate a demand letter dated November 30, 1993 which it alleged to have been personally served upon
Chua. However, See decided to have his house rented out such that he emptied it with all his belongings and
had it cleaned. It was during this time that he found the demand letter dated November 30, 1993. The
prosecution thus prayed that it be allowed to submit a supplemental offer of evidence to include said demand
letter dated November 30, 1993 as part of its evidence. Again, the records of the case bear no copy of an MeTC
Order or Resolution granting the aforesaid motion. Nevertheless, extant on records is a Formal Offer of
Evidence filed by the private prosecutor submitting the demand letter dated November 30, 1993 as additional
evidence. MeTC refused to take cognizance of the supplemental formal offer on the ground that the same was
filed by the private prosecutor without the conformity of the public prosecutor. Be that as it may, the demand
letter dated November 30, 1993 eventually found its way into the records of this case. MeTC convicted Chua of
54 counts of violation of BP 22. The RTC affirmed the ruling of the MeTC.

Before the CA, Chua argued against the probative value of the demand letter dated November 30, 1993 by
pointing out that: (1) for more than 10 years from the time the case was filed, the prosecution never adverted to
its existence. He thus surmised that this was because the document was not really missing but in fact
inexistent - a mere afterthought as to make it appear that the second element of the offense is obtaining in the
case; (2) the subject demand letter is not a newly discovered evidence as it could have been discovered earlier
through the exercise of due diligence; and, (3) his counsel's admission of the physical existence of the subject
demand letter and Chua's signature thereon does not carry with it the admission of its contents and his receipt
of the same. The CA likewise affirmed the decision of the lower court.

Issue:

Whether or not demand letter dated November 30, 1993 is a newly discovered evidence?

Held:

No. The question of whether evidence is newly discovered has two aspects: a temporal one, i.e., when was the
evidence discovered, and a predictive one, i.e., when should or could it have been discovered. It is to the latter
that the requirement of due diligence has relevance. In order that a particular piece of evidence may be
properly regarded as newly discovered to justify new trial, what is essential is not so much the time when the
evidence offered first sprang into existence nor the time when it first came to the knowledge of the party now
submitting it; what is essential is that the offering party had exercised reasonable diligence in seeking to locate
such evidence before or during trial but had nonetheless failed to secure it.

The Rules do not give an exact definition of due diligence, and whether the movant has exercised due diligence
depends upon the particular circumstances of each case. Nonetheless, it has been observed that the phrase is
often equated with "reasonable promptness to avoid prejudice to the defendant." In other words, the concept of
due diligence has both a time component and a good faith component. The movant for a new trial must not only
act in a timely fashion in gathering evidence in support of the motion; he must act reasonably and in good faith
Remedial Law I| Page 177

as well. Due diligence contemplates that the defendant acts reasonably and in good faith to obtain the evidence,
in light of the totality of the circumstances and the facts known to him.

Under the Rules of Court, the requisites for newly discovered evidence are: (a) the evidence was discovered after
trial; (b) such evidence could not have been discovered and produced at the trial with reasonable diligence; and (c) it
is material, not merely cumulative, corroborative or impeaching, and is of such weight that, if admitted, will
probably change the judgment."

In this case, the Court holds that the demand letter dated November 30, 1993 does not qualify as a newly discovered
evidence within the purview of the law. Per See's statements in his affidavit, the said evidence was already known
to him at the time he filed his complaint against Chua. It was also apparently available considering that it was just
kept in his house. Undeniably, had See exercised reasonable diligence, he could have promptly located the said
demand letter and presented it during trial.
Remedial Law I| Page 178

Padilla-Rumbaua vs. Rumbaua
G.R. No. 166738. August 14, 2009

Facts:

Petitioner (Rowena Padilla-Rumbaua) filed a petition for declaration of nullity of marriage against the respondent
(Edward Rumbaua) before the RTC on the ground of psychological incapacity. The Republic of
the Philippines (Republic), through the office of the Solicitor General (OSG), opposed the petition. The Regional
Trial Court nullified the marriage of the petitioner but on appeal to the CA by the OSG, the CA reversed the ruling
of the RTC and denied the nullification of the parties’ marriage. The petitioner moved to reconsider the decision, but
the CA denied her motion. Hence, this petition. The petitioner prays, among others, that the RTC’s and the CA’s
decisions be reversed and set aside, and the case be remanded to the RTC for further proceedings; in the event the
Court cannot grant this prayer, that the CA’s decision be set aside and the RTC’s decision be reinstated. She argues
that the inadequacy of her evidence during the trial was the fault of her former counsel, and asserts that remanding the case to
the RTC would allow her to cure the evidentiary insufficiencies.

Issue:

Whether or not the remand of the case for reception of additional evidence should be allowed?

Held:

No. A remand of the case to the RTC for further proceedings amounts to the grant of a new trial that is not procedurally proper at
this stage. Section 1 of Rule 37 provides that an aggrieved party may move the trial court to set aside a judgment or final
order already rendered and to grant a new trial within the period for taking an appeal. In addition, a motion for new
trial may be filed only on the grounds of (1) fraud, accident, mistake or excusable negligence that could not have
been guarded against by ordinary prudence, and by reason of which the aggrieved party’s rights have probably been
impaired; or (2) newly discovered evidence that, with reasonable diligence, the aggrieved party could not have
discovered and produced at the trial, and that would probably alter the result if presented.

In the present case, the petitioner cites the inadequacy of the evidence presented by her former counsel as basis for a
remand. She did not, however, specify the inadequacy. That the RTC granted the petition for declaration of
nullity prima facie shows that the petitioner’s counsel had not been negligent in handling the case.
Granting arguendo that the petitioner’s counsel had been negligent, the negligence that would justify a new trial
must be excusable, i.e. one that ordinary diligence and prudence could not have guarded against.

As cited by the Court in Uy v. First Metro Integrated Steel Corporation, it explained:

“Blunders and mistakes in the conduct of the proceedings in the trial court as a result of the ignorance, inexperience
or incompetence of counsel do not qualify as a ground for new trial. If such were to be admitted as valid reasons for
re-opening cases, there would never be an end to litigation so long as a new counsel could be employed to allege and
show that the prior counsel had not been sufficiently diligent, experienced or learned.”
Remedial Law I| Page 179

Senit vs. People


G.R. No. 192914. January 11, 2016

Facts:

Petitioner (Napoleon Senit) was charged with Reckless Imprudence Resulting to Multiple Serious Physical Injuries
and Damage to Property in an Amended Information which was filed with Branch 10 of the RTC in Malaybalay
City. Such information stemmed from a collision of the Super Five Nissan Bus driven by the petitioner and the
Toyota pick-up vehicle owned and driven by private respondent Toor, Sr., with his wife, son and housemaid riding
with him. Petitioner pleaded not guilty upon being arraigned. Trial thereafter ensued. However, after the initial
presentation of evidence for the petitioner, he resigned from his employment and transferred residence. His
whereabouts allegedly became unknown so he was not presented as a witness by his new counsel. The RTC rendered
its Decision in absentia convicting the petitioner of the crime charged.

The RTC issued a Promulgation, which included an order for the arrest of the petitioner. The petitioner then filed a
motion for new trial via registered mail on the ground that errors of law or irregularities have been committed
during trial that are allegedly prejudicial to his substantial rights. He claimed that he was not able to present
evidence during trial because he was not notified of the schedule. Likewise, he mistakenly believed that the case
against him has been dismissed as private complainant Toor, Sr. purportedly left the country. The motion for new
trial was denied by the lower court pronouncing that notices have been duly served the parties and that the reason
given by the petitioner was self-serving. On appeal to the CA, the CA affirmed the decision of the RTC and ruled,
among others, that the arguments of the petitioner to move for a new trial as baseless. The petitioner filed a motion
for reconsideration but was denied by the CA, hence this petition. The Petitioner theorizes that there was an error of
law or irregularities committed when the RTC promulgated a decision in absentia and deemed that he had waived
his right to present evidence resulting to denial of due process, a one-sided decision by the RTC, and a strict and
rigid application of the Revised Rules of Criminal Procedure against him.

Issue:

Whether or not a motion for new trial or to re-open the case to allow the petitioner to present evidence should be
allowed?

Held:

No. No errors of law or irregularities, prejudicial to the substantial rights of the petitioner, have been committed
during trial. The petitioner anchors his motion for new trial on Rule 121, Section 2(a) of the Revised Rules of
Criminal Procedure.

First, it must be noted that the petitioner had already been arraigned and therefore, the court a quo had already
acquired jurisdiction over him. In fact, there was already an initial presentation of evidence for the defense when his
whereabouts became unknown. Also, the petitioner clearly had previous notice of the criminal case filed against him
and was given the opportunity to present evidence in his defense. The petitioner was not in any way deprived of his
substantive and constitutional right to due process as he was duly accorded all the opportunities to be heard and to
present evidence to substantiate his defense, but he forfeited this right, through his own negligence, by not
appearing in court at the scheduled hearings. The negligence of the petitioner in believing that the case was already
terminated resulting to his failure to attend the hearings, is inexcusable. The Court finds that the negligence
exhibited by the petitioner, towards the criminal case against him in which his liberty is at risk, is not borne of
ignorance of the law as claimed by his counsel rather, lack of concern towards the incident, and the people who
suffered from it. While there was no showing in the case at bar that the counsel of the petitioner was grossly
negligent in failing to inform him of the notices served, the Court cannot find anyone to blame but the petitioner
himself in not exercising diligence in informing his counsel of his whereabouts.

A motion for new trial based on newly-discovered evidence may be granted only if the following requisites are met:
(a) that the evidence was discovered after trial; (b) that said evidence could not have been discovered and produced
at the trial even with the exercise of reasonable diligence; (c) that it is material, not merely cumulative,
Remedial Law I| Page 180

corroborative or impeaching; and (d) that the evidence is of such weight that, if admitted, it would probably change
the judgment. It is essential that the offering party exercised reasonable diligence in seeking to locate the evidence
before or during trial but nonetheless failed to secure it."

In the instant case, the Court finds no reason to waive the procedural rules in order to grant the motion for new trial
of the petitioner. There is just no legal basis for the grant of the motion for new trial. The Court believes that the
petitioner was given the opportunity to be heard but he chose to put this opportunity into waste by not being
diligent enough to ask about the status of the criminal case against him and inform his counsel of his whereabouts.
Remedial Law I| Page 181

People vs. Li Ka Kim
G.R. No. 148586. May 25, 2004

Facts:

The Regional Intelligence and Investigation Division (RIID) of PNP-Region IV received a report from an informer,
named "Boy," that a certain alias "Ed," known to be a drug dealer operating in the southern part of Metro Manila,
was looking for a buyer of shabu. PO2 Trambulo, an officer of RIID, made initial contact with Ed through a phone
call using Boy’s cellular phone. Boy introduced PO2 Trambulo to Ed as "Rollie," a buyer of shabu. The parties agreed
to meet at a mall in Parañaque City. The buy-bust operation was successfully carried-out where Ed was arrested.
An Information was then filed against Ed for violation of R.A. 6425, as amended.

Acting upon advice of counsel, Ed did not enter any plea during his arraignment; the court was thus constrained to
enter a plea of "not guilty" in his behalf. At the trial, PO2 Trambulo pointed to Ed as being the seller of the
confiscated shabu and positively identified the brown paper bag given to him by Ed containing the prohibited drug
which he marked "CVT.” On the other hand, Ed denied the allegations. The RTC convicted Ed and decreed the
penalty of death. It debunked Ed’s defense of denial. The court found it hard to believe that Ed would be singled out
by the police officers from scores of people at the mall. The court likewise noted that the car, as well as the license
plate, used by Ed had been stolen, and that he was an undocumented alien as so shown by the letter of the
Commissioner on Immigration and Deportation to the State Prosecutor.

Ed then filed a motion to remand the case for new trial. In his motion, he asked for a new trial, citing Section 14,
Rule 121, of the Rules on Criminal Procedure, because of newly discovered evidence, i.e., his passport which would
establish his true identity as Huang Xiao Wei, a Chinese National, and as having entered the Philippines as a
tourist.

Issue:

Whether or not the passport can be considered as newly discovered evidence as a ground for new trial?

Held:

No. The requisites of newly discovered evidence in order to justify a new trial are that - (a) the evidence is discovered
after trial; (b) such evidence could not have been discovered and produced at the trial even with the exercise of
reasonable diligence; and (c) the evidence is material, not merely cumulative, corroborative, or impeaching, and of
such weight that, if admitted, would likely change the judgment.

Not one of the requisites mentioned is attendant. Ed’s passport could have easily been presented and produced
during the trial. Then, too, the presentation of his passport, would hardly be material to the outcome of the case. Ed
was positively identified by the prosecution witnesses as being the perpetrator of the crime. Most importantly, Ed
even identified himself as Li Ka Kim at the trial and not as Huang Xiao Wei, that bolsters the conclusion that he
deliberately concealed his true identity in the nefarious enterprise. Also, the totality of evidence presented is
convincing and points to Ed as being a person engaged in the sale of illegal drugs. The testimony of the prosecution
witnesses identifying him to be a seller of illegal drugs appears to be categorical and unfabricated.
Remedial Law I| Page 182

Rules 40-45

Heirs of Arturo Garcia I (In substitution of Heirs of Melecio Bueno) vs. Municipality of Iba, Zambales G.R. No.
162217, July 22, 2015

Facts:

The late Melecio Bueno was the tenant-farmer beneficiary of an agricultural land located in Poblacion, Iba,
Zambales. Bueno brought an ejectment suit in the MTC of Iba against the Municipality of Iba, Province of Zambales,
claiming that in 1983, the Municipality of Iba had constructed the public market on a substantial portion of his land
without his consent; and that his repeated demands for the Municipality of Iba to vacate the property had remained
unheeded. The MTC ruled in favor of Bueno. The Municipality of Iba then filed its notice of appeal, but the MTC
denied the same. The Municipality of Iba thus filed its petition for certiorari in the RTC of Iba, Zambales to
assail the denial of due course by the MTC. The RTC granted the petition for certiorari. The herein petitioners-heirs,
who meanwhile substituted Bueno upon his death, moved for the reconsideration of the judgment granting the
petition for certiorari, but the RTC denied the same.

Aggrieved, the Heirs appealed to the Court of Appeals by petition for review under Rule 42 of the Rules of Court.
The CA dismissed the petition for review for not being the proper mode of appeal. It observed that the assailed
orders had been issued by the RTC in the exercise of its original jurisdiction. The Heirs filed a motion for
reconsideration but it was denied by the CA. Hence, the present case. Although admitting that their petition for
review under Rule 42 was inappropriate, the Heirs maintain that they substantially complied with the requirements
of an ordinary appeal under Rule 41, and pray that the Court exercise its equity jurisdiction.

Issue:

Whether or not a Petition for Review under Rule 42, despite being an improper remedy, may be treated as an
Ordinary Appeal under Rule 41?

Held:

No. An appeal brings up for review any error of judgment committed by a court with jurisdiction over the subject of
the suit and over the persons of the parties, or any error committed by the court in the exercise of its jurisdiction
amounting to nothing more than an error of judgment. It was, therefore, very crucial for the Heirs and their counsel
to have been cognizant of the different modes to appeal the adverse decision of the RTC in the special civil action for
certiorari brought by the Municipality of Iba. Such modes of appeal were well delineated in the Rules of Court, and
have been expressly stated in Section 2, Rule 41 of the Rules of Court since July 1, 1997, to wit:

Section 2. Modes of appeal.

(a) Ordinary appeal.- The appeal to the Court of Appeals in cases decided by the Regional Trial Court in the exercise
of its original jurisdiction shall be taken by filing a notice of appeal with the court which rendered the judgment or
final order appealed from and serving a copy thereof upon the adverse party. No record on appeal shall be required
except in special proceedings and other cases of multiple or separate appeals where the law or these Rules so require.
In such cases, the record on appeal shall be filed and served in like manner.

(b) Petition for review.- The appeal to the Court of Appeals in cases decided by the Regional Trial Court in the exercise
of its appellate jurisdiction shall be by petition for review in accordance with Rule 42.

(c) Appeal by certiorari.-In all cases where only questions of law are raised or involved, the appeal shall be to the
Supreme Court by petition for review on certiorari in accordance with Rule 45.

Pursuant to this rule, in conjunction with Section 3 and Section 4 of Rule 41, the Heirs should have filed a notice of
appeal in the RTC within the period of 15 days from their notice of the judgment of the RTC, and within the same
period, should have paid to the clerk of the RTC the full amount of the appellate court docket and other lawful fees.
Remedial Law I| Page 183

The filing of the notice of appeal within the period allowed by Section 3 sets in motion the remedy of ordinary appeal
because the appeal is deemed perfected as to the appealing party upon his timely filing of the notice of appeal. It is
upon the perfection of the appeal filed in due time, and the expiration of the time to appeal of the other parties that
the RTC shall lose jurisdiction over the case. On the other hand, the non-payment of the appellate court docket fee
within the reglementary period as required by Section 4, is both mandatory and jurisdictional, the non-compliance
with which is fatal to the appeal, and is a ground to dismiss the appeal under Section 1(c), Rule 50 of the Rules of
Court. The compliance with these requirements was the only way by which the Heirs could have perfected their
appeal from the adverse judgment of the RTC.

In contrast, an appeal filed under Rule 42 is deemed perfected as to the petitioner upon the timely filing of the
petition for review before the CA, while the RTC shall lose jurisdiction upon perfection thereof and the expiration of
the time to appeal of the other parties.

The distinctions between the various modes of appeal cannot be taken for granted, or easily dismissed, or lightly
treated. The appeal by notice of appeal under Rule 41 is a matter or right, but the appeal by petition for review
under Rule 42 is a matter of discretion. An appeal as a matter of right, which refers to the right to seek the review
by a superior court of the judgment rendered by the trial court, exists after the trial in the first instance. In contrast,
the discretionary appeal, which is taken from the decision or final order rendered by a court in the exercise of its
primary appellate jurisdiction, may be disallowed by the superior court in its discretion. Verily, the CA has the
discretion whether to give due course to the petition for review or not.

The procedure taken after the perfection of an appeal under Rule 41 also significantly differs from that taken under
Rule 42. Under Section 10 of Rule 41, the clerk of court of the RTC is burdened to immediately undertake the
transmittal of the records by verifying the correctness and completeness of the records of the case; the transmittal to
the CA must be made within 30 days from the perfection of the appeal. This requirement of transmittal of the
records does not arise under Rule 42, except upon order of the CA when deemed necessary.

As borne out in the foregoing, the Heirs' resort to the petition for review under Rule 42 was wrong. Hence, the CA
did not err in denying due course to the petition for review. The Heirs’ plea for liberality is also unworthy of any
sympathy from the Court, as the latter always looked at appeal as not a matter of right but a mere statutory
privilege. Moreover, the Heirs did not give any good reason or cause that could warrant the relaxation of the rules in
their favor.
Remedial Law I| Page 184

Estinozo vs. CA
G.R. No. 150276. February 12, 2008

Facts:

Cecilia Estinozo, a townmate of the complainants (Gaudencio Ang, et al), while in Sogod, Southern Leyte,
represented to them that she was one of the owners of Golden Overseas Employment and that she was recruiting
workers to be sent abroad. On the promised date of their departure, however, private complainants never left the
country. he then asked from the said complainants the payment of placement and processing fees
totaling P15,000.00. They were then informed by petitioner that there were no available plane tickets and that they
would leave by September of that year. After the failed promises of Estinozo, the complainants filed a case for estafa
against her which they eventually won, the trial court finding her guilty beyond reasonable doubt of estafa by false
pretenses.

The CA affirmed the decision of the trial court. within the 15-day reglementary period to file a motion for
reconsideration or a petition for review, Estinozo filed with the appellate court a Motion for Extension of Time to
File a Motion for Reconsideration. On June 28, 2001, the CA, in the challenged Resolution, denied the said motion
pursuant to Rule 52, Section 1 of the Rules of Court and Rule 9, Section 2 of the Revised Internal Rules of the Court
of Appeals (RIRCA). Estinozo then filed a Motion for Reconsideration of the June 28, 2001 Resolution of the CA. The
appellate court denied the same, on August 17, 2001, in the other assailed Resolution.

Displeased with this series of denials,Estinozo instituted the instant Petition for Certiorari under Rule 65, arguing,
among others, that: (1) her previous counsel, by filing a prohibited pleading, foreclosed her right to file a motion for
reconsideration of the CA’s decision, and consequently an appeal therefrom; (2) she should not be bound by the
mistake of her previous counsel especially when the latter’s negligence and mistake would prejudice her substantial
rights and would affect her life and liberty;

Issue:

Whether or not a petition for certiorari under Rule 65 was the proper remedy in assailing the CA’s decision denying
her appeal?

Held:

No. Considering that, in this case, appeal by certiorari was available to Estinozo, she effectively foreclosed her right
to resort to a special civil action for certiorari, a limited form of review and a remedy of last recourse, which lies only
where there is no appeal or plain, speedy and adequate remedy in the ordinary course of law. A petition for
review on certiorari under Rule 45 and a petition for certiorari under Rule 65 are mutually exclusive
remedies. Certiorari cannot co-exist with an appeal or any other adequate remedy.

Even granting arguendo that the instant certiorari petition is an appropriate remedy, still this Court cannot grant
the writ prayed for because there was no grave abuse of discretion committed by the CA in the challenged issuances.
The rule, as it stands now without exception, is that the 15-day reglementary period for appealing or filing a motion
for reconsideration or new trial cannot be extended, except in cases before this Court, as one of last resort, which
may, in its sound discretion grant the extension requested.

It is well to point out that with petitioner’s erroneous filing of a motion for extension of time and with her non-filing
of a motion for reconsideration or a petition for review from the CA’s decision, the challenged decision has already
attained finality and may no longer be reviewed by this Court. The instant Rule 65 petition cannot even substitute
for the lost appeal—certiorari is not a procedural device to deprive the winning party of the fruits of the judgment in
his or her favor. Relief will not be granted to a party who seeks to be relieved from the effects of the judgment when
the loss of the remedy at law was due to his own negligence, or to a mistaken mode of procedure.
Remedial Law I| Page 185

Heirs of Spouses Reterta vs. Spouses Mores


G.R. No. 159941. August 17, 2011.

Facts:

The Heirs of Reterta filed an action for quieting of title and reconveyance against the Spouses Mores in the RTC of
Trece Martires City averring that they were the true owners of the subject land having inherited the same from
their father who has been granted the land by virtue of his occupation and cultivation and for the continuous
possession by him and his predecessors-in-interest of the land. They also assert that they only discovered of an
affidavit purportedly signed by their father waiving his rights, interests and participation in the land in favor of one
Lorenzo Mores. Spouses Mores filed a Motion to Dismiss against the complaint citing lack of jurisdiction of the RTC
over the land, the same being a friar land and not part of public domain, the rightful jurisdiction is with the Director
of Lands. The RTC granted such motion. The Heirs of Reterta timely filed their MR but it was denied. Therefore, the
Heirs assailed such dismissal via petition for certiorari under Rule 65.

Issue:

Whether or not a petition for certiorari under Rule 65 was the proper remedy in assailing the RTC’s Order granting
a Motion to Dismiss?

Held:

No. The order granting the respondents’ motion to dismiss was a final, as distinguished from an interlocutory, order
against which the proper remedy was an appeal in due course. Certiorari, as an extraordinary remedy, is not
substitute for appeal due to its being availed of only when there is no appeal, or plain, speedy and adequate remedy
in the ordinary course of law. The order that the petitioners really wanted to obtain relief from was the order
granting the respondents motion to dismiss, not the denial of the motion for reconsideration. The fact that the order
granting the motion to dismiss was a final order for thereby completely disposing of the case, leaving nothing more
for the trial court to do in the action, truly called for an appeal, instead of certiorari, as the correct remedy.

Moreover, even Section 9 of Rule 37 of the Rules of Court, cited by the petitioners, indicates that the proper remedy
against the denial of the petitioner’s motion for reconsideration was an appeal from the final order dismissing the
action upon the respondents motion to dismiss. The restriction against an appeal of a denial of a motion for
reconsideration independently of a judgment or final order is logical and reasonable. A motion for reconsideration is
not putting forward a new issue, or presenting new evidence, or changing the theory of the case, but is only seeking
a reconsideration of the judgment or final order based on the same issues, contentions, and evidence.

HOWEVER, despite availing of improper mode of remedy, the Court of Appeals should have given due course to the
petition because (a) the broader interest of justice demanded that certiorari be given due course to avoid the
undeserved grossly unjust result that would befall the petitioners otherwise; and (b) the order of the RTC granting
the motion to dismiss on ground of lack of jurisdiction over the subject matter evidently constituted grave abuse of
discretion amounting to excess of jurisdiction.

On occasion, the Court has considered certiorari as the proper remedy despite the availability of appeal, or other
remedy in the ordinary course of law. In Francisco Motors Corporation v. Court of Appeals the Court has declared
that the requirement that there must be no appeal, or any plain speedy and adequate remedy in the ordinary course
of law admits of exceptions, such as: (a) when it is necessary to prevent irreparable damages and injury to a party;
(b) where the trial judge capriciously and whimsically exercised his judgment; (c) where there may be danger of a
failure of justice; (d) where an appeal would be slow, inadequate, and insufficient; (e) where the issue raised is one
purely of law; (f) where public interest is involved; and (g) in case of urgency. Specifically, the Court has held that
the availability of appeal as a remedy does not constitute sufficient ground to prevent or preclude a party from
making use of certiorari if appeal is not an adequate remedy, or an equally beneficial, or speedy remedy. It is
inadequacy, not the mere absence of all other legal remedies and the danger of failure of justice without the writ,
Remedial Law I| Page 186

that must usually determine the propriety of certiorari. A remedy is plain, speedy and adequate if it will promptly
relieve the petitioner from the injurious effects of the judgment, order, or resolution of the lower court or agency.
Remedial Law I| Page 187

Manaloto vs. Veloso III
G.R. No. 171365. October 6, 2010

Facts:

[PRIOR CASE] Ermelinda Manaloto, et al. (petitioners) filed an unlawful detainer case against Ismael Veloso III
(respondent). The MeTC ruled in favor of the petitioners and ordered respondent to vacate. The RTC reversed the
MeTC decision. After successive appeals, the case went up to the Supreme Court which affirmed the decision of the
RTC and that decision became final and executory.

Meanwhile, while respondent’s appeal of the MeTC’s decision was pending before RTC Branch 88, he filed before
RTC Branch 227 a Complaint for Breach of Contract (allegedly, the petitioners as lessors failed to make continuing
repairs on the property and keep it tenantable) and Damages (because he suffered embarrassment and humiliation
when petitioners distributed copies of the MeTC decision in the unlawful detainer case to the homeowners of
Horseshoe Village) against petitioners.

Petitioners filed an Omnibus Motion to Dismiss respondent’s complaint. RTC Branch 227 dismissed respondent’s
complaint for violating the rule against splitting of cause of action, lack of jurisdiction, and failure to disclose the
pendency of a related case. Respondent received a copy of the RTC-Branch 227 decision on September 26, 2003. He
filed a Motion for Reconsideration of said judgment on October 10, 2003, which RTC-Branch 227 denied in an
Order dated December 30, 2003. Respondent received a copy of the RTC-Branch 227 order denying his Motion for
Reconsideration on February 20, 2004, and he filed his Notice of Appeal on March 1, 2004. However, the RTC-
Branch 227, in an Order dated March 23, 2004, dismissed respondent’s appeal for being filed out of time.
Respondent received a copy of the RTC-Branch 27 order dismissing his appeal on April 30, 2004 and he filed a
Motion for Reconsideration of the same on May 3, 2004. The RTC-Branch 227, in another Order dated May 31, 2004,
granted respondent’s latest motion.

The CA then gave due course to the appeal and ruled in favor of respondent on his claim for damages and ordered
the petitioners to pay him moral damages. In a Petition for Review before the SC, petitioners now contend that
respondent’s appeal was filed out of time and the decision of the RTC which dismissed respondent’s complaint had
already attained finality.

Issue:

Whether or not Ismael Veloso (respondent) timely filed his appeal?

Held:

Yes. Jurisprudence has settled the “fresh period rule,” according to which, an ordinary appeal from the RTC to the
Court of Appeals, under Section 3 of Rule 41 of the Rules of Court, shall be taken within fifteen (15) days either
from receipt of the original judgment of the trial court or from receipt of the final order of the trial court dismissing
or denying the motion for new trial or motion for reconsideration.

In this case, respondent received a copy of the Resolution dated September 2, 2003 of the RTC-Branch 227
dismissing his complaint in Civil Case No. Q-02-48341 on September 26, 2003. Fourteen days thereafter, on
October 10, 2003, respondent filed a Motion for Reconsideration of said resolution. The RTC-Branch 227 denied
respondent’s Motion for Reconsideration in an Order dated December 30, 2003, which the respondent received on
February 20, 2004. On March 1, 2004, just after nine days from receipt of the order denying his Motion for
Reconsideration, respondent already filed his Notice of Appeal. Clearly, under the fresh period rule, respondent was
able to file his appeal well-within the prescriptive period of 15 days, and the Court of Appeals did not err in giving
due course to said appeal.
Remedial Law I| Page 188

Latorre vs. Latorre


G.R. No. 183926. March 29, 2010

Facts:

Generosa Latorre filed before the RTC a Complaint for Collection and Declaration of Nullity of Deed of Absolute
Sale with application for Injunction against her own son, Luis Latorre and one Ifzal Ali. Luis Latorre immediately
filed a Motion to Dismiss on the sole ground that the venue of the case was improperly laid. He stressed that while
the complaint was denominated as one for Collection and Declaration of Nullity of Deed of Absolute Sale with
application for Injunction, in truth the case was a real action affecting title to and interest over the subject property.
He insisted that all of petitioner's claims were anchored on her claim of ownership over one-half portion of the
subject property. Since the subject property is located in Makati City, respondent argued that petitioner should have
filed the case before the RTC of Makati City and not of Muntinlupa City. Ifzal also filed his motion to dismiss on the
ground of want of jurisdiction, asserting that he was immune from suit because he was an officer of ADB, an
international organization.

The RTC denied Luis’ motion to dismiss. He then filed an Answer Ad Cautelam. (RTC later on dismissed petitioner's
claim against Ifzal because the dispute was clearly between petitioner and respondent). The RTC thereafter ruled in
favor of Luis. Aggrieved, Generosa filed her Motion for Reconsideration, which the RTC denied. Generosa then
went up directly to the Supreme Court via a petition for review on certiorari claiming that the RTC erred in treating
the venue as jurisdiction and in treating her complaint as a real action.

Issue:

Whether or not a petition for review on certiorari under Rule 45 filed directly to the SC is proper?

Held:

No. Petitioner came directly to the Supreme Court on a Petition for Review on Certiorari under Rule 45, in relation
to Rule 41, of the Rules of Civil Procedure on alleged pure questions of law. There are three (3) modes of appeal from
decisions of the RTC, namely: (1) ordinary appeal or appeal by writ of error, where judgment was rendered in a civil
or criminal action by the RTC in the exercise of its original jurisdiction; (2) petition for review, where judgment was
rendered by the RTC in the exercise of its appellate jurisdiction; and (3) petition for review to the Supreme Court.

The first mode of appeal, governed by Rule 41, is brought to the Court of Appeals (CA) on questions of fact or mixed
questions of fact and law. The second mode of appeal, covered by Rule 42, is brought to the CA on questions of fact,
of law, or mixed questions of fact and law. The third mode of appeal, provided in Rule 45, is filed with the Supreme
Court only on questions of law.

A question of law arises when there is doubt as to what the law is on a certain state of facts, while there
is a question of fact when the doubt arises as to the truth or falsity of the alleged facts.

In her Reply to respondents Comment, petitioner prayed that this Court decide the case on the merits. To do so,
however, would require the examination by this Court of the probative value of the evidence presented, taking into
account the fact that the RTC failed to adjudicate this controversy on the merits. This, unfortunately, the SC cannot
do. It thus becomes exceedingly clear that the filing of the case directly with this Court ran afoul of the doctrine of
hierarchy of courts. Pursuant to this doctrine, direct resort from the lower courts to the Supreme Court will not be
entertained unless the appropriate remedy sought cannot be obtained in the lower tribunals. This Court is a court of
last resort, and must so remain if it is to satisfactorily perform the functions assigned to it by the Constitution and
by immemorial tradition.

Summary of other procedural lapses by the parties mentioned by the SC:

First. Petitioner filed her complaint with the RTC of Muntinlupa City instead of the RTC of Makati City, the latter
being the proper venue in this case. Actions affecting title to or possession of real property or an interest therein
(real actions) shall be commenced and tried in the proper court that has territorial jurisdiction over the area where
the real property is situated. The action in the RTC, other than for Collection, was for the Declaration of Nullity of
Remedial Law I| Page 189

the Deed of Absolute Sale involving the subject property, which is located at No. 1366 Caballero
St., Dasmarias Village, Makati City. The venue for such action is unquestionably the proper court of Makati City,
where the real property or part thereof lies, not the RTC of Muntinlupa City.

Second. The RTC also committed a procedural blunder when it denied respondent's motion to dismiss on the ground
of improper venue. The RTC insisted that trial on the merits be conducted even when it was awfully glaring that the
venue was improperly laid, as pointed out by respondent in his motion to dismiss. After trial, the RTC eventually
dismissed the case on the ground of lack of jurisdiction, even as it invoked, as justification, the rules and
jurisprudence on venue. Despite the conduct of trial, the RTC failed to adjudicate this case on the merits.

Third. When the RTC denied his Motion to Dismiss, respondent could have filed a petition for certiorari and/or
prohibition inasmuch as the denial of the motion was done without jurisdiction or in excess of jurisdiction or with
grave abuse of discretion amounting to lack of jurisdiction. However, despite this lapse, it is clear that respondent
did not waive his objections to the fact of improper venue, contrary to petitioner's assertion. Notably, after his
motion to dismiss was denied, respondent filed a Motion for Reconsideration to contest such denial. Even in his
Answer Ad Cautelam, respondent stood his ground that the case ought to be dismissed on the basis of improper
venue.
Remedial Law I| Page 190

Alfredo vs. Borras


G.R. No. 144225. June 17, 2003

Facts:

The subject matter of the controversy is a parcel of land measuring 81,524 square meters in Bataan. The original
owners are Spouses Godofredo and Carmen Alfredo (Spouses Alfredo). They allegedly sold the land to Spouses
Armando and Adelie Borras (Spouses Borras) for P15,000: P7,000 of which for the mortgage debt to release the
mortgage from Development Bank of the Philippines and the remaining to be paid to Spouses Alfredo. Spouses
Borras gave the money to Spouses Alfredo for the payment of the loan and subsequently, the mortgage was released
and the OCT was returned to Spouses Alfredo. Upon the payment of the balance of the purchase price, Spouses
Alfredo delivered to Spouses Borras the owner’s duplicate copy of OCT. Spouses Alfredo also introduced Spouses
Borras as the new owners of the land to their tenants.

In 1994, Spouses Borras learned that Spouses Alfredo had re-sold the portions of the subject land to several persons.
They then filed an adverse claim with the Register of Deeds in Bataan. They found out that Spouses Alfredo had
secured an owner’s duplicate copy of OCT No. 284 after filing a petition in court for the issuance of a new copy. They
wrote to Spouses Alfredo and complained about their acts but the latter did not reply. They then filed a Complaint
for Specific Performance. They amended their complaint to include additional defendants – the subsequent buyers.

In their Answer, Spouses Alfredo and the subsequent buyers argued that the action is unenforceable under the
Statute of Frauds. They pointed out that there is no written instrument evidencing the alleged contract of sale. They
also asserted that the subsequent buyers were buyers in good faith.

The RTC ruled in favor of Spouses Borras and found that there was a perfected contract of sale and the subsequent
buyers were not innocent purchasers.

The CA affirmed the ruling of the RTC and found the factual findings of the trial court well supported by evidence.
Hence, this petition.

Issue:

Whether or not the findings of the RTC and CA are binding on the Supreme Court?

Held:

Yes. In a petition for review on certiorari under Rule 45, the Court reviews only errors of law and not errors of facts.
The factual findings of the appellate court are generally binding on this Court. This applies with greater force when
both the trial court and the Court of Appeals are in complete agreement on their factual findings. In this case, there
is no reason to deviate from the findings of the lower courts. The facts relied upon by the trial and appellate courts
are borne out by the record. We agree with the conclusions drawn by the lower courts from these facts.

Valid and Enforceability of the Sale: The contract was consummated because the sellers and buyers have performed
their respective obligations under the contract: Spouses Borras had paid the price and the Spouses Alfredo delivered
the subject land to the former. The Statute of Frauds applies only to executory contracts and not to contracts either
partially or totally performed. Thus, the Statute of Frauds is not applicable in this case.

Note: There are many civil law issues under this case such as validity of alienation without the signature of the
husband, without the approval of the Secretary of Agriculture and Natural Resources, prescription, laches and
whether or not the subsequent buyers were buyer in good faith. The SC resolved them all in favor of Spouses Borras.
Remedial Law I| Page 191

Validity of alienation without the signature of the husband: The sale is not void in accordance with the Civil Code
(not Family Code), it is merely voidable. And the husband had ratified the alienation anyway.

Validity of alienation without the approval of the Secretary of Agriculture and Natural Resources: the approval after
the 5th year period is merely directory not mandatory.

Prescription and laches: The action has not prescribed since the action is one for reconveyance based on an implied
trust which does not prescribe as long as possession of the land is with the plaintiff.

Subsequent buyers were not buyers in good faith: since the Spouses Borras had caused the notice of adverse claim
prior to the sale and their registration, they cannot be deemed buyers in good faith since they had notice of the
adverse claim of the Spouses Borras.
Remedial Law I| Page 192

People vs. Corpuz


G.R. No. 148198. October 1, 2003

Facts:

Private complainants Belinda Cabantog, Concepcion San Diego, Erlinda Pascual and Restian Surio went to Alga-
Moher International Placement Services Corporation to apply for employment as factory workers in Taiwan. They
were accompanied by a certain “Aling Josie” who introduced them to the agency’s President and General Manager,
Mrs. Evelyn Gloria Reyes. Mrs. Reyes asked them to accomplish the application forms. Thereafter, they were told to
return to the office with P10,000.00 each as processing fee. Private complainants returned to the agency to pay the
processing fees. Mrs. Reyes was not at the agency that time, but she called Elizabeth Corpuz (appellant) on the
telephone to ask her to receive the processing fees. Thereafter, appellant advised them to wait for the contracts to
arrive from the Taiwan employers. Nothing happened with their applications so they filed their complaint with the
National Bureau of Investigation which led to the arrest and detention of appellant. Appellant was then charged
with the crime of Illegal Recruitment in Large Scale. For her part, appellant resolutely denied having a hand
in the illegal recruitment and claimed that she merely received the money on behalf of Mrs. Reyes, where she had
been working as secretary for three months prior to the incident. In compliance with the order of her employer and
since the cashier was absent, she received the processing fees of private complainants, which she thereafter remitted
to Mrs. Reyes. She had no knowledge that the agency’s license was suspended by the POEA.

The trial court convicted the appellant. Hence, this appeal.

Issue:

Whether or not the trial court erred in convicting the appellant?

Held:

Yes. The trial court convicted appellant based on its findings that despite the suspension of the agency’s license,
appellant still convinced the applicants to give their money with the promise to land a job abroad. Moreover, as the
registered secretary of the agency she had management control of the recruitment business. It is axiomatic that
findings of facts of the trial court, its calibration of the collective testimonies of witnesses and
probative weight thereof and its conclusions culled from said findings are accorded by this Court great
respect, if not conclusive effect, because of the unique advantage of the trial court in observing and
monitoring at close range, the conduct, deportment and demeanor of the witnesses as they testify
before the trial court. However, this principle does not apply if the trial court ignored, misunderstood
or misconstrued cogent facts and circumstances of substance which, if considered, would alter the
outcome of the case. The exception obtains in this case.

On July 30, 1998, appellant received the processing fees of the private complainants since the cashier was absent
that day. Her receipt of the money was in compliance with the order of her employer, Mrs. Reyes. She did not
convince the applicants to give her their money since they went to the agency precisely to pay the processing fees
upon the earlier advice of Mrs. Reyes. Moreover, as stated in the last sentence of Section 6 of RA 8042, the persons
who may be held liable for illegal recruitment are the principals, accomplices and accessories. In case of juridical
persons, the officers having control, management or direction of their business shall be liable.

In this case, the prosecution failed to establish that appellant, as secretary, had control, management or direction of
the recruitment agency. Appellant started her employment with the agency on May 1, 1998 and she was tasked to
hold and document employment contracts from the foreign employers. She did not entertain applicants and she had
no discretion over how the business was managed. The trial court’s finding that appellant, being the secretary of the
agency, had control over its business, is not only non sequitur but has no evidentiary basis. The culpability of the
employee hinges on his knowledge of the offense and his active participation in its commission. Where it is shown
that the employee was merely acting under the direction of his superiors and was unaware that his acts constituted
a crime, he may not be held criminally liable for an act done for and in behalf of his employer.

In this case, the prosecution failed to adduce sufficient evidence to prove appellant’s active participation in the
illegal recruitment activities of the agency. As already established, appellant received the processing fees of the
Remedial Law I| Page 193

private complainants for and in behalf of Mrs. Reyes who ordered her to receive the same. She neither gave an
impression that she had the ability to deploy them abroad nor convinced them to part with their money. More
importantly, she had no knowledge that the license was suspended the day before she received the money. Their
failure to depart for Taiwan was due to the suspension of the license, an event which appellant did not have control
of. Her failure to refund their money immediately upon their demand was because the money had been remitted to
Mrs. Reyes on the same day she received it from them.
Remedial Law I| Page 194

Philippine Airlines, Inc. vs. Court of Appeals
G.R. No. 127473. December 8, 2003

Facts:

Private respondents (Judy Amor, et al.) filed with the RTC a complaint for damages against petitioner (PAL) due to
the latter’s failure to honor their confirmed tickets. Private respondent Judy Amor purchased three plane tickets for
her and her infant son, Gian Carlo as well as her sister Jane Gamil for 7:10 a.m. flight bound for Manila from PAL.
They arrived at the Legaspi Airport at 6:20 a.m. Carlo Benitez was supposed to use the confirmed ticket of a certain
Dra. Emily Chua. Judy went to the office of the station manager to request that minor plaintiff Carlo Benitez be
allowed to use the ticket of Dra. Chua. Lloyd Fojas, the check-in clerk on duty, wrote something on the tickets which
they later read as "late check-in 7:05" and told them to proceed to the cashier to make arrangements. They pleaded
with Fojas, pointing out that it is only 6:45 a.m., but the latter did not even look at him or utter any word. Private
respondents were not able to board said flight. The plane left at 7:30 a.m., twenty minutes behind the original
schedule. They tried to catch an afternoon flight. Private respondents were told to wait for the 5:30 p.m. flight. They
checked-in their bags and were told to hand in their tickets. Later, a PAL employee at the check-in counter called
out the name of private respondent minor Carlo Benitez and was told by the PAL personnel that they cannot be
accommodated. Fojas who was also at the counter then removed the boarding passes inserted in private
respondents’ tickets as well as the tags from their luggages.

PAL contends that private respondents are not entitled to their claim for damages because they were late in
checking-in for the flight; and that they were only chance or waitlisted passengers for the afternoon flight and were
not accommodated because all confirmed passengers of the flight had checked-in. The RTC rendered judgment
upholding the evidence presented by private respondents. Aggrieved, petitioner appealed to the CA which affirmed
the judgment of the trial court. Hence, the present petition of PAL, raising the following issues: (1) whether or not
private respondents checked-in on time for the flight, and (2) whether or not the damages awarded are excessive.

Issue:

Whether or not the case falls under the exemptions when the Supreme Court can review factual findings of the
lower courts?

Held:

No. In petitions for review on certiorari under Rule 45 of the Rules of Court, the general rule is that only questions
of law may be raised by the parties and passed upon by this Court. Factual findings of the appellate court are
generally binding on us especially when in complete accord with the findings of the trial court. This is because it is
not the Court’s function to analyze or weigh the evidence all over again. However, this general rule admits of
exceptions, to wit:

(a) where there is grave abuse of discretion; (b) when the finding is grounded entirely on speculations, surmises or
conjectures; (c) when the inference made is manifestly mistaken, absurd or impossible; (d) when the judgment of the
Court of Appeals was based on a misapprehension of facts; (e) when the factual findings are conflicting; (f) when the
Court of Appeals, in making its findings, went beyond the issues of the case and the same are contrary to the
admissions of both appellant and appellee; (g) when the Court of Appeals manifestly overlooked certain relevant
facts not disputed by the parties and which, if properly considered, would justify a different conclusion; and, (h)
where the findings of fact of the Court of Appeals are contrary to those of the trial court, or are mere conclusions
without citation of specific evidence, or where the facts set forth by the petitioner are not disputed by the
respondent, or where the findings of fact of the Court of Appeals are premised on the absence of evidence and are
contradicted by the evidence on record.

Petitioner invokes exception (b). After review of the records, the Court found no reason to disturb the affirmance by
the CA of the findings of the trial court that the private respondents have checked-in on time; that they reached the
airport at 6:20 a.m., based on the testimonies of private respondent Judy Amor, and witnesses Salvador Gonzales
and Atty. Owen Amor who were consistent in their declarations on the witness stand and corroborated one another’s
Remedial Law I| Page 195

statements; and that the testimony of petitioner’s lone witness, Lloyd Fojas is not sufficient to overcome private
respondents’ evidence.

It is a well-entrenched principle that absent any showing of grave abuse of discretion or any palpable error in its
findings, this Court will not question the probative weight accorded by the lower courts to the various evidence
presented by the parties. As we explained in Superlines Transportation Co. Inc., vs. ICC Leasing & Financing
Corporation:

The Court is not tasked to calibrate and assess the probative weight of evidence adduced by the parties during trial
all over again…So long as the findings of facts of the Court of Appeals are consistent with or are not palpably
contrary to the evidence on record, this Court shall decline to embark on a review on the probative weight of the
evidence of the parties.
Remedial Law I| Page 196

Augusto vs. Risos
G.R. No. 131794. December 10, 2003

Facts:

Felisa Augusto and her siblings, Jose Augusto, Magdalena Augusto and Alfonso Augusto, all married, were the co-
owners of a parcel of land located in Barrio Mactan, Opon, Cebu. They sold the property to Guillermo Omolon. They
took possession of the property but it was still registered in the names of the Augusto siblings, under the original
certificate of title. Guillermo Omolon died intestate and was survived by Cleofe Omolon. Cleofe Omolon filed a
petition for the reconstitution of the OCT covering the lot before the RTC. The RTC granted the petition. However,
upon presentation of the aforesaid order to the Register of Deeds, Cleofe was informed that the owner’s copy had
already been issued to Ruben Augusto, pursuant to an order issued by the court and the same was in the possession
of Atty. Noel Archival. Cleofe filed a petition before the RTC alleging that as lawful co-owner and possessor of lot,
she had every right to have and hold the owner’s duplicate of the said OCT. She prayed that after due proceedings,
the respondents Ruben Augusto and Atty. Noel Archival be ordered to surrender the owner’s copy of the said title.

On the other hand, Ruben Augusto and Atty. Noel Archival alleged that the Deed of Absolute Sale executed by
Felisa, Magdalena, Alfonso and Jose, all surnamed Augusto, was falsified and fictitious, and, thus, null and void.

On October 22, 1997, the RTC issued an order directing Atty. Noel Archival to produce the owner’s copy of OCT to
allow the annotation of Cleofe’s interest, upon which the owner’s duplicate copy of the title may thereafter be
returned. The respondents therein filed a motion for a partial reconsideration of the order. However, the court
issued an order denying the motion of the respondents therein. The respondents filed a notice of appeal from the
said order to the Court of Appeals. The RTC issued an order denying due course therefor, on its perception that the
orders subject thereof were interlocutory; hence, not appealable. Petitioners argue that contrary to the ruling of the
public respondent, its October 22, 1997 Order was final and appealable, as the same disposed of the case.

Issue:

Whether or not the October 22, 1997 order of the RTC directing Atty. Archival to produce the owner’s copy of OCT
was a final order and hence appealable?

Held:

No. Section 1, Rule 41 of the Rules of Court provides that an appeal may be taken only from a final order, and not
from an interlocutory one. A final order is one which disposes of the whole subject matter or terminates a particular
proceeding or action, leaving nothing to be done but to enforce by execution what has been determined. An order or
judgment is deemed final if it finally disposes of, adjudicates, or determines the rights, or some right or rights of the
parties, either on the entire controversy or on some definite and separate branch thereof, and concludes them until it
is reversed or set aside. Where no issue is left for future consideration, except the fact of compliance with the terms
of the order, such order is final and appealable.8 In contrast, an order is interlocutory if it does not finally dispose of
the case.

In this case, the order of the public respondent directing the petitioners to produce the owner’s copy of OCT No. 3560
in the Office of the Register of Deeds for the annotation of the private respondent’s interest over the property is
merely interlocutory and not final; hence, not appealable by means of a writ of error. The public respondent had
not fully disposed of the case as it had not yet ruled on whether to grant the private respondent’s prayer for the
surrender of the owner’s copy of OCT No. 3560. As gleaned from the order of the respondent judge, he believed that
he had no jurisdiction to delve into and resolve the issue of ownership over the property and was disposed to dismiss
the petition. Before so doing, he believed it was necessary that the petitioner’s claim over the property be annotated
at the dorsal portion of the title before the institution of an ordinary motion for the resolution of the conflicting
claims of ownership over the property.
Remedial Law I| Page 197

Springfield Development Corporation, Inc. vs.


Presiding Judge, RTC, Misamis Oriental, Br. 40, Cagayan de Oro City
G.R. No. 142628. February 6, 2007

Facts:

Petra Capistrano Piit previously owned Lot which measured 123,408 square. Springfield Development Corporation,
Inc. (Springfield) bought with an area of 68,732 square meters, and another with an area of 49,778 square meters.
Springfield developed these properties into a subdivision project called Mega Heights Subdivision.

The Department of Agrarian Reform (DAR), through its Municipal Agrarian Reform Officer, issued a Notice of
Coverage, placing the property under the coverage of Republic Act (R.A.) No. 6657 or the Comprehensive Agrarian
Reform Law of 1988. DARAB Provincial Adjudicator Abeto A. Salcedo, Jr. rendered a decision declaring the nature
of the property as residential and not suitable for agriculture. The Regional Director filed a notice of appeal, which
the Provincial Adjudicator disallowed for being pro forma and frivolous. The decision became final and executory
and Springfield proceeded to develop the property.

The DAR Regional Director then filed a petition for relief from judgment of the DARAB Decision, docketed as
DARAB. In its Decision dated October 5, 1995, the DARAB granted the petition and gave due course to the Notice of
Coverage. It also directed the Municipal Agrarian Reform Office to proceed with the documentation, acquisition, and
distribution of the property to the true and lawful beneficiaries.

Springfield and the heirs of Piit (petitioners) filed with the RTC of Cagayan de Oro City, Branch 40, a petition for
annulment of the DARAB Decision dated October 5, 1995 and all its subsequent proceedings. Petitioners contend
that the DARAB decision was rendered without affording petitioners any notice and hearing.

On motion filed by the farmer-beneficiaries, the RTC issued an Order dismissing the case for lack of jurisdiction.
Petitioners filed with the Court of Appeals (CA) a special civil action for certiorari, mandamus, and prohibition with
prayer for the issuance of writ of preliminary injunction and/or temporary restraining order. Petitioners alleged that
the RTC committed grave abuse of discretion when it ruled that the annulment of judgment filed before it is actually
an action for certiorari in a different color. According to petitioners, what it sought before the RTC is an annulment
of the DARAB Decision and not certiorari, as the DARAB Decision is void ab initio for having been rendered without
due process of law. The CA dismissed the petition for lack of merit, ruling that the RTC does not have jurisdiction to
annul the DARAB Decision because it is a co-equal body.

Issue:

Whether or not the RTC has jurisdiction over the decisions of DARAB?

Held:

No. The DARAB is a quasi-judicial body created by Executive Order Nos. 229 and 129-A. R.A. No. 6657 delineated
its adjudicatory powers and functions. The DARAB Revised Rules of Procedure adopted on December 26, 198827
specifically provides for the manner of judicial review of its decisions, orders, rulings, or awards. Rule XIV, Section 1
states:

SECTION 1. Certiorari to the Court of Appeals. Any decision, order, award or ruling by the Board or its Adjudicators
on any agrarian dispute or on any matter pertaining to the application, implementation, enforcement or
interpretation of agrarian reform laws or rules and regulations promulgated thereunder, may be brought within
fifteen (15) days from receipt of a copy thereof, to the Court of Appeals by certiorari, except as provided in the next
succeeding section. Notwithstanding an appeal to the Court of Appeals the decision of the Board or Adjudicator
appealed from, shall be immediately executory.

Further, the prevailing 1997 Rules of Civil Procedure, as amended, expressly provides for an appeal from the
DARAB decisions to the CA. The rule is that where legislation provides for an appeal from decisions of certain
administrative bodies to the CA, it means that such bodies are co-equal with the RTC, in terms of rank and stature,
Remedial Law I| Page 198

and logically, beyond the control of the latter. Given that DARAB decisions are appealable to the CA, the inevitable
conclusion is that the DARAB is a co-equal body with the RTC and its decisions are beyond the RTC's control.

Note: There is no such thing as annulment of judgments of the decisions of quasi-judicial bodies. The remedy is a
Petition for Review under Rule 43.

De Vera vs. Santiago, Sr.


G.R. No. 179457. June 22, 2015

Facts:

Wilfredo De Vera, et al. filed an action for reconveyance of ownership or possession with damages against Santiago,
et al. before the MTC alleging that they have allegedly been in actual and continuous possession and occupation of
their respective portions of the land since 1967, without disturbance from any third person. Later on, however, they
discovered that their respective lots covered by Lot 7303 were already covered by Free Patent Titles in the names of
Santiago which were acquired through manipulation, misrepresentation, fraud and deceit. In their Answer,
Santiago, et al. specifically denied the material allegations in petitioners' complaint and countered that they are the
owners of the land. MTC ruled in favor of Santiago. On appeal, RTC reversed the decision of MTC.

Santiago filed with the CA a petition for review under Rule 42 of the Rules of Court. The CA granted the petition for
review, and annulled and set aside the Decisions of both the RTC and the MTC on the ground of lack of jurisdiction
(the assessed value of the land is more than 20,000 and therefore the CA ruled that the MTC had no jurisdiction
when it took first cognizance of the case and consequently, the RTC cannot render a valid judgment when the case
was appealed to it). For the same reason, the CA declined to resolve and deemed as moot and academic the other
factual issues raised in the petition.

The CA also ruled that assuming arguendo that the RTC had jurisdiction over the case, it nonetheless has no
authority to declare as null and void the Original Certificates of Title (Free Patents) registered in the name of
respondents because the said titles were issued four (4) years prior to the filing of the petitioners' complaint for
reconveyance. The CA denied De Vera's motion for reconsideration of its Decision. Hence, the petition for review on
certiorari.

Issue:

Whether or not the CA erred in annulling the decision of RTC for lack of jurisdiction?

Held:

Yes. Based on the Tax Declarations attached to their complaint, the disputed land has a total assessed value of
₱54,370.00. Therefore, the RTC has jurisdiction over petitioners' civil action involving title to a real property outside
Metro Manila with a total assessed value in excess of ₱20,000.00.

Thus, while the CA is correct in ruling that the MTC has no jurisdiction over the case for reconveyance and recovery
of ownership and possession of a land with an assessed value over ₱20,000.00, the same cannot be said of its ruling
with respect to the RTC. Under Section 8, Rule 40 of the Rules of Court, if the MTC tried a case on the
merits despite having no jurisdiction over the subject matter, its decision may be reviewed on appeal by
the RTC, to wit:

Sec. 8. Appeal from orders dismissing case without trial; lack of jurisdiction.

If an appeal is taken from an order of the lower court dismissing the case without a trial on the merits, the Regional
Trial Court may affirm or reverse it, as the case may be. In case of affirmance and the ground of dismissal is lack of
jurisdiction over the subject matter, the Regional Trial Court, if it has jurisdiction thereover, shall try the case on the
merits as if the case was originally filed with it. In case of reversal, the case shall be remanded for further
proceedings.

If the case was tried on the merits by the lower court without jurisdiction over the subject matter, the Regional Trial
Court on appeal shall not dismiss the case if it has original jurisdiction thereof, but shall decide the case in
Remedial Law I| Page 199

accordance with the preceding section, without prejudice to the admission of amended pleadings and additional
evidence in the interest of justice.

In Serrano v. Spouses Gutierrez, the Court explained that the first paragraph of Section 8, Rule 40 contemplates an
appeal from an order of dismissal issued without trial of the case on the merits, while the second paragraph deals
with an appeal from an order of dismissal but the case was tried on the merits. Both paragraphs, however, involve
the same ground for dismissal, i.e., lack of jurisdiction. Verily, the second paragraph refutes respondents' contention
that Section 8, Rule 40 refers solely to cases where the MTC dismissed a case filed therein without a trial on the
merits and an appeal to the RTC was taken from the order of dismissal. Therefore, the RTC correctly proceeded to
decide the case on the merits despite the MTC's lack of jurisdiction over the subject matter.

In contrast, the CA erroneously reversed and set aside the RTC Decision for lack of jurisdiction. Indeed, the RTC has
appellate jurisdiction over the case and its decision should be deemed promulgated in the exercise of that
jurisdiction. The RTC’s appellate jurisdiction, as contrasted to its original jurisdiction, is provided in Section 22 of
B.P. Blg.129, as amended, thus:

SECTION 22. Appellate jurisdiction.–Regional Trial Courts shall exercise appellate jurisdiction over all cases
decided by Metropolitan Trial Courts, Municipal Trial Courts, and Municipal Circuit Trial Courts in their respective
territorial jurisdictions.

The above-quoted provision vests upon the RTC the exercise of appellate jurisdiction over all cases decided by the
Metropolitan Trial Courts, Municipal Trial Courts, and Municipal Circuit Trial Courts in their respective territorial
jurisdictions. Clearly then, the amount involved is immaterial for purposes of the RTC’s appellate jurisdiction; all
cases decided by the MTC are generally appealable to the RTC irrespective of the amount involved. Hence, the CA
grossly erred in nullifying the RTC Decision for lack of jurisdiction, and in declaring as moot and academic the
factual issues raised in the respondents' petition for review when it should have proceeded to review on appeal the
factual findings of the RTC. This is because the RTC not only has exclusive original jurisdiction over petitioners'
action for reconveyance of ownership and possession with damages, but also appellate jurisdiction over the MTC
Decision itself.
Remedial Law I| Page 200

Philippine National Construction Corporation vs. Asiavest Merchant Bankers (M) Berhad
G.R. No. 172301. August 19, 2015

Facts:

PNCC and Asiavest Holdings (M) Sdn. Bhd. (Asiavest Holdings) caused the incorporation of an associate company
known as Asiavest-CDCP Sdn. Bhd. (Asiavest-CDCP), through which they entered into contracts to construct rural
roads and bridges for the State of Pahang, Malaysia. In connection with this construction contract, PNCC obtained
various guarantees and bonds from Asiavest Merchant Bankers (M) Berhad to guarantee the due performance of its
obligations. There was failure to perform the obligations under the construction contract, prompting the State of
Pahang to demand payment against Asiavest Merchant Bankers (M) Berhad's performance bonds. It "entered into a
compromise agreement with the State of Pahang by paying the reduced amount. Consequently, the corporation
demanded indemnity from PNCC by demanding the amount it paid to the State of Pahang.

Asiavest Merchant Bankers (M) Berhad filed a Complaint for recovery of sum of money against PNCC before the
RTC of Pasig. It based its action on Malaysian laws. PNCC filed Motions for extension of time to file its Answer. The
trial court granted these motions, with the last one set to expire on July 3, 1994. But PNCC filed a Motion for
another five-day extension. The trial court denied this Motion. On July 27, 1994, the trial court declared PNCC in
default for failure to file any responsive pleading, and allowed Asiavest Merchant Bankers (M) Berhad to present its
evidence ex parte. The Regional Trial Court rendered judgment in favor of Asiavest Merchant Bankers (M) Berhad.

The trial court also denied PNCC's Motion to Lift Order of Default. It also denied PNCC's Motion for
Reconsideration Ad Cautelam. PNCC brought its case before the Court of Appeals. The Court of Appeals dismissed
PNCC's appeal for raising pure questions of law exclusively cognizable by this court. It likewise denied the
reconsideration.

Issue:

Whether or not the CA correctly ruled in dismissing the appeal on the ground that it raised pure questions of law?

Held:

Yes. Section 9(3) of Batas Pambansa Blg. 129 enumerates the appellate jurisdiction of the Court of Appeals. This
section includes the proviso: "except those falling within the appellate jurisdiction of the Supreme Court[.]" The
Supreme Court's appellate jurisdiction is found in Article VIII, Section 5(2)(e) of the Constitution: SECTION 5. The
Supreme Court shall have the following powers: 2) Review, revise, reverse, modify, or affirm on appeal or
certiorari, as the law or the Rules of Court may provide, final judgments and orders of lower courts in…
e) All cases in which only an error or question of law is involved.

A question of law exists when the doubt or difference arises as to what the law is on a certain state of facts, while a
question of fact exists when the doubt or difference arises as to the truth or the falsehood of alleged facts. Questions
of fact require the examination of the probative value of the parties' evidence. This Petition originated from a default
judgment against petitioner. Petitioner was not able to present evidence before the trial court. Necessarily, the
errors raised from the trial court involved only questions of law.
Remedial Law I| Page 201

Maravilla vs. Rios
G.R. No. 196875. August 19, 2015

Facts:

Joseph Rios filed a criminal case against Teddy Maravilla for reckless imprudence resulting in serious physical
injuries before the MTCC of Himamaylan City, Negros Occidental. Rios accused Maravilla of recklessly driving his
jeep which caused it to collide with the motorcycle he (respondent) was then driving; as a result, respondent was
injured and incapacitated to work for more than ninety days. After trial, the MTCC rendered judgment acquitting
Maravilla of the crime charged against him. However, the court finds preponderance of evidence to hold him liable
for damages. Respondent interposed an appeal before the RTC which modified the decision of the court by deleting
the award of temperate damages in the amount of P20,000.00 and finding respondent liable to pay Rios the amount
of P256,386.25 as actual and compensatory damages.

Maravilla then filed a Petition for Review with the CA which the latter dismissed for failure of the petitioner to
comply with pertinent provisions of the Rules - "xxx b. Some relevant and pertinent pleadings and documents, which
are necessary for a better understanding and resolution of the instant petition, were not attached therein, in
violation of Section 2(d), Rule 42 of the Revised Rules of Court, to wit: (i) Copy of the information filed before the
municipal trial court; ii. Copy of the appellant’s brief filed before the RTC; iii. Copy of the appellee’s brief, if any; iv.
‘other pieces of evidence/documents adduced before the lower court.

Issue:

Whether or not the petition for review under Rule 42 should have been dismissed for failure of the petitioner to
attached certain documents?

Held:

Yes. Under Section 2, Rule 42 of the 1997 Rules of Civil Procedure (1997 Rules), a petition for review shall be
accompanied by, among others, copies of the pleadings and other material portions of the record as would support
the allegations of the petition. Section 3 of the same rule states that failure of the petitioner to comply with any of
the requirements regarding the contents of and the documents which should accompany the petition shall be
sufficient ground for the dismissal thereof.

In Galvez v Court of Appeals, this Court held that there are three guideposts in determining the necessity of
attaching pleadings and portions of the record to petitions under Rules 42 and 65 of the 1997 Rules, to wit: First, not
all pleadings and parts of case records are required to be attached to the petition. Only those which are relevant and
pertinent must accompany it. The test of relevancy is whether the document in question will support the material
allegations in the petition, whether said document will make out a prima facie case of grave abuse of discretion as to
convince the court to give due course to the petition. Second, even if a document is relevant and pertinent to the
petition, it need not be appended if it is shown that the contents thereof can also [be] found in another document
already attached to the petition. Thus, if the material allegations in a position paper are summarized in a
questioned judgment, it will suffice that only a certified true copy of the judgment is attached. Third, a petition
lacking an essential pleading or part of the case record may still be given due course or reinstated (if earlier
dismissed) upon showing that petitioner later submitted the documents required, or that it will serve the higher
interest of justice that the case be decided on the merits. The guideposts, which equally apply to a petition for review
filed in the CA under Rule 42, reflect that the significant determinant of the sufficiency of the attached documents is
whether the accompanying documents support the allegations of the petition.

In this case, while the petitioner submitted additional necessary attachments along with his Motion for
Reconsideration, he left out important parts of the record — excerpts of the transcript of stenographic notes, the
respondent’s formal offer of evidence, and the trial court’s Order admitting said formal offer of evidence — that
would support his claim that the trial court erred in awarding damages to respondent since the latter failed to
testify as to his hospital expenses and identify particular exhibits.
Remedial Law I| Page 202

Rule 38: Petition for Relief from Judgment

Escueta vs. Lim


G.R. No. 137162. January 24, 2007

Facts:

Respondent Rufina Lim filed an action to remove cloud on, or quiet title to, real property, with preliminary
injunction and issuance of [a hold-departure order] from the Philippines against Ignacio E. Rubio and the heirs of
Baloloy. As to petitioner Corazon Escueta, in spite of her knowledge that the disputed lots have already been sold by
Ignacio Rubio to respondent, it is alleged that a simulated deed of sale involving said lots was effected by Ignacio
Rubio in her favor; and that the simulated deed of sale by Rubio to Escueta has raised doubts and clouds over
respondent’s title. In their separate amended answers, petitioners denied the material allegations of the complaint.

The Baloloys failed to appear at the pre-trial. Upon motion of respondent, the trial court declared the Baloloys in
default. They then filed a motion to lift the order declaring them in default, which was denied by the trial court in an
order. Consequently, respondent was allowed to adduce evidence ex parte. Thereafter, the trial court rendered a
partial decision. The judgment is rendered in favor of [respondent] and against [petitioners, heirs] of Luz R. Baloloy.
Atty. Arsenio Villalon, Jr., the former counsel of record of the Baloloys received a copy of the partial decision on
April 5, 1994. On July 4, 1994, the Baloloys, through their new counsel, filed a petition for relief from judgment
which was denied by the trial court for being filed out of time.

Issue:

Whether or not the petition for relief from judgment was filed beyond the reglementary period?

Held:

Yes. The 60-day period for filing a petition for annulment of judgment is reckoned from the time the party acquired
knowledge of the order, judgment or proceedings and not from the date he actually read the same. Section 3 of Rule
38 of the Rules of Court states:

SEC 3. TIME FOR FILING PETITION; CONTENTS AND VERIFICATION - a petition provided for in either of the
preceding sections of this Rule must be verified, filed within 60 days after the petitioner learns of the judgment, final
order, or other proceeding to be set aside, and not mroe that 6 months after such judgment or final order was entered,
or such proceeding was taken; and must be accompanied with affidavits showing the fraud, accident, mistake, or
excusable negligence relied upon, and the facts constituting the petitioner's good and substantial cause of action or
defense as the case may be.

There is no reason for the Baloloys to ignore the effects of the above rule. the 60-day period is reckoned from the
time the party acquired knowledge of the order, judgment, or proceedings and not from the date he actually read the
same.

The evidence on record as far as this issue is concerned shows that Atty. Arsenio Villalon, Jr., the former counsel of
record of the Baloloys received a copy of the partial decision dated June 23, 1993 on April 5, 1994. At that time, said
former counsel is still their counsel of record. The reckoning of the 60-day period therefore is the date when the said
counsel of record received a copy of the partial decision which was on April 5, 1994. The petition for relief was filed
by the new counsel on July 4, 1994 which means that 90 days have already lapsed or 30 days beyond the 60-day
period. Moreover, the records further show that the Baloloys received the partial decision on September 13, 1993 as
evidenced by Registry return cards which bear the numbers 02597 and 02598 signed by Mr. Alejandrino Baloloy.

The Baloloys, apparently in an attempt to cure the lapse of the aforesaid reglementary period to file a petition for
relief from judgment, included in its petition the two Orders dated May 6, 1994 and June 29, 1994. The first Order
denied Baloloys’ motion to fix the period within which plaintiffs-appellants pay the balance of the purchase price.
The second Order refers to the grant of partial execution, i.e. on the aspect of damages. These Orders are only
consequences of the partial decision subject of the petition for relief, and thus, cannot be considered in the
determination of the reglementary period within which to file the said petition for relief.
Remedial Law I| Page 203

Further, no fraud, accident, mistake, or excusable negligence exists in order that the petition for relief may be
granted.
Remedial Law I| Page 204

Rule 47: Annulment of Judgment
Diona vs. Balangue
G.R. No. 173559, January 7, 2013
Facts:

Respondents (Romeo Balangue, et al) obtained a loan secured by a real estate mortgage from petitioner (Leticia
Diona). Because they failed to pay, the petitioner filed a complaint against them. The RTC ruled in favor of the
petitioner. Thereafter petitioner filed a motion for execution, alleging that, the respondent failed to timely appealed.
The respondent on the other hand filed a Motion to Set Aside Judgment claiming that not all of them were served
with summons. However, the RTC granted the motion for execution and imposed 5% monthly interest. Respondent
filed a motion to correct/amend judgment, alleging that in their agreement with the petitioner, the latter only
demands for 12% per annum, and not 5% monthly, which the RTC granted. The petitioner filed a petition for
certiorari, which was denied by the CA.

Issue:

Whether or not the decision of the RTC granting 5% monthly interest was properly annulled under Rule 47?

Held:

Yes. The award of 5% monthly interest violated their right to due process and, hence, the same may be set aside in a
Petition for Annulment of Judgment filed under Rule 47 of the Rules of Court.

A Petition for Annulment of Judgment under Rule 47 of the Rules of Court is a remedy granted only under
exceptional circumstances where a party, without fault on his part, has failed to avail of the ordinary remedies of
new trial, appeal, petition for relief or other appropriate remedies. Said rule explicitly provides that it is not
available as a substitute for a remedy which was lost due to the party’s own neglect in promptly availing of the
same. The underlying reason is traceable to the notion that annulling final judgments goes against the grain of
finality of judgment. Litigation must end and terminate sometime and somewhere, and it is essential to an effective
administration of justice that once a judgment has become final, the issue or cause involved therein should be laid to
rest.

While under Section 2, Rule 47 of the Rules of Court a Petition for Annulment of Judgment may be based only on
the grounds of extrinsic fraud and lack of jurisdiction, jurisprudence recognizes lack of due process as
additional ground to annul a judgment. In Arcelona v. Court of Appeals, this Court declared that a final and
executory judgment may still be set aside if, upon mere inspection thereof, its patent nullity can be shown for having
been issued without jurisdiction or for lack of due process of law.

In this case, the grant of 5% monthly interest is way beyond the 12% per annum interest sought in the Complaint
and smacks of violation of due process.
Remedial Law I| Page 205

Yuk Ling Ong vs. Co
G.R. No. 206653. February 25, 2015
Facts:

Petitioner Yuk Ling Ong, a British-Hong Kong national, and respondent Benjamin Co a Filipino citizen, were
married in the Philippines. Respondent filed a petition for declaration of nullity on the ground of psychological
incapacity before the RTC. Respondent indicated that petitioner’s address was 23 Sta. Rosa Street, Unit B-2
Manresa Garden Homes, Quezon City. The RTC issued summons. In his Server’s Return, process server Rodolfo
Torres, Jr. stated that, on August 1, 2002, substituted service of summons with the copy of the petition was effected
after several futile attempts to serve the same personally on petitioner. The said documents were received by Mr.
Roly Espinosa, a security officer. The RTC found respondent’s marriage with petitioner as void ab initio on the
ground of psychological incapacity under Article 36 of the Family Code. It stated that summons was served on
petitioner on August 1, 2002, but she failed to file her responsive pleading within the reglementary period. The
public prosecutor also stated that there were no indicative facts to manifest collusion. Thus, the RTC concluded that
petitioner was psychologically incapacitated to perform her essential marital obligations.

Consequently, petitioner filed a petition for annulment of judgment under Rule 47 of the Rules of Court before the
CA claiming that she was never notified of the cases filed against her. She prayed that the RTC decision be nullified
on the grounds of extrinsic fraud and lack of jurisdiction. CA found the petition for annulment of judgment to be
devoid of merit. It held that there was no sufficient proof to establish that respondent employed fraud to insure
petitioner’s non-participation in the trial of the aforementioned case. Petitioner moved for reconsideration, but her
motion was denied by the CA. Hence, this petition.

Issue:

Whether or not the judgment by the RTC rendered without acquiring jurisdiction over the person of the defendant
can be annulled under Rule 47?

Held:

Yes. Annulment of judgment is a recourse equitable in character, allowed only in exceptional cases as where there is
no available or other adequate remedy. Rule 47 of the 1997 Rules of Civil Procedure, as amended, governs actions
for annulment of judgments or final orders and resolutions, and Section 2 thereof explicitly provides only two
grounds for annulment of judgment, that is, extrinsic fraud and lack of jurisdiction. Annulment of judgment is an
equitable principle not because it allows a party-litigant another opportunity to reopen a judgment that has long
lapsed into finality but because it enables him to be discharged from the burden of being bound to a judgment that is
an absolute nullity to begin with.

Lack of jurisdiction on the part of the trial court in rendering the judgment or final order is either lack of jurisdiction
over the subject matter or nature of the action, or lack of jurisdiction over the person of the petitioner. The former is
a matter of substantive law because statutory law defines the jurisdiction of the courts over the subject matter or
nature of the action. The latter is a matter of procedural law, for it involves the service of summons or other
processes on the petitioner.

Since there was no valid service of summons upon the petitioner, the RTC’s decision must be voided for lack of
jurisdiction over the person of petitioner. The favorable judgment enjoyed by respondent cannot be categorized as a
genuine victory because it was fought against an adversary, who was ignorant of the existing dispute. Whatever
prize bestowed upon the victor in such a void decision must also be undone.
Remedial Law I| Page 206

Santos vs. Santos
G.R. No. 187061, October 08, 2014

Facts:

Ricardo Santos (husband), filed a petition for declaration of presumptive death of his wife for purposes of
remarriage. Ricardo claimed that it was almost 12 years from the date of his Regional Trial Court petition since
Celerina left to work as domestic helper in Honkong. He believed that she had passed away. His petition was
granted by the RTC. Celerina Santos (wife) claimed that she learned about Ricardo’s petition only sometime in
October 2008 when she could no longer avail the remedies of new trial, appeal, petition for relief, or other
appropriate remedies. Thus she filed a petition for annulment of judgment on the ground of extrinsic fraud and lack
of jurisdiction. She argued that she was deprived her day in court when Ricardo, despite his knowledge of her true
residence, misrepresented to the court that she was a resident of Tarlac City. The Court of Appeals dismissed
Celerina’s petition for being a wrong mode of remedy. According to the CA, the proper remedy was to file a sworn
statement before the civil registry, declaring her reappearance in accordance with Article 42 of the Family Code.

Issue:

Whether or not Annulment of Judgment under Rule 47 is the proper remedy in assailing the decision of the RTC
obtained by extrinsic fraud declaring the spouse presumptively dead?

Held:

Yes. The grounds for annulment of judgment are extrinsic fraud and lack of jurisdiction. For fraud to become a basis
for annulment of judgment, it has to be extrinsic or actual. It is intrinsic when the fraudulent acts pertain to an
issue involved in the original action or where the acts constituting the fraud were or could have been litigated, it is
extrinsic or collateral when a litigant commits acts outside of the trial which prevents a party from having a real
contest, or from presenting all of his case, such that there is no fair submission of the controversy.

The present petition was filed within the four-year period allowed by law in case of extrinsic fraud, and before the
action is barred by laches, which is the period allowed in case of lack of jurisdiction. There was also no other
sufficient remedy available to Celerina at the time of her discovery of the fraud perpetrated on her. The choice of
remedy is important because remedies carry with them certain admissions, presumptions, and conditions.

The Family Code provides that it is the proof of absence of a spouse for four consecutive years, coupled with a well-
founded belief by the present spouse that the absent spouse is already dead, that constitutes a justification for a
second marriage during the subsistence of another marriage. The Family Code also provides that the second
marriage is in danger of being terminated by the presumptively dead spouse when he or she reappears.

The provision on reappearance in the Family Code as a remedy to effect the termination of the subsequent marriage
does not preclude the spouse who was declared presumptively dead from availing other remedies existing in law.
This court had, in fact, recognized that a subsequent marriage may also be terminated by filing “an action in court to
prove the reappearance of the absentee and obtain a declaration of dissolution or termination of the subsequent
marriage.

Celerina does not admit to have been absent. She also seeks not merely the termination of the subsequent marriage
but also the nullification of its effects. She contends that reappearance is not a sufficient remedy because it will only
terminate the subsequent marriage but not nullify the effects of the declaration of her presumptive death and the
subsequent marriage.

For the purpose of not only terminating the subsequent marriage but also of nullifying the effects of the declaration
of presumptive death and the subsequent marriage, mere filing of an affidavit of reappearance would not suffice.
Celerina’s choice to file an action for annulment of judgment will, therefore, lie.
Remedial Law I| Page 207

Lasala vs. NFA


G.R. No. 171582, August 19, 2015.

Facts:

Alberto Lasala, through his company PSF Security Agency, used to provide security guard services to the National
Food Authority. Lasala's employees who were deployed to the NFA filed with the NLRC a complaint for
underpayment of wages and nonpayment of other monetary benefits. The NLRC ruled for the employees and held
Lasala and the NFA solidarily liable for the employees' adjudged monetary award. Consequently, the sheriff
garnished the NFA's P383,572.90 worth of bank deposits with the Development Bank of the Philippines.

Believing that it had no liability to Lasala's employees, the NFA filed with the RTC, Branch 220, Quezon City, a
complaint for sum of money with damages and an application for the issuance of a writ of preliminary attachment
against Lasala. In response, Lasala filed an answer with counterclaim and opposition to the prayer for preliminary
attachment. Lasala also filed a counterclaim against NFA.

Initially, the trial court granted the NFA's prayer for the issuance of a writ of preliminary attachment. However,
this writ was eventually nullified when Lasala questioned it with the Court of Appeals. The trial court dismissed the
NFA's complaint for failure of the lawyer deputized by the OGCC, Atty. Mendoza, to present the NFA's evidence-in-
chief, due to his repeated hearing absences. The NFA replaced Atty. Mendoza and administratively charged him
with dishonesty, grave misconduct, conduct grossly prejudicial to the best interests of the service, and gross neglect
of duty. It subsequently employed Atty. Atty. Cahucom as its new counsel.

Although the NFA's complaint was dismissed, Lasala's counterclaim remained, and he presented evidence to
support it. Interestingly, Atty. Cahucom, the NFA's new counsel, did not submit any evidence to controvert Lasala's
counterclaim evidence. When asked during trial, Atty. Cahucom simply waived his right to cross-examine Lasala
and did not exert any effort to counter his testimony.

Despite the huge award to Lasala, the NFA failed to appeal its case to the CA. Atty. Cahucom did not inform the
NFA's management about the trial court's adverse ruling. When asked to explain, he reasoned out that he only
discovered the decision after the lapse of the period for appeal.

Having lost its chance to appeal, the NFA filed with the trial court a petition for relief from judgment (petition for
relief) grounded on excusable negligence. In its petition, the NFA through Atty. Cahucom, attributed its failure to
appeal to one of the NFA's employees. Allegedly, this employee received the copy of the trial court's September 2,
2002 decision but did not inform Atty. Cahucom about it. It was only after the lapse of the period for the filing of a
motion for reconsideration and an appeal that the NFA learned about the adverse ruling. The trial court denied the
petition.

In the meantime, then NFA Administrator Arthur C. Yap had assumed his position. One of his first instructions was
the legal audit of all NFA cases. In doing this, the NFA management found out that the two lawyers (Attys.
Mendoza and Cahucom) assigned to the case against Lasala, grossly mishandled it; hence, causing a huge and
unjust liability to the NFA in the amount of P52,788,970.50.

Thus, on the grounds of lack of jurisdiction and extrinsic fraud, the NFA, now through the OGCC, filed with the CA
a petition and an amended petition for annulment of judgment of the trial court's September 2, 2002 decision which
had granted a substantially higher award than what Lasala originally prayed for in his counterclaim. The CA
granted the petition and annulled the trial court's September 2, 2002 decision holding that the RTC committed
grave abuse of discretion. Hence, this petition.

Issue:

Whether or not the negligence of NFA’s counsels constitutes valid ground to annul the decision of the trial court?

Held:
Remedial Law I| Page 208

Yes. The actions of Attys. Mendoza and Cahucom, under the unique circumstances of this case, amount to extrinsic
fraud that warrants the grant of NFA's petition for relief from judgment. The party in the present case, the NFA, is
a government agency that could rightly rely solely on its legal officers to vigilantly protect its interests. The NFA's
lawyers were not only its counsel, they were its employees tasked to advance the agency's legal interests. NFA's
lawyers acted negligently several times in handling the case that it appears deliberate on their part.

First, Atty. Mendoza caused the dismissal of the NFA's complaint against Lasala by negligently and repeatedly
failing to attend the hearing for the presentation of the NFA's evidence-in-chief. Consequently, the NFA lost its
chance to recover from Lasala the employee benefits that it allegedly shouldered as indirect employer. Atty.
Mendoza never bothered to provide any valid excuse for this crucial omission on his part.

For these failures, Atty. Mendoza merely explained that the NFA's copy of the adverse decision was lost and was
only found after the lapse of the period for appeal. Under these circumstances, the NFA was forced to file an
administrative complaint against Atty. Mendoza for his string of negligent acts.

Atty. Cahucom, Atty. Mendoza's successor in handling the case, notably did not cross-examine Lasala's witnesses,
and did not present controverting evidence to disprove and counter Lasala's counterclaim. Atty. Cahucom further
prejudiced the NFA when he likewise failed to file a motion for reconsideration or an appeal from the trial court's
September 2, 2002 decision, where Lasala was awarded the huge amount of P52,788,970.50, without any convincing
evidence to support it. When asked to justify his failure, Atty. Cahucom, like Atty. Mendoza, merely mentioned that
the NFA's copy of the decision was lost and that he only discovered it when the period for appeal had already lapsed.
The trial court's adverse decision, of course, could have been avoided or the award minimized, if Atty. Cahucom did
not waive the NFA's right to present its controverting evidence against Lasala's counterclaim evidence. Strangely,
when asked during hearing, Atty. Cahucom refused to refute Lasala's testimony and instead simply moved for the
filing of a memorandum.

The actions of these lawyers, that at the very least could be equated with unreasonable disregard for the case they
were handling and with obvious indifference towards the NFA's plight, lead to the conclusion that Attys. Mendoza's
and Cahucom's actions amounted to a concerted action with Lasala when the latter secured the trial court's huge
and baseless counterclaim award. By this fraudulent scheme, the NFA was prevented from making a fair
submission in the controversy.

Another issue:

Did the CA err in granting the petition for annulment of judgment based on RTC’s grave abuse of
discretion?

Yes. The court ruled that the CA violated the restrictive application of a petition for annulment; only extrinsic fraud
and/or lack of jurisdiction may annul a final judgment. By seeking to include acts committed with grave abuse of
discretion, the CA's ruling enlarged the concept of lack of jurisdiction as a ground for annulment. In a petition for
annulment based on lack of jurisdiction, the petitioner cannot rely on jurisdictional defect due to grave abuse of
discretion, but on absolute lack of jurisdiction. As we have already held, the concept of lack of jurisdiction as a
ground to annul a judgment does not embrace grave abuse of discretion amounting to lack or excess of jurisdiction.
Remedial Law I| Page 209

Mangubat vs. Morga-Seva,
G.R. No. 202611. November 23, 2015

Facts:

Gaudencio Mangubat and his wife Aurelia Rellora-Mangubat filed with the Regional Trial Court of Pili, Camarines
Sur a Complaint for Specific Performance with Damages against respondent Belen Morga-Seva Belen and two other
defendants. The RTC ruled in favor of the Mangubats. The case went up to the Supreme Court which affirmed the
RTC decision. That decision became final and executory.

Gaudencio and his children as heirs of the deceased Aurelia filed with the same court a Complaint for Revival of the
Decision. They averred that the writ of execution could not be implemented because Belen and her co-defendants
evaded service thereof. And since five years had already lapsed from the date of its entry, Gaudencio and the heirs
prayed for the revival of the RTC Decision. Gaudencio, assisted by Atty. Reynaldo L. Herrera (Atty. Herrera) and
Belen by Atty. Junnel M. Relativo, entered into a Compromise Agreement. The RTC approved the agreement and on
February 23, 2001 rendered a Decision in accordance therewith. Upon its finality, the Writ of Execution was ordered
issued by the said court.

Belen handed to Atty. Herrera her payment of P91,280.00 in accordance with the Compromise Agreement. Abner,
however, manifested that as far as he is concerned, Belen has not yet made any payment to the heirs as he was not
notified by Atty. Herrera of the same. Thus, Atty. Herrera reported to the court that out of the P91,280.00 handed to
him by Belen, he had turned-over the amount of P84,480.00 to the Clerk of Court and retained 6,800.00 as his
attorney's fee. This was duly noted by the RTC.

Subsequently and purportedly in behalf of all the heirs, Abner, through Atty. Vista-Gumba, filed a Motion to Declare
the Amicable Settlement Null and Void. It was alleged therein that Gaudencio acted only on his own behalf when he
entered into the compromise agreement with Belen, hence, the same is null and void for want of consent and
participation of the heirs who were indispensable parties.

RTC ruled on the Motion to Declare the Amicable Settlement Null and Void. Again purportedly on behalf of all the
heirs, Abner moved for the reconsideration but was denied by the RTC. When the same became final, Belen filed a
Motion for Execution of Specific Acts wherein she once more prayed that Abner be ordered to surrender to the RTC
the owner's copy of TCT No. 6337 and the Clerk of Court to execute in her favor and on behalf of the heirs a deed of
sale involving the lot covered by the said title. This was granted by the RTC in an Order dated July 14, 2006. Still,
Abner refused to comply. The afore-mentioned order became final on November 19, 2006. Pursuant thereto, the RTC
directed the Registrar of Deeds of Camarines Sur to transfer title to the property under TCT No. 6337 to Belen.

Abner filed a Petition for Annulment of Final Order with the CA. He contended that under the Compromise
Agreement, Belen was supposed to make her payment on or before June 30, 2001. However, the same was made
only on December 18, 2003 or way beyond the period agreed upon. Moreover, Abner argued that since the
February 23, 2001 RTC Decision approving the Compromise Agreement had long become final and
executory, the RTC had already lost its jurisdiction over the case when it issued the September 25, 2006
Order.

Issue:

Whether or not the petition for annulment of judgment may be granted on the ground that when the RTC issued the
assailed Order, it already lost its jurisdiction since the Order had long become final and executory?

Held:

No. It must be stressed that the remedy of annulment of judgment is only available under certain exceptional
circumstances as this is adverse to the concept of immutability of final judgments. Hence, it is allowed only on two
grounds, i.e., extrinsic fraud and lack of jurisdiction.
Remedial Law I| Page 210

Abner anchors his Petition for Annulment of Final Order on lack of jurisdiction. He posits that the RTC had lost
jurisdiction over the case when its February 23, 2001 Decision became final, hence, any issuance subsequent thereto
is made without any jurisdiction. The argument is, however, specious. "Lack of jurisdiction on the part of the trial
court in rendering the judgment or final order is either lack of jurisdiction over the subject matter or nature of the
action, or lack of jurisdiction over the person of the petitioner." Here, it is undisputed that the RTC acquired
jurisdiction over the person of Abner, he having asked for affirmative relief therefrom several times. As mentioned,
what Abner questions is the RTC's jurisdiction over the case.

"In a petition for annulment of judgment based on lack of jurisdiction, petitioner must show not merely
an abuse of jurisdictional discretion but an absolute lack of jurisdiction. Lack of jurisdiction means
absence of or no jurisdiction, that is, the court should not have taken cognizance of the petition
because the law does not vest it with jurisdiction over the subject matter. Jurisdiction over the nature of
the action or subject matter is conferred by law."

The RTC's jurisdiction over petitions for revival of judgment had already been upheld by the Court. It was held that
"[a]n action for revival of judgment may be filed either 'in the same court where said judgment was rendered or in
the place where the plaintiff or defendant resides, or in any other place designated by the statutes which treat of the
venue of actions in general.'" Here, the Complaint for revival of judgment was filed in the same court which
rendered the August 27, 1985 Decision in Civil Case No. P-279. Undoubtedly, the RTC has jurisdiction over the
action. There is therefore no valid ground for the Petition for Annulment of Final Order that Abner filed with the
CA.
Remedial Law I| Page 211

Sibal vs. Buquel
G.R. No. 197825. January 11, 2016

Facts:

Respondents Pedro Buquel, Santiago Buquel, Jr., Rosalinda Buquel and Francisco Buquel inherited from their
parents a parcel of land consisting of 81, 022 sq.m. Sometime in January 1999, petitioner Camilo Sibal and Tobi
Mangoba took possession of a portion of the property which belonged to Santiago, Sr. Thereafter, the Buquels made
several demands against Sibal and Mangoba for them to vacate and turn over the property, but the latter refused to
do so. Hence, they filed a complaint before the Tuguegarao RTC for recovery of possession and damages. The RTC
ruled in favor of the Buquels and ordered among others, the restoration to them of their peaceful possession of the
land in question, specifically on the share of Santiago Buquel; Jr. This judgment has become final and executory.
Consequently, Sibal filed a Petition for Annulment of the RTC Decision before the CA, where he raised lack of
jurisdiction and that the Buquels were guilty of extrinsic fraud. Sibal asserts that the negligence of his former
counsel in handling his defense during the proceedings in Civil Case No. 6429 resulted in violation of his right to
due process. The CA dismissed petition for lack of merit as well as the Motion for Reconsideration, hence, this
petition.

Issue:

Whether or not the decision of the RTC should be annulled because of the negligence of the petitioner’s counsel?

Held:

No. It must be emphasized that not every kind of fraud justifies the action of annulment of judgment. Only extrinsic
fraud does. Fraud is extrinsic when the unsuccessful party has been prevented from fully exhibiting his case, by
fraud or deception practiced on him by his opponent, as by keeping him away from court, a false promise of a
compromise; or where the defendant never had knowledge of the suit, being kept in ignorance by the acts of the
plaintiff; or where an attorney fraudulently or without authority connives at his defeat; these and similar cases
which show that there has never been a real contest in the trial or hearing of the case are reasons for which a new
suit may be sustained to set aside and annul the former judgment and open the case for a new and fair hearing. As a
ground for annulment of judgment, extrinsic fraud must arise from an act of the adverse party, and the fraud must
be of such nature as to have deprived the petitioner of its clay in court. The fraud is not extrinsic if the act was
committed by the petitioner's own counsel.

Sibal asserts that the negligence of his former counsel in handling his defense during the proceedings in Civil Case
No. 6429 resulted in violation of his right to due process. He claims that his counsel's inexcusable negligence denied
him of his clay in court. However, he admitted that he attended only one stage of the proceedings below, which was
the preliminary conference. He was not aware of the subsequent proceedings as he was totally dependent on his
former counsel and would merely wait for the latter to notify him if his attendance would be required. There was
likewise no indication that his counsel was in fact in cahoots with the Buquels to obtain the assailed judgment. Sibal
must therefore bear the unfortunate consequences of his actions. As a litigant, he should not have entirely left the
case in his counsel's hands, for he had the continuing duty to keep himself abreast of the developments, if only to
protect his own interest in the litigation. He could have discharged said duty by keeping in regular touch with his
counsel, but he failed to do so.
Remedial Law I| Page 212

Rule 39: Execution and Satisfaction of Judgments

A. Execution Pending Appeal

Bañez vs. Bañez


G.R. No. 132592. January 23, 2002

Facts:

The RTC of Cebu decreed the legal separation between petitioner Aida Bañez and respondent Gabriel Bañez on the
ground of the latter’s sexual infidelity; the dissolution of their conjugal property relations and the division of the net
conjugal assets; the forfeiture of respondent’s one-half share in the net conjugal assets in favor of the common
children; the payment to petitioner’s counsel of the sum of ₱100,000 as attorney’s fees to be taken from petitioner’s
share in the net assets; and the surrender by respondent of the use and possession of a Mazda motor vehicle and the
smaller residential house located at Maria Luisa Estate Park Subdivision to petitioner and the common children
within 15 days from receipt of the decision. While Gabriel filed a Notice of Appeal, Aida filed a motion for execution
pending appeal. The trial court only gave due course to petitioner’s motion for execution pending appeal where it
ordered the respondent (1) to vacate the premises of the small residential house situated in Maria Luisa Estate Park
Subdivision, Lahug, Cebu City and for (2) respondent to surrender the use and possession of the Mazda motor
vehicle together with its keys and accessories thereof to petitioner.

In turn, in a petition for certiorari, Gabriel elevated the case to the Court of Appeals. Petitioner Aida focused her
claimed on the residential house by asserting that justice requires that she and her children be allowed to occupy
and enjoy the house considering that during the entire proceedings before the trial court, she did not have the
chance to occupy it. However, respondent Gabriel in his comment, denied petitioner’s allegation that she did not
have the chance to occupy the residential house. He averred that she could have, had she chosen to. According to
him, as the inventory of the couple’s properties showed, petitioner owned two houses and lots and two motor vehicles
in the United States, where she is a permanent resident. Respondent contended that there was no compelling reason
for petitioner to have the judgment executed pending appeal. The CA set aside the RTC’s decision granting the
motion for execution pending appeal, hence, this petition.

Issue:

Whether or not the execution of judgment pending appeal was justified?

Held:

No. Execution pending appeal is allowed when superior circumstances demanding urgency outweigh the damages
that may result from the issuance of the writ. Otherwise, instead of being an instrument of solicitude and justice,
the writ may well become a tool of oppression and inequity. In this case, considering the reasons cited by petitioner,
there is no superior or urgent circumstance that outweighs the damage which respondent would suffer if he were
ordered to vacate the house. Petitioner did not refute respondent’s allegations that she did not intend to use said
house, and that she has two (2) other houses in the United States where she is a permanent resident, while he had
none at all. Merely putting up a bond is not sufficient reason to justify her plea for execution pending appeal. To do
so would make execution routinary, the rule rather than the exception.
Remedial Law I| Page 213

Santos vs. COMELEC


G.R. No. 155618. March 26, 2003

Facts:

Petitioner Edgar Santos and respondent Pedro Panulaya were both candidates for Mayor of the Municipality of
Balingoan, Misamis Oriental in the May 2001 elections. After the votes were counted and canvassed, the Municipal
Board of Canvassers proclaimed Panulaya as the duly elected Mayor. Santos filed an election protest before the
Regional Trial Court. After trial and revision of the ballots, the trial court found that Santos garnered 2,181 votes
while Panulaya received only 2,105. Thus, the trial court proclaimed and declared Santos as the duly elected
Municipal Mayor. Santos thereafter filed a motion for execution pending appeal. Meanwhile, before the trial court
could act on Santos’ motion, Panulaya filed with the COMELEC a petition for certiorari, docketed as SPR No. 20-
2002, assailing the decision of the trial court. Likewise Panulaya appealed the trial court’s decision to the
COMELEC, where it was docketed as EAC No. A-12-2002.

The COMELEC, in SPR No. 20-2002, issued a Writ of Preliminary Injunction, which effectively enjoined the trial
court from acting on Santos’ motion for execution pending appeal. Subsequently, the COMELEC dismissed SPR No.
20-2002. Hence, it directed the trial court to dispose of all pending incidents in SPL Election Protest No. 1-M(2001)
with dispatch. Thereafter, the trial court issued an Order which upholds and approves the Motion for Execution
Pending Appeal. After Santos posted the required bond, the trial court issued the Writ of Execution, thereby
installing Santos as Municipal Mayor of Balingoan, Misamis Oriental. Accordingly, Santos took his oath of office and
thereafter assumed the duties and functions of his office.

Later, Panulaya filed with the COMELEC a motion for reconsideration of the dismissal of his petition in SPR No.
20-2002, and later on filed a supplemental petition. Panulaya also filed another petition with the COMELEC
docketed SPR No. 37-2002. On September 2002, the COMELEC issued the assailed Order directing the parties to
maintain the status quo ante and enjoining Santos from assuming the functions of Mayor. Santos filed a motion for
reconsideration of the above Order. However, the COMELEC First Division did not refer the said motion to the
COMELEC En Banc. Hence, Santos brought the instant special civil action for certiorari with the Supreme Court.
On October 2002, the COMELEC issued a Resolution in SPR No. 37-2002 granting the petition, setting aside the
trial court’s Order granting Santos’ Motion for Execution pending Appeal as well the Writ of Execution and ordering
that Santos is enjoined from assuming the function of mayor of Balingoan, Misamis Oriental until the final
determination of the election appeal case.

Issue:

Whether or not Santos’ motion for execution pending appeal should be allowed?

Held:

Yes. The grant of execution pending appeal was well within the discretionary powers of the trial court. A valid
exercise of the discretion to allow execution pending appeal requires that it should be based upon good reasons to be
stated in a special order. The following constitute good reasons and a combination of two or more of them will suffice
to grant execution pending appeal: (1.) public interest involved or will of the electorate; (2.) the shortness of the
remaining portion of the term of the contested office; and (3.) the length of time that the election contest has been
pending.

The decision of the trial court in Election Protest No. 1-M(2001) was rendered on April 2, 2002, or after almost one
year of trial and revision of the questioned ballots. It found petitioner as the candidate with the plurality of
votes. Respondent appealed the said decision to the COMELEC. In the meantime, the three-year term of the Office
of the Mayor continued to run. The will of the electorate, as determined by the trial court in the election protest, had
to be respected and given meaning. The Municipality of Balingoan, Misamis Oriental, needed the services of a
mayor even while the election protest was pending, and it had to be the candidate judicially determined to have been
chosen by the people.
Remedial Law I| Page 214

City of Iligan vs. Principal Management Group, Inc.


G.R. No. 145260. July 31, 2003
Facts:

The City of Iligan represented by Mayor Quijano entered into a Memorandum of Agreement with Land Bank Realty
Development Corporation (LBRDC) as General Contractor and Principal Management Group, Inc. (PMGI) as
Developer - Financing Manager for the construction of a Sports Complex which upon completion and payment shall
be turned over to the City of Iligan. However, the construction was stopped because of the refusal of the occupants of
the property to vacate stating that they have not received their disturbance compensation yet. By then, PMGI has
already accomplished 78.27% of the contracted project equivalent to ₱10,957,800.00 of the total project cost of
₱14,000,000.00. Thereafter, PMGI requested from the City of Iligan the payment equivalent for such percentage of
completion but the City refused stating that what was constructed was only 52.89% or equivalent only to
₱6,958,861.59 based on the Accomplishment Report as of February 9, 1999 and that the agreement was to pay once
the project has been completed. Since it was not yet completed, they do not have to pay yet.

PMGI filed a complaint against the City of Iligan for rescission of the MOA and damages. After the filing of the City
of Iligan’s Answer, PMGI filed a Motion for Partial Summary Judgment claiming that there was no genuine issue as
to the fact of the obligation of the City of Iligan since it admitted the accomplishment of 52.89% or equivalent to
₱6,958,861.59 of PMGI and that the City of Iligan had not specifically denied under oath the genuineness of the
Letter of Credit and Memorandum of Agreement.

This Motion was granted by the trial court. The City’s MR being denied, it filed a Notice of Appeal. On the other
hand, PMGI filed a Motion for Execution Pending Appeal alleging that the appeal was clearly dilatory and therefore
an order for execution may be given by the Court in its discretion. The motion for execution pending appeal was
granted by the trial court stating that its adjudication is based on the City’s own admission and hence, it would only
serve to delay execution of the final order subject of the instant motion. This was upheld by the CA and thus, this
Petition for Review on Certiorari under Rule 45 was filed by the City of Iligan.

Issue:

Whether or not the issuance of the writ of execution pending appeal was proper?

Held:

Yes. There are three requisites for the execution of a judgment pending appeal: a) a motion must be filed by the
prevailing party with notice to the adverse party; b) there must be good reasons for execution pending appeal; and c)
the good reasons must be stated in a special order. Normally, execution cannot be obtained until and unless (a) the
judgment has become final and executory; (b) the right of appeal has been renounced or waived; (c) the period for
appeal has lapsed without an appeal having been filed; or (d) having been filed, the appeal has been resolved and
the records of the case have been returned to the court of origin -- in which case, execution shall issue as a matter of
right. On the other hand, when the period of appeal has not yet expired, the execution of a judgment should not be
allowed except if, in the court’s discretion, there are good reasons therefor.

Since the execution of a judgment pending appeal is an exception to the general rule, the existence of "good reasons"
is essential. These reasons must be stated in a special order, because unless these are divulged, it will be difficult to
determine on appeal whether judicial discretion has been properly exercised by the lower court.

Normally, the trial court is not allowed to assess its own judgment and to hold that an appeal may not prosper, or
that it would merely be dilatory. In the present case, however, there are circumstances that undisputedly serve as
cogent bases for arriving at such a conclusion.

First, it is not seriously disputed that the judgment is anchored upon material facts as follows: (1) there is a
Memorandum of Agreement (MOA) for the site development of Sports Complex Project No. 1 signed by the parties;
(2) petitioner failed to pay the occupants of the project site on time, thereby preventing respondent from fully
complying with its obligation under the MOA; (3) respondent admitted that the work accomplished was 52.89
percent, which was equivalent to ₱6,958,861.59. Obviously, there is no genuine issue as to any material fact on this
point.
Remedial Law I| Page 215

Second, Article 1191 of the Civil Code states that:

"The power to rescind obligations is implied in reciprocal ones, in case one of the obligors should not comply with
what is incumbent upon him.

By failing to pay the occupants of the project site within the time required for the completion of the project,
petitioner did not comply with what was incumbent upon it. Applying the law to the undisputed facts, the trial court
had prima facie bases for rendering its partial summary judgment holding that respondent was entitled to rescission
and to the payment of ₱6,958,861.59.
Remedial Law I| Page 216

Far East Bank and Trust Co. vs. Toh, Sr.


G.R. No. 144018, June 23, 2003
Facts:

Tomas Toh, Sr. filed a Complaint against Far East Bank and Trust Co. (FEBTC) seeking recovery of his bank
deposits in the amount of P2,560,644.68 plus damages. FEBTC allegedly debited the said amount from Toh’s
account without his consent and knowledge and applied the same as payment for the Letters of Credit availed of by
Catmon Sales International Corporation (CASICO). Toh Sr. filed a Motion for Summary Judgment which the
Regional Trial Court (RTC) granted. Toh Sr. then filed a Motion for Discretionary Execution by invoking Section 2 of
Rule 39 on the ground of old age (79 years old) and the probability that he may not be able to enjoy his money
deposited in FEBTC. While this Motion was pending, FEBTC filed a Notice of Appeal. The RTC granted the Motion
for Discretionary Execution. FEBTC then filed a special civil action for certiorari with the Court of Appeals (CA).
The CA affirmed the RTC.

Issue:

Whether or not old age constitutes a good reason to allow execution pending appeal?

Held:

Yes. Discretionary execution is permissible only when “good reasons” exist for immediately executing the judgment
before finality or pending appeal or even before the expiration of the time to appeal. “Good reasons” are compelling
circumstances justifying the immediate execution lest judgment becomes illusory, or the prevailing party may, after
the lapse of time, become unable to enjoy it, considering the tactics of the adverse party who may apparently have no
case except to delay. The Rules of Court does not state, enumerate, or give examples of “good reasons” to justify
execution. The determination of what is a good reason must, necessarily, be addressed to the sound discretion of the
trial court. In other words, the issuance of the writ of execution must necessarily be controlled by the judgment of
the judge in accordance with his own conscience and by a sense of justice and equity, free from the control of
another’s judgment or conscience. It must be so for discretion implies the absence of a hard and fast rule. In this
case, the trial court granted Toh’s motion for discretionary execution due to his advanced age.

Toh is already 79 years old. It cannot, by any stretch of imagination, be denied that he is already of advanced age.
Not a few might be fortunate to live beyond 79 years. But no one could claim with certainty that his tribe would be
always blessed with long life. It concluded that old age is a “good reason” to allow execution pending appeal as any
delay in the final disposition of the present case may deny private respondent of his right to enjoy fully the money he
has with FEBTC.

Here, Toh obtained a favorable judgment in the trial court. But that judgment in Civil Case No. MC-99-643 is still
on appeal before the Court of Appeals. It might even reach this Court before the controversy is finally resolved with
finality. As well said in Borja, “while we may not agree that a man of his years is practically moribund, the Court
can appreciate his apprehension that he will not be long for this world and may not enjoy the fruit of the judgment
before he finally passes away.
Remedial Law I| Page 217

Stronghold Insurance Company, Inc. vs. Felix


G.R. No. 148090, November 28, 2006
Facts:

Emerita Garon filed an action for sum of money against Project Movers Realty & Development Corporation and
Stronghold Insurance Company. RTC rendered a summary judgment in favor of Garon holding Project Movers and
Stronghold solidarily liable for the obligation. Garon then filed a Motion for execution pending appeal on the ground
of illness of her husband. RTC granted Garon’s Motion and it ordered Garon to post a bond of P20 million to answer
for any damage that Project Movers and Stronghold Insurance may sustain by reason of the execution pending
appeal. Stronghold Insurance filed a petition for certiorari before the Court of Appeals to assail the RTC’s Order
granting the Motion for Execution Pending Appeal. The CA affirmed the RTC’s Order and held that while it was not
Garon who was ill, Garon needed the money to support her husband’s medical expenses and to support her family.

Issue:

Whether or not the illness of Garon’s husband constitute as a good reason to allow execution pending appeal?

Held:

No. Execution pending appeal is an exception to the general rule. Execution pending appeal is an extraordinary
remedy, being more of the exception rather than the rule. This rule is strictly construed against the movant because
courts look with disfavor upon any attempt to execute a judgment which has not acquired finality. Such execution
affects the rights of the parties which are yet to be ascertained on appeal.

The requisites for the grant of an execution of a judgment pending appeal are the following:
(a) there must be a motion by the prevailing party with notice to the adverse party;
(b) there must be good reasons for execution pending appeal;
(c) the good reasons must be stated in the special order.

As a discretionary execution, execution pending appeal is permissible only when good reasons exist for immediately
executing the judgment before finality or pending appeal or even before the expiration of the period to appeal. Good
reasons, special, important, pressing reasons must exist to justify execution pending appeal; otherwise, instead of an
instrument of solicitude and justice, it may well become a tool of oppression and inequality. Good reasons consist of
exceptional circumstances of such urgency as to outweigh the injury or damage that the losing party may suffer
should the appealed judgment be reversed later.

In this case, Garon anchors the motion for execution pending appeal on the the ill health of her spouse and the
spouses’ urgent need for the funds owed to them by Project Movers and Stronghold Insurance constitute good
reasons for execution pending appeal; and Garon is ready and willing to post a bond to answer for any damage
Project Movers and Stronghold Insurance may suffer should the trial court’s decision be reversed on appeal. The
posting of a bond, standing alone and absent the good reasons required under Section 2, Rule 39 of the Rules, is not
enough to allow execution pending appeal. The mere filing of a bond by a successful party is not a good reason to
justify execution pending appeal as a combination of circumstances is the dominant consideration which impels the
grant of immediate execution. The bond is only an additional factor for the protection of the defendant’s creditor.

The exercise of the power to grant or deny a motion for execution pending appeal is addressed to the sound
discretion of the trial court. However, the existence of good reasons is indispensable to the grant of execution
pending appeal. Here, Garon failed to advance good reasons that would justify the execution pending appeal. It was
not Garon, but her husband, who was ill.
Remedial Law I| Page 218

B. Supervening Events

Villaruel, Jr. vs. Fernando


G.R. No. 136726. September 24, 2003

Facts:

Panfilo Villaruel was a former Assistant Secretary of the Air Transportation Office (ATO), Department of
Transportation and Communication (DOTC). Reynaldo Fernando, Modesto Abarca and Marilou Cleofas, were Chief,
Chief Administrative Assistant, and Administrative Assistant, respectively, of the Civil Aviation Training Center
(CATC) which is an adjunct office of the ATO. The controversy started when Villaruel issued a memorandum
addressed to Fernando et.al., detailing them under DOTC USec. Primitivo Cal. They then requested for the
reconsideration of such order but complied with the order. Without acting on such request, Villaruel placed Abarca
under preventive suspension 90 days without pay pending investigation for alleged grave misconduct. After the
lapse of 90 days, respondents requested from the DOTC Secretary Garcia to lift the order. They also sought the
intervention of the Ombudsman in their case. The Ombudsman then inquired with Sec. Garcia to which he replied
that he had issued a memorandum directing Villaruel to recall respondents to their mother unit. This went
unheeded by Villaruel. Thereafter, a Petition for Mandamus and Damages with Prayer for Preliminary Mandatory
Injunction against Villaruel was filed by respondents with the RTC of Pasay City. The trial court granted the prayer
for preliminary mandatory injunction by respondents. For the continuous defiance by Villaruel of the order, he was
cited in indirect contempt by the RTC. The trial court declared Villaruel in default for his failure to file an answer
and accordingly, respondents was allowed to adduce evidence ex-parte. The trial court rendered a judgment
declaring the mandamus permanent and ordering Villaruel to pay damages to respondents. Villaruel, through the
OSG filed an appeal before the CA. They were granted a period of until Dec. 13, 1996, non-extendible, within which
to file his memorandum. However, the OSG failed to file the memorandum. Subsequently, Solicitor Restituto
Tuando, Jr. who was handling the case was appointed Regional Trial Court judge of Dumaguete City. The case was
re-assigned to Assistant Solicitor Luciano Joson, Jr. The Court of Appeals issued a Resolution dismissing petitioners
appeal for failure to file the required memorandum. The Resolution became final and executory on 14 June 1997.
Consequently, the respondents filed a Motion for Execution with the trial court. Although served a copy of the
motion for execution, the OSG did not file any opposition. Acting on the motion for execution, the trial court issued a
Writ of Execution. The Sheriff issued a Notice of Sheriffs Sale setting on 23 February 1998 the sale of petitioner’s
real property.

Villaruel, through his new counsel, filed a Motion to Quash the Writ of Execution and to Suspend Sheriff’s Sale. In
his motion, petitioner alleged that the trial courts decision never became final and executory as the trial court
deprived him of his right to due process. He claims that the OSG failed to file his memorandum and that he was not
informed of the dismissal of the appeal and the granting of the motion for execution. He further asserts that the
Ombudsman’s resolution finding Abarca guilty of violation RA 6713 superseded the decision of the trial court. The
CA dismissed his special civil action for certiorari. Hence, this appeal.

Issue:

Whether or not the decision of the Ombudsman finding Abarca guilty of violation RA 6713 superseded the decision
of the trial court?

Held:

No. Settled is the rule that a judgment that has acquired finality becomes immutable and unalterable and may no
longer be modified in any respect except only to correct clerical errors or mistakes. True, this rule admits of certain
exceptions. One of these exceptions is whenever circumstances transpire after the finality of the decision rendering
its execution unjust and inequitable. This, however, is not the case here. In the present case, the Ombudsman issued
his Resolution prior to the finality of the trial courts decision. The Ombudsman issued his Resolution on 22 January
1997 while the trial courts decision became final and executory on 14 June 1997. Therefore, the resolution of the
Ombudsman is not a supervening event to warrant the stay of the execution of the decision of the trial court.
Remedial Law I| Page 219

The action filed by the petitioner before the Ombudsman is completely different from the action instituted by
respondents before the trial court. The two actions, which are clearly separate and distinct from each other,
presented two different causes of action. Petitioner’s cause of action arose from respondents alleged violation of
certain provisions of RA 6713 whereas respondents’ cause of action resulted from petitioner’s refusal to recall
respondents to their mother unit at CATC. In the administrative case before the Ombudsman, the issue was
whether respondents were guilty of violating RA 6713. The findings of the Ombudsman did not render the execution
of the trial courts decision unjust and inequitable. The resolution of the Ombudsman finding Abarca guilty of
violating Section 7(d) of RA 6713 did not state that petitioner had a valid reason to detail respondents to the Office
of Undersecretary Cal.
Remedial Law I| Page 220

Morta vs. Bagagñan


A.M. No. MTJ-03-1513. November 12, 2003

Facts:

Spouses Morta filed an administrative complaint against Judge Bagagan and Sheriff Matias charging them of gross
ignorance of the law, incompetence, bias and delay and gross ignorance of the law, negligence and connivance with
the defendants respectively. The case stemmed from an action for Damages with Prayer for a Writ of Preliminary
Injunction, filed by the Spouses against a certain Jaime Occidental. They allege that the MTC of Guinobatan, Albay
rendered a decision in their favor and this was appealed before the RTC which dismissed the action stating that it
was under the original and exclusive jurisdiction of DARAB as it was tenancy related. This was affirmed by the CA.
However, the First Division of the Supreme Court affirmed the MTC’s ruling and set aside the RTC and CA’s
decision. They now complain that despite the fact that the decision of the Supreme Court in the aforesaid case had
already become final and executory, the respondent Judge still refused to issue a writ of possession in their favor.

As against the respondent Sheriff, the complainants aver[red] that through his ignorance, negligence and
connivance with the defendants, he failed to execute in full the writ of execution that had been previously issued by
the court in Civil Case Nos. 481 and 482. Moreover, it took respondent Sheriff a long time before he finally
submitted his Sheriff's Return of Service on the Writ of Execution.

In his answer, Judge Bagagan stated that he had denied complainants Motion for the issuance of a writ of
possession because, by the time Civil Case Nos. 481 and 482 were finally decided by this Court on June 10, 1999,
they had already been ousted from the lots in question pursuant to the Decisions in DARAB Case No. 2413 and Civil
Case No. 1920. In Civil Case No. 1920, respondent judge ordered complainants to vacate the disputed lots. On the
other hand, the DARAB Decision, which became final and executory on October 27, 1998, directed them to cease and
desist from disturbing the peaceful possession of therein Petitioner Jaime Occidental.

Respondent Sheriff Matias admitted in his Comment that there was delay in the full implementation of the Writ of
Execution in Civil Case Nos. 481 and 482. Explaining that the delay was due to his heavy workload and thus
unintentional, he begged for compassion from this Court.
Issue:
(1) Whether or not Judge Bagagan acted correctly in not issuing the writ of possession?
(2) Whether Sheriff Matias is administratively liable?
Held:
(1) Yes. The respondent judge acted correctly in not issuing a writ of execution/possession. Besides, the latter’s
Order directing defendants not to molest complainants in their peaceful possession was rendered moot when they
were ousted from the disputed lots by virtue of the final and executory judgments in Civil Case No. 1920 and
DARAB Case No. 2413.
Indeed, the execution of a final judgment may be refused, as in this case, when there has been a change in the
situation of the parties that would make its execution inequitable.
(2) Yes. He was remiss in his duty to implement the Writ fully in Civil Case Nos. 481 and 482. Those tasked to
implement court orders and processes must see to it that the final stage in the litigation process -- the execution of
judgment -- be carried out promptly.
There is no mistaking the mandatory character of the period prescribed under Section 14 of Rule 39 of the Revised
Rules of Court on the Return of a Writ of Execution. A similar rule is stated in Administrative Circular No. 12
dated October 1, 1985, and incorporated in the Manual for Clerks of Court. According to this Circular, all sheriffs
and deputy sheriffs shall submit to the judge concerned a report on actions taken on all writs and processes assigned
to them within 10 days from receipt. Per the records of this case, a Writ of Execution was issued on November 22,
1999 in Civil Case Nos. 481 and 482. Respondent Sheriffs Return of Service of that Writ was filed only on May 25,
2000, however, or six months thereafter. There is nothing in the records showing that he submitted before then a
Remedial Law I| Page 221

periodic report on the actions he had taken on the Writ every 30 days from the date of receipt as required. On the
contrary, the Report indicates that the Writ was partially executed on December 15-28, 1999 and January 11, 2000;
and that the damages adjudged were partly paid in the amount of P3,500 plus one unit of Karaoke machine. But it
was only on May 25, 2000, that this matter was reported to the trial court.

C. Properties Exempt from Execution

D’ Armoured Security and Investigation Agency, Inc. vs. Orpia


G.R. No. 151325. June 27, 2005

Facts:

The security guards employed by DArmoured Security and Investigation Agency, Inc., assigned to Fortune Tobacco,
Inc. filed with the Labor Arbiter a complaint for illegal dismissal and various monetary claims against DArmoured
Security and Fortune Tobacco. The Labor Arbiter decided in favor of the security guards. Fortune Tobacco appealed
to the NLRC while DArmoured Security did not appeal. NLRC dismissed the complaint against Fortune Tabacco.
This Decision became final and executory. Thus, the award specified in the Decision of the Arbiter became the sole
liability of DArmoured Security. The records were then remanded to the Arbiter for execution. Upon the security
guard’s motion, the Arbiter issued a writ of execution. Eventually, the sheriff served a writ of garnishment upon the
Chief Accountant of Foremost Farms, Inc., a corporation with whom DArmoured Security has an existing services
agreement. Thus DArmoured Security’s receivables with Foremost were garnished.
DArmoured Security filed with the NLRC a Motion to Quash/Recall Writ of Execution and Garnishment claiming
that its monthly receivables are exempt from execution. The Arbiter denied the motion. DArmoured Security
motion for reconsideration was denied, hence it appealed to the NLRC. NLRC dismissed the appeal. Its motion for
reconsideration was also denied. Forthwith, DArmoured Security filed with CA a petition for certiorari and
prohibition with prayer for issuance of a writ of preliminary injunction. CA dismissed the petition. Hence, this
petition for review on certiorari under Rule 45.
Issue:
Whether or not the petitioner’s monthly receivables from the Foremost Farms, Inc. (garnishee) are exempt from
execution?
Held:
No. Section 13 of Rule 39 of the Rules of Court is plain and clear on what properties are exempt from execution:

(i) So much of the salaries, wages or earnings of the judgment obligor for his personal services within the
four months preceding the levy as are necessary for the support of his family.

The exemption under this procedural rule should be read in conjunction with the Civil Code, the substantive law
which proscribes the execution of employees wages, thus:

ART. 1708. The laborers wage shall not be subject to execution or attachment, except for debts incurred for food,
shelter, clothing and medical attendance.

Obviously, the exemption under Rule 39 of the Rules of Court and Article 1708 of the New Civil Code is meant to
favor only laboring men or women whose works are manual. In this context, exemptions under this rule are confined
only to natural persons and not to juridical entities such as petitioner.

It stands to reason that only natural persons whose salaries, wages and earnings are indispensable for his own and
that of his familys support are exempted under Section 13 (i) of Rule 39 of the Rules of Court. Undeniably, a
corporate entity such as petitioner security agency is not covered by the exemption.
Remedial Law I| Page 222

D. Revival of Judgment

Panotes vs. City Townhouse Development Corporation


G.R. No. 154739, January 23, 2007

Facts:

Rogelio Panotes filed a Complaint with the National Housing Authority (NHA) against Provident Village
Homeowners Association, Inc., against Provident Securities Corporation (PROSECOR), owner-developer of the
Provident Village in Marikina City because PROSECOR’s failure to provide an open space in the said subdivision.
During the proceedings before the NHA, an ocular inspection showed that the subdivision has no open space. The
NHA found, however, that Block 40, with an area of 22,916 square meters, could be utilized as open space. Thus,
NHA, in a Resolution, directed PROSECOR to provide an open space which is Block 40. Considering that
PROSECOR did not appeal from the NHA Resolution, it became final and executory. When Panotes filed a motion
for execution of the NHA Resolution, it was found that the records of the case were “mysteriously missing.” Hence,
his motion “was provisionally dismissed” without prejudice.

Meanwhile, PROSECOR sold to City Townhouse Development Corporation (CTDC), respondent, several lots in the
subdivision. Among the lots sold were those comprising Block 40. CTDC was unaware of the NHA Resolution
ordering PROSECOR to have Block 40 utilized as open space of Provident Village.

Eventually, Panotes was succeeded by Araceli Bumatay as president of the Provident Village Homeowners
Association, Inc. She then filed with the Housing and Land Use Regulatory Board (HLURB) a complaint for revival
of the NHA Resolution. HLURB ruled in favor of Bumatay. The Office of the President affirmed the HLURB
decision. However, the CA reversed the HLURB decision and dismissed Bumatay’s Complaint for revival of
judgment.

Issue:

Whether or not the NHA Resolution can be enforced against CTDC?

Held:

No. An action for revival of judgment is no more than a procedural means of securing the execution of a previous
judgment which has become dormant after the passage of five years without it being executed upon motion of the
prevailing party. It is not intended to re-open any issue affecting the merits of the judgment debtor’s case nor the
propriety or correctness of the first judgment.

Here, the original judgment or the NHA Resolution sought to be revived was between Rogelio Panotes and
PROSECOR, not between Araceli Bumatay and respondent CTDC.

CTDC purchased from PROSECOR Block 40 in the said village, not as an owner-developer like PROSECOR, but as
an ordinary buyer of lots. Even after the sale, CTDC did not become an owner- developer. The Deed of Sale executed
by CTDC, as buyer, and PROSECOR, as seller, shows that the subject matter of the sale is the unsold lots
comprising Block 40 within the subdivision to CTDC. The contract does not include the transfer of rights of
PROSECOR as owner-developer of the said subdivision. Clearly, there is no basis to conclude that CTDC is the
successor-in-interest of PROSECOR.

It bears stressing that when CTDC bought Block 40, there was no annotation on PROSECOR’s title showing that
the property is encumbered. In fact, the NHA Resolution was not annotated thereon. CTDC is thus a buyer in good
faith and for value, and as such, may not be deprived of the ownership of Block 40.

Furthermore, strangers to a case, like CTDC, are not bound by the judgment rendered by a court. It will not divest
the rights of a party who has not and never been a party to a litigation. Execution of a judgment can be issued only
against a party to the action and not against one who did not have his day in Court.
Remedial Law I| Page 223

E. Execution of Judgment by Motion

Rizal Commercial Banking Corporation vs. Serra


G.R. No. 203241. July 10, 2013

Facts:

Federico Serra is the owner of a 374 square meter parcel of land located in Masbate. Serra and RCBC entered into a
Contract of Lease with Option to Buy, wherein Serra agreed to lease his land to RCBC for 25 years. Serra further
granted RCBC the option to buy the land and improvement within 10 years. RCBC informed Serra of its decision to
exercise its option to buy the property. However, Serra replied that he was no longer interested in selling the
property. RCBC filed a Complaint for Specific Performance and Damages against Serra in the RTC Makati. The
RTC Makati initially dismissed the complaint. However, the RTC Makati reversed itself and ordered Serra to
execute and deliver the proper deed of sale in favor of RCBC. Serra appealed to the CA. Meanwhile, Serra donated
the property to his mother, Leonida Ablao. Ablao, then, sold the property to Liok. A new land title was issued in
favor of Liok. Thus, RCBC filed a Complaint for Nullification of Deed of Donation and Deed of Sale with
Reconveyance and Damages against Liok, Ablao and Serra before the RTC Masbate. Meanwhile, the CA, and later
the Supreme Court, affirmed the order of the RTC Makati in the Specific Performance case. The decision in the
Specific Performance case became final and executory upon entry of judgment.

The RTC Masbate ruled in favor of RCBC, declaring the donation in favor of Ablao and the subsequent sale to Liok
null and void. The CA affirmed the RTC Masbate decision. Thus, Liok filed a Petition for Review on Certiorari, while
Serra and Ablao filed a Petition for Certiorari, before this Court. This Court found neither reversible error nor grave
abuse of discretion on the CA’s part. RCBC moved for the execution of the decision in the Specific Performance case.
RCBC alleged that it was legally impossible to ask for the execution of the decision prior to the annulment of the
fraudulent transfers made by Serra. Thus, the period to execute by motion was suspended during the pendency of
the Annulment case. Serra filed his comment and opposition to the motion. Serra insisted that the motion for
execution was already barred by prescription and laches, and that RCBC was at fault for failing to register as lien in
the original title the Contract of Lease with Option to Buy. The RTC Makati denied RCBC’s motion for execution as
well as the motion for reconsideration. Thus, RCBC filed this petition.

Issue:

Whether or not RCBC is barred from having its 05 January 1989 decision executed through motion after the lapse of
five years?

Held:

No. The Rules of Court provide that a final and executory judgment may be executed by motion within five years
from the date of its entry or by an action after the lapse of five years and before prescription sets in. This Court,
however, allows exceptions when execution may be made by motion even after the lapse of five years. These
exceptions have one common denominator: the delay is caused or occasioned by actions of the judgment obligor
and/or is incurred for his benefit or advantage. Where the delays were occasioned by the judgment debtor’s own
initiatives and for her advantage as well as beyond the judgment creditor’s control, the five-year period allowed for
enforcement of the judgment by motion is deemed to have been effectively interrupted or suspended.

In the present case, there is no dispute that RCBC seeks to enforce the decision which became final and executory on
15 April 1994. This decision orders Serra to execute and deliver the proper deed of sale in favor of RCBC. However,
to evade his obligation to RCBC, Serra transferred the property to his mother Ablao, who then transferred it to Liok.
Serra’s action prompted RCBC to file the Annulment case. Clearly, the delay in the execution of the decision was
caused by Serra for his own advantage. Thus, the pendency of the Annulment case effectively suspended the five-
year period to enforce through a motion the decision in the Specific Performance case. Since the decision in the
Annulment case attained finality on 3 March 2009 and RCBC’s motion for execution was filed on 25 August 2011,
RCBC’s motion is deemed filed within the five-year period for enforcement of a decision through a motion.
Remedial Law I| Page 224

This Court has reiterated that the purpose of prescribing time limitations for enforcing judgments is to prevent
parties from sleeping on their rights. Far from sleeping on its rights, RCBC has pursued persistently its action
against Serra in accordance with law. On the other hand, Serra has continued to evade his obligation by raising
issues of technicality. While strict compliance with the rules of procedure is desired, liberal interpretation is
warranted in cases where a strict enforcement of the rules will not serve the ends of justice.
Remedial Law I| Page 225

F. Foreign Judgment

Fujiki vs. Marinay


G.R. No. 196049. June 26, 2013

Facts:

Minoru Fujiki (Fujiki) is a Japanese national who married Maria Paz Galela Marinay (Marinay) in the Philippines.
Marinay met another Japanese, Shinichi Maekara (Maekara). Without the first marriage being dissolved, Marinay
and Maekara married in the Philippines. Maekara brought Marinay to Japan. However, Marinay allegedly suffered
physical abuse from Maekara. She left Maekara and started to contact Fujiki. Fujiki and Marinay met in Japan and
they were able to reestablish their relationship. Fujiki helped Marinay obtain a judgment from a family court in
Japan which declared the marriage between Marinay and Maekara void on the ground of bigamy. Fujiki filed a
petition for Judicial Recognition of Foreign Judgment or Decree of Absolute Nullity of Marriage in the RTC. Fujiki
prayed that (1) the Japanese Family Court judgment be recognized; (2) that the bigamous marriage between
Marinay and Maekara be declared void ab initio under Articles 35(4) and 41 of the Family Code of the
Philippines; and (3) for the RTC to direct the Local Civil Registrar to annotate the Japanese Family Court judgment
on the Certificate of Marriage between Marinay and Maekara and to endorse such annotation to the NSO.

The RTC immediately issued an order dismissing the petition. The RTC took the view that only "the husband or the
wife," in this case either Maekara or Marinay, can file the petition to declare their marriage void, and not Fujiki.
Fujiki moved that the order be reconsidered. He argued that A.M. No. 02-11-10-SC contemplated ordinary civil
actions for declaration of nullity and annulment of marriage. Thus, A.M. No. 02-11-10-SC does not apply. The RTC
resolved to deny petitioner’s motion for reconsideration. The Court required respondents to file their comment on the
petition for review. The public respondents, the Local Civil Registrar of Quezon City and the Administrator and
Civil Registrar General of the NSO, participated through the Office of the Solicitor General. Instead of a comment,
the Solicitor General filed a Manifestation and Motion.

The Solicitor General agreed with the petition. He prayed that the RTC’s "pronouncement that the petitioner failed
to comply with x x x A.M. No. 02-11-10-SC x x x be set aside" and that the case be reinstated in the trial court for
further proceedings. The Solicitor General argued that Fujiki, as the spouse of the first marriage, is an injured party
who can sue to declare the bigamous marriage between Marinay and Maekara void. The Solicitor General
cited Juliano-Llave v. Republic which held that Section 2(a) of A.M. No. 02-11-10-SC does not apply in cases of
bigamy. The Solicitor General contended that the petition to recognize the Japanese Family Court judgment may be
made in a Rule 108 proceeding. Moreover, the Solicitor General argued that there is no jurisdictional infirmity in
assailing a void marriage under Rule 108, citing De Castro v. De Castro and Niñal v. Bayadog which declared that
"[t]he validity of a void marriage may be collaterally attacked."

Issue:

Whether or not a foreign judgment decreeing a marriage void may be recognized in the Philippines?

Held:

Yes. For Philippine courts to recognize a foreign judgment relating to the status of a marriage where one of the
parties is a citizen of a foreign country, the petitioner only needs to prove the foreign judgment as a fact under the
Rules of Court. To be more specific, a copy of the foreign judgment may be admitted in evidence and proven as a fact
under Rule 132, Sections 24 and 25, in relation to Rule 39, Section 48(b) of the Rules of Court. Petitioner may prove
the Japanese Family Court judgment through (1) an official publication or (2) a certification or copy attested by the
officer who has custody of the judgment. If the office which has custody is in a foreign country such as Japan, the
certification may be made by the proper diplomatic or consular officer of the Philippine foreign service in Japan and
authenticated by the seal of office.

A foreign judgment relating to the status of a marriage affects the civil status, condition and legal capacity of its
parties. However, the effect of a foreign judgment is not automatic. To extend the effect of a foreign judgment in the
Philippines, Philippine courts must determine if the foreign judgment is consistent with domestic public policy and
other mandatory laws. Article 15 of the Civil Code provides that "[l]aws relating to family rights and duties, or to the
status, condition and legal capacity of persons are binding upon citizens of the Philippines, even though living
Remedial Law I| Page 226

abroad." This is the rule of lex nationalii in private international law. Thus, the Philippine State may require, for
effectivity in the Philippines, recognition by Philippine courts of a foreign judgment affecting its citizen, over whom
it exercises personal jurisdiction relating to the status, condition and legal capacity of such citizen.

Section 48(b), Rule 39 of the Rules of Court provides that a foreign judgment or final order against a person creates
a "presumptive evidence of a right as between the parties and their successors in interest by a subsequent title."
Moreover, Section 48 of the Rules of Court states that "the judgment or final order may be repelled by evidence of a
want of jurisdiction, want of notice to the party, collusion, fraud, or clear mistake of law or fact." Thus, Philippine
courts exercise limited review on foreign judgments. Courts are not allowed to delve into the merits of a foreign
judgment. Once a foreign judgment is admitted and proven in a Philippine court, it can only be repelled on grounds
external to its merits, i.e. , "want of jurisdiction, want of notice to the party, collusion, fraud, or clear mistake of law
or fact." The rule on limited review embodies the policy of efficiency and the protection of party expectations, as well
as respecting the jurisdiction of other states.

There is therefore no reason to disallow Fujiki to simply prove as a fact the Japanese Family Court judgment
nullifying the marriage between Marinay and Maekara on the ground of bigamy. While the Philippines has no
divorce law, the Japanese Family Court judgment is fully consistent with Philippine public policy, as bigamous
marriages are declared void from the beginning under Article 35(4) of the Family Code. Bigamy is a crime under
Article 349 of the Revised Penal Code. Thus, Fujiki can prove the existence of the Japanese Family Court judgment
in accordance with Rule 132, Sections 24 and 25, in relation to Rule 39, Section 48(b) of the Rules of Court.
Remedial Law I| Page 227

G. Res Judicata

City of Cebu vs. Dedamo, Jr.


G.R. No. 172852. January 30, 2013

Facts:

The present controversy is an off-shoot of civil case for eminent domain over two (2) parcels of land owned by
spouses Spouses Dedamo, filed by the petitioner before the RTC of Cebu City. The petitioner immediately took
possession of the lots after depositing P51,156.00 with the PNB. During the pendency of the case, the petitioner and
Spouses Dedamo entered into a Compromise Agreement whereby the latter agreed to part with the ownership of the
parcels of land in favor of the former in consideration of P1,786,400.00 as provisional payment and just
compensation in an amount to be determined by a panel of commissioners. They recommend the sum
of P20,826,339.50 as just compensation and it was approved by the RTC. The RTC Order was affirmed by the CA
and then by the Court, when the matter was elevated for review. When the said decision became final and
executory, the case was remanded for execution to the RTC, before which, a motion for the issuance of a writ of
execution was filed by Spouses Dedamo. The RTC granted the motion and ordered the issuance of the writ. In the
meantime, Spouses Dedamo passed away and they were substituted in the case by herein respondent. The petitioner
paid the respondent the sum of P19,039,939.50 which is the difference between the just compensation due and the
provisional payment already made. The respondent filed a motion before the RTC to order the petitioner to pay
interest on the just compensation computed from the time of actual taking of the lands.

The RTC denied the motion and ruled that it can no longer amend a final and executory judgment that did not
specifically direct the payment of legal interest. Adamant, the respondent sought recourse before the CA asserting
that the petitioner is liable to pay: (a) 12% legal interest on the unpaid balance of the just compensation computed
from the time of actual taking of the property up to the date of payment of just compensation; and (b) 12% legal
interest from the time the decision awarding just compensation became final and executory on September 20, 2002
until its satisfaction on December 23, 2003.

The CA rejected the respondents first claim since the issue was belatedly raised and found the respondents second
contention meritorious. The CA awarded legal interest accruing from the time the RTC Order dated December 27,
1996 awarding just compensation was affirmed with finality by the Supreme Court up to the time of full payment
thereof in line with the ruling in Eastern Shipping Lines, Inc. v. Court of Appeals that when a court judgment
awarding a sum of money becomes final and executory, it shall earn legal interest of 12% per annum reckoned from
such finality until satisfaction. In G.R. No. 172942, the Supreme Court denied respondent’s petition on August 22,
2006 for failure to sufficiently show that the CA committed any reversible error in the questioned judgment. This
present petition, initiated by the City of Cebu, the petitioner seeks to set aside the same CA Decision awarding legal
interest to respondet.

Issue:

Whether or not the petition should be denied on the ground of res judicata in the mode of conclusiveness of
judgment?

Held:

Yes. A perusal of the allegations in the present case evidently shows that the petitioner broaches the issues
similarly raised and already resolved in G.R. No. 172942.

Under the principle of conclusiveness of judgment, when a right or fact has been judicially tried and determined by a
court of competent jurisdiction, or when an opportunity for such trial has been given, the judgment of the court, as
long as it remains unreversed, should be conclusive upon the parties and those in privity with them. Stated
differently, conclusiveness of judgment bars the re-litigation in a second case of a fact or question already settled in
a previous case.

The adjudication in G.R. No. 172942 has become binding and conclusive on the petitioner who can no longer
question the respondent’s entitlement to the 12% legal interest awarded by the CA. The Courts determination in
Remedial Law I| Page 228

G.R. No. 172942 on the reckoning point of the 12% legal interest is likewise binding on the petitioner who cannot re-
litigate the said matter anew through the present recourse. Thus, the judgment in G.R. No. 172942 bars the present
case as the relief sought in the latter is inextricably related to the ruling in the former.

H. Terceria

PSALM vs. Maunlad Homes


G.R. No. 215933. February 8, 2017
Facts:

Respondent Maunlad Homes, Inc. filed with the Municipal Trial Court in Cities (MTCC), Malolos City, Bulacan, an
unlawful detainer case against National Power Corporation (NPC). The MTCC issued its Decision, ordering NPC to
vacate the subject premises and surrender physical possession thereof to respondent. The NPC appealed the
decision to the Regional Trial Court (RTC) of Malolos City, Bulacan. However, it affirmed the decision of the MTCC.
Respondent filed a Motion for Execution which was opposed by the NPC. The NPC also filed a motion for
reconsideration of the RTC decision. RTC denied the MR and granted respondent's motion for execution. A Writ of
Execution pending appeal was issued. The sheriff served a Notice of Demandof payment to the NPC.

Respondent then filed an urgent motion for issuance of a Break Open Order since the sheriff who tried to implement
the writ of execution, by serving the notice of levy on the NPC Warehouse at Barangay Lagundi, Mexico, Pampanga,
was prevented by the security guards assigned therein. The NPC argued that the warehouse is being used both by it
and the Power Sector Assets and Liabilities Management Corporation (herein petitioner PSALM), an entity created
and existing by virtue of Republic Act No. 9136, the Electric Power Industry Reform Act of 2001 (EPIRA Law); that
the said law provides that the ownership and all generation assets, IPP contracts and other NPC disposable assets
are transferred to PSALM; and that as of the moment, the ownership of the said items stored in the said warehouse
cannot be established with certainty as they are in the process of determining what properties may be retained by
the latter.

The sheriff issued a Notice of Levy on execution pending appeal of personal properties/sale of seven (7) units
transformer radiator fins, one (1) unit power transformer and four (4) pieces angle bars.

Petitioner PSALM filed an Affidavit of third-party claim with the sheriff pursuant to Section 16, Rule 39 of the
Rules of Court, and alleging that it is the owner of the levied properties pursuant to the EPIRA Law. It also filed a
Manifestation with Urgent Ex Parte Motion for Issuance of Status Quo Order with the RTC arguing that it is the
owner of the subject properties pulled out by the sheriff by operation of law; that it is not a party to the instant case
and therefore cannot be bound by the judgment therein; that the obligation to pay respondent had not been
transferred to it. It also prayed for the nullification of the levy of its properties and restoring their immediate
possession to it. RTC denied the motion of PSALM.

Petitioner filed with the CA a petition for certiorari assailing Break Open Order, the notice of levy on execution
pending appeal, the Order dated denying the motion for issuance of Status Quo Order and the third-party claim, and
the notice of sale on execution of personal properties. It alleged that it has no adequate remedy available from the
writs and processes issued by the RTC, and that it acted without or in excess of jurisdiction in issuing the assailed
orders despite the fact that petitioner is the owner of the subject properties. The CA dismissed the petition for
certiorari for being an incorrect remedy. Petitioner filed a motion for reconsideration, which was denied by the CA.

Issue:

Whether or not the third-party claim of PSALM was proper?

Held:

No. The power of the court in executing judgments extends only to properties unquestionably belonging to the
judgment debtor alone. An execution can be issued only against a party and not against one who did not have his
day in court. The duty of the sheriff is to levy the property of the judgment debtor not that of a third person. For, as
the saying goes, one man’s goods shall not be sold for another man’s debts. Thus, if the property levied by virtue of a
writ of execution is claimed by a third person who is not the judgment obligor, Section 16 of Rule 39 of the 1997
Remedial Law I| Page 229

Rules of Civil Procedure provides for the remedy of such third party claimant. The third-party claimant may execute
an affidavit of his title or right to the possession of the property levied, and serve the same to the officer making the
levy and a copy thereof to the judgment creditor. This remedy is known as terceria. The officer shall not be bound to
keep the property, unless the judgment creditor files a bond approved by the court to indemnify the third-party
claimant in a sum not less than the value of the property levied on. An action for damages may be brought against
the officer within one hundred twenty (120) days from the date of the filing of the bond.

In Spouses Sy v. Hon. Discaya, the court held that the claim of ownership or right of possession to the levied
property by the third-party claimant must first be unmistakably established, thus:

x x x A third person whose property was seized by a sheriff to answer for the obligation of the judgment debtor may
invoke the supervisory power of the court which authorized such execution. Upon due application by the third
person and after summary hearing, the court may command that the property be released from the mistaken levy
and restored to the rightful owner or possessor. What said court can do in these instances, however, is limited to a
determination of whether the sheriff has acted rightly or wrongly in the performance of his duties in the execution of
judgment, more specifically, if he has indeed taken hold of property not belonging to the judgment debtor. The court
does not and cannot pass upon the question of title to the property, with any character of finality. It can treat of the
matter only insofar as may be necessary to decide if the sheriff has acted correctly or not. It can require the sheriff to
restore the property to the claimant's possession if warranted by the evidence. However, if the claimant's proofs do
not persuade the court of the validity of his title or right of possession thereto, the claim will be denied.

You might also like